You are on page 1of 207

APRESENTAO

O Governo do Estado da Paraba , por meio da Secretaria de Estado da Educao, desde


2011 estabeleceu como uma das suas metas, atender os alunos do Ensino Mdio no prvestibular social-Pbvest. A principal inteno que o Pbvest seja mais um instrumento de
incluso social de milhares de jovens e de adultos que frequentam o Ensino Mdio, no caminho
concreto do acesso aos melhores cursos e s melhores instituies de ensino superior. Este
projeto visa preparar, especialmente, os alunos da rede estadual de ensino para as provas do
Exame Nacional do Ensino Mdio ENEM.
No primeiro ano os resultados j comearam a aparecer com alunos aprovados em
2 lugar de Direito , 1 lugar em Engenharia Civil e Odontologia na UEPB, entre outros
excelentes resultados. Em 2012 foram mais de 1.300 aprovados, em diversas universidades.
Na UEPB, por exemplo, 80% dos aprovados ficaram entre os 20 primeiros lugares dos cursos
escolhidos.
Em 2013 nos superamos, 12 alunos aprovados em medicina, mais de 20 alunos
aprovados entre os trs primeiros colocados , nas principais instituies particulares de nosso
estado, alm de muitos aprovados em instituies federais de outros estados.
Em 2015 o Governo do Estado, estender o PBvest para todos os alunos do ensino
mdio da nossa rede.
As inscries so feitas atravs do portal PBvest, e a partir do incio das aulas, os
alunos podem acompanha-las atravs do portal nos polos mais prximos de sua regio ou at
mesmo em sua prpria residncia.
Assim o PBvest se constitui como uma das aes da Secretaria de Estado da Educao,
que solidifica a premissa fundamental de que a educao uma prioridade inequvoca do
processo de soerguimento social, econmico e cultural de um povo. Sabe-se que a educao
no pode tudo e que sem ela, pouco ou nada se pode fazer em prol de um processo
transformador de sociedade.

Alssio Trindade de Barros


Secretrio de Estado da Educao

SECRETARIA DE ESTADO DA EDUCAO

Paraba/Brasil, Janeiro de 2015

QUMICA

PROFESSORES COLABORADORES

CINCIAS DA NATUREZA
-ANTONIO AMRICO FALCONE DE ALMEIDA
-ANTONIO ALENCAR DINIZ
-PELGIO NERCIO
-SERGIO MELQUIOR

CINCIAS HUMANAS
-CAIO A. PEREIRA DE ALMEIDA
-MRIO AUGUSTO ROMERO
-WALDEMIR PINHEIRO
-DINHO ZMBIA

LINGUAGENS E CDIGOS
-SOCORRO ARRUDA
-SRGIO RODRIGUES
-ALTON MOTA
-LEONARDO

MATEMTICA
-GERALDO LACERDA
-RMULO LINS

SECRETARIA DE ESTADO DA EDUCAO

QUMICA

202

SECRETARIA DE ESTADO DA EDUCAO

QUMICA

SECRETARIA DE ESTADO DA EDUCAO

QUMICA

SECRETARIA DE ESTADO DA EDUCAO

QUMICA

SECRETARIA DE ESTADO DA EDUCAO

QUMICA

AULA 01: PRINCPIOS ELEMENTARES

EXERCCIOS DE FIXAO

Observe o esquema abaixo,


(I)

H , O , N

1) (Amrico-2015)A maioria dos materiais existentes ao nosso redor


no constituda de substncias puras, mas sim de misturas. Uma
substncia pura caracterizada por possuir propriedades fsicas,
como os pontos de fuso e ebulio, constantes. J as misturas so
caracterizadas exatamente pelo oposto:
Misturas so materiais compostos de duas ou mais
substncias, que no tm composio constante e no tm as
propriedades fsicas definidas.
Baseando-se no texto acima, podemos concluir que as
substncias que mantem temperatura fixa durante a fuso e a
ebulio , alm de ser formada por um nico tipo de tomo, so,
a) ar atmosfrico e gs oznio
b) gua mineral sem gs e bronze
c) gs carbnico e gs metano
d) ouro-18K e lingote de alumnio
e) gs nitrognio e iodo.

(..............)

(conjunto)
(II)

H2 , O3 , NH3

(..............)

(conjunto de )
Molculas IGUAIS

Molculas DIFERENTES

............

......................

H2

............... ...................
............................

.......................
(...................)

Observao:
Substncias puras mantm a temperatura constante durante
as mudanas de estado fsico
T(C)
V

2) (Amrico-2015)A calcita , mineral muito comum, corre como


massas rochosas sedimentares enormes e amplamente espalhadas,
nas quais o nico mineral preponderante, sendo o nico presente
em certos calcrios. um constituinte importante de margas
(calcrio contendo 35 a 60% de argila) e pelitos (formados com os
fragmentos ou detritos oriundos da destruio de outras
rochas) calcrios.
As rochas calcrias formam-se por processos orgnicos e
inorgnicos. No primeiro caso resulta da deposio em fundo
marinho, de grandes camadas de material calcrio, sob a forma de
carapaas e esqueletos de animais marinhos. Uma proporo menor
dessas rochas formam-se inorgnicamente pela precipitao direta
de carbonato de clcio(CaCO3 ) em solues aquosas.
O emprego mais importante da calcita na fabricao de
cimentos e cal para argamassa. Tambm usado como corretor de
pH em solos cidos
Considerando-se que o volume de uma amostra de calcita,
pesando 35,75g, 13cm3 , haver flutuao da amostra de calcita no
lquido,
a) tetracloreto de carbono (d = 1,60 g/ cm3 )
b) brometo de metileno (d = 2,50 g/ cm3 )
c) tetrabromoetano (d = 2,96 g/ cm3 )
d) etanol (d = 0,79 g/ cm3 )
e) triclorometano (d = 1,48 g/ cm3 )

L+V

Tebulio
S+L

Tfuso

S
tempo

Propriedades Fsicas da matria: Referem-se aos fenmenos


fsicos e no provocam alteraes na estrutura ntima das
substncias, como: ponto de fuso, ponto de ebulio, densidade.
sublimao
sublimao

3)(UEPB-adaptada) Na cincia, qualquer transformao que ocorre


num determinado sistema vista como um fenmeno , que para ser
descrito, necessrio comparar os estados inicial e final do sistema
em questo. Em alguns fenmenos ocorre alterao qumica da
substncia em questo, em outros no.
Com base no texto, podemos considerar um fenmeno fsico,
a)a respirao de um animal.
b)o avermelhamento da l de ao umedecida.
c)a queima da gasolina.
d)a decomposio de um corpo.
e)a extrao do leo do caroo do algodo.
anotaes

SLIDO
(gelo)

fuso
solidificao

LQUIDO
(gua)

vaporizao
(evaporao)
liquefao
(condensao)

GS
GS OU
VAPOR
(vapor d'gua)

ressublimao
(res)
sublim ao

Propriedades Qumicas da Matria: Referem-se aos fenmenos


qumicos e
provocam alteraes
na estrutura ntima das
substncias,
como
as
reaes
qumicas
de
combusto(........................)
,
oxidao
(.........................),
decomposio , etc.

SECRETARIA DE ESTADO DA EDUCAO

QUMICA

AULA 02: MTODOS DE SEPARAO

EXERCCIOS DE FIXAO
1) (AMRICO-2015) Associe as atividades dirias contidas na
primeira coluna com as operaes bsicas de laboratrio e
fenmenos contidos na segunda coluna.
COLUNA I
COLUNA II
(1) preparar um cafezinho
( ) sublimao
(2) coar nata do leite
( ) diluio
(3) adoar o leite
( ) filtrao
(4) usar naftalina na gaveta
( ) extrao
(5) adicionar gua no suco
( ) dissoluo
de limo para ficar mais fraco

A) MISTURAS HETEROGNEAS
A.I)SLIDO-SLIDO

...............................

..................................

A.II)SLIDO-LQUIDO

Os nmeros da segunda coluna, lidos de cima para baixo, so:


a) 3, 2, 5, 4, 1
d) 3, 2, 4, 5, 1
b) 1, 3, 4, 5, 2
e) 4, 5, 2, 1, 3
c) 4, 3, 2, 1, 5

............................

A. III)LQUIDO-LQUIDO

2)(UFPB-adaptada) Nas salinas, para se obter o sal cloreto de sdio


separado da gua do mar, usa-se a energia do sol e a ao dos
ventos. Alm do processo de evaporao, utilizado nas salinas para
separar este tipo de mistura, pode-se empregar o mtodo de
separao esquematizado ao lado. Observando-se o esquema e
considerando que uma soluo aquosa de cloreto de sdio encontrase no balo, correto afirmar:

...........................................

......................................
Condensador

B) MISTURAS HOMOGNEAS
B.1) SLIDO-LQUIDO

Erlenmeyer

a) O esquema representado corresponde ao processo de destilao


fracionada, pois os dois componentes so lquidos.
b) Uma soluo aquosa de cloreto de sdio no pode ser separada
em seus componentes, pois, uma mistura homognea.
c) O bico de gs fornece calor mistura que evapora e coletada no
Erlenmeyer.
d) Os vapores d`gua formados pelo aquecimento da mistura so
resfriados quando passam pelo condensador e coletados como gua
lquida no Erlenmeyer.
e) No h separao dos componentes da mistura homognea,
porque no h resfriamento dos vapores.
3)(Amrico-2015) 1) Abaixo temos o esquema simplificado
de uma Estao de Tratamento de gua(E.T.A.)

..........................................................................................................
B.2) LQUIDO-LQUIDO

III

Acerca do esquema acima podemos concluir que,


a) Em I, a gua no mais agitada e os flocos vo se depositando
no fundo, separando-se da gua, processo denominado de filtrao.
b) Em II, atravs da decantao o sulfato de alumnio separado da
soluo formada com a gua, atravs da decantao.
c) No tanque III, o carvo ativado utilizado para destilar a matria
orgnica indesejvel.
d) A gua filtrada est limpa, mas ainda pode conter microrganismos
causadores de doenas. Por isso, ela recebe um produto que contm
cloro, que mata os microrganismos, processo denominado de
clorao.
e) todos os processos descritos so qumicos.

..........................................................................................................

SECRETARIA DE ESTADO DA EDUCAO

II

QUMICA

EXERCCIOS DE FIXAO

AULA 3: O TOMO

1)(AMRICO-2015)
O alumnio um elemento qumico de smbolo Al de nmero
atmico 13 e nmero de massa 27u(*) . Na temperatura ambiente
slido, sendo o elemento metlico mais abundante da crosta
terrestre. Sua leveza, condutividade eltrica, resistncia corroso e
baixo ponto de fuso lhe conferem uma multiplicidade de aplicaes,
especialmente nas solues de engenharia aeronutica. Entretanto,
mesmo com o baixo custo para a sua reciclagem, o que aumenta
sua vida til e a estabilidade do seu valor, a elevada quantidade
de energia necessria para a sua obteno reduzem sobremaneira o
seu campo de aplicao, alm das implicaes ecolgicas negativas
no rejeito dos subprodutos do processo de reciclagem, ou mesmo de
produo do alumnio primrio.
dado a Friedrich Whler o reconhecimento do isolamento do
alumnio, em 1827.
Dado: (*) u unidade de massa atmica

A) O tomo de BOHR

DISTRIBUIO DE eK 2
L 8
M 18
N 32
O 32
P 18
Q 8
ELETROSFERA

B) Conceitos Bsicos
B.1) Nmero atmico (Z) : indica o nmero de ....................... contido
no ncleo atmico.

Representao:

Acerca do texto acima podemos concluir que,


a) a distribuio eletrnica do alumnio por camadas K-2;L-11
b) seu tomo neutro apresenta 14 eltrons
c) ele istopo do ction Al3+
d) seu nmero de nutrons 13
e) ele isoeletrnico do nion 9F12) (MACK-SP )Sabendo-se que dois elementos qumicos

6X8
3X3 A
B.2) Nmero de Massa (A) : indica a soma de.....................................
contido no ncleo atmico.

Representao:

ou

B.4) IsbAros :
.................... )
Exemplo:

2
1H

11Na

B.5) IstoNos :
.................... )
Exemplo:

3
1H

258
Md
tomos que apresentam o mesmo n 101
de
24

24

12Mg

258

Md

tomos que apresentam 101


o mesmo n
24
11Na

d)20 e 18.
e) 17 e 16

B.3) IstoPos :
tomos que apresentam o mesmo n
de
....................
portanto
pertencem
ao
mesmo
elemento
qumico(apresentam as mesmas propriedades ................................)
Exemplo:
1
1H

3x 20
2x 8 B

so isbaros podemos concluir que o nmero de nutrons de A e o


nmero atmico de B so, respectivamente,

a)15 e 32.
b)32 e 16.
c)15 e 17.

A = ........... + ............

257
Fm
100
de

24
12Mg

B.6) tomo Neutro : apresentam nmero de .................... igual ao


nmero de ..........................
Exemplo:
A=
A=
24
24
p =
e=
p =
e=
11Na
12Mg
N =
N =

3)(AMRICO-2015)
Considerando
os
elementos
qumicos
Mendelvio,
Frmio
e
Einstnio
abaixo
representados
respectivamente, com nome de seus descobridores ,
elemento

descobridores

258
101 Md

O mendelvio foi sintetizado pela primeira vez


257
252
em Fm
1955 por umaEs
equipe de cientistas norte100
99 por Albert Ghiorso.
americana liderada

257
100 Fm

Foi descoberto em 1955 por uma equipe liderada


252
porEs
Albert
Ghiorso.

produzido
pelo
99
bombardeamento de plutnio com nutron. Fora

da pesquisa bsica nenhum uso para o frmio foi


encontrado.
O elemento foi identificado pelo grupo de
252
pesquisa
formado
por
G.R.Choppin,
99
A. Ghiorso, B.G. Harvey e C.G. Thompson
nos destroos deixados pela exploso da
primeira bomba de hidrognio, em 1952.
De acordo com os dados e seus conhecimento em estrutura atmica,
podemos concluir que,
a)O on Md1+ istopo do elemento Fm.
b)O elemento Es istono do elemento Md.
c)O elemento Fm isbaro do on Md1+.
d)O elemento Md possui massa maior do que o elemento Fm devido
ao maior nmero de nutrons do Md.
e)O ction Md1+ isoeletrnico do tomo de frmio

Es

B.7) ON :
n de prtons

......... n de eltrons

Se n p > n e => .............


Se n p < n e => .............

p=
-2
8O

p=
10Ne

e=

p=
13Al

+3

e=

e=

..........................................................

SECRETARIA DE ESTADO DA EDUCAO

QUMICA

AULA 4: AS LIGAES QUMICAS

EXERCCIOS DE FIXAO

I) A Ligao Inica ocorre por transferncia de eltrons entre


....................(ctions doam eltrons e nions recebem eltrons)

1)(AMRICO-2015) O cloreto de sdio essencial para a vida


animal e tambm um importante conservante de alimentos e um
popular tempero. Sua obteno , a partir da gua do mar, um
processo eficiente e de baixo impacto ambiental, visto que se utiliza
da energia solar e dos ventos para evaporao da gua. Baseandose em seus conhecimentos em ligaes qumicas, podemos concluir
que o cloreto de sdio,
a) uma substncia covalente formada pela combinao de um
metal e um ametal.
b) uma substncia formada por ons que se ligam covalentemente.
c) Tem baixo ponto de fuso devido a pequena atrao entre seus
ons.
d) Apresenta compartilhamento de um conjunto desordenado de
eltrons.
e) condutor de eletricidade quando fundido ou dissociado em
gua.

Observaes:
1)Tendem a doar eltrons tomos que apresentam 1, 2 ou 3
eltrons na ltima camada(camada de valncia) e tendem a receber
eltrons , tomos que apresentam 5, 6 ou 7 eltrons na camada de
valncia.
Exemplos,

2) Compostos inicos so em sua absoluta maioria formado por


metais(como Na, K, Al, Mg, etc.) e ametais(como C,H,O,P,S,Se,N) e
apresentam-se slidos nas condies ambientes, possuem alto ponto
de fuso e ebulio e conduzem corrente eltrica quando no estado
lquido ou em soluo aquosa(dissociados)
II)A Ligao Covalente ocorre por emparelhamento de eltrons
entre ametais. Quando os ametais apresentam eletronegatividade
diferente a ligao dita covalente POLAR (HCl , BeF2) , quando as
eletronegatividades so iguais, covalente APOLAR (O3 , N2).
Observao:
Molculas apolares so aquelas cujo momento dipolo resultante
nulo ( como substncias simples, hidrocarbonetos, molculas que
contm berlio e boro e molculas do carbono tipo CX2 ou CX4).
Molculas polares apresentam momento dipolo resultante diferente
de zero, como H2O e NH3 , por exemplo.
Nos compostos orgnicos, molculas que apresentam muitos
tomos de carbono so insolveis em gua(apolares), j molculas
que apresentam grupos de alta polaridade como ( -OH , -NH2 , C=O),
So grupos polarizados e aumenta a solubilidade em gua. Assim
como tambm so polares os grupos inicos como COO-Na+ , por
exemplo.

2)(AMRICO-2015) A tabela abaixo relaciona o ponto de fuso e a


solubilidade em gua de algumas substncias,
Substncia
Ponto de fuso (C)
solubilidade
A
993
alta
B

-63,5

455

baixa
insolvel

Baseando-se nestas duas propriedades , podemos concluir que,


a) a substncia A covalente apolar.
b)a substncia B inica.
c)a substncia C covalente polar.
d)a substncia B um slido nas condies ambientes.
e)a substncia A um composto inico e dissociado em gua
conduzir eletricidade.
3)(AMRICO-2015) A molcula de DNA composta por uma fita
dupla de nucleotdeos. As cadeias de nucleotdeos so formadas
por uma pentose (acar de cinco carbonos) associada a um ou
mais grupos fosfato e a uma base nitrogenada.
As quatro bases nitrogenadas contidas no DNA so :
adenina(A) , citosina(C), guanina(G) e timina(T), abaixo
representadas:

SEMELHANTE DISSOLVE SEMELHANTE


III) FORAS INTERMOLECULARES As substncias que tem
ligaes covalentes podem ser, em temperatura ambiente, slida,
liquida ou gasosa. Isto mostra que as interaes entre estas
molculas
podem
ser
maiores
ou
menores.
Existem
trs
tipos
de
interaes
intermoleculares,
III.A) Foras de London: Esta interao intermolecular pode ser
chamada tambm de dipolo induzido ou Foras de Van der Waals.
a interao mais fraca de todas e ocorre todo tipo de molcula,
mas predomina em molculas apolares. Neste caso, no h
atrao eltrica entre estas molculas.
III.B) Dipolo-Dipolo: Esta interao intermolecular pode ser
chamada
tambm
de
dipolo
permanente
ou
dipolar.
Ocorre entre molculas polares como,

III.C) Pontes de Hidrognio : Esta interao intermolecular pode ser


chamada tambm de Ligaes de Hidrognio. Ocorre sempre entre
molculas que contenham o hidrognio e um tomo muito
eletronegativo, como flor, oxignio e nitrognio

Acerca do esquema acima , podemos concluir que,


a) s a guanina apresenta grupos polares.
b) S a citosina forma ligao de hidrognio.
c) todas as bases nitrogenadas apresentam ligaes inicas.
d) as duas cadeias unem-se atravs de pontes de hidrognio
entre as bases nitrogenadas dos nucleotdeos.
e) as bases nitrogenadas apresentam-se como molculas
apolares.

Pontes de hidrognio

SECRETARIA DE ESTADO DA EDUCAO

10

QUMICA

EXERCCIOS DE FIXAO

AULA 5: AS FUNES CIDO e BASE


I) FUNO CIDO SEGUNDO ARRHENIUS
cido toda substncia que , em soluo aquosa, libera ...............
Exemplo:

HX(aq)

(fenmeno da IONIZAO)

Quando em soluo aquosa um cido libera um H+ ,


denominado .................................., quando libera dois H+
denominado ..................., trs H+ ........................, quatro em
diante .................................
I.A) Nomenclatura de Hidrcidos(HxE) cidos no oxigenados
cido + ........................ + DRICO
Nome de E
So fortes apenas HCl, HBr e HI
Exemplos:
HCl .................................................. HCN.........................................
HBr...................................................
H2S.........................................
I.B)Nomenclatura de oxicido(HxEOy) cidos oxigenados
prefixo
sufixo
NOX de E
+1, +2
Hipo
oso
+3,+4
...
oso
+5,+6
...
ico
+7
(Hi)per
ico
Cuidado!

O B+3 e C+4 apresentam o sufixo ICO.

Nos cidos de frmula geral HxE2O7 usamos o prefixo


piro e sufixo ico.

Quando a diferena (nOxignio-nH+) for 2 o cido


forte, quando =1 moderado e =0 ou contenha carbono,
fraco.
Exemplos:
HClO ...................................................

Os anticidos neutralizam a hiperacidez gstrica atravs de uma


reao direta com o cido clordrico. Essa capacidade neutralizante
varia de acordo com a dosagem e associaes

H2SO4 ................................................. H4P2O7 ...................................


HMnO4 ...............................................
II) FUNO BASE SEGUNDO ARRHENIUS
Base toda substncia que , em soluo aquosa, libera ...............
Exemplo:

2)(UFBB) Os cidos so substncias qumicas sempre presentes no


cotidiano do homem. Por exemplo, durante a amamentao, era
comum usar-se gua boricada (soluo aquosa que contm cido
brico) para fazer a assepsia do seio da me; para limpezas mais
fortes da casa, emprega-se cido muritico (soluo aquosa de
cido clordrico); nos refrigerantes, encontra-se o cido carbnico;
e, no ovo podre, o mau cheiro devido presena do cido
sulfdrico). Estes cidos podem ser representados, respectivamente,
pelas seguintes frmulas moleculares:
a) H3BO3 , HCl , H2CO2 e H2SO4
b) H2BO3 , HCl , H2CO3 e H2S
c) H3BO3 , HClO3 , H2SO3 e H2CO2
d) H2BO3 , HClO4 , H2S
e H2CO3
e) H3BO3 , HCl , H2CO3 e H2S
3)(AMRICO-2015)Diz a bula de um medicamento genrico:

H3BO3 ..................................

HNO2 ................................................... H2CO3 ..................................

M(OH)x (aq)

1)(AMRICO-2015) Os sais do cido perclrico(HClO4) so


oxidantes poderosos que frequentemente so usados em
composies explosivas, incluindo propelentes de foguetes, o cido
fosfrico (H3PO4) utilizado Indstria de fertilizantes, indstria de
produo de sal mineral para alimentao animal, formulao de
detergentes e
refrigerantes, o cido brico (H3BO3)
frequentemente utilizado como inseticida relativamente atxico, para
matar baratas,
cupins, formigas, pulgas
e
tambm
como
antissptico, j o cido sulfrico (H2SO4) importante para o
processamento de minrios, refinao de petrleo e no tratamento de
guas residuais. Alm de ser um componente vital de muitos
produtos qumicos.
Considerando-se a fora crescente de acidez dos cidos citados
teremos,
a) HClO4 , H2SO4 , H3PO4 e H3BO3
b) H2SO4 , H3PO4 , H3BO3 e HClO4
c) H3BO3 , H3PO4 , H2SO4 e HClO4
d) H3BO3 , H3PO4 , HClO4 e H2SO4
e) HClO4 , H2SO4 , H3BO3 e H3PO4

(fenmeno da DISSOCIAO)

Observao : As bases de alcalinos (como Li, Na, K,...) so as


nicas fortes e solveis , a base da amnia NH4OH a nica fraca
e solvel.
II.A) Nomenclatura das bases : de metais alcalinos e alcalinos
terrosos (Li.Na,K, Mg, Ca,...) alm de metais como Zn+2 , Al+3 e Ag+1,
alm do on amnio,
, faz-se da seguinte forma:

Considerando-se que o fragmento da bula acima foi retida de um


medicamento que representa a funo base, e sabendo-se que o
medicamento est se apresentando como uma suspenso com a
gua mostrando a pouca solubilidade desta base em gua, alm de
ser considerada uma base fraca, podemos concluir que trata-se do:
a) hidrxido de sdio.
b) hidrxido de amnio
c) hidrxido de clcio.
d) hidrxido de alumnio.
e) hidrxido de potssio.
4)(AMRICO-2015)A alternativa que associa corretamente frmulanome ,
a) Cu(OH)2 - hidrxido de cobre
b) Zn(OH)2 - hidrxido de zinco(II)
c) Mg(OH)2 - hidrxido de mangans
d) Fe(OH)3 - hidrxido de ferro(III)
e) Pb(OH)2 - hidrxido de chumbo(2)

Hidrxido de ................................
Nome do ction
Quando o metal tem nox varivel como:
Fe+2 , +3 ; Cu e Hg +1,+2 ; Pb e Sn +2 , +4 ; Au+1,+3,
devemos indicar entre parnteses o nox do metal, vejamos:
NaOH ........................................ Fe(OH)2 .............................................
Al(OH3 ..................................... CuOH ..
Zn(OH)2.................................... Pb(OH)4

SECRETARIA DE ESTADO DA EDUCAO

11

QUMICA

EXERCCIOS DE FIXAO
1)(UFPB-adaptada)Considere as equaes representadas abaixo.

AULA 6 : AS FUNES SAL e XIDO


I)SAIS : So compostos inicos que possuem pelo menos um ction
diferente de H+ e um nion diferente de OH- .
Podem se obtidos pela reao de neutralizao de um cido
por uma base, vejamos:
Exemplo 1

H3PO4 ( l )

Exemplo 2
Mg(OH)2 + HCl
Mg
Cl
I.A) Nomenclatura de sais no oxigenados , considerando um sal
CnAm , formado por um ction C+m e um nion A-n , teremos,
(nome do nion + ETO) de (nome do ction)
Exemplos
NaCl ........................................

FeCl3 .........................................

AlBr3.........................................

CuS ...........................................

H 2O

Ca(OH)2(aq)

CaO( s )

H O

2
Mg(OH)2 ( s )

NH4Cl

NaOH + HCl
Na
Cl

H 2O

(s)

H 2O

+ 2

Com base nessas equaes, correto afirmar que os compostos


H3PO4 ( l )
, CaO( s )
, Mg(OH)2 ( s ) e NH4Cl (s)
pertencem,
respectivamente, s funes:
a)cido, base, xido e sal
d)sal, cido, base, xido
b)base, xido , sal, cido
e)xido, base, sal, cido
c)cido, xido, base, sal
2)(UFPB) Nas salinas, a gua do mar evaporada pela ao do
vento e do calor, obtendo-se o sal grosso. Em seguida, por
processos de separao, esse sal purificado, resultando no cloreto
de sdio cristalizado, que utilizado na indstria como matria-prima
para produo de diversos produtos qumicos, conforme
exemplificado no esquema abaixo.

I.B) Nomenclatura de sais oxigenados


LEMBRAR
.............................
.............................
.............................
.............................

.........................................
........................................
...................................
.....................................

Exemplos:
Al2(SO4)3 ...................................
Ca3(PO4)2 ..................................
AgNO3 .......................................

CuCO3 .......................................
NaHCO3 .....................................
KMnO4.........................................

II)XIDOS(ExOy) : so compostos binrios onde o elemento mais


eletronegativo o oxignio.

As substncias comercialmente conhecidas como soda custica (I),


gua sanitria (II) e barrilha(IV) so utilizadas como produtos de
limpeza e na fabricao de papel. A nomenclatura correta para
essas substncias respectivamente:
a) Carbonato de sdio, hipoclorito de sdio e hidrxido de sdio
b) Hipoclorito de sdio, carbonato de sdio e hidrxido de sdio
c) Hidrxido de sdio, hipoclorito de sdio e carbonato de sdio
d) Hidrxido de sdio, carbonato de sdio e hipoclorito de sdio
e) Hipoclorito de sdio, hidrxido de sdio e carbonato de sdio
3)(UFPB) Considerando os compostos listados abaixo, identifique a
alternativa que apresenta a correta correspondncia entre composto,
funo qumica e nomenclatura oficial:

II.a)A nomenclatura dos xidos covalentes


(mono,di,tri,...) xido de (di,tri,...) nome de E
Exemplos:
CO2 ..........................................

SO3.........................................

II.b) A nomenclatura dos xidos metlicos


xido de ...........................................
nome do ction metlico
Exemplos:
CaO ............................ Al2O3 ..................... FeO..................................
II.c) CLASSIFICAO

Anfteros (podem funcionar como cidos ou como bases)


os mais importantes so ............ , ............., .............

Neutros (no reagem com cidos, bases nem gua) os


mais importantes so: ........... , .............. , ............

Perxidos (apresentam o grupo [ -O O -]) e frmula geral


(alcalino)2O2 como H2O2 por exemplo.

Bsicos( em contato com a gua formam bases)so


xidos alcalinos em sua grande maioria como Na2O e
CaO (muito utilizado para correo da acidez dos solos)

cidos (em contato com gua formam cidos) so os


principais responsveis pela chuva cida, os mais
importantes so: CO2 , NO2 , SO2 e SO3.

SECRETARIA DE ESTADO DA EDUCAO

12

4)(AMRICO-2015) Calagem uma etapa do preparo do solo para


cultivo agrcola na qual se aplica determinada substncia para
corrigir o pH do solo, para um desenvolvimento satisfatrio das
culturas.
Chuva cida a designao dada chuva, ou qualquer outra
forma de precipitao atmosfrica, cuja acidez seja substancialmente
maior do que a resultante do dixido de carbono (CO2) atmosfrico
dissolvido na gua precipitada
Dois xidos que podem estar contidos nos processos
descritos, so eles, na ordem dada,
a) CaO e CO
d) SO3 e MgO
b) CaO e SO2
e) MgO e Al2O3
c) N2O e ZnO

QUMICA

AULA 7 : ESTEQUIOMETRIA
I)Massa Atmica(M.A.) : o nmero que indica quantas vezes um

do istopo-12 do carbono. As

tomo mais pesado que

massas atmicas de todos os elementos encontram-se na Tabela


Peridica e a unidade utilizada a unidade de massa atmica(u)
II)Massa Molecular : o somatrio das massas atmicas.
III)Massa Molar(MM) : indica o valor da massa atmica ou massa
molecular, expressa em gramas/mol. Portando , podemos dizer que a
massa molar representa o valor da massa atmica ou massa
molecular de um mol (6x1023) tomos ou molculas com valor
expresso em gramas.
Exemplo
Sabendo-se que as massa molares em g/mol do:
C=12
H=1
O=16
a)

2)(AMRICO-2015)A maior parte de cido clordrico preparada por


aquecimento de cloreto de sdio, NaCl, com cido sulfrico H 2SO4
concentrado segundo a equao no balanceada:
NaCl + H2SO4 Na2SO4 + HCl.
A massa de cido sulfrico, em quilogramas necessrios para
produzir 1,46t de cido clordrico concentrado de,
a) 1,96x103
d) 9,8x103
b) 196
e) 19,6
c) 98

Qual a massa de um mol de tomos de Carbono?


1mol de tomos de C = 12g

b)

Qual a massa de um mol de molculas de CO2


12g de Carbono
+

Um mol de molculas de CO2

EXERCCIOS DE FIXAO
1)(AMRICO-2015) Tambm chamada chumbo negro , a grafite tem
mltiplas e importantes aplicaes industriais, embora seja mais
conhecida popularmente por sua utilizao como lpis ou mina de
lpis.
A grafite corresponde a uma das quatro formas alotrpicas
do carbono. As outras so o diamante, o fulereno e o grafeno.
Um risco a lpis consome em mdia 0,1mg/cm 2 de grafite (C).
Dados: massa molar do carbono = 12g/mol
Constante de Avogadro = 6 x 1023
O n de tomos contidos em um trao de10 cm 2 de,
a) 5 x 1023
d) 5 x 1019
b) 6 x 1023
e) 5 x 1021
c) 6 x 1020

2x16 = 32g de Oxignio


44g
IV)Clculo Estequiomtrico

3)(AMRICO-2015) Talvez a reao de neutralizao mais comum


de seu cotidiano a que ocorre depois da ingesto de alimentos
calricos.
O que normalmente ocorre aps aquele almoo de
domingo cujo prato principal era feijoada? Tudo comea com uma
sensao de queimao no estmago, mais conhecida como azia,
que nada mais que um excesso de produo de cido clordrico,
HCl , secretado pelo nosso estmago, que contribui no processo
digestivo desse rgo. A soluo para esse problema ingerir um
anticido estomacal cujo princpio ativo desse medicamento nada
mais que do que uma base, como por exemplo, o hidrxido de
magnsio , Mg (OH)2 .
w.w.w. brasilescola.com.nov.2014-adaptado

Sabendo-se que durante a digesto so produzidos cerca de


100 mL de suco gstrico, cuja concentrao em cido clordrico
0,01mol/L, a massa
de hidrxido de magnsio , em mg,
necessria para neutralizar completamente o cido clordrico
produzido durante a digesto de,
Dados: Massa molar em g/mol: Mg=24 ; O=16 ; H=1
a) 78,0
Uma relao estequiomtrica pode ser feita entre mol-mol,
massa-massa , massa-mol , etc.
Vamos entender estas relaes em uma equao equilibrada,
(MM=16g/mol)

1 CH4
Em mol

1 mol

Em gramas

..............

(MM=32/mol)

(MM=44g/mol)

(MM=18g/mol)

2 O2

1 CO2

2 mols

1 mol

2 mols

...........

................

...............

2 H2O

b) 39,0

c) 156,0

d) 2,0

e) 15,6

4)(AMRICO-2015) O alumnio produzido, basicamente, a partir


da bauxita. Trata-se de um processo de produo difcil, pois exige
muita energia eltrica. A bauxita de cor marrom-avermelhada deve
sofrer um processo de purificao para que se possa extrair a
alumina (Al2O3).A partir da eletrlise da alumina, obtm-se o
alumnio, segundo a reao:
1Al2O3 2Al + 3O2
Sabendo-se que o rendimento do processo de purificao da
bauxita de 50%, a quantidade aproximada de alumnio, em
toneladas , obtida quando se trata 3,06t de bauxita de,
a)0,8
b)1,6
c)0,4
d)1,5
e)0,7
anotaes

Exemplo:
Qual a massa de dixido de carbono obtido na combusto de 3
mols de G.L.P.(C4H10)?

SECRETARIA DE ESTADO DA EDUCAO

13

QUMICA

EXERCCIOS DE FIXAO

AULA 08 : SOLUES

Dado o Coeficiente de solubilidade do nitrato de prata abaixo,


C.S.(AgNO3) 330g/100mL H2 O a 25C
Podemos concluir que:
a) Sero necessrios .............. gramas de AgNO3 para saturar
100mL de gua a 25oC.
b) Sero necessrios .............. gramas de AgNO3 para saturar 50mL
de gua a 25oC.
c) Adicionando-se 329g de AgNO3 a 100mL de gua , a 25oC ,
obteremos uma soluo ......................................
d)Adicionando-se 331g de AgNO3 a 100mL de gua, a 25oC,
obteremos uma soluo ........................... e um .................................
II)CONCENTRAO DAS SOLUES representa uma relao
entre,
Concentrao de uma soluo =
Exemplo 1
Considere a adio de
80g de hidrxido de sdio,
NaOH(MM=40g/mol) , em gua suficiente para preparar 500mL de
soluo.
Pede-se a concentrao da soluo em,

500
450
400
350
300
250
200
150
100
50
0
0

10

20

30
40
50
60
Temperatura (C)

70

80

90

A partir das informaes contidas no grfico, podemos concluir que,


a)A dissoluo da sacarose em gua no favorecida pelo aumento
da temperatura.
b)Uma soluo que contm 250g de sacarose em 100mL de gua a
20C saturada com 25g de precipitado.
c)Uma soluo que contm 200g de sacarose em 100mL de gua a
20C saturada.
d)Uma soluo que contm 302g de sacarose em 100mL de gua a
650C insaturada.
e) Uma soluo contendo 250g de sacarose em 200mL de gua a
45oC , saturada.
2)(UFPB-adaptado)Substncias orgnicas, presentes na gua, se
degradam sob a ao de bactrias, consomem o oxignio(O2),
MM=32g/mol, dissolvido e provocam a morte de lagos e rios.
Considerando-se que a concentrao de oxignio necessria para
manuteno da vida aqutica de 1,25 x 10-4 mol/L, essa
concentrao expressa em outra unidade :
a)4g/L
b) 4mg/L
c) 2mg/dm3
d) 8mg/L
e) 8mg/cm3

a) g/L

b)mg/mL

c)

1)(UFPB-adaptado)O grfico abaixo descreve a variao da


solubilidade da sacarose em gua em funo da temperatura.

Solubilidade (g/100mL)

I)COEFICIENTE DE SOLUBILIDADE(C.S.) a quantidade


necessria de uma substncia para saturar uma quantidade padro
de solvente, em determinada temperatura e presso.
Exemplo:

mol/L

Exemplo 2
Qual a massa de soluto contida em 200g de uma soluo 5%m/m?

Exemplo 3
gua sanitria uma mistura de hipoclorito de sdio , NaClO,
e gua (na proporo de 2% a 2,5%m/v) , considerando-se um
recipiente de 1L, qual a massa de hipoclorito contida em uma gua
sanitria cuja concentrao 2%m/V?

3)(AMRICO-2015) O Peeling de cido retinico consiste na


aplicao desse cido sobre a pele a fim de promover a descamao
progressiva da derme (chegando a atingir a epiderme em
concentraes maiores) e consequente estmulo da produo de
colgeno (substncia que responsvel pela firmeza da pele),
reorganizando as fibras elsticas danificadas pela exposio solar e
melhorando a irrigao e hidratao da derme.
Um dermatologista passou para um paciente , 200mg do
produto dermatolgico a 5%m/m de cido retinico, a quantidade do
cido contida no medicamento ,
a)10mg
b) 100mg
c) 0,5mg
d) 1mg
e) 2mg

4) Um qumico deseja preparar 1500mL de uma soluo 1,4mol/L de


cido clordrico (HCl), diluindo uma soluo 2,8mol/L do mesmo
cido. O volume de soluo que havia na primeira soluo a ser
diluda de,
a) 500mL
b) 750mL
c) 200mL
d) 1000mL
e) 350mL
anotaes

Exemplo 4
Diluindo-se 100mL de soluo 0,1mol/L de cido clordrico com
900mL de gua destilada , obtm-se uma nova soluo com que
concentrao?

SECRETARIA DE ESTADO DA EDUCAO

14

QUMICA

AULA 09 : TERMOQUMICA/CINTICA
I)TERMOQUMICA: estuda o calor envolvido nas reaes qumicas,
quando uma reao ocorre com LIBERAO DE CALOR dizemos
que ela ........................................Quando ela ocorre com absoro
de calor dizemos que ela ......................................
A grandeza termodinmica que mede a quantidade de calor
liberada ou absorvida em uma reao a ENTALPIA(H). E a variao
de entalpia de uma determinada reao determinada por:
=

CaO(s) + H2O(l) Ca(OH)2(aq)

Portanto, a variao de entalpia de uma reao no estado


padro (H0) depende apenas dos estados final e inicial.
Observaes
a)

A entalpia das substncias simples em seu estado mais


estvel nas condies padro(250C e 1atm) vale ZERO.
b) A entalpia de formao (
o calor liberado ou
absorvido na formao de 1mol de uma substncia a partir
de suas substncias simples no estado padro.
c) A variao de entalpia uma propriedade extensiva, ou seja
depende da quantidade de matria.
Exemplos:
a) Calor de formao padro do benzeno lquido.
6 C(s)

+ 3 H2(g)

1 C6H6(l) - 49kJ/mol

EXERCCIOS DE FIXAO
1) (UEL-adaptada) Por longo tempo, Constantinopla (atual Istambul)
foi assediada pelos muulmanos, antes de cair em 1453. Uma das
armas utilizadas pelos defensores da cidade era o "fogo grego", que
consistia em uma mistura viscosa injetada no mar e que se inflamava
ao contato com a gua, queimando violentamente. Ainda hoje no se
conhece a exata composio desta mistura, mas uma suposio
possvel de que continha p de cal misturado com o petrleo bruto,
dois materiais conhecidos na poca. O cal reage com a gua
segundo a equao,
O calor desprendido poderia inflamar o petrleo, que, por ter
densidade menor que a gua, se espalharia na superfcie do mar,
provocando a queima da frota invasora.
Dados
Entalpias de formao:
CaO(s) = -635kJ/mol; H2O(l) = -328kJ/mol; Ca(OH)2(aq) = -1002 kJ/mol
De acordo com o texto e seus conhecimentos em qumica,
conclui-se que
a) a reao descritas endotrmica.
b) o xido de clcio pertence a funo xido, classificado como
anftero.
c) o petrleo no se mistura com a gua porque polar.
d) a variao de entalpia da reao vale -39kJ/mol de hidrxido de
clcio produzido.
e)a adio de um catalisador ir diminuir a variao de entalpia da
reao, pois diminuir a energia de ativao.

2)(ENEM)Nas ltimas dcadas, o efeito estufa tem-se intensificado


de maneira preocupante, sendo esse efeito muitas vezes atribudo
intensa liberao de CO2 durante a queima de combustveis fsseis
para gerao de energia. O quadro abaixo traz as entalpias-padro
de combusto a 25oC
) do metano, do butano e do octano.

b) Calor molar de combusto do etanol lquido.


1 C2H5OH(l) + 3 O2(g) 2 CO2(g) + 3 H2O(l) + 326,7Kcal/mol

c) Calor de combusto de 2 mols de etanol


2 C2H5OH(l) + O2(g) CO2(g) + H2O(l)

Se Hp < Hr => H ..... 0 ( reao ...........)


Em Resumo
Se Hp > Hr => H ..... 0 ( reao ...........)
II)CINTICA QUMICA estuda a velocidade envolvida nas reaes
qumicas, que determinada pelo quociente,

Vreao

II.1)FATORES QUE AFETAM A VELOCIDADE DAS REAES


II.1.a)Temperatura :
Aumento da temperatura aumento da velocidade

medida que aumenta a conscincia sobre os impactos ambientais


relacionados ao uso da energia, cresce a importncia de se criar
polticas de incentivo ao uso de combustveis mais eficientes. Nesse
sentido, considerando-se que o metano, o butano e o octano sejam
representativos do gs natural, do gs liquefeito do petrleo(G.L.P.) e
da gasolina, respectivamente , ento, a partir dos dados fornecidos,
possvel concluir que, do ponto de vista da quantidade de calor
obtido por mol de CO2 gerado, a ordem crescente desses trs
combustveis :
a)gasolina ,GLP e gs natural
d) gs natural, GLP e gasolina
b) gs natural, gasolina e GLP
e) GLP, gs natural e gasolina
c) gasolina, gs natural e GLP
3) A obteno de CO2 gasoso, em laboratrio, pode ser feita pela
reao entre carbonato de clcio (slido) e soluo aquosa de cido
clordrico. Considere os conjuntos de condies experimentais
descritas na tabela a seguir

II.1.b)Superfcie(rea) de Contato:
Aumento da superfcie de contato aumento da velocidade
II.1.c)Concentrao dos Reagentes:
Aumento da concentrao dos reagentes aumento da velocidade
II.1.d)Catalisador
Diminui a energia de ativao
reao, sem participar da mesma.

aumentando a velocidade da

Utilizando-se uma certa massa de carbonato e excesso de cido,


obtm-se CO2 mais rapidamente quando se utiliza o conjunto de
condies experimentais descrito em
a) I.
b) II.
c) III.
d) IV.
e) V

SECRETARIA DE ESTADO DA EDUCAO

15

QUMICA

EXERCCIOS DE FIXAO

AULA 10 : EQUILBRIOS
I)EQUILBRIO QUMICO em sistemas Homogneos : a situao
em que a proporo entre os reagentes e produtos de uma reao
qumica (reversvel ocorrendo em recipiente fechado) se mantm
constante ao longo do tempo.
II)CONSTANTE DE EQUILBRIO(Keq) =
Observaes :

(UFPB-adaptada) Texto para as questes 1 e 2


No mundo atual, so produzidas milhes de toneladas de
compostos nitrogenados, entre os quais os fertilizantes so os mais
importantes pelo papel que desempenham na produo de
alimentos. Esses adubos agrcolas nitrogenados so fabricados a
partir da amnia, que produzida industrialmente atravs da sntese
de Haber-Bosch, descrita pela seguinte equao:
3H2(g) + 1N2(g)

1) [ ] representa a concentrao em mol/L.


2) em equilbrios heterogneos s consideramos o estado gasoso.

2NH3(g)

H = -92kJ

2HI
1) Podemos concluir que a constante de equilbrio em funo das
concentraes para a reao dada , :

Exemplos
H2(g) + I2(g)

2HI(g)

Keq =

2HI
C(s) + O2(g)

CO2(g)

a) Kc = [NH3]2

d) Kc =

b) Kc = [H2]3.[N2]

e) Kc =

Keq =

2HI
III) DESLOCAMENTO DO EQUILBRIO
Em 1884 o cientista francs Henri Louis Le Chatelier enunciou
um princpio geral conhecido pelo nome Principio de fuga ante a
fora, ou simplesmente, princpio de Le Chatelier, que defendia a
teoria de que quando se exerce uma ao num sistema em equilbrio
(variao de presso, temperatura, concentrao), o sistema se
desloca no sentido da reao que neutraliza esta ao.
III.a)VARIAO DA PRESSO
O aumento da presso total sobreo sistema, desloca o
equilbrio para o lado de menor n de mols de substncias no
estado gasoso
Exemplo
No equilbrio : 3H2(g) + 1N2(g)

2NH3(g)

2HI

n=
n=
um aumento da presso total desloca o equilbrio para o lado
.......................................................

III.b)VARIAO DA TEMPERATURA
A adio de calor, segundo o princpio de Le Chatelier,
deslocar o equilbrio de modo que o calor seja absorvido, o que
favorece a reao endotrmica ; inversamente, a retirada de
calor favorece a reao exotrmica.
Exemplo
2NH3(g) H = -92kJ

No equilbrio : 3H2(g) + 1N2(g)

2HI
um aumento da temperatura desloca o equilbrio para o lado
.......................................................

c) Kc =

2)Considerando a equao e a condio de equilbrio do sistema,


correto afirmar que o rendimento da produo de amnia
a) diminui com a remoo de NH3
b) aumenta com a elevao da temperatura.
c) aumenta com o aumento da concentrao de H2
d) permanece inalterado com a diminuio da concentrao de N2
e) permanece inalterado com a reduo do volume do reator.

3)(UFPB-adaptada) O estilo de vida atual provoca uma carga de


estresse elevada em grande parte da populao, aumentando o
ndice de pacientes com transtornos de ansiedade. A respirao e
seus mecanismos de controle exercem um papel fundamental
nesses transtornos, especificamente, o do pnico. O ataque de
pnico inicia-se com uma repentina sensao de terror, associada
com a condio de hiperventilao, que consiste na respirao
anormal, rpida e ofegante, expelindo maior quantidade de gs
carbnico. O efeito dessa condio pode ser avaliado atravs do
equilbrio que ocorre no plasma sanguneo, conforme equao:

CO2(g)

H2O(l)

2HI
De acordo com essas informaes e considerando o efeito da
hiperventilao no plasma sanguneo, correto afirmar:
a) A concentrao de CO2 aumenta durante a hiperventilao.
b) A hiperventilao provoca um aumento na concentrao de H+.
c) A concentrao de CO2 permanece constante.
d) A respirao rpida e ofegante desloca o equilbrio para a
esquerda.
e) A variao da presso total no afeta este equilbrio.
anotaes

III.c)VARIAO DA CONCENTRAO
O aumento da concentrao desloca o equilbrio para o
lado oposto ao adicionado ( afim de consumir o excesso) ,
enquanto que a diminuio da concentrao desloca o equilbrio
para o mesmo lado (afim de repor a perda)
Exemplo
2NH3(g) H = -92kJ

No equilbrio : 3H2(g) + 1N2(g)

2HI
*um aumento da [H2] , desloca o equilbrio para o lado .....................
*uma diminuio da [N2], desloca o equilbrio para o lado ...................

SECRETARIA DE ESTADO DA EDUCAO

16

QUMICA

EXERCCIOS DE FIXAO

AULA 11 : pH e pOH
I) pH de solues cidas e bsicas:
HX
[H+] =
Para cidos fortes
Para cidos fracos Ka =

1)(UFPB-adaptada) O cido ltico (CH3-CH(OH)-COOH)


est
presente na composio do leite e tambm produzido nos
msculos aps a prtica de exerccios fsicos muito intensos,
provocando dolorosas cimbras. Sabendo que seu grau de ionizao
de 1%, correto afirmar que, em uma soluo aquosa 0,01 mol/L
desse cido,
a) a [H+] na soluo 0,01mol/L.
b) sua constante de ionizao Ka vale 1,0 x 10-3.
c) a [H+] na soluo igual a [OH-].
d) o pOH da soluo vale 3.
e) o pH da soluo vale 4.
2)(UFPB)Um qumico resolveu implementar um programa de
gerenciamento de resduos no laboratrio de sua empresa,
comeando pela identificao de solues antigas cujos rtulos
estavam danificados e ilegveis. Sabendo-se previamente que eram
solues aquosas, aproximadamente 0,1 mol/L, de cido actico,
cido clordrico, hidrxido de sdio, hidrxido de amnio e cloreto de
sdio, o qumico resolveu fazer medidas de pH, para auxiliar na
distino dessas solues. Os resultados das medidas de pH, em
valores aproximados, so mostrados na tabela abaixo,

pH = .........................

MeOH
[OH- ] =
Para bases fortes
Para cidos fracos Kb =

pOH = .........................
ESCALA DE pH
Com base nesses resultados, o qumico concluiu que as solues de
1 a 5 correspondiam respectivamente a

pH + pOH = 14
II) pH de solues salinas (Hidrlise)
MeX
Sal derivado de:
a)

a) CIDO FORTE + Base fraca


pH.........7
a) CI

b)cido fraco + BASE FORTE


pH.........7
c)cido e base fracos
pH.........7

Observao:
Sais formados por cidos e bases fortes no sofrem hidrlise,
apenas se dissociam, porm suas solues aquosas apresentam
pH=7

III) SOLUO TAMPO : so solues que atenuam a variao dos


valores de pH (cido ou bsico), mantendo-o aproximadamente
constante, mesmo com adio de pequenas quantidades de cidos
ou bases. As solues tampo podem ser formadas por um cido
fraco e um sal derivado deste cido, ou, ento, por uma base fraca e
um sal derivado desta base.

SECRETARIA DE ESTADO DA EDUCAO

17

3)(AMRICO-2015)O gs contido no refrigerante que consumimos


o gs carbnico (dixido de carbono CO2). Geralmente o
refrigerante formado por uma soluo aquosa de um tipo de xarope
e esse gs. Mas antes de misturar o gs no xarope os fabricantes
misturam a gua e o gs num parelho chamado carbonizador,
gerando o cido carbnico, que tem forma lquida. Nesse tipo de
bebida existe, ento, o seguinte equilbrio qumico:
2 H2O(l) + 1 CO2(g) 1 H3O+(aq) + 1 HCO1-(aq)
O princpio de Le Chatelier diz que quando causada alguma forma
de perturbao em um sistema qumico, o seu equilbrio se desloca
no sentido de reduzir essas perturbaes.
Quando bebemos o refrigerante vrias alteraes so feitas nesse
sistema, deslocando equilbrio qumico de diversas formas.
Por exemplo, no nosso estmago existe o suco gstrico que
formado principalmente pelo cido clordrico (HCl). Esse cido, como
todos os outros, caracterizado pela presena de ons H+ em meio
aquoso.
Considerando-se o texto, podemos afirmar que,
a) a presena do cido no estmago aumenta a concentrao de
H3O+ no equilbrio descrito e, consequentemente, desloca o equilbrio
descrito para o lado direito.
b) a presena do cido no estmago significa que haver menos
formao de gs carbnico.
c) considerando que este refrigerante esteja contido em uma garrafa
PET, ao amassar a garrafa o deslocamento favorece liberao do
CO2.
d) A ingesto de bicarbonato de sdio, baixa o pH por diminuio da
concentrao de H+.
e) o Leite de Magnsia , Mg(OH) 2 pode ser usado para neutralizar a
ao do cido estomacal, aumentando o pH do meio.

QUMICA

EXERCCIOS DE FIXAO

AULA 12 : ELETROQUMICA
I) ELETROQUMICA:
um ramo da qumica que estuda as
reaes e fenmenos onde h transferncia de carga eltrica.
Este campo cientfico abrange todos os processos
qumicos que envolvam transferncia de eltrons entre substncias,
a PILHAS ou BATERIAS so exemplos deste processo que
transformam energia qumica em energia eltrica.
II) CONCEITOS BSICOS
II.a) Oxidao : indica a perda de eltrons, como por exemplo:
Fe0 Fe2+ + 2eII.b) Reduo : indica o ganho de eltrons, como por exemplo:
Al3+ + 3e Al0
II.c) Potencial de Reduo (
) : indica o ction com maior
tendncia de se reduzir.
Exemplo:
Considerando-se as semirreaes abaixo:
Cu2+ + 2e- Cu0 E0 = 0,34V
Ag+ + e- Ag0 E0 = 0,80V
Fe2+ + 2e- Fe0 E0 = - 0,44V
Mg2+ + 2e- Mg0 E0 = - 2,37V

1)(AMRICO-2015) O ferro, utilizado em cascos de navio, em contato


com a gua do mar, se oxidaria muito facilmente se no houvesse um
metal de sacrifcio. Considerando que substituir plaquetas deste
metal muito mais barato do que substituir a estrutura de ferro, fica
clara a vantagem da sua utilizao.
De acordo com a tabela abaixo,
semirreao
E0(Volts)
+3
0
Al + 3e Al
- 1,66
Zn2+ + 2e- Zn0
-0,76
Fe+2 + 2e- Fe0
-0,44
Mg2+ + 2e- Mg0
-2,37
Cu+2 + 2e- Cu0
+0,34
Entre os metais citados , o que melhor funciona como metal de
sacrifcio para o ferro o,
a) Mg+2
b) Al
c) Mg
d) Cu
e) Zn
2)(AMRICO-2015) Considere a clula eletroqumica a seguir e os
potenciais das semirreaes:

O ction com maior facilidade de se reduzir o ................,


enquanto que o metal com maior facilidade de se oxidar o ..............
II.d) Metal de sacrifcio: qualquer metal utilizado em estruturas
submetidas a ambientes oxidantes, com o objetivo de ser oxidado em
seu lugar. Esse metal deve possuir menor poder de reduo do
que o material utilizado na estrutura, para que possa ser "sacrificado"
e proteg-la.
Por exemplo, considerando-se os metais abaixo
Cu2+ + 2e- Cu0 E0 = 0,34V
Ag+ + e- Ag0 E0 = 0,80V
Fe2+ + 2e- Fe0 E0 = - 0,44V
Mg2+ + 2e- Mg0 E0 = - 2,37V
Qual o melhor metal de sacrifcio? .......................................................
III) ELETRLISE: todo o processo qumico no espontneo
provocado por uma corrente eltrica proveniente de um gerador .
Trata-se de um processo qumico inverso ao da pilha, ou seja, a
energia eltrica provoca a reao qumica.

Sobre o funcionamento da pilha, e fazendo uso dos potenciais dados,


podemos concluir que,
a) os eltrons caminham espontaneamente, pelo fio metlico, do
eletrodo de cobre para o de niquel.
b) O polo positivo ou ctodo, onde ocorre oxidao o eletrodo de
nquel.
c) a reao andica : Ni+2 + 2e- Ni0
d) o eletrodo de nquel aumenta a sua massa com o passar do
tempo.
e) a reao espontnea que ocorre na pilha e sua d.d.p. so,
Ni0 + Cu2+ Ni2+ + Cu0 E0 = 0,59V

CUIDADO!
Fenmeno
Eletroqu[imica
Eletrlise
No Esquea:

Polo positivo

Polo negativo

3)(UFRN) A produo industrial de alumnio pela eletrlise da bauxita


fundida um processo industrial que consome grande quantidade de
energia eltrica. A semirreao de reduo do alumnio dada por:
Al+3 + 3e Al0. Para se produzir 2,7g de alumnio metlico, a carga
eltrica necessria, em coulombs, :
Dado: massa molar do alumnio(g/mol)= 27.
a) 9650
b) 28950
c) 32160
d) 289500 e) 96500

d.d.p.(E0)

O NODO onde ocorre a .............................


O CTODO onde ocorre a .............................

LEI DE FARADAY
Em 1843, Michael Faraday

props as leis que


regem a eletrlise. So as leis que relacionam as massas
das substncias produzidas nos eletrodos e as
quantidades de energia gastas na eletrlise, em resumo
podemos dizer que a massa depositada em um eletrodo
aps o fenmeno da eletrlise, em gramas, dada por:

ANOTAES

Onde:
*Q a carga , em Coulomb(C) que atravessa a soluo , que pode
ser determinada pela frmula:
Q = i. t , onde i = intensidade em Ampres e t = tempo(segundos)
*F constante de Faraday e indica a quantidade de carga contida
em um mol de eltrons e vale 96.500C
*E Equivalente-grama da espcie qumica(normalmente o metal) a
ser eletrolisado, e dado por:

SECRETARIA DE ESTADO DA EDUCAO

18

QUMICA

EXERCCIOS DE FIXAO

AULA 13 : RADIOATIVIDADE
I)CONCEITO: um fenmeno natural ou artificial, pelo qual algumas
substncias ou elementos qumicos, chamados radioativos, so
capazes de emitir radiaes.
II) AS EMISSES RADIOATIVAS
A radiao liberada na radioatividade natural pode ser separada
por um campo eltrico ou magntico em trs tipos distintos. A figura a
seguir ilustra tal separao atravs de utilizao de um campo
eltrico.

1) (PUC ) O fenmeno da radioatividade foi descrito pela primeira vez


no final do sculo passado, sendo largamente estudado no incio do
sculo XX. Aplicaes desse fenmeno vo desde o diagnstico e
combate de doenas, at a obteno de energia ou a fabricao de
artefatos blicos.
Duas emisses radioativas tpicas podem ser representadas pelas
equaes:
U 234Th + e

Th 234Pa +

238

234

carga ......, denominada emisso


1

nome

carga

representao

2
3

A radiao o ncleo do tomo de hlio, possuindo 2 prtons


e 2 nutrons, que se desprende do ncleo do tomo radioativo. A
radiao um eltron, proveniente da quebra de um nutron,
formando tambm um prton, que permanece no ncleo. A equao
que representa o decaimento radioativo do istopo do 238U at o
istopo estvel 206Pb ,

III) 1 LEI (EMISSO ALFA)

2) (MACK)Quando a massa de nuvens de gs e poeira de uma


nebulosa se adensa, a temperatura aumenta, atingindo milhes de
graus Celsius. Ento, tomos de hidrognio se fundem, gerando gs
hlio, com liberao de quantidades fantsticas de energia. A
fornalha est acesa. Nasce uma estrela. Uma das equaes que
representa esse fenmeno :

Exemplo

IV) 2 LEI (EMISSO BETA)

Exemplo

V) FISSO NUCLEAR : quebra do ncleo de um tomo em ncleos


menores, com liberao de uma grande quantidade de energia
.Princpio utilizado em reatores nucleares e armas nucleares como a
bomba atmica.

+ 16,55x108 kJ/mol de He

A respeito da reao nuclear dada, correto afirmar que:


a) uma reao de fisso nuclear.
b) uma reao de fuso nuclear.
c) uma reao endotrmica.
d) um fenmeno fsico.
e) h liberao de prtons.
3) (Unifesp) O decaimento do tecncio-99, um istopo radioativo
empregado em diagnstico mdico, est representado no grfico
fornecido a seguir.

VI)FUSO NUCLEAR : Consiste na sntese (reunio) de ncleos,


dando origem a um ncleo maior e mais estvel, e na emisso de
grande quantidade de energia , MAIOR QUE A FISSO. So
necessrias altas temperaturas para que ocorra a fuso nuclear.

VII)MEIA-VIDA ( t1/2): tempo necessrio para desintegrar metade da


massa de uma amostra radioativa.
Exemplo

SECRETARIA DE ESTADO DA EDUCAO

19

Uma amostra tpica de tecncio-99 usada em exames apresenta


uma atividade radioativa inicial de 2x107 desintegraes por
segundo. Usando as informaes do grfico, pode-se prever que
essa amostra apresentar uma atividade de 2,5 x106 desintegraes
por segundo aps,
a) 3,5 horas.
b) 7 horas.
c) 10 horas.
d) 18 horas.
e) 24 horas.

QUMICA

EXERCCIOS DE FIXAO

AULA 14-INTRODUO A QUMICA ORGNICA


1)(AMRICO-2015)Qual
estrutura plana?

I)PROPRIEDADES DO TOMO DE CARBONO


I.a)TETRAVALNCIA : o carbono TETRAVALENTE, e pode se
ligar a outros tomos da seguinte forma,

das

molculas

abaixo

no

a) O=C=O

d)

b)

e) CH4

apresenta

c) HC CH
2)(AMRICO-2015)A molcula abaixo da famlia dos nanoKids e
seu nome oficial
1-ciclipentil-bis-(2,2-dimetil)propil-3,5-ciclopropil-cicloexano
(Tetradrico)
(Lineares)
(Trigonal plano)

I.b)ENCADEAMENTO: Os tomos de carbono tm a propriedade de


se unir, formando estruturas denominadas cadeias carbnicas. Essa
propriedade a principal responsvel pela existncia de milhes de
compostos orgnicos. Existem outros tomos que tambm podem
aparecer nas cadeias carbnicas, denominados organgenos como
N, O, S , Cl, Br, etc.
O nmero de carbonos tercirios presentes em sua estrutura ,
a) 6
b) 8
c)10
d) 7
e)5
3) Identifique a cadeia carbnica ramificada, homognea e
saturada:

I.c)CLASSIFICAO DO TOMO DE CARBONO


p
p

q
p

t
p

p primrio
s secundrio
t tercirio
q quaternrio

I.d)CLASSIFICAO DAS CADEIAS CARBNICAS


ABERTA ou ACCLICA
FECHADA ou CCLICA

NORMAL ou LINEAR

RAMIFICADA

4)A "fluxetina", presente na composio qumica do Prozac,


apresenta a frmula estrutural:

SATURADA

INSATURADA

HOMOGNEA

HETEROGNEA

Com relao a esse composto, correto afirmar que apresenta:


a) cadeia carbnica alicclica e heterognea.
b) cadeia carbnica aromtica e homognea.
c) cadeia carbnica insaturada e homognea.
d) somente tomos de carbonos primrios e secundrios.
e) frmula molecular C16H16ONF

Cadeias fechadas podem se denominar ,


a)Aromticas : quando apresentarem pelo menos um anel
aromtico

b) Alicclicas : nos demais casos

SECRETARIA DE ESTADO DA EDUCAO

20

QUMICA

AULA 15-HIDROCARBONETOS
I)CONCEITO: compostos pertencentes a funo HIDROCARBONETO
so formados exclusivamente por carbono e hidrognio.
II)PROPRIEDADES FSICAS

So gases (de 1 a 4 carbonos) , lquidos (de 5 a 17


carbonos e slidos ( acima de 17 carbonos)

So insolveis em gua j que suas molculas so


apolares .

So menos densos que a gua

O P.F. e P.E. aumenta com o aumento da cadeia, quando


as cadeias apresentam o mesmo nmero de carbonos,
quanto mais ramificaes, menor o P.F. e P.E.

EXERCCIOS DE FIXAO
1)(UFPB-adaptada)Gigantes reservas de petrleo foram encontradas
recentemente no Brasil. Essas reservas situam-se em regies de
grandes profundidades em guas ocenicas e abaixo de uma
camada de sal, por isso, denominadas de pr sal. Com a explorao
dessas reservas, o Brasil aumentar significativamente a produo
de petrleo. Aps a extrao, o petrleo transportado at as
refinarias, onde passar por uma srie de processos de purificao
denominada de refino. A figura abaixo representa um desses
processos, em que o petrleo entra na fornalha, aquecido e segue
para a torre de destilao, onde sero separadas as diversas
fraes.

III)NOMENCLATURA IUPAC
Nome dos radicais

Prefixo grego indicando

em ordem alfabtica

O n de carbonos da

Cadeia principal

AN- ligaes simples


EN- ligao dupla

+ o

IN- ligao tripla

Observaes
1) Os radicais alquil (as) mais importantes so,

2) Radicais com anel aromtico importantes:


Fenil

benzil

orto-toluil

meta-toluil

para-toluil

2) Prefixos gregos
1C MET
2C ET
3C PROP

4C BUT
5C PENT
6C HEX

7C HEPT
8C OCT
9C NON

10C DEC
11C UNDEC
12C DODEC

3) A numerao da cadeia principal inicia da extremidade mais


prxima da ramificao. Se a cadeia for insaturada, a numerao inicia
da extremidade mais prxima da dupla ou da tripla.
4) O nome da cadeia principal deve ser precedido do menor nmero
do carbono onde se encontra a insaturao, quando for o caso.
Exemplos:

..................................
.

....................................

...............................................

O petrleo constitudo por diversos hidrocarbonetos cujos


tamanhos, massas e pontos de ebulio diferentes, acerca dos
hidrocarbonetos ,
a)O querosene apresenta-se gasoso nas condies ambientes.
b) O ponto de ebulio do hidrocarboneto ser maior quanto menor
for sua massa molecular.
c) A gasolina uma mistura de diversos hidrocarbonetos, onde o
predominante o 2,2,4-trimetiloctano que pertence a funo alcano.
d) Metxi-terciobutano(CH3OC4H9) antidetonante usado na gasolina,
um hidrocarboneto saturado.
e) Metil-benzeno (C7H8) um hidrocarboneto aromtico.
2)(UFPB-adaptado) As protenas so macromolculas com massas
moleculares que vo de 5000 a alguns milhes de unidades de
massa atmica, sendo essas protenas constitudas por monmeros,
que so os aminocidos. Nesse sentido, considere os aminocidos
a seguir,

(I)

(III)

....................................

(II)

(IV)

Considerando-se apenas as estruturas assinaladas ,


nomenclaturas enquanto radicais, so de I a IV, nesta ordem,
a) metil, propil , fenil e etil.
b) metil, isopropil , fenil e etil.
c) metil, isopropil, benzil e etil.
d) etil, propil , benzil e metil.
e) etil, propil ,fenil e metil

suas

............................
............................
SECRETARIA DE ESTADO DA EDUCAO

21

QUMICA

AULA 16:FUNES ORGNICAS


A)FUNES OXIGENADAS
FUNO
LCOOL
FENOL

TER

GRUPO FUNCIONAL
R OH

NOMENCLATURA
(Nome do hidrocarboneto + OL)

Ar OH

(d-se o nome dos radicais) + benzeno

R O R'

(nome do menor R + OXI) (nome do maior R)

ALDEDO

CETONA

EXEMPLOS
CH3OH
CH3CH2OH CH3CH2CH2OH
................. .................. ...........................

.......................... ou ..................
CH3 O CH2CH3
CH3CH2-)- O CH2CH3
.

...............................

....................

(nome do hidrocarboneto correspondente) + AL

(nome do hidrocarboneto correspondente) + ONA


ou
nome dos radicais em ordem alfabtica + CETONA
......................
ou
........................

cido
Carboxlico

cido (nome do hidrocarboneto correspondente) + ICO

............................
STER

.......................
ou
.................................

............................

(nome de R + ATO) de (nome do radical R)

b)FUNES NITROGENADAS
Amidas: As amidas derivam dos cidos orgnicos por
substituio do grupo OH da carboxila por NH2. Sua
frmula geral R CONH2 , sua nomenclatura feita da
seguinte forma:
(nome do hidrocarboneto correspondente)+ AMIDA
Exemplo,
CH3 CH2 C = O ...............................
|
NH2

c) Outras funes importantes


Haletos ..................
Exemplo

Nitro compostos..................
Exemplo

cidos sulfnicos ..................


Exemplo

EXERCCIO DE FIXAO
Identifique todas as funes presentes nas molculas abaixo,

SECRETARIA DE ESTADO DA EDUCAO

22

QUMICA

AULA 17-ISOMERIA
A)CONCEITO:
A palavra isomeria vem do grego (iso = mesma(s) meros = partes),
o fenmeno pelo qual duas substncias compartilham a
mesma frmula molecular , mas apresentam estruturas diferentes.

EXERCCIO DE FIXAO
1) Considerando os compostos orgnicos numerados de I a IV

A.1)ISOMERIA PLANA

Considerando-se os conhecimentos em isomeria plana, podemos


concluir que,
a) I e II no so ismeros planos. d) I e II so ismeros de cadeia.
b) I e III so ismeros de posio. e)III e IV so ismeros de cadeia.
c) II e IV so ismeros de funo.
2)(Amrico-2015) O metxi-metano ou dimetil ter (Dme) um
gs de frmula CH3OCH3 utilizado primordialmente em aerossis.
Tambm pode ser utilizado como combustvel automotivo,
substituindo o GLP ou o diesel. Pode ser obtido a partir do gs
natural, do carvo ou da biomassa (o que tecnicamente o torna um
biocombustvel).
J etanol ou lcool etlico ou simplesmente lcool uma
substncia orgnica de frmula C2H5OH, obtida da fermentao
de acares , encontrado em bebidas alcolicas, bem como na
indstria de perfumaria. No Brasil, tal substncia tambm muito
utilizada como combustvel automotivo, constituindo assim um
mercado em ascenso para um combustvel obtido de maneira
renovvel e o estabelecimento de uma indstria de qumica de base,
sustentada na utilizao de biomassa de origem agrcola e
renovvel.
Estes dois compostos so ismeros ,
a) de funo.
d) de compensao
b) de cadeia.
e) por tautomeria.
c) de posio.

A.2)ISOMERIA ESPACIAL
a)GEOMTRICA

3)(Amrico-2015) A presena da vitamina A na dieta alimentar


importante porque, entre outras coisas, ela est relacionada
manuteno de uma boa viso. Dentro do organismo, a vitamina A
se converte em retinal, participando de um conjunto de reaes
qumicas que ocorrem nos olhos e sendo responsvel pelas
informaes visuais que so emitidas para o crebro. A frmula
estrutural do retinal

Exemplos

CIS

TRANS

Observaes :
1) Quando os ligantes dos carbonos das duplas so diferentes ,
classifica-se pelo tamanho dos radicais( massa molar).
2) Ismeros geomtricos diferem nas propriedades fsicas.
b)PTICA
Na isomeria ptica os ismeros so compostos assimtricos
(quirais) que, apesar de possurem as mesmas propriedades fsicas
e qumicas , diferem apenas no desvio da luz polarizada. H trs
maneiras de uma molcula ser assimtrica: possuindo carbono
assimtrico (por definio, o tomo de carbono que possui 4
ligantes diferentes e que tambm pode receber o nome de
carbono quiral) a mais comum.
Os ismeros pticos podem ser separados em dois grupos, os
levogiros e os dextrogiros, que formam um par de ENENTIMEROS,

Considerando-se o composto acima e seus conhecimentos sobre


isomeria espacial, podemos afirmar que,
a)o retinal apresenta carbono quiral.
b)o retinal apresenta-se como um par de enantimeros.
c)o retinal no apresenta isomeria geomtrica.
d)a parte cclica da cadeia encontra-se na forma cis.
e)considerando-se os carbonos 6 e 7, encontramos isomeria
geomtrica na forma trans.
4)(UFPB-adaptada) A isomeria ptica uma propriedade que ocorre
em molculas assimtricas ,observe a estrutura do composto abaixo:

Levogiros ("" ou "-") so os ismeros que desviam a luz


polarizada para a esquerda.
Dextrogiros ("d" ou "+") so os ismeros que desviam a
luz polarizada para a direita.
Em relao a esse composto, podemos concluir que,
a)a molcula aquiral.
b)o nmero de ismeros pticos ativos igual a dois
c) esto presentes as funes cido carboxlico, haleto e enol.
d) o nmero de racematos igual a um.
e)enantimeros possuem propriedades fsicas iguais, exceto o desvio
da luz polarizada.

O nmero de ismeros pticos dado por : 2n

R1

R2

R3

SECRETARIA DE ESTADO DA EDUCAO

R4

23

QUMICA

AULA 18: CLASSIFICAO DAS REAES ORGNICAS


I)SUBSTITUIO
aquela onde um tomo (ou grupo de tomos) da molcula
orgnica SUBSTITUDA por outro tomo (ou grupo de tomos).

EXERCCIO DE FIXAO
1) (AMRICO-2015) Associando a segunda coluna de acordo com a
primeira:
REAO
CLASSIFICAO

Exemplos,
a)HALOGENAO radicalar(LUZ)

luz
(1) CH4 + Br2
CH3Br + HBr
1700 / H
(2) CH3-CH2OH
CH2=CH2 + H2O
(3) C2H4 + 3O2 2CO2 + 2H2O

CH4

luz

Cl2 (Cl Cl )

b)ALQUILAO eletrfila(AROMTICOS)

+ -

AlCl3

+ CH3Cl

(4)

(5) C2H4 + Cl2 C2H4Cl2

II)ADIO
Ocorrem quando se ADICIONA um tomo (ou grupo de tomos) a
uma molcula orgnica.
Exemplos,
a)HIDROGENAO CATALTICA
H2C = CH2 + H2

/
+ H2 Ni

, Pd /
Ni
, Pt

b)HALOGENAO

( ) adio
( ) eliminao
( ) reduo
( ) substituio
( ) oxidao

a sequncia correta de cima para baixo, :


a)4-2-5-1-3
d) 1-2-3-4-5
b)5-3-4-1-3
e) 5-4-3-2-1
c)3-2-5-4-1
2)(AMRICO-2015)O benzeno o mais conhecido dos compostos
aromticos. Esta substncia pode sofrer reaes de substituio,
bem como de adio, como no caso da hidrogenao .Qual das
substncias abaixo um produto da clorao do benzeno?
a) hidrxibenzeno
d)hexano
b) cicloexano
e)clorobenzeno
c) tolueno

H2C = CH2 + Cl2


Observao : QUANDO O REAGENTE FORMADO POR
TOMOS ( OU GRUPO DE TOMOS) DIFERENTES TIPO : H X ,
H2O(H OH ), o hidrognio (H) ser adicionado ao carbono da dupla
mais hidrogenado.(REGRA DE MARKOVNICOV).
Exemplos
a)REAO COM HALOGENIDRETO
H2 C = CH CH3 + H-Cl

4) O ster obtido pela desidratao intermolecular entre um cido


carboxlico e um lcool, qual dos steres abaixo foi obtido da reao
entre o cido etanoico(actico) e o etanol?
a) acetato de metila.
d) metanoato de metila.
b) etanoato de etila
e) butanoato de metila.
c) metanoato de propila.

b) HIDRLISE CIDA
H+

H2 C = CH CH3 + H2O (H-OH)


3)ELIMINAO
So aquelas nas quais alguns tomos( ou grupo de tomos) so
eliminados da molcula orgnica. Geralmente ocorre aumentando a
insaturao e/ou formando pequenas molculas.
A) DESIDRATAO INTRAMOLECULAR

/H
CH2 CH CH CH3 170

|
|
|
Hprimrio OH Hsecundrio
0

REGRA DE SAYTZEFF:
A preferncia da eliminao : Htercirio>Hsecundrio>Hprimrio
B) DESIDRATAO INTERMOLECULAR
CH3 CH2 OH

/H
140

CH3 CH2 O H
4)OXIDAO/REDUO
A OXIDAO em molculas orgnicas ocorre atravs da adio de
oxignio ou diminuio do nmero de hidrognios.
A REDUO , ser o caso inverso.
Exemplos,
CH3CH2OH
primrio
[O]

CH3CHCH3
|
OH

3)(AMRICO-2015) Segundo a Regra de Markovnikov diz que o


hidrognio de um composto do tipo (HX), se liga ao carbono de um
composto orgnico insaturado que possuir maior quantidade de
hidrognios. A hidrlise do propeno em meio cido e posterior
oxidao, produzir ,
a) 2-bromopropano
d) cido propanoico
b) 1-bromopropano
e) propanona
c) 2-propanol

[O]

[O]

[H]

[H]

5)(AMRICO-2015) Outro produto preparado a partir de gorduras


vegetais hidrogenadas a margarina, surgida na Frana do sculo
XIX, durante o Imprio de Napoleo III. Em um perodo de forte crise
econmica, Napoleo III ofereceu um prmio a quem descobrisse um
substituto para a manteiga de origem animal que fosse mais
acessvel financeiramente classe mais pobre e aos militares do
exrcito. O qumico Hippolyte Mge-Mouris fez vrios testes at
descobrir a margarina ao emulsificar sebo bovino com leite. O
sucesso foi to grande que, mesmo aps a crise financeira, a
margarina tornou-se mais popular que a manteiga.
Com o desenvolvimento do processo de hidrogenao de leos na
primeira metade do Sculo XX, a gordura vegetal hidrogenada
passou a substituir o sebo na formulao das margarinas.
interessante salientar que essa substituio ocorreu tanto por
questes econmicas quanto por ser a gordura vegetal hidrogenada
considerada poca mais saudvel que o sebo bovino, devido
ausncia de colesterol. Assim, a partir da segunda metade do Sculo
XX, o processo de fabricao de margarina se consolidou como a
emulsificao de uma fase aquosa (leite, sal e conservantes) e uma
oleosa (gordura vegetal hidrogenada).
Considerando-se a hidrogenao cataltica abaixo,

Podemos concluir que o produto obtido o,


a) 2-buteno.
b) 2-butino.
c) 1-butino.
d) butano.
e) etano.

[O]

[H]

lcool secundrio
LCOOL TERCIRIO NO SE OXIDA

SECRETARIA DE ESTADO DA EDUCAO

24

QUMICA

EXERCICIOS DE FIXAO
1.As imagens abaixo representam o comportamento das hemcias
em solues de diferentes concentraes.

- No esquema I, a soluo hipotnica, ou seja: tem menor


concentrao de solutos.
- No esquema II, a soluo isotnica, ou seja: as concentraes
do meio interno e externo so similares.
- No esquema III, a soluo hipertnica, ou seja: tem maior
concentrao de solutos.
A............................................................................................................
B............................................................................................................
C............................................................................................................
D............................................................................................................
E............................................................................................................
F............................................................................................................

Esto corretas:
A) Todas as alternativas.
B) Somente a primeira alternativa.
C) Somente a segunda alternativa.
D) Somente a terceira alternativa.
E) Todas as alternativas esto
corretas

TRANSPORTE ATRAVS DA MEMBRANA


Passivo..............................................................................Osmose........................de um meio....................para um meio ......................
Difuso Simples-.........................de um meio ................para um
meio..........................
Difuso facilitada -O transporte de aminocidos por exemplo
permeada por...........................de transporte denominadas
..................................................................................................Ativo
Ocorre.................o gradiente de concentrao necessitando de
energia(......)
Bomba de sdio e potssio- sdio bombeado para o
meio...................................e sdio para o meio
......................................
Transporte em bloco . Transporte de grandes molculas em forma
de partculas...................(fagocitose) e
gotculas....................(pinocitose) o conjunto de englobamento
denomina-se ..........................
Exocitose. Processo inverso a endocitose onde a clula
excreta(........................) ou exporta substncias(............................).
Especializaes a membrana. As membranas apresentam
especializaes de ...........................como os
........................................Inter digitaes, zona.......................e zona
.......................... e outras de absoro denominada...........................

SECRETARIA DE ESTADO DA EDUCAO

3. Em relao s clulas animais, denominam-se glicoclix:


a) os microvilos das clulas que revestem o intestino delgado;
b) todas as clulas acinosas de glndulas secretoras;
c) o colo do espermatozide;
d) a estrutura lipoprotica das membranas celulares;
e) uma camada com protenas e carboidratos que recobre a
membrana plasmtica
4. Assinale a alternativa INCORRETA:
a) A difuso simples um tipo de transporte passivo atravs da
membrana plasmtica sem haver gasto de energia.
b) A difuso facilitada utiliza protenas carregadoras para o
transporte de acares simples e aminocidos atravs de membrana
constituindo, por essa razo, um processo de transporte ativo.
c) A membrana plasmtica formada por uma camada bimolecular
de fosfolipdeos onde esto dispersas molculas de protenas
d) Qualquer processo de captura por meio do envolvimento de
partculas chamado endocitose.
e) Na fagocitose a clula engloba partculas slidas para atravs da
emisso de pseudpodes que as englobam formando um vacolo
alimentar denominado fagossomo

25

QUMICA

BIOLOGIA

Aula 2: BIOQUMICA CELULAR:* Molculas, clulas e tecidos

EXERCCIOS DE FIXAO

INTRODUO
gua, sais minerais, carboidratos, oxignio, protenas, aminocidos,
vitaminas, nucleotdeos entre outras molculas, so parte no
universo de estudos na Bioqumica celular e esto os elementos
essenciais presentes em todas as clulas vivas. Essas molculas
desempenham as mais diversas funes, como a regulao da
temperatura, transporte de substncias, lubrificao, regulao
hormonal entre tantas outras funes. Essas molculas servem como
base para a regulao e manuteno da harmonia celular.
*A gua
o componente qumico mais abundante da matria viva, a gua
atua como .................................. funcionando, portanto, como
dispersante da maior parte dos compostos ................. e ....................
da natureza.
Quanto mais jovem e metablico , mais gua
Encfalo de embrio 92,0 %
Msculos 83,0 %
Rins 60,8 %
Ossos 48,2%
Dentina 12,0 %
* Os Sais Minerais
So substncias inorgnicas essenciais a manuteno da vida. So
encontrados nos seres vivos sob duas formas bsicas: livres
como..........e fixos como o ............dos ossos. Principais sais
minerais:
Ferro..........................................................................................
Clcio........................................................................................
Magnsio...................................................................................
Fsforo......................................................................................
Zinco..........................................................................................
Sdio e Potssio.......................................................................
*SUBSTNCIAS ORGNICAS
Os Carboidratos(glicdios / acares / glucdios / glcides
/ hidratos de carbono).
Funes:
.............................
a
principal(glicose),
estrutural......................(celulose) e reserva(.........................)
Classificao dos Carboidratos
Monossacardeos: formados de ...........tomos de carbono
E.:........................(pentoses).......................(hexoses)
Oligossacardeos: formados de ..... ....monossacardeos
EX.: Maltose, formado por ....molculas de .............................
Polissacardeos. Formados por mais de.......molculas de glicose.
Ex.: Amido, glicognio
*Lipdeos (= lpides ou gorduras)
Os lipdios so substncias insolveis em gua e solveis em
solventes orgnicos, tais como lcool, ter e clorofrmio.
Tipos de Lipdios
a) Simples. Formados por lcool de cadeia curta (3 carbonos, o
glicerol) denominados glicerdeos sendo divididos em leos e
gorduras e quando formados por lcool de cadeia longa denomina-se
Cerdeos com funes de proteo e impermeabilizao.
b) Compostos. So formados por lipdios associados a elementos
qumicos, tendo como exemplos os .............................que so
lipdios associados ao fsforo presente nas membranas celulares, e
os ............................encontrados na bainha de mielina
c) Complexos. So lipdios associados a anis carbnicos, formando
os hormnios.................a vitamina......e o colesterol
d) Carotenoides. So pigmentos de origem lipdica encontrados nas
frutas e legumes sendo representados pelo Beta Caroteno

SECRETARIA DE ESTADO DA EDUCAO

26

1.Os sais minerais existem nos seres vivos de forma imobilizada ou


dissociados em ons. Pequenas variaes nas porcentagens de
ons podem modificar profundamente a permeabilidade,
irritabilidade e viscosidade de clula. Analise as propostas
apresentadas.
( ) Magnsio (Mg) presente na clorofila , portanto, necessrio
fotossntese.
( ) Clcio (Ca) necessrio para a ao de certas enzimas em
importantes processos fisiolgicos.
( ) Ferro (Fe), presente na hemoglobina, faz parte de pigmentos
importantes na respirao (citocromos).
( ) Fosfato (PO4) o principal ction extra e intracelular.
( ) Cloreto (Cl) importante ction presente tanto na hemoglobina
quanto na clorofila.
2. Observe as assertivas
01) A gua uma molcula polar, por apresentar zonas positivas e
negativas, em lados opostos.
02) A frmula geral dos monossacardeos (CH2O)n, em que o
valor de n varia de 3 a 7.
04) O colesterol um esteroide presente na composio qumica da
membrana plasmtica de animais, alm
de atuar como substncia precursora dos hormnios testosterona e
progesterona.
08) Os fosfolipdios possuem caractersticas qumicas semelhantes
a detergentes, apresentando uma cauda hidrofbica (parte apolar)
e uma cabea hidroflica (parte polar).
16) Os dissacardeos no so solveis em gua, mas so
imediatamente aproveitveis como fonte de energia pelos
organismos vivos.
Soma (...........)
3.Associe as colunas:
A) Amido
( ) Funo estrutural. Encontrado em animais.
B) Glicognio ( ) Funo de reserva. Encontrado em plantas.
C) Celulose
( ) Funo estrutural. Encontrado em plantas.
D) Quitina.
( ) Funo de reserva. Encontrado em animais.
4.A energia que usamos para realizar os movimentos provm da
degradao dos alimentos que ingerimos. Entre os nutrientes
que ingerimos, indique o mais utilizado na produo desta energia:
a) protena;
b) carboidrato;
c) lipdio;
d) sais minerais
5.(Unifesp 2002) Um ser humano adulto tem de40 a 60% de sua
massa corprea constituda por gua. A maior parte dessa gua
encontra-se localizada
a) no meio intracelular.
b) no lquido linftico.
c) nas secrees glandulares e intestinais.
d) na saliva.
e) no plasma sanguneo.
6.O colesterol tem sido considerado um vilo nos ltimos tempos,
uma vez que as doenas cardiovasculares esto associadas a altos
nveis desse composto no sangue. No entanto, o colesterol
desempenha importantes papis no organismo.
Analise os itens a seguir.
I. O colesterol importante para a integridade da membrana celular.
II. O colesterol participa da sntese dos hormnios esteroides.
III. O colesterol participa da sntese dos sais biliares.
Da anlise dos itens, correto afirmar que:
a) somente I verdadeiro.
b) somente II verdadeiro.
c) somente III verdadeiro.
d) somente I e II so verdadeiros.
e) I, II e III so verdadeiros.

QUMICA

BIOLOGIA

Aula 3 : BIOQUMICA CELULAR: Molculas ,clulas e tecidos

EXERCCIOS DE FIXAO

INTRODUO

II

III

Proteinas -so.................formadas pela unio de aminocidos


atravs de ligaes ...........................apresentando as funes :
plsticas,
hormonais,............................,transporte
e..............................(enzimas).
Na
formao
de
protenas
temos
12
aminocidos...........................que

So produzidas no tecido......................e os essenciais que so


obtidos pela dieta em nmero de ......
As enzimas so protenas especiais com funo.............................
Que atuam ..................as reaes qumicas, sendo termo lbeis
atuam de acordo com: concentrao de ...................Ph
e.......................
Protenas Estruturais.....................................e.................................,
biocatalizadora
......................
.de
defesa............................de
transporte..........
Vitaminas so substncias essenciais, obtidas atravs da
alimentao, que estimulam e regulam atividades metablicas dos
organismos. So divididas em: hidrossolveis (C, B1, B2, B6, B12,
entre outras) e lipossolveis (A, D, E e K). essa expresso se refere
a substncias orgnicas que o corpo precisa, em pequenas
concentraes, para o seu bom funcionamento,
cidos nucleicos so as molculas com a funo de
armazenamento e expresso da ....................... gentica. Existem
basicamente
dois
tipos
de
cidos
nucleicos:
cido
desoxirribonucleico (.........) e cido ribonucleico (..............). So
formados pela unio de .......................................

De seus trs componentes (acar, radical fosfato e base orgnica


nitrogenada) apenas o radical fosfato no varia no nucleotdeo. Os
aucares e as bases nitrogenadas so variveis. Quanto aos
aucares, dois tipos de pentoses podem fazer parte de um
nucleotdeo: ribose e desoxirribose(assim chamada por ter um
tomo de oxignio a menos em relao ribose. J as bases
nitrogenadas pertencem a dois grupos:.................: adenina (A) e
guanina (G);com .......anis e as ....................timina (T), citosina (C) e
uracila (U),com apenas um anel. da associao dos diferentes
nucleotdeos que se formam as macromolculas dos dois tipos de
cidos nuclicos: o cido ribonucleico (RNA) e o cido
desoxirribonucleico (DNA). Eles foram assim chamados em funo
dos acar presente em suas molculas: O RNA contm o acar
ribose e o DNA contm o acar desoxirribose. O DNA
..............................pela hereditariedade e...........................celular,
enquanto o RNA se divide em trs tipos: O que leva a
mensagem:.............................................o
que
encaixa
os
...................................(RNA transportador) o que reconhece a
protena a ser produzida. Na formao do DNA .................se liga a
.............e a citosina se liga a.........................No RNA ocorre a troca
de Adenina por ...........................

1. Considere as quatro frases seguintes.


I Enzimas so protenas que atuam como catalisadores
de reaes qumicas.
II Cada reao qumica que ocorre em um ser vivo,
geralmente, catalisada por um tipo de enzima.
III A velocidade de uma reao enzimtica independe
de fatores como temperatura e pH do meio.
IV As enzimas sofrem um enorme processo de desgaste
durante a reao qumica da qual participam.
So verdadeiras as frases:
a) I e III, apenas.
b) III e IV, apenas.
c) I e II, apenas.
d) I, II e IV, apenas.
e) I, II, III e IV.
2. Muitas estruturas do nosso organismo possuem em sua
estrutura o colgeno. Quimicamente, o colgeno pertence
ao grupo de:
a) carboidratos
b) lipdeos
c) protenas
d) glicdios
e) cidos nuclicos
3) Muitas estruturas do nosso organismo possuem em sua
estrutura o colgeno. Quimicamente, o colgeno pertence
ao grupo de:
a) carboidratos
b) lipdeos
c) protenas
d) glicdios
e) cidos nucleicos
4. Sobre o RNA, INCORRETO afirmar que
a) possui a base nitrogenada uracila.
b) formado a partir do DNA.
c) apresenta como acar o monossacardeo ribose.
d) formado por uma nica cadeia de nucleotdeos.
e) apresenta as mesmas bases nitrogenadas encontradas no
DNA.
5. O DNA e o RNA quanto sua estrutura qumica, so
classificados como:
a) polipeptdios.
b) nucleoprotenas.
c) polissacardeos.
d) fosfatdeos.
e) poli nucleotdeos.
6.Uma cadeia de RNA foi produzida tendo como molde
o filamento superior da molcula de DNA esquematizada.
Marque a alternativa que designa corretamente a
sequncia de bases deste RNA.
ACATGACT --> FITA MOLDE
TGTACTGA
a) U G U T C U G A
b) T G T U C A C U
c) U C A T G A C T
d) T G U T G A C U
e) U G U A C U G A
7. O cido ascrbico, tambm conhecido como vitamina C, est
presente em grande quantidade de frutas, verduras e legumes.
essencial para a nossa sade e cumpre
importantes funes em nosso organismo, como
a) fornecer grande quantidade de energia.
b) possibilitar a absoro de clcio.
c) contribuir para o bom funcionamento do sistema urinrio.
d) participar da manuteno dos capilares sanguneos.
e) evitar a perda de massa ssea

SECRETARIA DE ESTADO DA EDUCAO

27

QUMICA
BIOLOGIA

Aula 5: BIOQUMICA CELULAR: Molculas, clulas e tecidos

EXERCCIOS DE FIXAO

Citoplasma e organelas
Organelas de uma clula animal e suas funes:

1-Organelas de uma clula animal e suas funes:


a)Citoplasma : nele est um fludo chamado ..........., O citoplasma
tem

a funo de ................as organelas e favorecer seus

movimentos
citoplasma

alm

das

reaes.................A

denominada......................e

parte

parte

liquida

do

slida

representada pelas.................................
b)Retculo endoplasmtico liso: conjunto de membranas tubulares
que

..............................todo

citoplasma

transportando..........................produzindo ..........................esteroides
e promovendo a ....................celular.
c)Ribossomos

organela........membranosa

formada

por

1.De acordo com os estudos citolgicos, correto afirmar que:


(01) As mitocndrias so responsveis pela respirao celular tanto
em clulas animais como em clulas
vegetais.
(02) O centrolo orienta a formao do fuso mittico nas clulas dos
vegetais superiores.
(04) possvel diferenciar o DNA do RNA pela base pirimdica e pela
pentose.
(08) O principal componente do ncleo a cromatina que
constituda por DNA e protenas.
(16) O nuclolo uma estrutura caracterstica das clulas
eucariontes, visvel na interfase.
(32) O complexo de Golgi est relacionado a vrias funes
celulares, sendo a secreo celular uma delas.
Soma ( )
2. (UFRS) Em um experimento em que foram injetados aminocidos
radioativos em um animal, a
observao de uma de suas clulas mostrou os seguintes resultados:
aps 3 minutos, a radioatividade estava localizada na organela X
(demonstrando que a sntese de protenas ocorria naquele local);
aps 20 minutos, a radioatividade passou a ser observada na
organela Y; 90 minutos depois, verificou-se a presena de grnulos
de secreo de radioativos, uma evidncia de que as protenas
estavam prximas de serem exportadas.
As organelas X e Y referidas no texto so, respectivamente,
a) o complexo golgiense e o lisossomo.
b) o retculo endoplasmtico liso e o retculo endoplasmtico rugoso.
c) a mitocndria e o ribossomo.
d) o retculo endoplasmtico rugoso e o complexo golgiense.
e) o centrolo e o retculo endoplasmtico liso.

.......................e RNA ................. com funo de produo de


protenas
d)Retculo endoplasmtico rugoso : participa da sntese e
transporte de protenas de ......................alm de sntese de
..............................
e)Complexo de Golgi conjunto de ....................achatados e
......................cujas unidades so chamadas..................... relacionase

produo

do

........................nos

espermatozoides.............................mdia

nos

vegetais

armazenamento, maturao e ......................de zimognios alm de


produzir os ........................
f)Lisossomos : participam da digesto................... de substncias
orgnicas.

pode

ser

externa....................de

organelas............................ou da prpria clula.......................


g)Mitocndrias:

Organelas

com

dupla

membrana

capacidade...............................responsvel

e
pela

respirao................................
h)Plastos :Organelas que da mesma forma que as mitocndrias
apresenta dupla membrana e capacidade auto......................se
coloridos

so

........................denominados

cromoplastos

se

incolores so ......................denominados...........................
i)Centrolos. Organelas forma as por microtbulos em conjunto de
......grupos

de

.....

formados

.........................participando

do

pela

protena

..................

de

nome

da

..........................celular.(mitose e meiose) alm de ...................e


..........................................
j)Peroxissomos: vesculas que contm ......................oxidativas

SECRETARIA DE ESTADO DA EDUCAO

28

3.Associe as organelas com suas respectivas funes:


1) Complexo de Golgi
2) Lisossoma
3) Peroxissoma
4) Ribossoma
5) Centrolo
( ) - responsvel pela desintoxicao de lcool e decomposio de
perxido de hidrognio.
( ) - local de sntese proteica.
( ) - modifica, concentra, empacota e elimina os produtos sintetizados
no Retculo Endoplasmtico
Rugoso.
( ) - vescula que contm enzima fortemente hidrolticas formadas
pelo Complexo de Golgi.
( ) - responsvel pela formao de clios e flagelos.
Assinale a sequncia correta:
a) 3; 4; 1; 2; 5
d) 1; 3; 2; 4; 5
b) 2; 3; 1; 5; 4
e) 3; 4; 2; 5; 1
c) 2; 1; 3; 4; 5
4. observe trs etapas: (1) Uma clula proto-eucarionte hospedou
uma bactria aerbia, obtendo assim a mitocndria; (2)Esta clula
proto-eucarionte hospedou uma espiroqueta obtendo assim clios,
flagelos e citoesqueleto;(3) Finalmente, esta clula proto-eucarionte
hospedou uma cianobactria e obteve assim os Plastos. I. Esta
hiptese chamada Teoria Endossimbiontica muito improvvel
porque a simbiose raramente ocorre na Natureza.
II. A sntese proteica em mitocndrias e cloroplastos no ocorre na
presena de substncias inibidoras de procariontes, como
estreptomicina e cloranfenicol.
III. A membrana que envolve as mitocndrias e Plastos dupla, o
que sugere que a bactria endossimbionte foi fagocitada pela clula
proto-eucarionte.
IV. Houve a aquisio de complexidade na estrutura e funo da
clula eucaritica em relao
clula procaritica, inclusive permitindo a maturao de protenas.
V. As organelas de eucariontes, mitocndrias e Plastos, no tm
DNA prprio e, portanto no podem
fazer diviso autnoma.
Indique a alternativa em que todas as afirmativas so verdadeiras.
a) II, III e IV b) I, II e IV c) I, II, IV e V d) IV e V

QUMICA
BIOLOGIA

Aula 5: BIOQUMICA CELULAR: Molculas, clulas e tecidos

EXERCCIOS DE FIXAO
1.Entre as frases a seguir, em relao diviso celular por mitose,
uma incorreta. Aponte-a:
a)
Na metfase, todos os cromossomos, cada um com duas
cromtides, encontram-se no equador da clula em maior grau de
condensao.
b)
A clula me d origem a duas clulas filhas com metade do
nmero de cromossomos.
c)
As clulas filhas so idnticas s clulas me.
d)
Ocorre nas clulas somticas, tanto de animais como de
vegetais.
e)
um processo muito importante para o crescimento dos
organismos.
2.Marque a alternativa que completa as frases abaixo.
I.
A ___________ uma fase marcada pela
condensao dos cromossomos que se tornam progressivamente
mais curtos e grossos.
II.
Na ____________ os cromossomos se descondensam
e uma nova carioteca surge ao redor de cada conjunto
cromossmico.
III.
A _____________ inicia-se aps a desagregao da
carioteca, que liberta os cromossomos altamente condensados.
IV.
A ______________ a fase em que as cromtidesirms se separam, puxadas para polos opostos pelo encurtamento
dos microtbulos do fuso.
a)
Telfase, anfase, prfase e metfase.
b)
Anfase, metfase, prfase e telfase.
c)
Prfase, telfase, metfase e anfase.
d)
Prfase, telfase, anfase e metfase.
e)
Metfase, prfase, telfase e anfase.
3. Os esquemas I, II e III representam diferentes estgios da mitose.

Com relao a estes estgios afirma-se:


1) O esquema III representa o estgio de anfase.
2) O esquema II representa o estgio de metfase.
3) durante o estgio representado pelo esquema II que ocorre o
rompimento do envoltrio nuclear.
4) durante o estgio representado pelo esquema I que ocorre a
separao das cromtides irms.
5) Durante a diviso celular, o estgio representado pelo esquema III
precede o estgio representado pelo esquema I.
6) Durante a diviso celular, o estgio representado pelo esquema II
precede o estgio representado pelo esquema III.
Quais so as afirmaes corretas:
a) apenas 3, 4, 5 e 6
b) todas
c) apenas 1, 2, 3, e 5
d) nenhuma
e) apenas 1, 3, 5 e 6

SECRETARIA DE ESTADO DA EDUCAO

29

QUMICA

BIOLOGIA

Aula 6: BIOQUMICA CELULAR: Molculas, clulas e tecidos

EXERCCIOS DE FIXAO
1.

a) Temos meiose em A, B e C.
b) Temos mitose em A e meiose em B e C.
c) Temos meiose em A e mitose em B e C.
d) Temos meiose em A e B e mitose em C.
e) Temos mitose em A e B e meiose em C.
2. O esquema ao lado apresenta um ciclo vital:

Os nmeros I, II e III representam, respectivamente:


a) mitose, fecundao, meiose;
b) meiose, fecundao, mitose;
c) mitose, meiose, fecundao;
d) meiose, mitose, fecundao;
e) fecundao, mitose, meiose.
3. Uma clula com 8 cromossomos sofre meiose e origina:
a) 2 clulas com 4 cromossomos;
b) 2 clulas com 8 cromossomos;
c) 4 clulas com 2 cromossomos;
d) 4 clulas com 4 cromossomos;
e) 4 clulas com 8 cromossomos;
4. (UFPB) A quantidade de DNA de uma clula somtica em
metfase mittica X. Clulas do mesmo tecido, mas nas fases G1 e
G2, devem apresentar uma quantidade de DNA, respectivamente,
igual a:
a) X e X
2
b) X e X
2
c) 2X e X
d) X e 2X
e) X e 2X
5.. (UFSC 2011) Assinale a(s) proposio(es) CORRETA(S).
O ciclo celular dividido em duas etapas: Diviso Celular e Interfase.
Nesta ltima etapa (Interfase), subdividida em G1 S G2, a clula
realiza seu metabolismo e se prepara, quando necessrio, para a
etapa da diviso celular.
1) O tempo que as clulas permanecem na etapa chamada de
Interfase o mesmo entre os diferentes tipos celulares.
2) No perodo do desenvolvimento embrionrio, o ciclo celular
compreende etapas de Interfases longas e etapas de divises
celulares rpidas.
4) Na fase S da Interfase ocorre o processo de duplicao do DNA.
8) Durante a Interfase o DNA est em plena atividade, formando o
RNA com as informaes para a sntese proteica.
16) Em G2 a quantidade de DNA a mesma que em G1.

SECRETARIA DE ESTADO DA EDUCAO

30

QUMICA
BIOLOGIA

Aula 7: BIOQUMICA CELULAR: Molculas, clulas e tecidos

HISTOLOGIA ANIMAL
Ramo da biologia que estuda os.................................ou seja, o
conjunto de clulas .............................com uma mesma origem
............................e que exercem uma mesma funo no organismo.
Obs.: A diferenciao se dar por silenciamento gnico, ou seja, todas
as clulas possuem os mesmos conjuntos gnicos, s que em umas
atuam determinados conjuntos gnicos.
Origem Embrionria
Ectoderma- Tecido epitelial e nervoso
Endoderma- epitelial
Mesoderma- epitelial ,muscular e conjuntivo
Tecido epitelial- se caracteriza por apresentar clulas.....................
Com pouco ou..............................material intercelular e so ..............
.......................nutridos
pelo
conjuntivo................................................................. a sua funo
bsica o revestimento e secreo sendo denominado
respectivamente de epitlio de ............................e..............................
Conjuntivo- Tecido com ..................substancia ..................fibras e
clulas especiais, vascularizados e ........................................As fibras
so divididas em colgenas,.................................e............................
As
clulas
so
os
fibroblastos(produtoras
da
....................)Macrfagos(
Defendem o tecido atravs da.......................) Plasmcitos( defendem
o tecido atravs dos...............................) Mastcitos (produtoras de
................... e ..................................)a relacionando a processos
alrgicos e os adipcitos que armazenam gorduras. Apresentam
grande variedade, sendo classificado em ....................dito e ...............
Este ltimo se divide em (..................., cartilaginoso,.........................
e sanguneo)

Muscular- tecido contrtil que unido aos..................alavanca o


esqueleto, atravs de..........................Formados por duas protenas,
a ......................e a ..........................que se encontra em clulas
especiais denominadas
fibras ou......................................ou
mimeros. So de trs tipos: Lisa de contrao....................
encontrada
nas
vsceras
ocas
e
vasos
sanguneos,
estriada.................
De contrao........................sendo as mais abundantes e
estriada........................representada
pelo..........................
No
processo de contrao as fibras de ...........................deslizam sobre
as de .........
...............encurtando o ........................esse processo ocrre com gasto
de energia e ons.........................
NervosoFormado por clulas altamente especializadas, os...............................
que so auxiliados pelas clulas da..................So responsveis pela
...............................,integrao, elaborao e resposta aos estmulos
provenientes do meio.................................
.

SECRETARIA DE ESTADO DA EDUCAO

31

nervos so vrios agrupamentos de feixes de axnios e dendritos.


Em torno dos axnios existem a bainha de mielina, coberta pela
bainha de Schwann
Arco reflexo- ao nervosa que envolve trs neurnio, um
.......................outro associativo e um outro...................O crebro no
toma conhecimento do.............para sua realizao, ele fica
nvel...........................
EXERCICIOS DE FIXAO
1.As clulas nervosas apresentam especializaes que as
diferenciam das demais no organismo. Desta forma, a alternativa que
apresenta as principais clulas que compem o tecido nervoso :
a) mastcito e macrfago
d) neurnio e glicitos
b) linfcito e plasmcito
e) neutrfilo e basfilo
c) mitose e meiose
2.O sistema nervoso possui trs componentes bsicos, encfalo,
medula e nervos. Estes ltimos fazem a ligao das outras duas
estruturas com as diversas partes do corpo. Se desfiarmos um
desses nervos, veremos que so constitudos de estruturas
cilndricas muito finas denominadas axnios, prolongamentos dos
neurnios. Os outros dois componentes estruturais dos neurnios
so:
a) o corpo celular e os dendritos
b) o corpo celular e as sinapses
c) os dendritos e as sinapses
d) os dendritos e as clulas da glia
e) as sinapses e as clulas da glia
3.as clulas que constituem os organismos dos metazorios foram se
especializando no decorrer do processo evolutivo, se agrupando em
tecidos e estes formando os rgos, que por sua vez, formam os
sistemas.
Assinale a alternativa que apresenta as afirmativas corretas.
I - Nos animais, os estmulos do ambiente so captados por rgos
sensoriais, como o olho e o ouvido. Nesses rgos, encontramos
neurnios sensitivos, encarregados de receber os estmulos e
transform-los em impulsos nervosos.
II - O tecido epitelial reveste o corpo humano, forra as cavidades
internas e forma as glndulas. Apesar de no possuir terminaes
nervosas, esse tecido nutrido por vasos sangneos.
III - O sangue pode ser considerado um tecido muito especial, porque
possui uma substncia intercelular no estado lquido, o plasma. Nele
esto mergulhadas as clulas do sangue, os glbulos sangneos.
IV - O msculo liso possui fibras mononucleadas, sem estrias
transversais. encontrado na parede dos rgos ocos (tubo
digestivo, tero, artrias e brnquios) e apresentam contrao
involuntria.
a) Todos os itens esto corretos.
b) Os itens I e II esto corretos
c) Os itens II, III e IV esto corretos
d) Os itens I, II e III esto corretos
e) Os itens I, III e IV esto corretos
4. (UFPB) O tecido que est amplamente distribudo pelo corpo e
desempenha
funes
bastante
variadas,
tais
como,
sustentao, defesa do organismo e transporte de substncias, o
tecido
a) muscular.
b) sseo
c) conjuntivo.
d) sanguneos
e) epitelial.

QUMICA
BIOLOGIA

Aula 8: BIOQUMICA CELULAR: Molculas, clulas e tecidos

EXERCICIOS DE FIXAO

3.(FUVEST) Enzimas de restrio so fundamentais Engenharia


Gentica porque permitem:
a) a passagem de DNA atravs da membrana celular;
b) inibir a sntese de RNA a partir de DNA;
c) inibir a sntese de DNA a partir de RNA;
d) cortar DNA onde ocorrem sequncias especficas de bases;
e) modificar sequncias de bases do DNA.
4. A Engenharia Gentica consiste numa tcnica de manipular
genes, que permite, entre outras coisas, a fabricao de produtos
farmacuticos em bactrias transformadas pela tecnologia do DNA
recombinante. Assim, j possvel introduzir em bactrias o gene
humano que codifica insulina, as quais passam a fabricar
sistematicamente essa substncia. Isto s possvel porque:

Ferramentas e procedimentos

a) Enzimas de restrio- Tambm conhecidas como endonucleases


de restrio...........................o DNA em pontos...................................
b) Placa de eletroforese-Utilizada para separar
os.............................de dna nos exames de identificao de...........
c)Clonagem- produo em srie de .....................ou.....................com
fins.............................................................ou Teraputicos.

a) o cromossomo bacteriano totalmente substitudo pelo DNA


recombinante;
b) as bactrias so seres eucariontes;
c) os ribossomos bacterianos podem incorporar o gene humano que
codifica insulina, passando-o para as futuras linhagens;
d) as bactrias possuem pequenas molculas de DNA circulares
(plasmdeos), nas quais podem ser incorporados genes estranhos a
elas, experimentalmente;
e) as bactrias so seres muito simples, constitudos por um nico
tipo de cido nuclico (DNA).
5. (MACKENZIE) Atualmente deixou de ser novidade a criao de
plantas transgnicas, capazes de produzir hemoglobina. Para que
isso seja possvel, essas plantas recebem:
a) o fragmento de DNA, cuja sequncia de nucleotdeos determina a
sequncia de aminocidos da hemoglobina;
b) o RNAm que carrega os aminocidos usados na sntese de
hemoglobina;
c) somente os aminocidos usados nessa protena;
d) os anticdons que determinam a sequncia de aminocidos nessa
protena;
e) os ribossomos utilizados na produo dessa protena.
6. A clonagem molecular :

d) DNA Finger Print- Obteno de padres especficos do........ de


um organismo.
e) Transgnicos- Organismos que apresentam no seu .........material
gentico de outra........................funcionando.

f) Projeto Genoma- Sequenciamento de bases que compe o......


de um organismo.
g) Clulas tronco. So clulas indiferenciadas com capacidade de se
..........................nas diversas linhagens celulares, inclusive formando
os anexos..........................(Totipotentes)

SECRETARIA DE ESTADO DA EDUCAO

32

a) a tcnica que emprega bactrias como multiplicadores de um


fragmento de DNA;
b) o mecanismo para se obter resistncia a antibiticos;
c) a fabricao de produtos farmacuticos;
d) o processo utilizado para cortar o DNA;
e) a enzima utilizada na Gene terapia.
7. Os avanos de Engenharia Gentica permitem que um ser vivo
fornea genes a outro de espcie diferente, sem alterar as principais
caractersticas que os diferenciam. O seu que recebe o gene
denominado:
a) clone
b) parasitado
c) mutante
d) transgnico
e) mutagnico

QUMICA
BIOLOGIA

EXERCICIOS DE FIXAO
Aula 9: BIOQUMICA CELULAR: Molculas, clulas e tecidos

1 anlise de trs grupos de ovos em desenvolvimento apresentou as

EMBRIOLOGIA
a cincia que estuda a..............................e o desenvolvimento do
embrio, desde a formao dos ............................gametognese, at
o nascimento.
Gametognese- a formao dos gametas masculino
(espermatognese) e feminino(..............................)
Tipos de ovos
Os ovos dos animais possuem um material nutritivo denominado
vitelo, cuja concentrao e distribuio diferem conforme a espcie.
Podemos classificar os ovos em isolcitos, mesolcitos, megalcitos
e centrolcitos.
Isolcitos ou oligolcitos so ovos que contm pequena quantidade
de vitelo uniformemente distribudo pelo citoplasma. So
caractersticos de porferos, equinodermos, protocordados e
mamferos (nestes so chamados de alcitos).
Mesolcitos ou heterolcitos so ovos que apresentam cerca da
metade do volume citoplasmtico (plo vegetativo) ocupado pelo
vitelo; o ncleo situa-se no plo oposto (plo animal). So
caractersticos dos platelmintos, aneldeos, moluscos, anfbios e
algumas espcies de peixes.
Megalcitos ou telolcitos so ovos nos quais a quantidade de vitelo
to grande que ocupa quase todo o citoplasma (plo vegetativo),
enquanto que o ncleo ocupa um espao mnimo na periferia (plo
animal ou disco germinativo). So caractersticos de aves, rpteis e
algumas espcies de peixes.
Centrolcitos so ovos nos quais o ncleo central, envolvido pelo
citoplasma. So caractersticos dos artrpodes.

seguintes caractersticas:
Grupo I: ovos com distribuio razovel de vitelo,
Grupo II: ovos ricos em vitelo,
Grupo III: ovos pobres em vitelo, Os ovos desses grupos
poderiam ser, respectivamente:
a)sapo, galinha e ourio do mar
b) galinha, sapo e macaco
c) cachorro, galinha e sapo
d)galinha, ourio do mar e homem
e e)ourio do mar, macaco e sapo.
B 2.A notocorda uma estrutura embrionria que:
a)
b)

d origem aos somitos


persiste no adulto de todos os vertebrados

c c)

desaparece no anfioxo adulto

d)

substituda pela coluna vertebral nos vertebrados

e)

d origem coluna vertebral dos mamferos.

3.No desenvolvimento embrionrio dos animais, existem etapas


caractersticas nas quais ocorrem processos mais ou menos

DESENVOLVIMENTO EMBRIONRIO

semelhantes. Observe que h uma sequncia cronolgica igual para

Aps a fecundao, o ovo.............................. a se dividir.


As divises(...........................) prosseguem at formar-se um
aglomerado macio de clulas denominado ...................................
(64 clulas). Apesar do maior nmero de clulas, a mrula tem um
volume quase igual ao do zigoto que a originou.
Em seguida, as clulas da mrula vo-se posicionando na poro
perifrica enquanto secretam ....................... que se instala
no.........................., ocupando uma cavidade. O estgio embrionrio
nessa fase denomina-se ...................... nesse estgio de
desenvolvimento que, nos seres humanos, o ovo chega cavidade
uterina. Geralmente por volta do sexto dia aps a fecundao .A
seguir ocorre a.................... ou formao da gstrula. Um dos polos
se dobra para dentro formando duas camadas de clulas: o
ectoderma e o endoderma,........................, blastporo e crescimento
do .....................embrionrio .Nos seres mais evoludos, a gstrula
evolui para um novo estgio. Surge um terceiro folheto embrionrio,
o mesoderma. Nos animais vertebrados, ocorre a neurulao ou
formao da nurula , onde se formar o ...............................e a
notocorda que daro origem ao sistema nervoso central e coluna
vertebral, respectivamente.

todos os grupos zoolgicos, traduzindo a "origem comum" dos


metazorios. Os principais "momentos" pelos quais passam os
embries de diferentes grupos so:
1.

segmentao

2.

Mrula

3.

Blstula

4.

gstrula

5.

nurula.
Para voc visualizar o que ocorre em cada uma dessas fases, basta
relacion- las corretamente com os eventos abaixo.

A.

Formao de tubo neural

B.

Proliferao do ovo originando os blastmeros

C.

Micrmeros e macrmeros envolvendo pequena cavidade central

D.

Intensas modificaes dos blastmeros originando trs folhetos

ANEXOS EMBRIONRIOS

embrionrios

Os anexos embrionrios so .................... ligadas ao embrio de


rpteis, aves e mamferos, relacionados com a ................... desses
vertebrados ao ambiente terrestre.
mnio bolsa contendo lquido......................, favorecendo
ambiente mido ao desenvolvimento do embrio, amortecendo os
choques trmicos e mecnicos.
Crio membrana de natureza celular. Envolvendo o embrio e o
saco vitelino, protegendo-os
Alantoide bolsa membranosa que realiza o armazenamento das
,,,,,,,,,,,,,,,,,,,,,,do...................... Em rpteis e aves, a membrana do
alantoide une-se a do crio, constituindo o alantocrio, exercendo
funo respiratria

E.

Formao de uma estrutura esfrica e macia.

Assinale a alternativa que contm a sequncia correta dos


"momentos" do desenvolvimento embrionrio.
a)

1E, 2B, 3A, 4D, 5C

b b)

1B, 2E, 3C, 4D, 5A

c c)

1A, 2C, 3E, 4B, 5D

d d)

1C, 2D, 3A, 4E, 5B

e e)

1D, 2A, 3E, 4B, 5C

Saco vitelnico bolsa contendo substncias de ...................


energtica (vitelo), responsvel pela nutrio do embrio, nos
mamferos .....................pela placenta.

SECRETARIA DE ESTADO DA EDUCAO

33

QUMICA
BIOLOGIA

Aula 10 : ZOOLOGIA

EXERCICIOS DE FIXAO

PORFEROS
So denominados................pois no possuem..............verdadeiros
apresentam o corpo simples formado por ........,trio e ................,
so assimtricos ou,,,,,,,,,,,,, ,Eles se dividem em trs classes,
scon, Scon e Lucon. So todos aquticos e................(fixos ao
substrato).Em geral no tem esqueleto, mas possuem estruturas
chamadas espculas que exercem essa funo. No apresentam
nenhum sistema.
CNIDRIOS
So..............................radiais, ssseis.............................ou livres
nadantes................................ apresentam os primeiros sistemas:
Sistema digestivo - .......................................................
Sistema circulatrio - Ausente, distribuio pela................
Sistema respiratrio - Ausente, trocas gasosa com o ambiente
Sistema excretor - Ausente, liberadas pelas clulas no ambiente
Sistema nervoso -.........................................................
Reproduo - Ocorre de forma sexuada ou assexuada,
Desenvolvimento - Indireto, fase larvria chamada de plnula
PLATIELMINTOS
So os primeiros vermes, que se caracterizam por apresentar o
corpo...............dorso..................so............................,acelomados
de
vida............ou
.............................,
divididos
em
trs
classes:
Tuberlrios-(........................)Trematdeos(.............................).e
cestdeos(.............................................)
Sistema digestivo - .......................................................
Sistema circulatrio - Ausente, distribuio pela................
Sistema respiratrio - Ausente, trocas gasosa com o ambiente
Sistema excretor - ...................................................................
Sistema nervoso -....................................................................

2.As verminoses representam um grande problema de sade,


principalmente
nos pases subdesenvolvidos. A falta de redes de gua e de esgoto, de
campanhas de
esclarecimento pblico, de higiene pessoal e de programas de combate
aos transmissores, leva
ao aparecimento de milhares de novos casos na populao brasileira.
Dentre as verminoses humanas causadas por nemtodos, citam-se,
corretamente,
a) tenase, ascaridase e ancilostomose.
b) filariose, ancilostomose e ascaridase.
c) esquistossomose, ascaridase e ancilostomose.
d) esquistossomose, filariose e oxiurose.
e) tenase, filariose e esquistossomose.
3. Nematdeos so animais vermiformes de vida livre ou parasitria,
encontrados em plantas e animais, inclusive no homem. Sobre as
caractersticas presentes em nematdeos,
considere as afirmativas a seguir.
I. Corpo no-segmentado coberto por cutcula.
II. Trato digestrio completo.
III. rgos especializados para circulao.
IV. Pseudoceloma.
Esto corretas apenas as afirmativas:
a) I e III.
b) I e IV.
c) II e III.
d) I, II e IV.
e) II, III e IV.
4. Indique as caractersticas que tornam os organismos do filo Porifera
bem diferentes daqueles de outros filos animais:

Reproduo - Ocorre de forma sexuada(............................... ou


assexuada(....................................)
NEMATELMINTOS
So
vermes..................................afilados
nas
.........................que
apresentam trs folhetos , protostomia, simetria..........................j
apresentam
uma
cavidade
interna
cheia
de
lquido
denominada......................................so parasitas de...........................
e......................................................................
Sistema digestivo - .......................................................
Sistema circulatrio - Ausente, distribuio pela................
Sistema respiratrio - Ausente, trocas gasosa com o ambiente
Sistema excretor - ...................................................................
Sistema nervoso -....................................................................
Reproduo - Ocorre de forma sexuada(.............................................. )
MOLUSCOS
So animais de...........mole, com ou........concha, triblsticos,
bilaterais,...............e celomados, so terrestres ou .........................de
grande importncia econmica.
Sistema digestivo - ...................................................................
Sistema circulatrio - ................................................................
Sistema respiratrio - ...............................................................
Sistema excretor - Ausente, liberadas pelas clulas no ambiente
Sistema nervoso -.........................................................
Reproduo - Ocorre de forma sexuada, Desenvolvimento - Indireto,
fase larvria chamada de trocfora e vliger

SECRETARIA DE ESTADO DA EDUCAO

1.(UFPI) Os moluscos constituem um grupo abundante e diversificado de


animais que apresentam
corpo mole, com ou sem concha, simetria bilateral, sendo triblsticos e
celomados. Assinale a
alternativa que indica corretamente todos os possveis habitats desses
animais.
a) Ambientes aquticos e terrestres.
b) Ambiente marinho.
c) Ambientes aquticos: marinho e dulccola.
d) Ambientes marinho e terrestre.
e) Ambientes dulccola e terrestre.

34

a) No podem se reproduzir.
b) As formas adultas so ssseis.
c) No respondem a estmulos externos.
d) Alimentam-se atravs de mecanismos de filtrao.
e) Suas clulas no so organizadas em tecidos.
5. (UFSJ) A digesto dos Porferos (esponjas) intracelular e realizada
por clulas chamadas:
a) arquecitos.
b) porcitos.
c) coancitos.
d) pinaccitos.
e) amebcitos.
6. (FURG/2008) Assinale a alternativa que apresenta a funo dos tipos
celulares de Porfera.
a) A digesto do alimento realizada pelos coancitos, e os nutrientes
so distribudos pelos pinaccitos.
b) Os coancitos so responsveis pela fagocitose das partculas
alimentares.
c) Os amebcitos so responsveis somente pela produo das
espculas.
d) Os porcitos so as clulas que circundam a abertura do sculo, por
onde entra a gua para a espongiocele.
e) Os pinaccitos formam o tecido presente entre a parede externa do
corpo e a parede da espongiocele.

QUMICA
BIOLOGIA

Aula 11 ZOOLOGIA

EXERCICIOS DE FIXAO

ANELIDEOS
So animais triblsticos, bilaterais, protostnmios e .......................
Que apresentam o corpo formado por,,,,,,,,,,,,,ou...............................
So divididos em trs classes de acordo com as.............quitinosas:
Oligoquetos-..............cerdas, como as minhocas, Poliquetos-muitas
cerdas como nas.....................marinhas e aquetos...........cerdas
quitinosas. Importante na agricultura e indstria de.............................
Sistema digestivo - .......................................................
Sistema circulatrio - Ausente, distribuio pela................
Sistema respiratrio - Ausente, trocas gasosa com o ambiente
Sistema excretor - ...................................................................
Sistema nervoso -....................................................................
Reproduo - Ocorre de forma sexuada(............................... )
ARTRPODOS
So os...............................de maior disperso e........................ocupando
todos os habitats, so divididas em 5 classes: Insecta,
Crustcea,Aracnidea, Chilpoda e Diplpoda, crescem diferentes dos
outros invertebrados pois devem se livrar do exoesqueleto, tambm
sofrem metamorfose(...................................)
Sistema digestivo - .......................................................
Sistema circulatrio - Ausente, distribuio pela................
Sistema respiratrio - Ausente, trocas gasosa com o ambiente
Sistema excretor - ...................................................................
Sistema nervoso -....................................................................
Reproduo - Ocorre de forma sexuada(............................... )
EQUINODERMOS
So exclusivamente...........................com
simetria......................,deuterostomia e celoma enteroclico, sua principal
caracterstica espinhos na pele.
Sistema digestivo - .......................................................
Sistema circulatrio - Ausente, distribuio pela................
Sistema respiratrio - Ausente, trocas gasosa com o ambiente
Sistema excretor - ...................................................................
Sistema nervoso -....................................................................
Reproduo - Ocorre de forma sexuada(............................... )

1.A ausncia de bexiga urinria e a eliminao continua de fezes pode


ser considerada (em aves) uma adaptao (ao):
a) homeotermia
b) heterotermia
c) vida arborcola
d) postura de ovos
e) vo
2. Assinale a alternativa que no correta em relao s aves:
a) o rgo do canto denominado siringe;
b) a glndula uropigiana a nica presente na pele, produzindo secreo
oleosa;
c) o esterno pode ou no ter uma quilha, cuja funo auxiliar no vo;
d) sob as duas plpebras h uma fina e quase transparente membrana
denominada nictitante que protege os olhos durante o vo;
e) as hemcias so anucleadas.
3.(MED. - Taubat) Vermes segmentados marinhos, lmnicos e
terrestres, triblsticos, celomados, com segmentao metamrica e
aparentados com os artrpodes. Trata-se de:
a) nematides
b) cestides
c) aneldeos
d) platielmintes
e) celenterados
4 .O gafanhoto, a aranha e o camaro tm em comum:
a) um par de antenas;
b) o corpo dividido em cabea, trax e abdmen;
c) a respirao traqueal;
d) pertencerem ao filo dos artrpodos;
e) possurem 3 pares de patas articuladas.
5.A cobra-cega, a cobra-de-duas-cabeas e a cobra-de-vidro pertencem:

CORDADOS
So os animais mais evoludos do planeta divididos em dois grupos:
Protocordados e ..............................Ocupam todos os habitats do planeta
sendo divididos em 5 classes: Peixes, anfbios, rpteis, aves e mamferos
Sistema digestivo - .......................................................
Sistema circulatrio - Ausente, distribuio pela................
Sistema respiratrio - Ausente, trocas gasosa com o ambiente
Sistema excretor - ...................................................................
Sistema nervoso -....................................................................
Reproduo - Ocorre de forma sexuada(............................... )

a) todas aos lacertlios;


b) aos anfbios, ofdios e lacertlios, respectivamente;
c) todas aos ofdios;
d) aos lacertlios, anfbios e anfbios, respectivamente;
e) aos lacertlios, anfbios e lacertlios, respectivamente.
6. (FAAP) Dos animais abaixo, os nicos que apresentam respirao
pulmonar so:
a) minhoca, sapo e peixe;
b) golfinho, barata e cobra;
c) peixe-boi, jacar e pato;
d) baleia, aranha e peixe;
e) tartaruga, jacar e tubaro.
7. (FATEC) Comparando-se evolutivamente, o animal mais prximo do
homem :
a) o lagarto
b) o pingim
c) o sapo
d) o rato
e) o cao
8. (MED. CATANDUVA) Associe:
( ) preguia
1. xenarto
( ) elefante-marinho
2. roedor
( ) paca
3. sirnio
( ) peixe-boi
4. marsupial
( ) gamb
5. carnvoro
a) 2, 3, 4, 5, 1
b) 4, 2, 3, 5, 1
c) 1, 5, 2, 3, 4
d) 3, 5, 2, 1, 4
e) 1, 5, 2, 3, 4

SECRETARIA DE ESTADO DA EDUCAO

35

QUMICA
BIOLOGIA

Aula 12 :EVOLUO

EXERCICIOS DE FIXAO
1.(UNIFESP/2004) Leia os trechos seguintes, extrados de um texto
sobre a cor de pele humana.
A pele de povos que habitaram certas reas durante
milnios adaptou-separa permitir a produo de vitamina D.
medida que os seres humanos comearam a se movimentar pelo
Velho Mundo h cerca de 100 mil anos, sua pele foi se
adaptando s condies ambientais das diferentes regies. A cor da
pele das populaes nativas da frica foi a que teve mais tempo para
se adaptar porque os primeiros seres humanos surgiram
ali.
(Scientific American Brasil, vol.6,
novembro de 2002).
Nesses dois trechos, encontram-se subjacentes ideias:
a) da Teoria Sinttica da Evoluo.
b) darwinistas.
c) neodarwinistas.
d) lamarckistas.
e) sobre especiao.

Se baseia em duas leis: A do uso e.......................e a da transmisso


hereditria dos .......................adquiridos.

2.O hbito de colocar argolas no pescoo, por parte das mulheres


de algumas tribos asiticas, promove o crescimento desta estrutura,
representando nestas comunidades um sinal de beleza. Desta forma
temos que as crianas, filhos destas mulheres j nasceriam com
pescoo maior, visto que esta uma tradio secular.
A afirmao acima pode ser considerada como defensora de qual
teoria evolucionista:
a) Teoria de Lamarck
b) Teoria de Malthus
c) Teoria de Wallace
d) Teoria de Darwin
e) Teoria de Mendel
3. (UFC/2004) O ambiente afeta a forma e a organizao dos
animais, isto , quando o ambiente se torna muito
diferente, produz ao longo do
tempomodificaes correspondentes na forma e organizao dos
animais... As cobras adotaram o hbito de se arrastar no solo e se
esconder na grama; de tal maneira que seus corpos, como
resultados de esforos repetidos de se alongar, adquiriram
comprimento considervel....
O trecho citado foi transcrito da obra Filosofia Zoolgica de um
famoso cientista evolucionista.
Assinale a alternativa que contm, respectivamente, a ideia
transmitida pelo texto e o nome do seu autor.
a) Seleo natural Charles Darwin.
b) Herana dos caracteres adquiridos Jean Lamarck.
c) Lei do transformismo Jean Lamarck.
d) Seleo artificial Charles Darwin.
e) Herana das caractersticas dominantes Alfred Wallace.
4. Considerando diferentes hipteses evolucionistas, analise as
afirmaes abaixo e as respectivas justificativas.
A O Urso Polar BRANCO porque vive na NEVE!
B O Urso Polar vive na NEVE porque BRANCO!
As afirmaes A e B podem ser atribudas, respectivamente, a:
a) Lamarck e Darwin.
b) Pasteur e Lamarck.
c) Pasteur e Darwin.
d) Darwin e Wallace.
e) Wallace e Darwin.
5.Quais as caractersticas presentes nos indivduos de uma espcie
afim de que possamos afirmar que os mesmos so mais adaptados
em comparao a outros indivduos da mesma espcie:
a) so maiores e solitrios.
b) comem mais e apresentam cores vibrantes.
c) vivem mais e reproduzem mais.
d) apresentam mais membros como pernas ou patas.
e) so mais fortes.

SECRETARIA DE ESTADO DA EDUCAO

36

QUMICA
BIOLOGIA

Aula 13 : Botnica

EXERCICIOS DE FIXAO

o ramo da biologia que estuda os .........................sendo estes os


organismos...................................
Embrifitos, cormfitos com clorofila.... e ......, amido como SRN e
celulose na ...........................celular.

1.As plantas, assim como todos os demais seres vivos, possuem


ancestrais aquticos e desta forma
sua histria evolutiva encontra-se relacionada ocupao progressiva do
ambiente terrestre. Para que isso
pudesse acontecer algumas caractersticas foram selecionadas e dentre
elas podemos destacar:
I - Sistema vascular
II - Esporfito dominante
III - Filoides
IV - Esporfito no ramificado
So caractersticas prprias de pteridfitas e brifitas, respectivamente:
a) I e II; III e IV
b) I e III; II e IV
c) II e IV; I e III
d) III e IV; I e II

Ciclo evolutivo.

O
O Reino Plantae de extrema importncia manuteno da vida na
Terra, e no somente a ns, seres humanos. As plantas so,
primordialmente, as responsveis pela nutrio de todos os seres vivos.
Alm disso, fornecem oxignio e tambm matria-prima.
O Reino Plantae, apresenta os cormfitos(folhas,.............e ..................)

2. (UFRS) Brifitas e pteridfitas apresentam vrias caractersticas em


comum, mas tambm diferem em
muitos aspectos.
Assinale a caracterstica que pertence a apenas um desses grupos de
plantas.
a) Crescer preferencialmente em solos midos e sombreados.
b) Necessitar de gua para reproduzir-se.
c) No ter flores, sementes e frutos.
d) Ser criptgama.
e) Ser portadora de tecidos de transporte.
3. Alternncia de geraes ou ciclo haplntico-diplntico, isto , uma
gerao
haploide que produz gametas (Gametfito) e uma outra diploide que
produz esporos (Esporfito), ocorre
a) apenas em angiospermas.
b) apenas em gimnospermas e em angiospermas.
c) apenas em pteridfitas, em gimnospermas e em angiospermas.
d) apenas em brifitas, em pteridfitas, em gimnospermas e em
angiospermas.
e) em algumas algas, em brifitas, em pteridfitas, em gimnospermas e
em angiospermas.

- Criptgamas , cujas estruturas de reproduo no se apresentam


visveis; brifitas e pteridfitas;
- Fanergamas, cujas estruturas de reproduo se apresentam visveis;
gimnospermas e angiospermas.
Plantas avasculares :representado pelas brifitas que
apresentam..................porte , como , por exemplo , musgos.
Plantas vasculares : representados pelos vegetais que
conquistaram o ambiente terrestre, so ...........................de acordo
com
a
presena
ou
...........................de
sementes.
- Sem semente :Pteridfitas primeiras plantas a conquistarem o
ambiente terrestre, porm com dependncia da gua para reproduo
- Com semente,
sem
frutos (gimnospermas):
so
vegetais
independentes da gua para reproduo pois apresentam
tubo...................................... e gros de...........................................
- Com
semente,
com
frutos
e
flores (angiospermas):
As angiospermas so as plantas que predominam em nosso planeta;
sendo as........................a apresentarem frutos de extrema...................na
disperso
das.........................se
dividem
em
.......................e
................................sendo consideradas pertencentes a um grupo
monofiltico, dentro da Botnica.

SECRETARIA DE ESTADO DA EDUCAO

37

4- (UFES) No trecho a seguir, os autores destacam algumas


caractersticas do trabalhador brasileiro:
Rancho de Goiabada
(Joo Bosco e Aldair Blanc)
Os boias-frias
Quando tomam umas birita
Espantam a tristeza
Sonham com bife a cavalo,
Batata-frita e sobremesa
goiabada-casco com muito queijo.
Depois do caf, cigarro e um beijo
De uma mulata chamada Leonor
Ou Dagmar...
Do ponto de vista botnico, podemos afirmar que
a) o caldo-de-cana, do qual feita a birita (cachaa), constitudo de
uma soluo orgnica em que predominam acares solveis
acumulados no floema do colmo cheio da cana-de-acar.
b) a batata-frita, alimento muito apreciado, corresponde raiz tuberosa
principal de plantas de batata(Solanum tuberosum), que armazena
substncias de reserva, principalmente o amido.
c) a goiabada-casco tem pedaos (casco) do fruto da goiabeira, que
correspondem ao epicarpo seco originado do receptculo floral.
d) o famoso cafezinho brasileiro tem como um dos seus principais
constituintes a cafena, que possui acentuada ao estimulante e
produzida pelo tegumento da semente.
e) o cigarro, feito a partir do meristema apical das folhas de fumo
(Nicotiana tabacum), tem altaconcentrao de nicotina, alcaloide que
pode provocar o cncer de pulmo.

QUMICA
BIOLOGIA

Aula 14 : FISIOLOGIA VEGETAL

EXERCICIOS DE FIXAO

ABSORO
A gua e os sais minerais (................ inorgnicas) so absorvidas nos
pelos absorventes e atingem o................ no cilindro central por duas vias:
Depende de transporte .......... e osmose, atinge os vasos lenhosos
(xilema) de forma mais lenta.
Ao chegarem nos vasos xilemticos a seiva bruta ser distribuda das
razes at as folhas.

1.(Fuvest-2000) Entre as plantas vasculares, uma


caracterstica que pode ser usada para diferenciar grupos :
a) presena de xilema e de floema.
b) ocorrncia de alternncia de geraes.
c) dominncia da gerao diploide.
d) desenvolvimento de sementes.
e) ocorrncia de meiose.

Transporte
Conduo da Seiva Bruta (gua e sais minerais) = Nutrientes Inorgnicos
As clulas do xilema que conduzem a seiva bruta so mortas. Estas
formam longos tubos de dimetro reduzido (..................)
Presso positiva ou Impulso da raiz, ........................e teoria da CoesoTenso-Adeso (Teoria de ..................)

2. Nas grandes rvores, a seiva bruta sobe pelos vasos lenhosos, desde
as razes at as folhas:
a) bombeada por contraes rtmicas das paredes dos vasos.
b) apenas por capilaridade.
c) impulsionada pela presso positiva da raiz.
d) por diferena de presso osmtica entre as clulas da raiz
e as do caule.
e) sugada pelas folhas, que perdem gua por transpirao.

Em dias frios e midos, estando o solo encharcado de gua, as razes


podem, excepcionalmente, empurrar seiva bruta at copa onde esta, no
podendo ser evaporada por causa da baixa temperatura e da saturao
de umidade do ambiente, sai pelos bordos da folha atravs de pequenas
aberturas que so os hidatdios (estmatos que perderam a capacidade
de fechar). Este fenmeno denominado de sudao ou gutao.
O cintamento num caule interrompe-se o fluxo descendente da seiva
orgnica que vai se acumulando acima do corte, mesmo assim a seiva
bruta sobe. Depois de suas reservas esgotadas as razes morrem, com
isso o transporte de gua para e a planta morre.

Fotossntese
Fotossntese a capacidade do vegetal transformar energia
............... em energia ................., utilizando-se de ...........e luz, que
fornecero a energia para fixar o CO2 na forma de .................. A
energia luminosa transferida para as ligaes entre dois tomos de
carbono, at formar a glicose.

3.A gua transportada por vasos lenhosos at a folha e, nas clulas


desse rgo, fornece hidrognio para a realizao de um processo
bioqumico, por meio do qual produzido um gs que poder ser
eliminado para o ambiente e tambm participar de um outro
processo bioqumico naquelas mesmas clulas.
A estrutura que NO tem associao com a descrio
a) cloroplasto.
b) mitocndria.
c) floema.
d) xilema.
e) estmato
4.A contribuio da seiva bruta para a realizao da fotossntese nas
plantas vasculares a de fornecer
a) glicdios como fonte de carbono.
b) gua como fonte de hidrognio.
c) ATP como fonte de energia.
d) vitaminas como coenzimas.
e) sais minerais para captao de oxignio.
5. Das frases abaixo, assinale a que est
correta:
a) a fotossntese o processo de converso da energia
qumica em energia luminosa feito pelas plantas.
b) os hetertrofos so a base das cadeias alimentares.
c) uma planta que tem uma taxa fotossinttica maior que
sua taxa de respirao consegue acumular matria orgnica.
d) a glicose produzida no processo de respirao a fonte
energtica dos organismos clorofilados.
e) a fermentao o principal meio dos organismos
obterem energia.
6) Acerca desse desenvolvimento e da ao dos hormnios vegetais
foram feitas as seguintes afirmaes:
I. Esse crescimento resultado da ao direta de hormnios
vegetais conhecidos como citocininas, que estimulam as divises
celulares e o desenvolvimento das gemas laterais, fazendo com que
o caule cresa em direo fonte de luz.
II. Nessa planta, o hormnio conhecido como auxina fica mais
concentrado do lado menos iluminado, o que faz com que as clulas
desse lado alonguem-se mais do que as do lado exposto luz,
provocando a curvatura da planta para o lado da janela entreaberta.
III. O movimento de curvatura apresentado por essa planta
denominado tigmotropismo e pode ser explicado pela ao do
hormnio giberelina, que se concentra no lado iluminado do
caule, como uma resposta ao estmulo luminoso da janela
entreaberta.
Dessas afirmaes , somente est (esto) correta(s):
a) I.
b) II.
c) III.
d) I e II.
e) I e III.

SECRETARIA DE ESTADO DA EDUCAO

38

QUMICA
BIOLOGIA

Aula 15 : GENTICA

EXERCICIOS DE FIXAO

rea biolgica que ...........


as bases da
..........................Fundamentada por .................depois do seu
aperfeioamento tornou-se uma grande...................para as vrias
reas do conhecimento...........................
Conceitos bsicos
Caritipo Conjunto de ........................ de cada clula de um
organismo.
Herana Biolgica (hereditariedade) Transmisso das
informaes genticas de ...................... para filhos durante a
reproduo.
Genes Seguimento da molcula de DNA que contm uma
instruo ....................... codificada para a sntese de uma protena.
Gentipo

Constituio
gentica
de
um
indivduo
Fentipo Caractersticas ou conjunto de caractersticas fsicas,
fisiolgicas ou comportamentais de um ser vivo, associao
do.........................associado
ao
.........................
Cromossomo Cada um dos longos filamentos presentes no
ncleo das clulas eucariticas, constitudos basicamente por DNA e
protenas.
Cromossomos Homlogos Cada membro de um par de
cromossomos geneticamente equivalentes, presentes em uma clula
diploide, apresentando a mesma sequncia de lcus gnico.
Lcus Gnico Posio ocupada por um gene no cromossomo.
Homozigtico Indivduo em que os dois genes alelos so
idnticos.
Heterozigticos Indivduos em que os dois alelos de um gene
so
diferentes
entre
si.
Dominncia Propriedade de um alelo (dominante) de produzir o
mesmo fentipo tanto em condio homozigtica quanto
heterozigtica.
1 Lei. Se a planta possui o fator V mas tambm o v, ela ser
amarela. O verde s aparecer na ausncia do fator dominante,
como representado abaixo:

1) Resultam das modificaes produzidas pelo meio ambiente, que


no chegam a atingir os gametas, no sendo por isso transmissveis.
O texto acima refere-se aos caracteres:
a) hereditrios
d) adquiridos
b) dominantes
e) recessivos
c) genotpicos
2) (PUCC-SP) O caritipo est relacionado com:
a) nmero de cromossomos
b) forma dos cromossomos
c) tamanho dos cromossomos
d) todas alternativas anteriores
e) volume do cromossomo
3) (UNESP) Em organismos diploides sexuados, cada par de
cromossomos formado por um cromossomo de origem paterna e
um de origem materna. Qual a probabilidade de os espermatozoides
conterem apenas os cromossomos de origem materna em um
organismo com quatro pares de cromossomos?
a)
b)
c) 1/8
d) 1/16
e) 1/32

4) (UFSCAR) Relativamente primeira lei de Mendel, s no


correto dizer que:
a) o gameta recebe apenas um dos genes de cada carter, sendo,
portanto, sempre puro.
b) entre outros nomes, esta lei recebe o nome de lei da pureza dos
gametas
c) os genes alelos so segregados e passam aos gametas em
propores fixas e definidas
d) cada gameta puro, isto , s contm um alelo de cada par de
genes
e) cada carter determinado por um par de genes, que se une na
formao dos gametas.
5)Cruzando-se ervilhas verdes vv com ervilhas amarelas Vv, os
descendentes sero:
a)
100% vv, verdes;
b)
100% VV, amarelas;
c)
50% Vv, amarelas; 50% vv, verdes;
d)
25% Vv, amarelas; 50% vv, verdes; 25% VV, amarelas;
e)
25% vv, verdes; 50% Vv, amarelas; 25% VV, verdes.

Ento podemos definir a 1 Lei de Mendel: Cada caracterstica


determinada por dois fatores que se separam na formao dos
gametas, onde ocorrem em dose simples, isto , para cada gameta
encaminhado apenas um fator.
E a descendncia esperada de 3 para 1 no fentipo e 1:2:1 no
gentipo o que caracteriza uma dominncia completa.

SECRETARIA DE ESTADO DA EDUCAO

39

6)Um gato da cor marrom foi cruzado com duas fmeas. A primeira
fmea era da cor preta, e teve 7 filhotes da cor preta e 6 filhotes da
cor marrom. J a outra fmea, tambm era da cor preta, e teve 14
filhotes, sendo todos eles da cor preta. A partir desses cruzamentos
marque a opo que contm os gentipos do macho, da primeira e
da segunda fmea respectivamente.
a)
Aa, aa, aa.
d)
aa, Aa, AA.
b)
AA, aa, aa.
e)
Aa, AA, Aa.
c)
aa, AA, aa.

QUMICA
BIOLOGIA

Aula 16 : Gentica 2- Grupos Sanguneos

EXERCICIOS DE FIXAO

Cada pessoa pertence a um desses grupos sanguneos. Nas


hemcias humanas podem existir dois tipos de protenas:
o aglutinognio A e o aglutinognio B. De acordo com a presena ou
no dessas hemcias, o sangue assim classificado:
Grupo A possui somente o aglutinognio A; Grupo B possui
somente o aglutinognio B;

Grupo AB possui somente o aglutinognio A e B;


Grupo O no possui aglutinognios.
No plasma sanguneo humano podem existir duas protenas,
chamadas aglutininas: aglutinina anti A e aglutinina anti-B.
Grupo
sanguneo

Aglutinognio

Aglutinina

anti-B

2) (FUVEST) Um banco de sangue possui 5 litros de sangue tipo


AB, 3 litros tipo A, 8 litros tipo B e 2 litros tipo O. Para transfuses em
indivduos dos tipos O, A, B e AB esto disponveis, respectivamente:
a) 2, 5, 10 e 18 litro.
d) 18, 8, 13 e 5 litros.
b) 2, 3, 5 e 8 litro.
e) 7, 5, 10 e 11 litros.
c) 2, 3, 8 e 16 litros.
3)(FUND. CARLOS CHAGAS) Qual o nmero de autossomos
existentes em um vulo de um animal que tem 14 pares de
cromossomos?
a) 28
b) 26
c) 14
d) 13
e) 1

anti-A

AB

AeB

No possui

No possui

anti-A e anti-B

1) (FUVEST) Um homem do grupo sanguneo AB casado com uma


mulher cujos avs paternos e maternos pertencem ao grupo
sanguneo O. Esse casal poder ter apenas descendentes:
a) do grupo O;
b) do grupo AB;
c) dos grupos AB e O;
d) dos grupos A e B;
e) dos grupos A, B e AB.

Quadro de doaes sanguineas

4) (PUC) O sexo feminino, quanto aos cromossomos sexuais,


chamado:
a) homogamtico
b) heterogamtico
c) diploide
d) haploide
e) genoma
Determinao
sexo

gentica

do
5) O corpsculo de Barr, tambm chamado de cromatina sexual,
encontrado:
a) nas clulas sexuais do homem;
b) nas clulas sexuais da mulher;
c) ligado ao nuclolo nas clulas somticas da mulher;
d) junto membrana nuclear nas clulas somticas da mulher.
e) junto membrana plasmtica nas clulas somticas da mulher.
6)(FEI-SP) Qual a seqncia mais provvel dos genes A, B, C, D,
localizados no mesmo cromossomo, apresentando as seguintes
freqncias de recombinao:
AB - 17% CD - 30% AC - 5% AD - 35% BD - 18%
a) A - B - C - D
b) A - C - B - D
c) A - B - D -C
d) C - A - B - D
e) C - A - D B

Mapa gentico, uma representao grfica das distncias entre


genes e de suas posies relativas em um cromossomo. A distancia
entre os genes dado pela soma dos recombinantes.
Exemplo :Porcentagem de recombinao entre genes A e B: 19%
Porcentagem
de
recombinao
entre
A
e
C:
2%
Porcentagem de recombinao entre B e C: 17%

SECRETARIA DE ESTADO DA EDUCAO

40

7) Os gens a e b encontram-se num mesmo cromossoma, sendo a


distncia entre eles de 17 unidades. A frequncia de
gametas AB formados por um indivduo AB/ab de:
a) 8,5%
b) 17%
c) 34%
d) 41,5%
e) 83%

QUMICA
BIOLOGIA

Aula 17 : Ecologia

EXERCICIOS DE FIXAO

Introduo.
o ramo da biologia que ........................as relaes entre os seres
vivos e em relao ao ...................................so distribudos em

1)Assinale a alternativa CORRETA:


a) Em Ecologia, a COMUNIDADE inclui grupos de indivduos de
uma mesma espcie de organismos.
b) Em Ecologia, a POPULAO inclui todos os indivduos de uma
mesma rea, pertencentes ou no a vrias espcies.
c) Em Ecologia, o ECOSSISTEMA a poro da terra
biologicamente habitada.
d) Em Ecologia, a BIOSFERA o conjunto formado pela comunidade
de indivduos vivos e o meio ambiente inerente.
e) Nenhuma das anteriores.
2)(CESCEM) So ecossistemas
EXCETO:
a) uma astronave.
b) uma lagoa.
c) um pasto.
d) uma colnia de corais.
e) o solo.

nveis.

todos

os

exemplos

abaixo,

Estudos dos ecossistemas

3)Com referncia cadeia alimentar, marque a alternativa correta:


a) a energia contida no produtor diminui gradualmente, ao passar de
consumidor a consumidor.
b) a energia do produtor aumenta gradualmente nos diferentes
elementos da cadeia alimentar.
c) o potencial energtico do produtor no sofre nenhuma alterao
nos diferentes segmentos da cadeia alimentar.
d) na cadeia alimentar, como os animais so de tamanho diferentes ,
a quantidade energtica do produtor sofre oscilaes.
e) a complexidade da teia alimentar no interfere com a quantidade
de energia transferida, que permanece inalterada.

Cadeia alimentar. Fluxo de matria e.........que passa dos


produtores, at os ...................diretamente pelos
.............................

Teia alimentar. O fluxo de matria e energia

segue

por.........................opcionais ou seja so.................cadeias

Sucesso ecolgica.
Processo lento e .............que ocorre nos ecossistemas saindo de
uma comunidade inicial(..................), as intermedirias(................) e
a final( ...............)

4) (CESGRANRIO) Qual das alternativas a seguir responde


corretamente a seguinte questo: Que resultaria se desaparecesse
do mar o fitoplncton ?
a) o equilbrio ecolgico desse ecossistema no sofreria alterao,
visto que o fitoplncton constitudo por seres apenas microscpios.
b) o zooplncton ocuparia o seu lugar na cadeia alimentar, mantendo
assim o equilbrio ecolgico do ecossistema.
c) a cadeia alimentar do ecossistema perderia o elo principal, pois do
fitoplncton depende praticamente toda a matria orgnica
necessria aos demais componentes biticos.
d) o ecossistema no seria afetado visto que o plncton apenas um
consumidor na cadeia alimentar.
e) o equilbrio ecolgico no seria alterado visto que nem todos os
organismos marinhos se alimentam do fitoplncton.
5) Suponhamos que o charco onde viva a pre e os outros seres
vivos de nossa histria, com o decorrer dos tempos, v lentamente
se modificando. O espelho dgua diminua, pela progressiva invaso
de novas plantas, assoreamento e a instalao de plantas mais
duradouras; pelo surgimento de novos animais e desaparecimento
de outros. O cenrio se modifica. Esse fenmeno denomina-se:
a) comunidade clmax.
b) comunidade em equilbrio dinmico.
c) comunidade em equilbrio esttico.
d) sucesso ectona.
e) sucesso ecolgica.
6)(CESGRANRIO) Existiu, em uma determinada regio, uma lagoa
com gua lmpida contendo apenas plncton. O acmulo de matria
fornecida pelo plncton permitiu que se formasse um fundo capaz de
sustentar plantas imersas. Surgiram depois, alguns animais que
comearam a povoar a lagoa, e o fundo acumulou detritos e nele se
desenvolveram plantas emergentes. A lagoa ficou cada vez mais
rasa. Surgiram os juncos nas margens, que foram fechando a
lagoa. Ela se transformou num brejo, e a comunidade que o habitava
j era bem diferente. Posteriormente, o terreno secou e surgiu um
pasto que poder ser substitudo, no futuro, por uma floresta.
Este texto apresenta um exemplo de:
a) irradiao adaptativa
d) pioneirismo
b) disperso de espcies
e) domnio ecolgico
c) sucesso ecolgica

SECRETARIA DE ESTADO DA EDUCAO

41

QUMICA
BIOLOGIA

AULA 18 : Biomas Brasileiros

EXERCICIOS DE FIXAO
1) (FUVEST) A concentrao de gs na atmosfera vem aumentando
de modo significativo desde meados do sculo XIX; estima-se que se
quadruplicou no ano 2000. Qual dos fatores abaixo o principal
responsvel por esse aumento?
a) ampliao da rea de terras cultivadas;
b) utilizao crescente de combustveis fsseis;
c) crescimento demogrfico das populaes humanas;
d) maior extrao de alimentos do mar;
e) extino de muitas espcies de seres fotossintetizantes.

As regies fitogeogrficas do............se divide em biomas que


representam a ......................e os......................da nossa fauna.

2) Considere os seguintes itens:


I. agrotxicos que destroem ovos;
II. agrotxicos que matam larvas;
III. vrus que atacam larvas.
O controle biolgico de pragas pode ser feito atravs de, apenas:
a) I
b) II
c) III
d) I e III
e) II e III
3)O poluente atmosfrico que se liga permanentemente s
molculas de hemoglobina impossibilitando-as de transportar
oxignio s clulas o:
a) oznio
b) hidrocarbonato
c) dixido de enxofre
d) dixido de carbono
e) monxido de carbono
4)O processo que chamamos de ______________ consiste na
transformao de _________ em ___________. Esse processo
devolve o ___________ para a atmosfera e feito por bactrias
_____________.
a) Nitrificao, amnia, nitrato, nitrito, nitrificantes.
b) Desnitrificao, nitratos, gs nitrognio, nitrognio,
desnitrificantes.
c) Amonificao, nitrognio atmosfrico, amnia, nitrognio,
amonificantes.
d) Fixao, nitrognio, amnia, nitrato, fixadoras.
e) Fixao do nitrognio, nitrognio, amnia, nitrognio, fixadoras de
nitrognio.

Ciclos Biogeoqumicos
Ciclo do Nitrognio

5)Em relao ao ciclo hidrolgico nas grandes cidades, julgue as


afirmativas com V ou F.
I.( ) A devoluo de gua para a atmosfera atravs da transpirao
reduzida.
II.( ) O abastecimento do lenol fretico favorecido pela facilidade
de infiltrao de gua no solo.
...............................................................................................................
...............................................................................................................
..............................................................................................................
Ciclo da gua

III.( ) O volume de gua que escoa superficialmente aumenta em


funo das construes urbanas.IV.( ) A existncia de grande
quantidade de poos de bombeamento de gua no exerce
influncia sobre a vazo dos rios.
V.( ) O assoreamento dos canais fluviais contribui para o
agravamento das enchentes.
VI.( ) A concentrao dos poluentes e as altas temperaturas
favorecem a formao de ncleos de condensao, o que pode
provocar um aumento das precipitaes.

SECRETARIA DE ESTADO DA EDUCAO

42

QUMICA
BIOLOGIA

Aula 19: Fisiologia e problemas cotidianos

EXERCICIOS DE FIXAO

3)Assinale a alternativa que apresenta doenas causadas por


bactrias, fungos, vrus e protozorios, nessa ordem:
a)micose malria dengue gonorreia
b)raiva blastomicose chagas tuberculose
c)sfilis herpes micose doena do sono.
d)Micose gripe clera esquistossomose
e)Ttano sapinho bucal caxumba ulcera de Bauru

Relacione doena ao tipo de parasita


HPV......................................................................................................
Cisticercose........................................................................................
Malria.................................................................................................
Tuberculose.........................................................................................
Ebola....................................................................................................
Leishmaniose......................................................................................
Ascaridase..........................................................................................
Pediculose...........................................................................................
Rubola................................................................................................
Candidase..........................................................................................
Tenase.................................................................................................
Ancylostomose......................................................................................
Gonorria.............................................................................................
Imunizao
Vacinas
Usam...................estimulam a produo de ......................................,
imunizao........................................................e

duradoura.

4)(MACKENZIE-SP) A contaminao do reservatrio de gua de


Santa Isabel do Ivai (PR) provocou 375 casos de toxoplasmose. A
doena fugiu ao controle das autoridades sanitrias do municpio de
9154 habitantes, que teve 375 casos confirmados e 743 notificaes.
Essa doena pode causar aborto, deficincias neurolgicas e visuais.
Alguns gatos tinham um ninho na estao de tratamento de gua da
cidade.
Adaptado de Folha de So Paulo, 24/02/2002
A ___ I ___, na referida cidade, pode ter sido provocada por
contaminao do sistema desabastecimento de gua. Os gatos,
encontrados na estao, agem como hospedeiros ___ II ___ do
causador da doena, que ___ III ___.Os espaos I, II e III devem
ser preenchidos, correta e respectivamente por
a) epidemia, definitivos e um protozorio.
b) epidemia, definitivos e uma bactria.
c) endemia, intermedirios e um protozorio.
d) endemia, definitivos e um protozorio.
e) epidemia, intermedirios e uma bactria.

Soros
Utilizam.........................................para combater o ...........................,
imunizao.........................................e

passageira

SECRETARIA DE ESTADO DA EDUCAO

43

QUMICA
BIOLOGIA

AULA 20 : Fisiologia aplicada ao cotidiano.

EXERCCIOS DE FIXAO

Atividades fsicas, nutrio e doenas relacionadas


Utilizao de esterides- uso de ........................que estimulam o
trabalho......................e crescimento ................................................
Obesidade- excesso de peso correspondente a IMC superior a .........
Desnutrio- Falta de .........................que atuam na construo dos
tecidos e manunteno das atividades...........................................
Risco coronariano- so fatores que ...........................o indivduo ser
vtima de um......................agudo do miocrdio,so
eles:.....................,................................, uso de
...............................ilcitas, tabagismo e ............................................
Drogas
O consumo de substncias psicoativas um fenmeno civilizatrio,
ou seja, sempre existiu em todas as culturas humanas o uso de
substncias que alteram os estados de conscincia.

Descobrimentos arqueolgicos:

5000 AC vinho na regio do atual Ir;

4000 AC maconha (cnhamo) na China;

3000 AC folhas de coca na Amrica do Sul;

Drogas e Movimentos Culturais:

Surrealismo - Absinto incio Sc. XX

Movimento Hippie Maconha e LSD anos


1960
As caractersticas desse consumo modificaram-se significativamente
nas ltimas dcadas:
colocando em risco a vida de muitas pessoas,
sendo um dos fatores estressantes a espelhar o sistema
econmico contemporneo e seu ciclo da sociedade de
consumo;
As drogas lcitas e ilcitas so hodiernamente um dos principais
problemas de sade pblica mundial:
As drogas, principalmente as lcitas (lcool e tabaco),
carregam os maiores ndices de mortalidade, bem como os
de violncia urbana e no trnsito, bem como so as
primeiras responsveis pelo absentesmo ao trabalho e
seu custo social;
MEDICAMENTO = Toda droga com utilidade teraputica
cientificamente comprovada.
REMDIO = Tudo o que provoca alvio de um sinal e/ou sintoma.
DROGA DE ABUSO = Droga utilizada com finalidade intoxicante.
Geralmente utilizada de forma descontrolada, leva ao uso de risco ou
dependncia.
Depressoras do SNC: drogas que tm ao relaxante ou calmante.
Exemplos: lcool, ansiolticos
Estimulantes: drogas que estimulam as funes psquicas. Tm
ao tnica, revigorante, euforizante.
Exemplos: tabaco, anfetaminas, ecstasy, cocana,

SECRETARIA DE ESTADO DA EDUCAO

44

3. (UFOP-JUNHO/2008) Sobre o processo de reproduo humana,


assinale a alternativa correta:
a) Alguns mtodos contraceptivos hormonais, como a plula
anticoncepcional, so eficientes porque mantm os nveis
hormonais femininos constantes e com isso evitam a ovulao.
b) O DIU (dispositivo intrauterino) um mtodo contraceptivo que
impede a implantao do embrio, sendo muito eficiente na
preveno da gravidez uterina e tubria.
c) A formao de gmeos diferentes (dizigticos) s possvel se
houver a fecundao de um vulo por dois espermatozoides
diferentes, ou seja, com materiais genticos diferentes.
d) Um exame de farmcia feito com a urina de uma mulher indicou
a presena do hormnio prolactina, o que significa que esta mulher
est grvida.
4. (UFV/2008) Correlacione alguns dos mtodos de anticoncepo
com seus respectivos mecanismos de ao.
Mtodos de anticoncepo
(I) Diafragma
(III) Dispositivo intrauterino
(II) Laqueadura
(IV) Vasectomia
Mecanismos de ao
( ) impede a liberao dos gametas da gnada para a uretra.
( ) impede a nidao no endomtrio.
( ) impede que os gametas cheguem ao tero distal das tubas.
( ) impede a passagem dos gametas da vagina para o tero.
Assinale a alternativa que apresenta a sequncia CORRETA:
a) I, II, III, IV.
d) III, II, IV, I.
b) IV, III, II, I.
e) III, I , IV , II
c) II, III, I, IV.

QUMICA
BIOLOGIA

AULA 01 CINEMTICA ESCALAR

EXERCCIOS

MOVIMENTO UNIFORME (MU):

1. Joo Maria levou seu carro a um lava rpido que utiliza


uma mquina de escovas rotativas. Os rolos giram e se
deslocam sobre o carro e o motorista permanece no interior
do veculo. Num determinado momento, Joo Maria teve a
impresso de o carro ter-se deslocado. Ao olhar para uma
placa, fixada na entrada do prdio do lava rpido, observou
que em relao a ela o carro no se movimentou. Concluiu,
ento, que

Funes:
-

MOVIMENTO UNIFORMEMENTE VARIADO(MUV):


Funes:
-

MOVIMENTO VERTICAL NO VCUO:

a) o carro deslizou devido existncia do xampu utilizado na


lavagem.
b) em relao placa, o carro realizou um movimento
retilneo e uniforme.
c) em relao aos rolos, o carro est em movimento.
d) a sensao de movimento se deve gua jogada sobre o
carro.
e) os conceitos de movimento e repouso independem do
referencial adotado.

2. Uma das leis sobre segurana no trnsito, principalmente para os

caminhes que transitam carregados com pedriscos, obriga que a


carga seja coberta com lona, para evitar a queda de pedras soltas
pela traseira, colocando em risco veculos que transitam atrs do
caminho. Considere que um caminho, carregado com essas
pedras e sem a cobertura de lona, est transitando em uma pista
plana e horizontal e que, num certo instante, cai uma pedra da
traseira do caminho de uma altura h com relao ao solo. Considere
tambm que um observador em repouso, ao lado da pista, v o
caminho movimentando-se da direita para a esquerda no momento
da queda da pedra. Assinale corretamente qual dos esboos abaixo
melhor representa a trajetria da pedra vista pelo observador.
Despreze efeitos de resistncia do ar.

Funes:
Grficos da Cinemtica

3. Dois carros A e B fazem uma viagem entre duas cidades X


e Y. O carro A percorre a primeira metade do percurso com
velocidade constante de 70 km/h e a metade restante com
velocidade constante de 30 km/h. J o carro B faz a primeira
metade da viagem a 60 km/h e a metade restante a 40 km/h.
Nessas condies, podemos garantir que
a) Os carros A e B fazem a viagem no mesmo intervalo de
tempo.
b) Na viagem, a velocidade escalar mdia de A e maior do
que a de B.
c) O carro B tem velocidade escalar mdia duas vezes maior
do que o carro A.
d) O carro A completa a viagem em um tempo maior do que o
carro B.
e) O carro B completa a viagem em um tempo maior do que o
carro A.

SECRETARIA DE ESTADO DA EDUCAO

45

QUMICA
FSICA

AULA 02 CINEMTICA VETORIAL

EXERCCIOS

Lanamento Horizontal:

1. Um naturalista, na selva tropical, deseja capturar um


macaco de uma espcie em extino, dispondo de uma arma
carregada com um dardo tranquilizante. No momento em que
ambos esto a 45 m acima do solo, cada um em uma rvore,
o naturalista dispara o dardo. O macaco, astuto, na tentativa
de escapar do tiro se solta da rvore. Se a distncia entre as
rvores de 60m, a velocidade mnima do dardo, para que o
macaco seja atingido no instante em que chega ao solo, vale
2
em m/s: Adote g = 10 m/s .
a) 45
b) 60
c) 10
d) 20
e) 30

Funes
2. Em uma partida de futebol, o goleiro bate um tiro de meta
com a bola no nvel do gramado. Tal chute d bola uma
velocidade inicial de mdulo 20m/s e um ngulo de
lanamento de 45. Nessas condies, a distncia mnima
que um jogador deve estar do ponto de lanamento da bola,
para receb-la no seu primeiro contato com o solo, :

a) 30m
b) 40m
c) 20m
d) 10m
e) 5m

Lanamento Oblquo:

(Enunciado para as questes 3 e 4 )

Numa partida de futebol, um jogador nos presenteou com um


lindo gol, no qual, ao correr para receber um lanamento de
um dos atacantes, o goleador parou a bola no peito do p e a
chutou certeira ao gol. Analisando a jogada pela TV, verificase que a bola chutada pelo armador da jogada a partir do
cho com uma velocidade inicial de 20,0 m/s, fazendo um
ngulo com a horizontal de 45 para cima.

Dados: g = 10,0 m/s e

Funes

2 = 1,4

3. A distncia horizontal percorrida pela bola entre o seu


lanamento at a posio de recebimento pelo artilheiro foi
de

a) 40 m
b) 20 m
c) 10 m
d) 50 m
e) 30 m

SECRETARIA DE ESTADO DA EDUCAO

46

QUMICA
FSICA

AULA 03 - DINMICA

EXERCCIOS DE FIXAO
. O Cdigo de Transito Brasileiro (Lei n.. 9.503/97) deter
mina:
Art. 64. As crianas com idade inferior a dez anos devem ser
transportadas nos bancos traseiros, salvo excees
regulamentadas pelo CONTRAN (Conselho Nacional de
Trnsito).
Art. 65. obrigatrio o uso do cinto de segurana para
condutor e passageiros em todas as vias do territrio
nacional, salvo em situaes regulamentadas pelo
CONTRAN.
As orientaes desses dois artigos da lei visam minimizar os
efeitos de um acidente, pois, em caso de uma brecada
brusca ou coliso frontal de um automvel,
a) o cinto de segurana reage contra o impulso dado pelo
carro aos passageiros.
b) as crianas sentadas no banco traseiro automaticamente
passam a ter velocidade.
c) os passageiros tendem a continuar o movimento que
estavam realizando, por inrcia.
d) os passageiros ganham um impulso, transmitido pelo
carro, para fora do veiculo.
e) as crianas sentadas no banco traseiro, por serem mais
leves, no ganham impulso.

Fora:

1 Lei de Newton:

2 Lei de Newton:

3 Lei de Newton:

2. As regras de trnsito tm por objetivo garantir o mximo de


segurana para os motoristas; por exemplo, o limite de
velocidade em uma curva deve garantir que os carros no
deslizem. Sabendo-se que o coeficiente de atrito esttico
entre os pneus e o asfalto 0,8 e que a acelerao da
2
gravidade tem mdulo g = 10 m/s , pode-se afirmar que a
velocidade escalar mxima de um carro que faz uma curva
horizontal com raio R = 12,5 m, sem deslizar, de:

Peso de um corpo:

Reao Normal do apoio:

Fora de Atrito:

a) 10 km/h

Fora Elstica:

b) 24 km/h c) 36 km/h

d) 64 km/h e) 72 km/h

3. Em uma partida de Curling, uma jogadora arremessa uma


pedra circular de 18 kg (ver figura abaixo), que desliza sobre
o gelo e para a 30 m da arremessadora. Sabendo que o
coeficiente de atrito cintico entre a pedra e o gelo de
0,015, correto afirmar que a pedra foi lanada com
velocidade de:
Fora Centrpeta:

a) 2 m/s
b) 3 m/s
c) 4 m/s
d) 5 m/s
e) 6 m/s

SECRETARIA DE ESTADO DA EDUCAO

47

QUMICA

FSICA

AULA 04 TRABALHO E ENERGIA

EXERCCIOS DE FIXAO
1. Para a coleta de entulho de construo, tornou-se comum
o uso de caambas. Suponha que uma dessas caambas
cheia de entulho tenha massa total de 2,0 toneladas. Atrelada
ao brao do guindaste, este necessita de 40 s para
posicionar a caamba sobre o caminho, a 80 cm do solo.
Admitindo-se que a acelerao da gravidade tem mdulo
2
igual a 10 m/s , Qual a potncia, em W, necessria para que
o guindaste leve a caamba do solo para sua posio sobre o
caminho?

ENERGIA CINTICA

ENERGIA POTENCIAL GRAVITACIONAL

ENERGIA POTENCIAL ELSTICA

ENERGIA MECNICA

a) 400 W

b) 500 W

c) 200 W d) 600 W e) 800 W

2. Durante um treinamento de salvamento, um bombeiro de


massa 80 kg cai verticalmente, sem velocidade inicial, do alto
de um prdio de 5,0 m de altura sobre uma plataforma
circular rgida, de peso e espessura desprezveis, que est
ligada ao solo por uma mola de constante elstica 9,6 x 103
N/m, verticalmente postada, conforme se observa na figura
abaixo. O bombeiro cai no centro da plataforma e a mola
sofre compresso mxima de 0,9cm, ocorrendo aquecimento
da superfcie pelo impacto. Desprezando-se os efeitos da
resistncia do ar durante o salto, pode-se estimar que o valor
da energia mecnica dissipada nesse aquecimento foi de

Trabalho de uma fora constante:

Trabalho do Peso :

Trabalho da fora de Atrito:

Trabalho da Fora elstica:

TEOEMAS TRABALHO ENERGIA

a) 112 J

c) 420 J d) 832 J e) 1080 J

3. Em um incndio, os bombeiros posicionam uma cama


elstica para receber uma pessoa de massa 80 kg que cair
verticalmente, a partir do repouso, de uma altura H = 20m
acima da cama elstica. A acelerao da gravidade tem
2
mdulo g = 10m/s e admite-se que no haja dissipao de
energia mecnica. A pessoa atinge a cama com uma
velocidade de mdulo V e, at a sua velocidade se anular, a
cama sofre uma deformao x = 1,0m. Admita que a pessoa
caiu bem no centro da cama elstica, que se comporta como
uma mola elstica ideal.
Pretende-se calcular com que velocidade escalar V a pessoa
atinge a cama elstica e qual a constante elstica k
associada cama elstica. Os valores encontrados so mais
prximos de:
4
a) V = 20m/s e k = 3,2 . 10 N/m
4
b) V = 20m/s e k = 3,4 . 10 N/m
4
c) V = 10m/s e k = 3,2 . 10 N/m
4
d) V = 10m/s e k = 3,4 . 10 N/m
4
e) V = 8,0m/s e k = 3,0 . 10 N/m

POTNCIA MDIA

POTNCIA INSTATNEA

SECRETARIA DE ESTADO DA EDUCAO

b) 250 J

48

QUMICA
FSICA

AULA 05 DINMICA IMPULSIVA

EXERCCIOS DE FIXAO

1. Um ginasta de 60 kg de massa, exercitando-se sobre uma


cama elstica, deseja saltar cada vez mais alto. Sabe-se que,
aps atingir a altura de 0,8 m acima do nvel da cama, o
ginasta cai sobre a mesma e sobe at a altura de 1,25 m.
Nesse contexto, correto afirmar que, para esse ltimo salto,
o mdulo do impulso transmitido pela cama elstica ao atleta
foi de:

IMPULSO DE UMA FORA CONSTANTE:

MOMENTO LINEAR OU QUANTIDADE DE MOVIMENTO:

a) 60 kg m/s
b) 100 kg m/s
c) 150 kg m/s
d) 270 kg m/s
e) 540 kg m/s
TEOREMA DO IMPULSO:

PRINCPIO DA
MOVIMENTO.

CONSERVAO

2. Um disco de 0,03 kg de massa move-se sobre um colcho


de ar com velocidade de 4 m/s na direo i. Um jogador, com
auxlio de um taco, bate o disco imprimindo-lhe um impulso
de 0,09 kg m/s na direo j.
Desta forma, correto dizer que o mdulo da velocidade final
do disco ser:

DA

QUANTIDADE

DE

a) 1 m/s
b) 2 m/s
c) 3 m/s
d) 5 m/s
e) 7 m/s

3.
Dois
motoristas
imprudentes
passam direto
em um cruzamento, provocando uma coliso. Um dos
motoristas dirigia uma caminhonete de 1200 kg de massa a
60 km/h na direo - j (sentido negativo), enquanto o outro
dirigia um carro pequeno de 800 kg de massa a 120 km/h na
direo i (sentido positivo). Sabendo que, logo aps a
coliso, os carros passam a se mover conjuntamente,
correto afirmar que a velocidade dos carros, em km/h, passou
a ser de:

COLISO INELSTICA
-

a) 120 i - 60 j
b) 60 i + 120 j
c) 48 i - 36 j
d) 36 i + 48 j
e) 120 i 36 j

COLISO ELSTICA

4. Um jogador chuta uma bola com massa 450 g a qual est


sobre uma superfcie horizontal com atrito desprezvel. A bola
choca-se contra um objeto de 0,9 kg, inicialmente em
repouso, apoiado sobre a superfcie. Aps o choque, o objeto
passa a se mover com velocidade 10 m/s, e a bola retorna
com uma velocidade de 4 m/s.
Admitindo que o choque frontal, correto afirmar que o
impulso transmitido pelo jogador bola, devido ao chute,
de:

COLISO PARCIAMENTE ELSTI A

a) 16,0 kg m/s
b) 10,8 kg m/s
c) 9,0 kg m/s
d) 7,2 kg m/s
e) 3,6 kg m/s

SECRETARIA DE ESTADO DA EDUCAO

49

QUMICA
FSICA

AULA 06 GRAVITAO UNIVERSAL

LEI DA RAVITAO UNIVERSAL (Newton)

LEIS DE KEPLER
1 LEI (Lei das rbitas) -

CAMPO GRAVITACIONAL

2 LEI (Lei das reas) -

CORPOS EM RBITA

FG Fc

3 LEI (Lei dos perodos) -

VELOCIDADE DE ESCAPE

r
o
nh
mi a
Ca fug
de

rbita circular
rbita elptica

SECRETARIA DE ESTADO DA EDUCAO

50

QUMICA
FSICA

EXERCCIOS DE FIXAO
1. Os satlites artificiais so uma conquista da tecnologia moderna e
os seus propsitos so variados. Existem satlites com fins militares,
de comunicao, de monitoramento etc. e todo satlite tem uma
rbita e uma velocidade orbital bem determinadas. Nesse contexto,
considere um satlite de comunicao que descreve uma rbita
circular em torno da Terra com um perodo de revoluo de 8x10 4 s.
Com base nessas informaes e desprezando o movimento da
Terra, correto afirmar que esse satlite gira em torno da Terra com
uma velocidade orbital de:
a) 1.000 m/s
d) 3.000 m/s
b) 1.500 m/s
e) 3.500 m/s
c) 2.000 m/s
2. )Em um sistema planetrio distante, os planetas X e Y descrevem
rbitas circulares em torno de uma estrela. Sabendo que o raio da
rbita do planeta X quatro vezes maior que o da rbita do planeta Y
e que o mdulo da velocidade do planeta X de 12 km/s, correto
afirmar que o mdulo da velocidade, em km/s, do planeta Y de:
a) 3
b) 6
c) 12
d) 24
e) 48

AULA 07 ESTTICA DOS SLIDOS


EQUILBRIO DE UM PONTO MATERIAL

MOMENTO DE UMA FORA OU TORQUE

3. Um foguete de 5 toneladas de massa move-se da Lua para a


Terra. Em um determinado momento da viagem, observa-se que o
foguete encontra-se ao longo da linha reta que une os centros da
Terra e da Lua. Nesse contexto, considere as seguintes informaes:
- o zero da energia potencial gravitacional tomado em um ponto
muito distante da Terra e da Lua, isto , no infinito;
- a massa da Terra cerca de 80 vezes maior que a massa da Lua;
- energia potencial gravitacional entre o foguete e a Terra
denotada por UT e entre o foguete e a Lua, por UL;
-a distncia entre o foguete e o centro da Terra denotada por dT e
entre o foguete e o centro da Lua, por dL;
-O mdulo da fora entre o foguete e a Terra denotado por FT e
entre o foguete e a Lua, por FL.
Diante do exposto, identifique as afirmativas corretas:
I. Se UL= UT ento dL = dT
II. Se o foguete for deixado em repouso no ponto onde UL= UT ento
ele permanecer em repouso.
III. Se FL= FT ento dL< dT.
IV. Se FL= FT, ento UL= UT
V. O ponto onde UL= UT no depende da massa do foguete.
4.Certa emissora de televiso que opera em canal fechado utiliza,
em suas transmisses, satlites de comunicao do tipo
geoestacionrio, como o mostrado na figura. Satlites desse tipo
orbitam no plano equatorial terrestre e tm perodo igual ao de
rotao da Terra. Sabendo-se que a Lua gasta aproximadamente 27
dias para dar uma volta completa em torno da Terra e que o raio da
sua rbita R, a expresso que representa o raio da rbita de um
satlite geoestacionrio

EQUILBRIO DE UM CORPO EXTENSO

TIPOS DE EQUILBRIO

a) R/27
b) R/9
c) R/3
d) R/(27)1/2

e) R/(3)

ESTVEL

1/2

SECRETARIA DE ESTADO DA EDUCAO

51

INSTVEL

INDIFERENTE

QUMICA
FSICA

EXERCCIOS DE FIXAO
1. Quatro foras, de mesmas intensidades, so aplicadas em
pontos diferentes (A, B, C, D) de uma barra homognea,
presa superfcie de uma mesa por um pino localizado no
ponto B, conforme a figura ao lado.
Considerando que as distncias AB = BC = CD, acerca dos
torques (T) relativos ao ponto B, causados pelas foras
aplicadas nos diferentes pontos, correto afirmar:

AULA 08 HIDROSTTICA
DENSIDADE ABSOLUTA OU MASSA ESPECFICA

PRESSO EXERCIDA POR UMA FORA NORMAL

PRESSO HIDROSTTICA

a) T > T > T > T


D

b) T > T > T > T


C

PARADOXO HIDROSTTICO

c) T > T > T > T


B

d) T > T > T > T


A

e) T > T > T > T


D

2. Uma balana constituda por um contrapeso de 4 kg que


pode ser movimentado sobre uma barra de 1 m de
comprimento e massa desprezvel. A extremidade esquerda
da barra pode girar livremente em torno de um piv fixo. Uma
corda de massa desprezvel amarrada a outra extremidade
da barra, passando por uma polia que pode girar sem atrito,
sustenta um bloco cuja massa se deseja medir, conforme
figura abaixo.Sabendo que o sistema encontra-se em
equilbrio com a barra na horizontal e que o contrapeso est
a 0,6 m da extremidade direita da barra, correto afirmar que
a massa do bloco de:

PRESSO ATMOSFRICA
Vcuo

TEOREMA DE STEVIN

a) 1,0 kg
b) 1,4 kg
c) 1,6 kg
d) 2,0 kg
e) 2,4 kg

p . g . h

3. Durante uma partida de sinuca, aps vrias bolas serem


encaapadas, sobram trs bolas de mesma massa sobre a
mesa. A mesa tem 2,0 metros de comprimento por 1,2
metros de largura e os centros das bolas esto localizados
nas posies representadas na figura ao lado. A partir dessas
informaes, correto afirmar que a distncia, em metros, do
centro de gravidade das bolas ao ponto O, no vrtice inferior
esquerdo indicado na figura, de:

a) 1,4
b) 1,2
c) 1,0
d) 0,8
e) 0,6

SECRETARIA DE ESTADO DA EDUCAO

VASOS COMUNICANTES

PRINCPIO DE PASCAL

52

QUMICA
FSICA

PRENSA HIDRULICA

2. Um carro de peso P1 est sobre uma plataforma que se


comunica por meio de um lquido incompressvel a uma
balana de mola. Antes da presena do carro na plataforma,
a balana indicava um peso P0. A rea da plataforma vale A1
e a rea de contato do lquido com a balana vale A2.

p1 p 2
F1
F
2
A1 A 2
F1
R12

F1
R12

F2
R 22
F2
R 22

PRINCPIO DE ARQUIMEDES

A indicao da balana, P2, com o carro presente, dada


por:

EXERCCIOS DE FIXAO
1. A figura abaixo representa um mecanismo hidrulico ideal
e isolado usado em elevadores hidrulicos. Uma fora
constante F1 foi aplicada sobre o mbolo esquerdo at que
ele descesse h1. Como consequncia, o mbolo direito subiu
h2, exercendo uma fora F2 para cima. O trabalho realizado
por F1 foi W 1 e por F2, W 2. As sees retas dos mbolos
esquerdo e direito tm reas A1 e A2, respectivamente, com
A1 < A2.

a) P2 = P0
b) P2 = P0 + P1
c) P2 = P0 + P1A2 / A1
d) P2 = P0 + P1A1 / A2
e) P2 = P0 P1A2 / A1
3. Um estudante realizou a seguinte experincia: colocou no
prato de uma balana de ponteiro uma vasilha contendo gua
e verificou que a balana marcou 1,5kg; em seguida,
3
mergulhou sua mo, de volume igual a 500cm , na gua
contida na vasilha (figura a seguir).

F1

Desta experincia, o estudante verificou que

Considerando-se essas informaes e com base no Princpio


de Pascal, assinale a opo correta.

a) a balana continuou marcando 1,5kg, pois ele no tocou


com a mo o fundo da vasilha.
b) a balana passou a marcar 1,0kg por causa do empuxo
provocado pelo deslocamento de gua produzido pela mo.
c) a balana passou a marcar 2,0kg por causa do empuxo
provocado pelo deslocamento de gua produzido pela mo.
d) a balana continuou marcando 1,5kg, pois o deslocamento
da gua compensado pela mo que passa a ocupar seu
lugar.
e) a balana passou a marcar 2,0kg porque, sendo a massa
igual a (densidade x volume), a gua aumentou sua massa
ao ter seu volume aumentado.

a) W 1 > W 2; h1 < h2; F1 < F2.


b) W 1 < W 2; h1 > h2; F1 > F2.
c) W 1 = W 2; h1 < h2; F1 > F2.
d) W 1 = W 2; h1 > h2; F1 < F2.
e) W 1 = W 2; h1 = h2; F1 = F2.

SECRETARIA DE ESTADO DA EDUCAO

53

QUMICA
FSICA

AULA 09 TERMOMETRIA E DILATAO

EXERCCIOS DE FIXAO
1. Durante uma temporada de frias na casa de praia, em
certa noite, o filho caula comea a apresentar um quadro
febril preocupante. A me, para saber, com exatido, a
temperatura dele, usa um velho termmetro de mercrio, que
no mais apresenta com nitidez os nmeros referentes
escala de temperatura em graus Celsius. Para resolver esse
problema e aferir com preciso a temperatura do filho, a me
decide graduar novamente a escala do termmetro usando
como pontos fixos as temperaturas do gelo e do vapor da
gua. Os valores que ela obtm so: 5 cm para o gelo e 25
cm para o vapor. Com essas aferies em mos, a me
coloca o termmetro no filho e observa que a coluna de
mercrio para de crescer quando atinge a marca de 13 cm.
Com base nesse dado, a me conclui que a temperatura do
filho de:
a) 40,0 C
b) 39,5 C
c) 39,0 C
d) 38,5 C
e) 38,0 C

TEMPERATURA
ENERGIA TRMICA
EQUILBRIO TRMICO
CALOR

ESCALAS TERMOMTRICAS

2. Em uma conferncia pela internet, um meteorologista


brasileiro conversa com trs outros colegas em diferentes
locais do planeta. Na conversa, cada um relata a temperatura
em seus respectivos locais. Dessa forma, o brasileiro fica
sabendo que, naquele momento, a temperatura em Nova
o
o
Iorque TNI=33,8 F, em Londres, TL=269 K, e em Sidnei,
o
TS=27 C. Comparando essas temperaturas, verifica-se:
a) TNI >TS >TL
b) TNI >TL >TS
c) T L>TS >TNI
d) TS >TN I>TL
e) TS >TL >TNI

DILATAO TRMICA DOS SLIDOS

3. Os materiais utilizados na construo civil so escolhidos


por sua resistncia a tenses, durabilidade e propriedades
trmicas como a dilatao, entre outras. Rebites de metal
(pinos de formato cilndrico), de coeficiente de dilatao linear
6
1
9,810 oC , devem ser colocados em furos circulares de
uma chapa de outro metal, de coeficiente de dilatao linear
5 o 1
2,010
C . Considere que, temperatura ambiente
o
2
(27 C), a rea transversal de cada rebite 1,00cm e a de
2
cada furo, 0,99cm . A colocao dos rebites, na chapa
metlica, somente ser possvel se ambos forem aquecidos
at, no mnimo, a temperatura comum de:
o
a) 327 C
o
b) 427 C
o
c) 527 C
o
d) 627 C
o
e) 727 C

DILATAO TRMICA DOS LQUIDOS

dilatao
aparente

t
t
Oo
O

t2

4. Cada uma das barras de ao (chamadas trilhos), que


compe uma estrada de ferro, tem comprimento de 20m
o
quando a temperatura de 30 C. Esses trilhos so fixados
sobre dormentes, mantendo-se um espaamento de 10 mm
entre trilhos consecutivos. Sendo o coeficiente de dilatao
-5
-1
oC , qual a maior temperatura a
que eles podem ser submetidos sem risco de deformao?
o
a) 50 C
o
b) 60 C
o
c) 70 C
o
d) 80 C
o
e) 90 C

COMPORTAMENTO ANMALO DA GUA

SECRETARIA DE ESTADO DA EDUCAO

54

QUMICA
FSICA

AULA 10 CALORIMETRIA

CONDUO TRMICA
LEI DA CONDUO TRMICA - FOURIER

CALOR SENSVEL

CALOR LATENTE

MUDANAS DE FASE
CONVECO TRMICA
IRRADIAO TRMICA

Calor especfico sensvel da H2O:


cs = 0,5 cal/g oC ; cL = 1,0 cal/g oC ; cG = 0,4 cal/g oC

EXERCCIOS
1. Em uma fbrica, utiliza-se uma barra de alumnio de 80 cm 2 de
seo reta e 20 cm de comprimento, para manter constante a
temperatura de uma mquina em operao. Uma das extremidades
da barra colocada em contato com a mquina que opera
temperatura constante de 400 oC, enquanto a outra extremidade est
em contato com uma barra de gelo na sua temperatura de fuso.
Sabendo que o calor latente de fuso do gelo de 80 cal/g, que o
coeficiente de condutibilidade trmica do alumnio de 0,5
cal/s.cm.oC e desprezando as trocas de calor do sistema mquinagelo com o meio ambiente, correto afirmar que o tempo necessrio
para derreter 500 g de gelo
a) 10 s
b) 20 s
c) 30 s
d) 40 s
e) 50 s

Calor especfico Latente da H2O:


LFS = 80 cal/g ; LVC = 540 cal/g

2. Uma maneira bastante prtica e rpida de aquecer gua atravs


de um aquecedor eltrico de nome popular mergulho. Uma dona
de casa costuma usar um mergulho que fornece 25 kcal de energia
por minuto, para aquecer gua. Desprezando o calor absorvido pelo
recipiente que contm a gua e o calor perdido para a atmosfera,
identifique as afirmativas corretas:
I. O mergulho gasta 3 minutos para elevar, de 25 C at 100 C, a
temperatura de um litro de gua.
II. O mergulho gasta 3 minutos para elevar, de 25 C at 50 C, a
temperatura de trs litros de gua.
III. O mergulho gasta 6 minutos para elevar, de 25 C at 100 C, a
temperatura de um litro de uma determinada substncia lquida, cujo
calor especfico igual metade do calor especfico da gua, porm
de igual densidade.
IV. O mergulho gasta meio minuto para elevar, de 20 C at 45 C,
a temperatura de um litro de gua.
V. O mergulho leva um minuto para elevar em 50 C a temperatura
de uma determinada substncia de capacidade trmica 5x10-1
kcal/C.

PRINCPIO DAS TROCAS DE CALOR


DIAGRAMA DE FASE (H2O)

3. Um engenheiro testa materiais para serem usados na fabricao


da carroceria de um automvel. Entre outras propriedades,
desejvel a utilizao de materiais com alto calor especfico. Ele
verifica que, para aumentar em 3 C a temperatura de 32g do
material A, necessrio fornecer 24cal de calor a esse material. Para
obter o mesmo aumento de temperatura em 40g do material B,
preciso 24cal. J 50g do material C necessitam 15cal para sofrer o
mesmo acrscimo de temperatura. Os calores especficos dos
materiais A, B e C so respectivamente:
a) cA=0 ,25cal/g C ; cB=0 ,20cal/g C ; cC=0 ,10cal/g C
b) cA=0 ,20cal/g C ; cB=0 ,35cal/g C ; cC=0 ,15cal/g C
c) cA=0 ,30cal/gC ; c B=0 ,10cal/g C ;cC=0 ,20cal/g C
d) cA=0 ,35cal/g C ; cB=0 ,20cal/g C ; cC=0 ,10cal/g C
e) cA=0 ,10cal/g C ; cB=0 ,30cal/g C ; cC=0 ,25cal/g C

EXPERINCIA DE TYNDALL (REGELO)

SECRETARIA DE ESTADO DA EDUCAO

55

QUMICA
FSICA

AULA 10 TERODINMICA

EXERCCIOS DE FIXAO
1. Ao chegar a um posto de gasolina, um motorista vai ao calibrador

LEI GERAL DOS GASES PERFEITOS:

e infla os pneus do seu carro, colocando uma presso de 30bars


5

(considere 1 bar igual a 10 N/m ). Nesse momento, o motorista


0

verifica que a temperatura dos pneus de 27 C. Depois de dirigir por

EQUAO DE CLAPEYRON:

algum tempo, a temperatura dos pneus sobe para 81 C.


Desprezando-se o pequeno aumento no volume dos pneus e
tratando o ar no seu interior como um gs ideal, correto afirmar
que, em bar, a presso nos pneus passar a ser:
a) 35,4
b) 90,0
c) 45,5
d) 70,0
e) 54,5

TRANSFORMAES GASOSAS
ISOTRMICA
ISOBRICA

2. Uma mquina trmica opera usando um gs ideal monoatmiico,


de acordo com o ciclo representado na figura abaixo.

ISOCRICA
Sabendo que a temperatura de operao da mquina no ponto B
de 500 K, identifique as afirmativas corretas:
I. O trabalho realizado pela mquina trmica em um ciclo de 4 x 105
J.
II. A eficincia dessa mquina igual eficincia de uma mquina
operando segundo o ciclo de Carnot.
III. A menor temperatura atingida durante o ciclo de operao da
mquina de 100 K.
IV. Para uma mquina trmica ideal que trabalhe entre as
temperaturas de operao do ciclo representado na figura, a maior
eficincia possvel de 0,7.
V. A variao de energia interna em um ciclo completo nula.

TRABALHO DE UM GS NA TRANSFORMAO ISOBRICA

3. Uma mquina trmica ideal realiza um trabalho de 750 J por ciclo


(de Carnot), quando as temperaturas das fontes so 400K e 100K.
Nesse sentido, para que uma mquina trmica real apresente a
mesma eficincia e realize, por ciclo, o mesmo trabalho que a
mquina ideal, o calor recebido e o calor rejeitado so
respectivamente:
a) 1000 J e 250 J
d) 850 J e 150 J
b) 750 J e 500 J
e) 950 J e 350 J
c) 1250 J e 50 J

ENERGIA INTERNA

LEI DE JOULE

4. Certa quantidade de gs ideal monoatmico levada do estado A


para o estado C atravs de uma transformao isotrmica AB,
seguida de uma transformao isobrica BC, como indicado no
grfico. No processo completo ABC, o gs recebe 2 J de calor do
meio ambiente. Sabemos, tambm, que a variao da energia
interna no processo BC de 0,6 J.

1 LEI DA TERMODINMICA

RENDIMENTO DA MQUINA TRMICA


Com relao s transformaes
identifique as afirmativas corretas:

nesse processo,

I. A variao da energia interna no processo AB nula.


II. O trabalho realizado pelo gs no processo BC de 0,4 J.
III. O trabalho realizado pelo gs no processo AB de 1,0 J.
IV. A variao da energia interna no processo ABC de 0,8 J.
V. O calor absorvido no processo BC de 1 J.

RENDIMENTO MXIMO - CICLO DE CARNOT

SECRETARIA DE ESTADO DA EDUCAO

realizadas

56

QUMICA
FSICA

AULA 11 PTICA GEOMTRICA I

IMAGENS NO ESPELHO CONVEXO

IMGENS NO ESPELHO CNCAVO


Objeto alm do centro de
curvatura.

Objeto sobre o centro de


curvatura.

Objeto entre o centro de


curvatura e o foco.

Objeto entre o foco e o


vrtice

EQUAO DE GAUSS

ESPELHOS ESFRICOS

AUMENTO LINEAR TRANSVERSAL

REFRAO

SECRETARIA DE ESTADO DA EDUCAO

57

QUMICA
FSICA

PRISMAS

INDICE DE REFRAO

LEIS DA REFRAO

DESVIO MNIMO

LAMINA DE FACES PARALELAS

2i A

PRISMAS DE REFLEXO TOTAL

REFLEXO TOTAL

DISPESERSO LUMINOSA

SECRETARIA DE ESTADO DA EDUCAO

58

QUMICA
FSICA

AULA 11 PTICA GEOMTRICA II


LESTES ESFRICAS

EXERCCIOS
1. Uma usina solar uma forma de se obter energia limpa. A
configurao mais comum constituda de espelhos mveis
espalhados por uma rea plana, os quais projetam a luz solar
refletida para um mesmo ponto situado no alto de uma torre. Nesse
sentido, considere a representao simplificada dessa usina por um
nico espelho plano E e uma torre, conforme mostrado na figura ao
lado. Com relao a essa figura, considere:
A altura da torre de 100 m;
A distncia percorrida pela luz do espelho at o topo da torre de
200 m;
A luz do sol incide verticalmente sobre a rea plana;
As dimenses do espelho E devem ser desprezadas.
Nessa situao, conclui-se que o ngulo de incidncia de um feixe de
luz solar sobre o espelho E de:
a) 900
b) 600
c) 450
d) 300
e) 00

TIPOS DE LENTES

RAIOS NOTVEIS NA LENTE DIVERGENTE

2. A figura abaixo mostra dois espelhos planos, E1 e E2, que formam


um ngulo de 140 entre eles. Um raio luminoso R1 incide e
refletido no espelho E1, de acordo com a figura abaixo.
Nessa situao, para que o raio refletido R2 seja paralelo ao espelho
E2, o ngulo de incidncia de R1 no espelho E1 deve ser de:

IMAGENS NAS LENTES DIVERGENTES

a) 20
b) 30
c) 40
d) 50
e) 60

3. Um objeto O colocado em frente a um espelho cncavo


esfrico, como mostra a figura abaixo, sendo C o centro de curvatura
e F o foco do espelho.
Neste caso, pode-se afirmar que a imagem deste objeto ser

EQUAO DE GAUSS

AUMENTO LINEAR TRANSVERSAL


a) real, invertida, e menor que o objeto.
b) real, direita, e menor que o objeto.
c) real, direita, e maior que o objeto.
d) virtual, invertida, e menor que o objeto.
e) virtual, direita, e maior que o objeto.

EQUAO DOS FABRICANTES DE LENTES

4. Em um experimento de ptica, em sala de aula, uma rgua de


30,0cm de comprimento, quando colocada perpendicular ao eixo
principal e a 24,0cm do vrtice de um espelho esfrico cncavo,
produz uma imagem invertida de 10,0cm de altura.
Nessas circunstncias, a distncia focal do espelho, em cm, :

RAIOS NOTVEIS NA LENTE CONVERGENTE

a) 2
b) 3
c) 4
d) 5
e) 6

SECRETARIA DE ESTADO DA EDUCAO

59

QUMICA
FSICA

IMAGENS NAS LENTES CONVERGENTES

PRESBIOPIA - Apresenta como defeito o endurecimento do cristalino


com perda da capacidade de acomodao visual. No defeito
congnito, mais decorrente da idade. conhecida vulgarmente como
"vista cansada". A correo feita com o uso de lentes
convergentes.

AMETROPIAS E LENTES DE CORREO

PONTO PRXIMO ( PP ) de um globo ocular a posio mais


prxima que pode ser vista nitidamente, realizando esforo mximo
de acomodao. Na pessoa normal, situa-se, convencionalmente, a
25 cm.
ACOMODAO VISUAL o mecanismo pelo qual o olho humano
altera a vergncia do cristalino, permitindo pessoa normal
enxergar nitidamente desde uma distncia de aproximadamente 25
cm at o infinito.
PONTO REMOTO ( PR ) de um globo ocular a posio mais
afastada que pode ser vista nitidamente, sem esforo de
acomodao. Na pessoa normal, este ponto est situado no infinito.
AMETROPIAS
MIOPIA - Apresenta como defeito o achatamento do globo ocular,
provocando um alongamento no eixo ptico. O mope tem
dificuldade de enxergar objetos mais distantes. A imagem
formada antes da retina. A correo feita atravs de lentes
divergentes

EXERCCIOS DE FIXAO
1. Um projetor de slide um dispositivo bastante usado em
salas de aula e/ou em conferncias, para projetar, sobre uma
tela, imagens ampliadas de objetos. Basicamente, um
projetor constitudo por lentes convergentes. Nesse sentido,
considere um projetor formado por apenas uma lente
convergente de distncia focal igual a 10 cm. Nesse contexto,
a ampliao da imagem projetada, em uma tela a 2 m de
distncia do projetor, de:
a) 20 vezes b) 19 vezes c) 18 vezes b) 19 vezes e) 16
vezes
2. Nas figuras abaixo, esto representados por O os
objetos e por I, suas respectivas imagens, para lentes
convergentes e divergentes. Em cada uma das figuras,
identifique com R, quando a imagem for real e com V,
quando for virtual.

A sequncia correta da esquerda para direita, :


a) RRR
b) RRV
c) RVV
d) VVV e) VVR
5. Indivduos com viso normal tm distncia mxima de viso
distinta (ponto remoto) infinitamente grande (d) e distncia
mnima de viso distinta (ponto prximo) igual a 25 cm. As imagens
de um olho emtrope (normal) so projetadas pelo cristalino sobre a
retina. Considere uma pessoa A que, sem usar lentes de correo,
s consegue ver, nitidamente, objetos colocados em distncias alm
de 40 cm de seus olhos. Considere, tambm, uma pessoa B que tem
o ponto remoto a 5 metros de seus olhos. Sobre essas duas pessoas
podemos afirmar corretamente que:

HIPERMETROPIA - Apresenta como defeito o encurtamento


do globo ocular. Dificuldade de enxergar objetos mais
prximos. corrigida com o uso de lentes convergentes.

ASTIGMATISMO - Apresenta defeito na crnea, com raios de


curvatura irregulares. Ocasiona uma viso manchada dos
objetos. A correo feita com o uso de lentes cilndricas.

SECRETARIA DE ESTADO DA EDUCAO

60

I- A pessoa A portadora de miopia e a pessoa B de


hipermetropia.
II- A pessoa A e B so acometidas das ametropias mostradas
nas figuras 1 e 2 respectivamente.
III- Para a correo da ametropia da pessoa A um
oftalmologista prescreve uma lente convergente de vergncia
1,5 dioptrias.
IV- Para que a pessoa B possa ter viso normal deve usar
lentes divergentes de 0,2 dioptrias.
V- As pessoas A e B devem usar para corrigir suas
ametropias as lentes X e Y respectivamente.

QUMICA
FSICA

AULA 12 ONDAS

FENMENOS ONDULATRIOS
REFLEXO

Definio de Onda:
Ondas mecnicas:

Ondas eletromagntica:

REFRAO

Ondas transversais:
.

propagao

Ondas longitudinais:

DIFRAO

ELEMENTOS GEOMTRICOS DAS ONDAS PERDICAS

POLARIZAO

EQUAO FUNDAMENTAL DAS ONDAS

FRMULA DE TAYLOR

SECRETARIA DE ESTADO DA EDUCAO

61

QUMICA
FSICA

INTERFERNCIA

EXERCCIOS DE FIXAO
1. Sonares so dispositivos frequentemente usados na indstria
naval. Os navios possuem sonares para detectar obstculos no
fundo do mar, detectar cardumes etc. Um determinado sonar de um
navio produz ondas sonoras progressivas, com comprimento de onda
de 2,0 m e frequncia 200 Hz. Nesse caso, um obstculo a 80 m do
sonar ser detectado pelo navio em um intervalo de tempo de:
a) 0,4 s
b) 1,0 s
c) 1,2 s
d) 1,6 s
e) 2,0 s
2. Uma onda pode ser analisada a partir do seu comportamento
espacial ou temporal. Os grficos I e II abaixo representam uma onda
que se propaga em uma corda. No grfico I, v-se a forma da corda
em um determinado instante de tempo, e no grfico II, v-se o
deslocamento de um ponto da corda em funo do tempo.

RESSONNCIA

.
ACSTICA
Analisando-se estes grficos, pode-se concluir que a velocidade de
propagao da onda
a) 2 m/s
b)15 m/s
c) 5 m/s
d) 20 m/s
e) 10 m/s

Limites de audibilidade
O eco

3. Em um dado instante, a forma de uma corda por onde se propaga


uma onda indicada na figura abaixo:
Qualidades Fisiolgicas do Som
A intensidade

Com base nos dados obtidos da figura e sabendo-se que a


velocidade de propagao da onda de 120 cm/s, pode-se concluir
que seu comprimento de onda e freqncia so dados,
respectivamente, por:
a) 6 cm e 40 Hz
b) 3 cm e 40 Hz
c) 9 cm e 10 Hz
d) 6 cm e 20 Hz
e ) 3 cm e 20 Hz

A altura

4. Em um trecho reto de determinada estrada, um fusca move-se do


ponto A para o ponto B com velocidade de 20 m/s. Dois outros carros
esto passando pelos pontos A e B, com velocidade de 20 m/s,
porm com sentido contrrio ao do fusca, conforme ilustrado na
figura abaixo.

O timbre
Efeito Doppler

Nesse momento, o motorista do fusca comea buzinar e o som


emitido pela buzina tem frequncia f. Denominando as frequncias
ouvidas pelos motoristas dos carros que passam pelos pontos A e B
de f A e fB , respectivamente, correto afirmar que
a) f A = fB > f b) f A = fB < f c) f A > fB > f d) f A < f < fB e) f A = fB = f
5. A sirene de uma fbrica emite um som de frequncia f. Nesse
momento, dois funcionrios encontram-se nas seguintes situaes. O
funcionrio A, que est de sada da fbrica, move-se, afastando-se,
com uma velocidade v. O funcionrio B, que est chegando para o
seu turno de trabalho, tambm se move, aproximando-se, com
velocidade v. Sendo f A e fB, respectivamente, as frequncias que os
funcionrios escutam, correto afirmar:
a) fB < f A <f
b) f A < f< fB
c) f A < fB <f
d) fB < f < f A
e) f< f A < fB

SECRETARIA DE ESTADO DA EDUCAO

62

QUMICA
FSICA

AULA 13 ELETROSTTICA

CAMPO ELTRICO UNIFORME

CARGA ELTRICA

PRINCPIOS DA ELETROSTTICA-

POTENCIAL ELTRICO

QUANTIZAO DA CARGA ELTRICA:

POTENCIAL ELTRICO DE UMA CQRGA PUNTIFORME


ELETRIZAO:

TRABALHO ELTRICO

FORA ELTRICA LEI DE COULOMB

CAMPO ELTRICO

TRABALHO DO CAMPO ELTRICO UNIFORME

CAMPO DE UMA CARGA PUNTIFORME:

SUPERFCIES EQUIPOTENCIAIS NO CAMPO DE UMA CARGA

LINHAS DE FORA

SUPERFCIES EQUIPOTENCIAIS NO CAMPO UNIFORME

SECRETARIA DE ESTADO DA EDUCAO

63

QUMICA
FSICA

AULA 14 ELETRODINMICA

EXERCCIOS DE FIXAO
1. Duas partculas no laboratrio tm cargas eltricas + q e q,
respectivamente. Qual dos grficos abaixo melhor representa a
variao do campo eltrico produzido por estas cargas, em funo da
coordenada z, medida ao longo da reta mediatriz do segmento que
une as cargas?

CORRENTE ELTRICA

INTENSIDADE DE CORRENTE ELTRICA ELETRNICA MDIA (i)

2. Para carregar uma bateria de 12 V, o carregador deve mover 4,5


105 C de carga do polo positivo para o polo negativo. Sendo o
mdulo da carga elementar do eltron e = 1,6 10-19 C, a
quantidade de energia armazenada na bateria e o nmero de
eltrons transferidos so,
respectivamente:

TENSO ELTRICA OU DDP (U)

POTNCIA ELTRICA (P)

3. A figura mostra as linhas de fora de um campo eltrico que esto


no plano da folha. A densidade de linhas de fora proporcional
intensidade do campo eltrico. Uma partcula, carregada
positivamente, lanada nesse campo, perpendicularmente s linhas
de fora.

RESISTNCIA ELETRICA

1 LEI DE OHM

2 LEI DE OHM
Considere desprezvel a ao do campo gravitacional sobre a
partcula. medida que a partcula atravessa o campo, ela adquire
uma acelerao:
a) constante no sentido sul para norte.
b) crescente no sentido sul para norte.
c) constante no sentido norte para sul.
d) crescente no sentido oeste para leste.
e) crescente no sentido leste para oeste.
4. A superfcie interna de uma membrana celular coberta por um
excesso de nions, enquanto sua camada externa contm o mesmo
nmero de ctions. Devido pequena espessura da membrana, esta
podeser descrita, aproximadamente, como composta por duas placas
paralelas e infinitas, carregadas com cargas eltricas de sinais
opostos e separados por uma distncia d, conforme representado na
figura abaixo:Sabendo que esta disposio de cargas gera um
campo eltrico E uniforme entre as placas, correto afirmar que o
potencial eltrico na superfcie interna, relativo superfcie externa,
bem como o trabalho realizado pela fora eltrica sobre um on
de carga q, que penetra na clula atravessando a membrana,
valem, respectivamente:

ASSOSCIAO DE RESISTORES EM SRIE

ASSOSCIAO DE RESISTORES EM PARALELO

ASSOCIAO MISTA DE RESISTORES

a) Ed
b) qEd
c) Ed
d) qEd
e) Ed

e qEd.
e
Ed.
e qEd.
e
Ed.
e qEd.

SECRETARIA DE ESTADO DA EDUCAO

64

QUMICA
FSICA

AMPERMETRO, VOLTMETRO, OHMMETRO

EXERCCIOS DE FIXAO
1. Apesar do amplo emprego do Sistema Internacional de Unidades,
algumas unidades do sistema ingls ainda so utilizadas, como, por
exemplo, btu (british thermal unit). Usualmente, a potncia de
aparelhos de ar-condicionado expressa em btu/h, sendo1 btu/h =
0,293 W. Assim, um condicionador de ar de 15.000 btu/h emprega
potncia aproximada de 4,40 kW e em 6,00 h a energia eltrica
consumida ser, EM kWh,
a) 26,4
b) 36,2
c) 48,5
d) 75,1
e) 94,3

GERADOR ELTRICO
2. Em uma certa residncia, existe um chuveiro eltrico (A) de
indicao nominal (4.400 W/6.600 W 220 V). Esse chuveiro possui
uma chave reguladora que possibilita dispor-se de gua morna na
posio Vero e de gua mais quente na posio Inverno.
Entretanto, existe tambm um outro chuveiro (B), de mesma
finalidade, que possui a inscrio nominal (4.400 W/6.600 W 110
V). Comparando-se o consumo de energia eltrica dos dois
chuveiros, para dois banhos idnticos, conclumos que:
a) o chuveiro A consome o dobro da energia eltrica consumida pelo
chuveiro B.
b) o chuveiro B consome o dobro da energia eltrica consumida pelo
chuveiro A.
c) o chuveiro A consome uma vez e meia a energia eltrica
consumida pelo chuveiro B.
d) o chuveiro B consome uma vez e meia a energia eltrica
consumida pelo chuveiro A.
e) os dois chuveiros consomem a mesma quantidade de energia.

ASSOCIAO DE GERADORES EM SRIE

3. Um chuveiro eltrico tem seletor que lhe permite fornecer duas


potncias distintas: na posio vero o chuveiro fornece 2.700 W:
na posio inverno fornece 4.800 W. Jos, o dono desse chuveiro
usa-o diariamente na posio inverno, durante 20 minutos.
Surpreso com o alto valor de sua conta de luz, Jos resolveu usar o
chuveiro com o seletor na posio vero, pelos mesmos 20 minutos
dirios. Supondo-se que o preo do quilowatt-hora seja de R$ 0,20,
isso representar uma economia diria de aproximadamente:
a) R$ 0,14
b) R$ 0,20
c) R$ 1,40 d) R$ 2,00 e) R$ 20,00

ASSOCIAO DE GERADORES EM PARALELO

4. Comercialmente, os resistores tm seus valores de resistncia


identificados a partir de um cdigo de trs cores, impressas sob a
forma de anis no prprio corpo do resistor.
As cores utilizadas nos anis A, B e C correspondem aos nmeros
indicados na seguinte tabela:
Nessa conveno, A e B so, respectivamente, os algarismos da
dezena e da unidade e C a potncia de 10 do valor da resistncia
em ohms. Considere 1 cal 4,2 J.
A resistncia do aparelho usado por uma bailarina para ferver a gua
para o caf deve ser substituda. Tal resistncia, ao ser atravessada
por uma corrente de 1,0 A dissipa uma potncia de 600 W. Calcule o
valor da resistncia e indique a seqncia de cores CBA que um
resistor comercial, com esse valor de resistncia, deve apresentar.
a) 300 ; marrom preta laranja
b) 400 ; marrom preta amarela
c) 500 ; marrom preta verde
d) 600 ; marrom preta azul
e) 700 ; marrom preta violeta

ASSOCIAO MISTA DE GERADORES

RECEPTOR ELTRICO

5. Por recomendao de um eletricista, o proprietrio substituiu a


instalao eltrica de sua casa, e o chuveiro, que estava ligado em
110 V, foi trocado por outro chuveiro de mesma potncia, ligado em
220 V. A vantagem dessa substituio est
a) no maior aquecimento da gua que esse outro chuveiro vai
proporcionar.
b) no menor consumo de eletricidade desse outro chuveiro.
c) na dispensa do uso de disjuntor para o circuito desse outro
chuveiro.
d) no barateamento da fiao do circuito desse outro chuveiro, que
pode ser mais fina.
e) no menor volume de gua de que esse outro chuveiro vai
necessitar.

LEIS DE KIRCHHOFF

LEI DOS NS

LEI DAS MALHAS

SECRETARIA DE ESTADO DA EDUCAO

65

QUMICA
FSICA

AULA 15 ELETROMAGNETISMO I

EXERCCIOS DE FIXAO

IMS so xidos de ferro (Fe2O3), encontrados originalmente


numa regio da sia menor chamada Magnsia, que exercem
atraes em materiais a base de ferro e exibem fenmenos
especiais. Quando suspensos ou pendurados pelo centro de massa,
alinham-se com a direo norte sul do lugar.
PRINCPIO DA ATRAO E REPULSO
.

INSEPARABILIDADE DOS PLOS MAGNTICOS

1. Os eletroms, formados por solenoides percorridos por correntes


eltricas e um ncleo de ferro, so dispositivos utilizados por
guindastes eletromagnticos, os quais servem para transportar
materiais metlicos pesados. Um engenheiro, para construir um
eletrom, utiliza um basto cilndrico de ferro de 2,0 metros de
comprimento e o enrola com um fio dando 4106 voltas. Ao fazer
passar uma corrente de 1,5 A pelo fio, um campo magntico
gerado no interior do solenoide, e a presena do ncleo
de ferro aumenta em 1.000 vezes o valor desse campo. Adotando
para a constante o o valor 4 107 T. m/ A , correto afirmar que,
nessas circunstncias, o valor da intensidade do campo magntico,
no interior do cilindro de ferro, em tesla, de:
a) 24 102 b) 12 102 c) 6 102 d) 3 102 e) 102

CAMPO MAGNTICO DOS MS

CAMPO MAGNTICO TERRESTRE

EXPERINCIA DE OERSTED

2. Os antigos navegantes usavam a bssola para orientao em alto


mar, devido sua propriedade de se alinhar de acordo com as linhas
do campo geomagntico. Analisando a figura onde esto
representadas estas linhas, podemos afirmar que:
a) o polo sul do ponteiro da bssola aponta para o Polo Norte
geogrfico, porque o norte geogrfico corresponde ao sul
magntico.
b) o polo norte do ponteiro da bssola aponta para o Polo Sul
geogrfico, porque o sul geogrfico corresponde ao sul
magntico.
c) o polo sul do ponteiro da bssola aponta para o Polo Sul
geogrfico, porque o sul geogrfico corresponde ao sul
magntico.
d) o polo norte do ponteiro da bssola aponta para o Polo Sul
geogrfico, porque o norte geogrfico corresponde ao norte
magntico.
e) o polo sul do ponteiro da bssola aponta para o Polo Sul
geogrfico, porque o norte geogrfico corresponde ao sul magntico.

CAMPO MAGNTICO DAS CORRENTES - LEI DE BIOT-SAVART

Regra da Mo Direita

CAMPO MAGNTICO DE UM CONDUTOR RETILNEO INFINITO

CAMPO MAGNTICO NO CENTRO DE UMA ESPIRA CIRCULAR

CAMPO MAGNTICO NO INTERIOR DE UM SOLENIDE

SECRETARIA DE ESTADO DA EDUCAO

66

QUMICA
FSICA

AULA 16 ELETROMAGNETISMO II

EXERCCIOS DE FIXAO
1. O tubo de televiso possui um canho eletrnico que faz a
varredura da tela fotoluminescente numa sucesso de linhas da
esquerda para a direita e de cima para baixo. Tal varredura feita
com a rapidez suficiente para que nossos olhos no percebam o
desaparecimento de uma linha e o surgimento de outra e, alm
disso, nos d a sensao de movimento da imagem. Sobre a fora
responsvel por esse movimento de varredura da tela de TV,
correto afirmar:
a) uma fora eletrosttica que atua na direo do feixe eletrnico.
b) uma fora magntica que atua na direo perpendicular ao feixe
eletrnico.
c) uma fora eletrofraca que atua nos neutrinos do feixe eletrnico.
d) uma fora eltrica que atua nos neutrinos do feixe eletrnico.
e) uma fora eletromagntica que atua nos nutrons do feixe
eletrnico.

FORA MAGNTICA

FORA MAGNTICA SOBRE CARGA ELTRICA MVEL

SENTIDO DA FORA MAGNTICA SOBRE CARGAS MVEIS


(REGRA DA MO)

MOVIMENTOS
MAGNTICO

DA

CARGA

NO

INTERIOR

DO

CAMPO

Se a velocidade da partcula for paralela ao campo


magntico, no atuar fora magntica e o movimento
ser retilneo e uniforme.

Se a velocidade da partcula for perpendicular ao campo


magntico, o movimento ser circular e uniforme.

Se a velocidade da partcula for obliqua ao campo


magntico, o movimento ser helicoidal e uniforme.

FORA MAGNTICA SOBER CONDUTOR PERCORRIDO POR


CORRENTE

FORA
MAGNTICA
ENTRE
CONDUTORES
PARALELOS PERCORRIDO POR CORRENTE

RETOS,

2. Uma partcula eletricamente carregada, inicialmente em


movimento retilneo uniforme, adentra uma regio de campo
magntico uniforme, perpendicular trajetria da partcula. O plano
da figura ilustra a trajetria da partcula, assim como a regio de
campo magntico uniforme, delimitada pela rea sombreada.
Se nenhum outro campo estiver presente, pode-se afirmar
corretamente que, durante a passagem da partcula pela regio de
campo uniforme, sua acelerao :
a) tangente trajetria, h realizao de trabalho e a sua
cintica aumenta.
b) tangente trajetria, h realizao de trabalho e a sua
cintica diminui.
c) normal trajetria, no h realizao de trabalho e a sua
cintica permanece constante.
d) normal trajetria, h realizao de trabalho e a sua
cintica aumenta.
e) normal trajetria, no h realizao de trabalho e a sua
cintica diminui.

energia
energia
energia
energia
energia

3 . O funcionamento de alguns instrumentos de medidas eltricas,


como, por exemplo, o galvanmetro, baseia-se no efeito mecnico
que os campos magnticos provocam em espiras que conduzem
correntes eltricas, produzindo o movimento de um ponteiro que se
desloca sobre uma escala. O modelo acima mostra, de maneira
simples, como campos e correntes provocam efeitos mecnicos. Ele
constitudo por um fio condutor, de comprimento igual a 50 cm,
suspenso por uma mola de constante elstica igual a 80 N/m e
imerso em um campo magntico uniforme, de intensidade B igual a
0,25 T, com direo perpendicular ao plano desta folha e sentido de
baixo para cima, saindo do plano da folha. Calcule a corrente eltrica
i que dever percorrer o condutor, da esquerda para a direita, para
que a mola seja alongada em 2,0 cm, a partir da posio de equilbrio
estabelecida com corrente nula.
a) 2,8 A
b) 4,2 A c) 10 A
d) 11, 2 A e) 12, 8 A
4. Peter Barlow (1776-1862), cientista e engenheiro ingls, foi um
dos primeiros a inventar um motor a corrente contnua,
esquematizado no desenho abaixo:
O circuito eltrico fecha-se no encontro da ponta de um raio da roda
com o mercrio. Devido ao campo magntico produzido pelo m, de
polos C e D, a roda gira, mantendo sempre um raio em contato com
o mercrio. Assim, v-se a roda girando no sentido:
a) horrio, se C for polo norte e a corrente fluir, no contato, do raio
para o mercrio.
b) horrio, se C for polo norte e a corrente fluir, no contato, do
mercrio para o raio.
c) horrio, se C for polo sul e a corrente fluir, no contato, do mercrio
para o raio.

d) anti-horrio, se C for polo norte e a corrente fluir, no


contato, do mercrio para o raio.
e) anti-horrio, se C for polo sul e a corrente fluir, no contato,
do raio para o mercrio.

SECRETARIA DE ESTADO DA EDUCAO

67

QUMICA
FSICA

AULA 17 INDUO ELETROMAGNTICA

EXERCCIOS DE FIXAO
1. Um carto de crdito consiste de uma pea plstica na qual h
uma faixa contendo milhes de minsculos domnios magnticos
mantidos juntos por uma resina. Cada um desses domnios atua
como se fosse um minsculo im com sentido de polarizao nortesul bem definido. Um cdigo contendo informaes particulares de
uma pessoa (como nome, nmero do carto, data de validade do
carto) pode ser gravado na faixa atravs de um campo magntico
externo que altera o sentido de polarizao dos domnios em alguns
locais selecionados. Quando o carto desliza atravs de uma fenda
de um caixa eletrnico ou equipamento similar, os domnios
magnticos passam por um cabeote de leitura, e pulsos de
voltagem e corrente so induzidos segundo o cdigo contido na
faixa. Esse processo de leitura do carto por induo magntica tem
seus fundamentos nas
a) Lei de Coulomb e Lei de Lenz.
b) Lei de Faraday e Lei de Lenz.
c) Lei de Biot-Savart e Lei de Gauss.
d) Lei de Faraday e Lei de Coulomb.
e) Lei de Coulomb e Lei de Ampre.

FLUXO MAGNTICO

2. Um dos dispositivos utilizados como detector de veculos nas


lombadas eletrnicas conhecido como lao indutivo. Quando um
veculo em movimento passa por um lao indutivo, a plataforma
metlica inferior do veculo (chassis) interage com um campo
magntico preexistente no local, induzindo uma corrente eltrica num
circuito ligado ao processador de dados. O sistema lao indutivo e a
plataforma metlica em movimento geram um sinal eletromagntico
obedecendo lei de Faraday, que pode ser enunciada da seguinte
maneira:
a) campo magntico que varia no tempo fonte de campo eltrico.
b) massa fonte de campo gravitacional.
c) campo eltrico que varia no tempo fonte de campo magntico.
d) carga eltrica fonte de campo eltrico.
e) corrente eltrica fonte de campo magntico.

LEI DE FARADAY

3. Geradores de energia eltrica valem-se do fenmeno da induo


eletromagntica para produzir quase toda a energia consumida. Um
gerador, basicamente, constitudo por espiras de um material
condutor que ir gerar uma fora eletromotriz em seus terminais,
quando essas espiras forem submetidas a um
a) fluxo magntico varivel.
b) fluxo magntico invarivel.
c) campo eltrico.
d) campo eletromagntico invarivel.
e) campo magntico invarivel.

LEI DE LENZ

4. No final do sculo XIX, uma disputa tecnolgica sobre qual a


corrente eltrica mais adequada para transmisso e distribuio da
energia eltrica, gerada em usinas eltricas, tornou clara a vantagem
do uso da corrente alternada, em detrimento da corrente contnua.
Um dos fatores decisivos para essa escolha foi a possibilidade da
utilizao de transformadores na rede de distribuio de eletricidade.
Os transformadores podem aumentar ou diminuir a tenso a eles
fornecida, permitindo a adequao dos valores da intensidade da
corrente transmitida e reduzindo perdas por efeito Joule, mas s
funcionam em corrente alternada. O princpio fsico em que se baseia
o funcionamento dos transformadores e a caracterstica da corrente
alternada que satisfaz a esse princpio so, respectivamente:

O sentido da corrente eltrica induzida tal que gera uma


oposio variao do fluxo magntico que lhe deu origem.

SECRETARIA DE ESTADO DA EDUCAO

68

a) a conservao da carga e o movimento oscilante dos portadores


de carga eltrica.
b) a induo eletrosttica e o movimento contnuo dos portadores de
carga eltrica.
c) a induo eletrosttica e o movimento oscilante dos portadores de
carga eltrica.
d) a induo eletromagntica e o movimento contnuo de portadores
de carga eltrica.
e) a induo eletromagntica e o movimento oscilante dos portadores
de carga eltrica.

QUMICA
FSICA

AULA 18 FSICA MODERNA

SECRETARIA DE ESTADO DA EDUCAO

69

QUMICA
FSICA

SECRETARIA DE ESTADO DA EDUCAO

70

QUMICA

SECRETARIA DE ESTADO DA EDUCAO

71

QUMICA

EXERCCICIOS DE FIXAO

Aula 1 : Cidadania na antiguidade Grcia

1) A democracia continua criando polmicas e atraindo mudanas


polticas. Na poca de Clstenes, na Grcia Antiga, a democracia
conseguiu espaos de poder importantes. Nos tempos de Clstenes,
a democracia:

Atenas
- Bero da democracia
* Regime implantado pelo legislador Clstenes (509 a.C);
* Apogeu: sculo V ( Sculo de Pricles);
* Cidadania (participao poltica): homens livres, maiores de
18 anos e filhos de pai e me atenienses;
* Segmentos excludos da cidadania: mulheres, escravos e
estrangeiros;
* Principal instituio poltica: Assembleia Popular ( Eclsia);

a) firmou-se com propostas descentralizadoras, ampliando a


cidadania e evitando a existncia do trabalho escravo, defendido pelo
filsofo Aristteles.
b) facilitou a participao no governo dos cidados mais pobres,
chegando a remunerar os cargos polticos e reorganizando a
administrao da cidade de Atenas.
c) anulou a lei que defendia o exlio poltico, por ser opressiva e
privilegiar a nobreza dona das grandes propriedades rurais.
d) considerou as mulheres como participantes da cidadania,
renovando as tradies e combatendo a corrupo muito comum na
poca da tirania.
e) defendeu a aplicao das teorias polticas de Plato, organizando
uma Repblica onde prevalecia o poder das Assembleias Populares.

* Ostracismo: exlio por dez anos e consequente perda dos


direitos polticos caso o cidado fosse condenado pela
Assembleia Popular por corrupo ou atentado contra a
democracia.
Esparta
- Caractersticas:
* Oligarquia
* Militarismo
* Xenofobia
* Eugenia
- Regime atribudo ao legislador Licurgo;
- Controle poltico dos esparciatas (cidados soldados);
- Principal instituio poltica: Eforato.

2)Qual das alternativas abaixo apresenta aspectos do legado romano


para as civilizaes posteriores?

Roma

a) O direito romano, presente at os dias de hoje na cultura do


Ocidente, assim como o latim, que deu origem a lngua portuguesa,
francesa, italiana e espanhola.
b) A religio politesta romana que at hoje predominante no
mundo ocidental.
c) As tcnicas de construo de pirmides e a Medicina, atravs do
processo de mumificao de corpos.
d) A lngua inglesa, a democracia e a educao voltada para as artes
e cultura.
e) As bases da filosofia ocidental, rompendo o mito e inaugurando
o imprio da razo.

- Bero da Repblica (509 a.C)


* Principal instituio poltica: Senado;
* Controle poltico dos patrcios (aristocracia);
* Lutas sociais: patrcios x plebeus;
* Conquistas da plebe:
Tribunato ;
Lei das XII Tbuas;
Lei Canuleia (igualdade civil);
Lei Licnia Sextia(aboliu a escravido por dvidas);
Plebiscito; Lei Oglnia(igualdade religiosa);

- Quais as diferenas entre a democracia grega e a


democracia praticada nos dias de hoje ?
______________________________________________________
______________________________________________________

______________________________________________________

SECRETARIA DE ESTADO DA EDUCAO

72

QUMICA
HISTRIA

Aula 2 : O mundo feudal


-O modo de produo feudal, predominante durante a idade
mdia, caracterizou-se por uma base econmica agrcola
,relaes de interdependncia pessoal e trabalho servil.
*Feudo: unidade de produo agrcola de subsistncia
composta de vila senhorial, mansos reas comunais
(bosques);
*Campons preso terra mediante o pagamento de
obrigaes(tributos): corveia, talha, banalidades, mo morta,
dentre outras;

EXERCCICIOS DE FIXAO
1) Uma das caractersticas a ser reconhecida no feudalismo europeu
:
a) A sociedade feudal era semelhante ao sistema de castas.
b) Os ideais de honra e fidelidade vieram das instituies dos hunos.
c) Viles e servos estavam presos a vrias obrigaes, entre elas o
pagamento anual de capitao, talha e banalidades.
d) A economia do feudo era dinmica, estando voltada para o
comrcio dos feudos vizinhos.
e) As relaes de produo eram escravocratas.

*Origens do feudalismo: crise do Imprio romano e


instituies brbaras;
* Sociedade estamental, rigidamente hierarquizada e
baseada nas relaes de suserania e vassalagem entre os
nobres;
* Pacto feudo vasslico: homenagem, juramento de
fidelidade e investidura;
* Igreja: o poder mais alto;
* Combate s heresias (ideias e/ou atitudes que
contrariavam os dogmas e as regras impostas pelo clero):
Tribunal do Santo Ofcio (Inquisio);
2) Nas relaes de suserania e vassalagem dominantes durante o
feudalismo europeu, possvel observar que:

* Cultura: marcada pelo teocentrismo;


* Civilizaes de destaque: rabe e Bizantina;
* Unificao religiosa e poltica dos rabes: Maom
(islamismo);
* Guerra Santa (Jihad) : expanso rabe do sculo VII;
* Imprio Bizantino: Cesaropapismo(teocracia);Questo
iconoclasta(destruio das imagens) e Grande Cisma do
Oriente(diviso da Igreja Crist em catlica e ortodoxa);
* Contribuies cientficas e culturais dos rabes para
o mundo ocidental: matemtica(difuso dos algarismos
hindus, desenvolvimento da lgebra);qumica(processo de
destilao do lcool e descoberta do nitrato de prata,
carbonato de sdio e salitre);anatomia (Tratado de
Avicena);culturas
do
caf
e
da
cana-de-acar;
filosofia(Averris).

a) a servido representou, sobretudo na Frana e na pennsula


Ibrica, um verdadeiro renascimento da escravido conforme existia
na Roma imperial.
b) os suseranos leigos, formados pela grande nobreza fundiria,
distinguiam juridicamente os servos que trabalhavam nos campos
dos que produziam nas cidades.
c) mesmo dispondo de grandes propriedades territoriais, os
suseranos eclesisticos no mantinham a servido nos seus
domnios, mas sim o trabalho livre.
d) o sistema de impostos incidia de forma pesada sobre os servos. O
imposto da mo morta, por exemplo, era pago pelos herdeiros de um
servo que morria para que continuassem nas terras pertencentes ao
suserano.
e) as principais instituies sociais que sustentavam as relaes
entre senhores e servos eram de origem muulmana, oriundos da
longa presena rabe na Europa Ocidental.

*
Contribuies
do
Imprio
Bizantino:
arquitetura(Catedral de Santa Sofia);mosaicos; direito(Cdigo
de Justiniano, Corpus Juris Civilis).
*
Crise do feudalismo: guerras; fome;
camponesas ;peste negra; declnio da servido;

revoltas

* Transio para o feudalismo: renascimento comercial e


urbano.
QUAIS
OS
ASPECTOS
QUE COMPROVAM
HISTORICAMENTE O PODER DA IGREJA SOBRE A
SOCIEDADE MEDIEVAL ?
-

______________________________________________________
______________________________________________________
______________________________________________________
______________________________________________________
______________________________________________________

SECRETARIA DE ESTADO DA EDUCAO

73

QUMICA
HISTRIA

AULA 3 : Amrica Pr-Colombiana

EXERCCICIOS DE FIXAO

*Os Maias ( Pennsula de Yucatn, Amrica Central)


- Confederao de cidades estados teocrticas
- Base econmica agrcola e servido coletiva
- Escrita baseada em ideogramas
-Politesmo
- Arquitetura monumental
- Dominao asteca
*Os Astecas (Planalto do Mxico)
-Imprio teocrtico
- Expansionismo
- Prtica de rituais sangrentos
- Capital: Tenotitln ( atual cidade do Mxico)
- Economia de base agrcola e servido coletiva
- Dominao espanhola (1519 sob a liderana de
Ferno Cortz)

1) Alejandro Toledo foi o primeiro indgena eleito presidente do Peru.


Durante a campanha eleitoral, foi chamado de Pachactec, numa
aluso ao imperador incaico que consolidou um imprio nos Andes
centrais. Sobre a sociedade incaica, correto afirmar que:
a) o fato de constituir uma das mais significativas sociedades prcolombianas tem como base a negao da cultura dos povos
dominados;
b) a sua economia tinha por base a agricultura, com a distribuio de
terras pelo Estado e a prtica do sistema de regadio;
c) o que a diferenciava das demais culturas pr-colombianas era a
ideia de uma sociedade igualitria;
d) o comrcio interno era significativo, tendo no guano um dos
produtos mais valorizados;
e) a sua cultura desapareceu com o processo da dominao
espanhola.

*Os Incas ( Cordilheira andina)


- Imprio teocrtico
- Capital: Cuzco ( Umbigo do Mundo)
- Agricultura diversificada e uso de animais como meio
de transporte e fornecimento da l( a exemplo das
lhamas e alpacas).
- Principal divindade: O sol (Inti)
- Avanado sistema de contabilidade: Quipos
- Conquista espanhola (1533 sob a liderana de
Francisco Pizarro)
A colonizao da Amrica Espanhola
- Definida pelos princpios mercantilistas : explorao,
monoplio, balana comercial favorvel, protecionismo e
metalismo
- Base: minerao (prata e ouro)
- Servido coletiva: mita e encomienda
- Diviso administrativa em vice-reinos e capitanias
gerais
- Rigoroso controle tributrio e alfandegrio
- Sociedade colonial marcada por forte hierarquizao:
chapetones , criollos, negros, ndios e mestios.
*A Colonizao da Amrica Portuguesa
- Monoplio garantido pelo pacto colonial
- Base: plantation aucareira
- Escravido indgena e negra
- Diviso em capitanias hereditrias (juridicamente
institudas pela carta de doao e foral)
- Forte centralizao administrativa : Governo Geral
- Sociedade colonial: rural, aristocrtica e patriarcal

2) "Os guerreiros constituam um dos grupos mais importantes na


sociedade asteca. No incio, eram escolhidos entre os indivduos
mais corajosos e valentes do povo. Com o tempo, entretanto, a
funo de guerreiro comeou a ser passada de pai para filho, e
apenas algumas famlias, privilegiadas, mantiveram o direito de ter
guerreiros entre os seus membros." (KARNAL, Leandro. A
CONQUISTA DO MXICO. So Paulo: FTD, 1996. p. 13.)
O texto faz referncia sociedade asteca, no sculo XV, a qual era:
a) guerreira e sacerdotal, formada de uma elite poltica que
governava com tirania a massa de trabalhadores escravos negros. b)
igualitria e guerreira, no reconhecendo outra autoridade seno a
sacerdotal, que tambm era guerreira. c) comunal, com estruturas
complexas, sendo dirigida por um Estado que contava com um
aparelho administrativo, judicirio e militar. d) hierarquizada e
guerreira, visto que o Imperador era, ao mesmo tempo, o general do
exrcito asteca e o sumo pontfice sacerdotal. e) igualitria, guerreira
e sacerdotal: todo guerreiro era um sacerdote e todo sacerdote era
um guerreiro.

*A Colonizao da Amrica Inglesa


-Povoamento (ao dos puritanos ingleses do
Mayflower,1620)
- Mdias e pequenas propriedades, trabalho livre
(servido por contrato),policultura e estruturao de um
mercado consumidor interno
- Ideologia do Destino Manifesto ( Calvinismo)
- Experincia indita de autogoverno
- Colnias de explorao ao sul: produo algodoeira
e escravido negra
QUAIS OS EFEITOS DA COLONIZAO EUROPEIA SOBRE
AS CIVILIZAES PR- COLOMBIANAS ?

SECRETARIA DE ESTADO DA EDUCAO

74

QUMICA
HISTRIA

AULA 4 : O RENASCIMENTO CULTURAL

EXERCCICIOS DE FIXAO
1) Erasmo de Rotterdam (1467-1536) foi um dos pensadores mais
influentes de sua poca, sobretudo porque em sua obra ELOGIO DA
LOUCURA defendeu, entre outros aspectos,

- Itlia: Bero da Renascena.


*
*
*
*
*
*
*

Causas :
Desenvolvimento comercial e urbano;
Ascenso da burguesia;
Fuga dos sbios bizantinos;
Forte presena da cultura clssica na Itlia;
Atuao dos Mecenas (patrocinavam os artistas);
Desenvolvimento dos estudos humanistas;
Advento da imprensa.

a) a tolerncia, a liberdade de pensamento e uma teologia baseada


exclusivamente nos Evangelhos.
b) a restaurao da teologia nos termos da ortodoxia escolstica, na
linha de Toms de Aquino.
c) a reforma eclesistica da Igreja segundo a proposta de
Savonarola, conforme sua pregao em Florena.
d) o comunismo dos bens, teoria que influenciaria o pensamento de
Rousseau no sculo XVIII.
e) a supremacia da razo do Estado sobre as regras definidas nos
princpios da moral crist.

- Caractersticas :
* Humanismo (valorizao do homem);
* Apego ao racionalismo;
* Estmulo investigao cientfica;
* Empirismo (experimentao e comprovao cientfica dos
fatos);
* Hedonismo (exaltao do prazer);
* Classicismo (reinterpretao da cultura clssica;
* Esprito crtico;
* Individualismo;
* Valorizao dos idiomas nacionais;
* Naturalismo;
* Universalismo;
* Antropocentrismo (o homem como centro do universo).
- Inovaes no campo da produo artstica :
*
*
*
*
*
*

Representao frontal das figuras humanas;


Humanizao das figuras representadas;
Recurso anatomia na concepo das esculturas;
Apreo pela esttica;
Perspectiva geomtrica aplicada pintura;
Uso da tinta a leo.

- Os grandes expoentes da produo renascentista:


*Literatura: Dante Alighieri (Divina Comdia); Cames
(Os Lusadas); Cervantes (D. Quixote);Giovanni Boccaccio
(Decamero);
*Pintura e escultura: Leonardo Da Vinci; Michelangelo;
Donatello ; Rafael Snzio ;Sandro Botticelli;
* Cincias: Nicolau Coprnico, Galileu Galilei e Kepler
(astronomia); Falpio, Miguel de Servet e Andr Veslio
(medicina); Francis Bacon (mtodo experimental).

2) Sobre o Renascimento, pode-se afirmar:


a) pode ser visto como uma revoluo religiosa, resultado das
profundas transformaes que ocorreram na transio entre o
feudalismo e o capitalismo;
b) Florena e Roma, Pequim e Bagd foram centros de irradiao do
movimento renascentista;
c) o Renascimento valorizava o anonimato e fortalecia o sentimento
nacionalista;
d) o Renascimento foi um movimento artstico, literrio e cientfico
defensor do humanismo, baseado no antropocentrismo e no esprito
crtico em oposio ao teocentrismo;
e) o Renascimento fez renovar toda tradio islmica da pennsula
Ibrica reprimida pelas Cruzadas.

Com suas ideias renovadoras, o movimento renascentista


contribuiu de modo marcante para a afirmao dos valores
da burguesia nascente e superao do obscurantismo..
- O Renascimento buscou uma fuso entre a cincia e a
arte. Em que medida, na atualidade, isso possvel ?

______________________________________________________
______________________________________________________
______________________________________________________
_____________________________________________________

SECRETARIA DE ESTADO DA EDUCAO

75

QUMICA
HISTRIA

EXERCCIOS DE FIXAO

AULA 5 : REFORMA PROTESTANTE

1) O Rei Henrique VIII, aclamado defensor da f pela Igreja Catlica,


rompeu com o Papa Clemente VII em 1534, por:

- Causas :
* Ascenso das monarquias nacionais;
* Desenvolvimento do comrcio e da burguesia;
* Transformaes provocadas pelo esprito renascentista;
* Prtica da simonia (venda de objetos e relquias santas);
* Concubinato dos padres;
* Intolerncia do clero;
* Venda das indulgncias.
- Expoentes do reformismo:
* Martinho Lutero, monge alemo.
- Proco em Wittemberg (Saxnia);
- Condenou publicamente o comrcio das indulgncias;
-Foi excomungado pelo Papa e reagiu publicando as 95
teses (1517);
-Recebeu apoio de parte da nobreza (interessada em dispor
das grandes extenses de terra que o clero possua);
- Redige a Confisso de Augsburgo (sntese da doutrina
protestante);
-Revolta dos Anabatistas (os camponeses condenam a
aliana de Lutero com os nobres);
- Paz de Augsburgo (1555): diviso da Alemanha entre
catlicos e protestantes.
* Joo Calvino, sacerdote francs
Na Suca,deu continuidade ao reformismo de Ulrico
Zwinglio;
- Obteve apoio da burguesia;
- Defendeu a predestinao absoluta;
- Estabeleceu rgidas regras morais (puritanismo);
Instituiu o Consistrio (Cmara de fiscalizao dos
costumes e comportamento social);
- tica protestante e capitalismo.
*
-

Henrique VIII, rei da Inglaterra


Criador da Igreja Anglicana;
Rei como Chefe da Igreja (Ato de Supremacia)
Refora o absolutismo monrquico.

a) opor-se ao Ato de Supremacia que submetia a Igreja Anglicana


autoridade
do
Papa.
b) rever todos os dogmas da Igreja Catlica, incluindo a
indissolubilidade do sagrado matrimnio, atravs do Ato dos Seis
Artigos.
c) aceitar as 95 teses de Martinho Lutero, que denunciavam as
irregularidades
da
Igreja
Catlica.
d) ambicionar assumir as terras e as riquezas da Igreja Catlica e
enfraquecer
sua
influncia
na
Inglaterra.
e) defender que o trabalho e a acumulao de capital so
manifestaes da predestinao salvao eterna como professava
Santo Agostinho.

2) No contexto dos diversos conflitos religiosos que eclodiram na


Europa, ao longo do sculo XVI, identificamos a convocao pela
Igreja Catlica, a partir de 1545, do Conclio de Trento. Dentre suas
determinaes,
destacamos
corretamente
o
(a):
a) reconhecimento da autoridade poltica e teolgica da Igreja
anglicana frente ao papado, encerrando os conflitos provocados na
Inglaterra devido luta de Henrique VIII contra o Vaticano.
b) fim do clero regular como soluo para conter os abusos
cometidos pela Igreja, tais como a venda de indulgncias e
sacramentos.
c) oficializao da doutrina calvinista que admitia o lucro comercial
como uma ddiva divina e no mais como um pecado usurrio, como
um
novo
dogma
catlico.
d) submisso da Igreja catlica aos Estados imperiais laicos e a
validade
da
livre
interpretao
da
Bblia.
e) reafirmao da hierarquia eclesistica catlica e a reativao do
tribunal do Santo Ofcio da Inquisio.

* Contrarreforma
- Reao da Igreja Catlica ao avano protestante;
- Conclio de Trento (1545) Decises:
* Proibio da venda de indulgncias;
* Publicao do ndex (lista dos livros proscritos);
* Revitalizao dos Tribunais inquisitoriais;
* Estmulo ao trabalho missionrio;
* Criao de Seminrios.

- Atualmente a Igreja Catlica atravessa um perodo de renovao


sob a liderana do Papa Francisco. Quais os aspectos que podem
exemplificar essa postura ?

_______________________________________________________

SECRETARIA DE ESTADO DA EDUCAO

76

QUMICA
HISTRIA

AULA 6: AS REVOLUES LIBERAIS

EXERCCIOS DE FIXAO

- Inspiradas no iderio iluminista e em sintonia com os anseios da


burguesia, contriburam para o fim do Antigo Regime e triunfo do
liberalismo poltico e econmico;

1) As grandes revolues burguesas do sculo XVIII refletem, em


parte, algumas ideias dos filsofos iluministas, dentre as quais
podemos destacar a que
a) apontou a necessidade de limitar a liberdade individual para
impedir que o excesso degenerasse em anarquismo.
b) acentuou que o Estado no possui poder ilimitado, o qual nada
mais do que a somatria do poder dos membros da sociedade.
c) visou defender a tese de que apenas a federalizao poltica
compatvel com a democracia orgnica.
d) mostrou que, sem centralizao e dependncia dos poderes ao
Executivo, no h paz social.
e) procurou salientar que a sociedade industrial somente se
desenvolver a partir de minucioso planejamento econmico.

* Revolues inglesas (sculo XVII)


- Puritana (1649) : Liderada por Oliver Crowmell, foi responsvel
pelo fim do absolutismo e abriu caminho para a industrializao;
- Gloriosa (1688): Liderada por Guilherme de Orange ,impediu o
retorno do absolutismo e estabeleceu uma monarquia
parlamentarista (Declarao de Direitos).
* Revoluo industrial (sculo XVIII)
- Significado histrico : substituiu a manufatura pela produo
mecanizada e consolidou o capitalismo;
- Pr-condies:
* Fim do absolutismo;
* Consolidao da burguesia;
* Disponibilidade de capitais;
* Grandes reservas de carvo;
* Oferta de mo de obra (cercamentos);
* Rede fluvial altamente navegvel;
* Advento da mquina a vapor (1759,James Watt) .
*
*
*
*
*

Consequncias:
Diviso do trabalho;
Produo em srie;
Formao da classe operria (proletariado);
Inglaterra como potncia hegemnica.
Revoluo americana (1776)

- Independncia dos Estados Unidos;


* Causas:
- Guerra dos 7 anos (1756-1763): Inglaterra e Frana disputam o
atual territrio do Canad;
- Arrocho fiscal sobre as colnias;
- Lei do Ch (1773) : A Inglaterra decreta o monoplio da
comercializao do produto;
- Leis intolerveis : interdio do Porto de Boston;
- Congresso Continental de Filadlfia (1775): emancipao das
colnias; Guerra de independncia;
-Tratado
de
Versalhes
(1783):
Reconhecimento
da
independncia.
*
-

Revoluo Francesa (1789)


Fim do absolutismo e dos privilgios de classe;
Queda da Bastilha (14 de julho de 1789);
Declarao dos Direitos do Homem e do Cidado;
Execuo do rei Lus XVI (1793);

Girondinos X Jacobinos:

Terror ( Robespierre) : radicalizao;


Golpe 18 Brumrio: Napoleo Bonaparte.

2) A consolidao do processo de industrializao na Inglaterra,


ocorrida na primeira metade do sculo XIX, relaciona-se
corretamente com a (o):
a) extino do processo de cercamento dos campos ("enclousures").
b) supremacia da ideologia liberal.
c) fortalecimento da produo atravs das corporaes de ofcio.
d) surgimento do capitalismo financeiro e oligopolista.
e) xodo da mo de obra especializada das cidades para o campo.
3) No incio do sculo XIX, a independncia da Amrica Espanhola
ocorreu num contexto poltico internacional marcado por fatos.
Dentre os fatos que favoreceram a independncia da Amrica
Espanhola, podemos mencionar.
a) A Revoluo Industrial Espanhola.
b) A derrota dos americanos na guerra de independncia dos
Estados Unidos.
c) O Despotismo Esclarecido.
d) O triunfo do absolutismo de direito divino na Espanha.
e) As guerras napolenicas.

- Qual a importncia da Revoluo Francesa na construo das


democracias contemporneas ?
_______________________________________________________
_______________________________________________________

SECRETARIA DE ESTADO DA EDUCAO

77

QUMICA
HISTRIA

AULA 7 :

O BRASIL IMPRIO

EXERCCIOS DE FIXAO

a) PRIMEIRO REINADO ( 1823-1831)


- Constituio de 1824:
* Outorgada por D .Pedro I
* Excessiva centralizao (poder moderador)
* Voto censitrio (baseado na renda do eleitor)
* Senado vitalcio
*Padroado (subordinao do clero ao Estado)
- Confederao do Equador:
* Movimento separatista e republicano
*Provncias do Nordeste lideradas por Pernambuco
* Represso (fuzilamento do Frei Caneca)
* Abdicao (crise poltica)
b) PERODO REGENCIAL (1831-1840)
* Criao
da
Guarda
Nacional( brao
armado
dos
Coronis);Ato Adicional de 1834 (descentralizao); Revoltas
regenciais:
* Cabanagem (PA,1835-40): Rebelio popular em meio s disputas
entre as elites locais, as quais estavam interessadas na reduo dos
impostos. Foi proclamada a independncia do Par. A represso
matou mais de 30 mil pessoas.
* Revolta dos Mals(BA,1835): Rebelio de escravos oriundos do
Sudo (muulmanos) na tentativa de fundar um Estado teocrtico
em Salvador. Rapidamente sufocada.
* Balaiada (MA,1838-1841) :Revolta popular que envolveu negros
escravos, brancos pobres e mestios contra a misria e a
excluso. O ento coronel Lus Alves de Lima e Silva foi nomeado
Presidente da Provncia e debelou a rebelio.
*Sabinada(BA,1837-1838): Movimento liberal envolvendo segmentos
da classe mdia urbana liderado pelo mdico Francisco Sabino
lvares contra a centralizao poltica. Proclamada a Repblica
Bahiense (de carter transitrio),logo dissolvida pelo governo
regencial.
* Guerra dos Farrapos ( RS,1835-1845) : Envolveu gachos e
catarinenses
e
foi
motivada pelo
descontentamento dos
estancieiros com a poltica de tributao sobre o charque e em
defesa de maior autonomia para as provncias. considerada a mais
longa guerra civil do Brasil.
c) SEGUNDO REINADO (1840-1889)
- Partidos polticos: liberal e conservador
- Parlamentarismo s avessas (controle do Imperador)
- Expanso da economia cafeeira (Oeste paulista)
- Fim do trfico negreiro ( Lei Eusbio de Queirs, 1850)
- Novas formas de trabalho (parceria e assalariamento)
Lei de Terras (1850, restringindo o acesso propriedade
agrria)
- Surto industrial e modernizao
Revoluo Praieira (PE,1848) : Inspirada nos movimentos
revolucionrios da Europa, o movimento de carter liberal teve
como causa as difceis condies econmicas e sociais da regio. O
Manifesto ao Mundo tinha como reivindicaes o voto universal
masculino, a nacionalizao do comrcio e a liberdade de imprensa.
-Guerra do Paraguai (1861-1865);
-Crise ( questo abolicionista, militar e religiosa).

1) Processo poltico de emancipao do Brasil desenvolveu-se


dentro de condies bastante especiais, dentre as quais correto
assinalar:
a) a presena de D. Pedro I, como regente do trono, estabelecia a
possibilidade de uma separao entre Portugal e Brasil, sem,
contudo, romper radicalmente com o regime monrquico.
b) as primeiras notcias chegadas ao Brasil dos acontecimentos do
Porto deflagraram, em todas as provncias brasileiras, movimentos
de repdio revoluo lusa, formando-se Juntas Constitucionais.
c) a Revoluo do Porto, fundamentada em ideias liberais, tinha
entre seus objetivos a reforma constitucional portuguesa e a
emancipao poltica das suas colnias, entre elas, o Brasil.
d) nas Juntas Constitucionais formadas por brasileiros e
portugueses, nas quais os brasileiros eram em maior nmero, havia
a firme deciso de no se acatarem as resolues tomadas pelas
cortes em Lisboa, o que contrariava os interesses lusos.
e) Com relao ao Brasil, os revolucionrios portugueses do Porto,
mantinham a coerncia com os postulados liberais, mostrando-se
intransigentes defensores da emancipao poltica brasileira.

2) O Reinado de D. Pedro II foi marcado por aes que


demonstravam o interesse da Monarquia em estimular o crescimento
intelectual da nao. Considerando-se essa informao e outros
conhecimentos sobre o assunto, CORRETO afirmar que, entre as
principais
aes
nesse
sentido,
se
destaca:
a) a criao de instituies de ensino como a Escola de Minas de
Ouro Preto, que, embora voltada formao das elites, cumpriu
importante funo na pesquisa e na prospeco de minerais.
b) a fundao do Museu da Inconfidncia um museu-escola , que
representou um ato de reparao aos mineiros pela perda, no
processo de devassa da Inconfidncia Mineira, de seus ilustres
intelectuais.
c) o financiamento da vinda da Misso Artstica Francesa, que se
props estimular e ensinar as mais diversas formas de expresso
artstica
a
artistas
brasileiros.
d) o resgate e proteo do Barroco Mineiro e, consequentemente,
valorizao da produo cultural brasileira.

- Relacione o desenvolvimento da cafeicultura com o surto industrial


do Brasil Imprio:
_______________________________________________________

_______________________________________________________
_______________________________________________________
_______________________________________________________
_______________________________________________________

SECRETARIA DE ESTADO DA EDUCAO

78

QUMICA
HISTRIA

AULA 8 : AS REVOLUES SOCIALISTAS

EXERCCIOS DE FIXAO

a) Russa (1917)
- Significado histrico: Implantou o primeiro Estado socialista da
Histria.
* Inspirada no marxismo.
* Causas :
Despotismo monrquico (czarismo); pobreza dos
camponeses e operrios; derrota da Rssia para o Japo na disputa
pelo norte da China(1904),agravando a situao econmica e social.
* Bolcheviques X Mencheviques (diviso da oposio ao czarismo)
* Revoluo Branca (fevereiro,1917) : A desastrosa participao
russa na Primeira Guerra Mundial leva deposio do Czar Nicolau
II e instalao de um governo de orientao liberal (menchevique)
liderado por Alexandre Kerensky.
* Revoluo Vermelha (outubro,1917) : Os Sovietes (conselhos
operrios organizados e liderados pelos bolcheviques) exigem a
implantao de um governo representativo do povo. O governo
provisrio de Kerensky deposto e Lnin assume sob o lema Paz,
Terra e Po .

1) A Revoluo Socialista na Rssia, em 1917, foi um dos


acontecimentos mais significativos do sculo XX, uma vez que
colocou em xeque a ordem socioeconmica capitalista. Sobre o
desencadeamento do processo revolucionrio, correto afirmar que:
a) os mencheviques tiveram um papel fundamental no processo
revolucionrio por defenderem a implantao ditadura do
proletariado.
b) os bolcheviques representavam a ala mais conservadora dos
socialistas, sendo derrotados, pelos mencheviques , nas jornadas de
outubro.
c) foi realimentado pela participao da Rssia na Primeira Guerra
Mundial, o que desencadeou uma srie de greves e revoltas
populares em razo da crise de abastecimento de alimentos.
d) foi liderada por Stalin, a partir de outubro, que estabeleceu a tese
da necessidade da revoluo em um s pas, em oposio a Trotsky,
lder do exrcito vermelho.
e) o Partido Comunista conseguiu superar os conflitos que existiam
no seu interior quando estabeleceu a Nova Poltica Econmica que
representava os interesses dos setores mais conservadores.

- Governo de Lnin (1917-1924) :


* A Rssia retira-se da guerra (Tratado de Brest-Litovsk)
*
Estatizao dos meios de produo (comunismo de guerra)
*Exrcito Vermelho (liderado por Trotski) consolida o poder
bolchevique na guerra civil (1921)
* Tratado das Nacionalidades (concesso de autonomia s
Repblicas)
* Criao da URSS (Unio das Repblicas Socialistas Soviticas
* NEP (Nova Poltica Econmica) : concesses livre iniciativa
- Governo de Stlin (1924-

):

*
Implantao da ditadura (expurgos,prises,deportaes e
assassinatos)
* Militarismo e expansionismo
* Planos Quinquenais
* Industrializao e infraestrutura
b) Chinesa (1949)
Revoluo Chinesa:
* Antecedentes - Repblica (Sun Yat-sen ), " os senhores da
guerra ";
* Nacionalismo Chins : Kuomintang ;
* Comunismo Chins : Mao Ts-tung ;
*Grande Marcha ;
*Guerra Civil ;
*Diviso : Nacionalista (Taiwan) e Comunista(Repblica Popular)

2) Em abril de 1917, o lder bolchevique Lenin, exilado em Zurique


(Sua), voltou Rssia lanando as Teses de Abril. Nesse programa
poltico incorreto afirmar que Lenin propunha a/o:
a) formao de uma Repblica de sovietes;
b) concesso defesa nacional, dando total apoio ao governo
provisrio;
c) nacionalizao dos bancos e das propriedades privadas;
d) reconstituio da Internacional;
e) controle da produo pelos operrios.

c) Cubana ( 1959)
*
Causas : Ditadura (Fulgncio Batista); ingerncia dos EUA
(anulando a soberania do Pas); profundas desigualdades sociais.
*
Tentativa fracassada de derrubada do governo: assalto ao
Palcio de Moncada, 26 de julho de 1953 (origem da guerrilha
marxista do MR-26).
*
Incio da guerrilha Derrubada de Fulgncio Batista pelo
MR-26. Fidel Castro assume o governo e estabelece o socialismo.
- Dos anos 80 para c o cenrio internacional mudou muito. A partir
dessa perspectiva, quais foram as transformaes ocorridas na
URSS, China e CUBA ?
_______________________________________________________
_______________________________________________________
_______________________________________________________
_______________________________________________________
_______________________________________________________

SECRETARIA DE ESTADO DA EDUCAO

79

QUMICA
HISTRIA

AULA 9 : O NEOCOLONIALISMO

(IMPERIALISMO)

EXERCCIOS DE FIXAO

- Significado histrico: A segunda revoluo industrial gerou


necessidades para as potncias europeias tais como novos
mercados consumidores; reas de investimento e enquadramento
populacional e demanda por mo de obra. Da a corrida imperialista
em direo aos continentes africano e asitico, acirrando as
rivalidades j existentes e abrindo caminho para o armamentismo e a
guerra.
- Pases pioneiros: Blgica (domnio sobre o Congo) e Frana
(conquista da Arglia).
- Conferncia de Berlim (1885):
* Partilha da frica;
* Fim da escravido no continente negro;
* Bacia do Congo transformada em rea de livre navegao.

1) A "partilha do mundo" (1870 -1914) resultou do interesse das


potncias capitalistas europeias em:
a) investir seus capitais excedentes nas colnias, obter mercados
fornecedores de matrias-primas e reservar mercados para seus
produtos industrializados;
b) desenvolver a produo de gneros alimentcios nas colnias,
visando suprir as deficincias de gros existentes na Europa na
virada do sculo;
c) buscar "reas novas" para a emigrao, uma vez que a presso
demogrfica na Europa exigia uma soluo para o problema;
d) promover o desenvolvimento das colnias atravs da aplicao de
capitais excedentes em programas sociais e educacionais;
e) favorecer a atuao dos missionrios catlicos junto aos pagos e
assegurar a livre concorrncia comercial.

- Pases mais beneficiados: Inglaterra e Frana;


- Ideologia da misso civilizatria;
- Darwinismo social
- Guerras e conflitos:
* Revolta dos Cipaios( ndia,1857) : nacionalistas hindus contra o
protetorado ingls;
* A ndia torna-se colnia do Imprio britnico;
* A rainha Vitria coroada imperatriz da ndia;
* Guerra do pio (China,1839) : O pio produzido na ndia era
contrabandeado para a China;
- Governo chins reage e as potncias invadem a China;
-A China se v obrigada a abrir 11 portos para o comrcio exterior;
* Guerra dos Beres ( frica do Sul, 1880) : Ops colonos de
origem holandesa (chamados beres), ao exrcito britnico, que
pretendia se apoderar das minas de diamante e ouro encontradas
naquele territrio.
* Em consequncia da guerra, os beres ficaram sob o domnio
britnico, com a promessa de autogoverno.
- Herana do neocolonialismo:
a)Instabilidade poltica representada por conflitos intertribais e
sucessivos golpes militares em diversos pases da frica;
b)Desertificao e fome;
c) Doenas, a exemplo da AIDS e causada pelo vrus ebola;
d)Atraso econmico e social;
conflitos entre etnias rivais; doenas ;fome; atraso econmico;
- Os EUA foraram a abertura dos portos japoneses (1853);
*
*
*
*

Restaurao Meiji: desenvolvimento industrial e modernizao;


Abolio do sistema feudal;
Abertura ao capital estrangeiro;
Investimento em tecnologia blica;

Domnio francs sobre o territrio da Indochina


correspondente s Repblicas do Vietn, Camboja e Laos;

2) A industrializao acelerada de diversos pases, ao longo do


sculo XIX, alterou o equilbrio e a dinmica das relaes
internacionais. Com a Segunda Revoluo Industrial emergiu o
Imperialismo, cuja caracterstica marcante foi o(a):
a) substituio das intervenes militares pelo uso da diplomacia
internacional.
b) busca de novos mercados consumidores para as manufaturas e
os capitais excedentes dos pases industrializados.
c) manuteno da autonomia administrativa e dos governos nativos
nas reas conquistadas.
d) procura de especiarias, ouro e produtos tropicais inexistentes na
Europa.
e) transferncia de tecnologia, estimulada por uma poltica no
intervencionista.

(hoje

- O apartheid na frica do Sul e o genocdio ocorrido em Ruanda


nos anos 90 guardam estreita relao com o imperialismo europeu
no continente negro. Explique:
_______________________________________________________
_______________________________________________________
_______________________________________________________
_______________________________________________________
_______________________________________________________

SECRETARIA DE ESTADO DA EDUCAO

80

QUMICA
HISTRIA

Aula 10: A PRIMEIRA GUERRA MUNDIAL

EXERCCIOS DE FIXAO

* Causas :
- Disputas territoriais
- Nacionalismos:
- revanchismo francs: herana da guerra franco-prussiana(1871);
- pan-eslavismo: ideia da Grande Srvia;
- pan-germanismo: ideia da Grande Alemanha;
- paz armada : corrida armamentista;
* Focos de tenso:
a) Alemanha X Inglaterra : corrida navalista;
b)Alemanha X Rssia : construo da ferrovia Berlim-Bagd;
c)ustria X Srvia: questo balcnica (anexao da Bsnia pelo
Imprio Austro-hngaro.
*Alianas militares :
- Trplice Aliana ( Alemanha, ustria e Itlia)
- Trplice Entente ( Inglaterra, Frana e Rssia)

1) "Foi em 1994 que acabou o sculo XIX (...) De 1815 a 1914, a


Europa (...) desfrutara um sculo de paz (...) Nesse sculo, a
burguesia pde consolidar o seu poder (...) E o imperialismo
colonialista ia bem, obrigado, na frica e sia. A guerra de 1914 caiu
como uma bomba NESTE (PARASO)."
A Primeira Guerra Mundial descortinou uma srie de conflitos
camuflados neste "paraso" tais como:
a) a luta pelas terras conquistadas na Amrica e a manuteno do
trfico de escravos.
b) a disputa de mercados mundiais pelas naes europeias
imperialistas como a Inglaterra, Alemanha e Frana e a opresso aos
movimentos nacionalistas na frica e na sia.
c) a difuso do movimento socialista em pases como a Inglaterra e
Frana com o advento da Revoluo Russa.
d) a disputa dos mercados consumidores europeus pelas naes
independentes da frica e da sia.
e) a luta dos americanos e brasileiros pelo controle dos mercados
fornecedores de matrias-primas japoneses e africanos.

*1917: ano decisivo para os rumos do conflito:


- Os EUA entram no conflito em favor da Trplice Entente
-A Rssia, em guerra civil, sai do conflito e assina a Paz de BrestLitovski com a Alemanha;
d) Consequncias:
- Desmembramento dos Imprios Austro-Hngaro e Otomano;
- Surgimento de novas Naes, a exemplo da ustria, Hungria,
Polnia, Iugoslvia e Tchecoslovquia
- Criao da Liga das Naes (organismo destinado a manter o
equilbrio internacional e zelar pela soberania dos povos). No
contou com a participao dos EUA, Rssia e Alemanha.
- Emergncia dos EUA como potncia mundial.
*
Tratado de Versalhes (1919, a Alemanha foi obrigada a pagar
pesada indenizao aos vencedores, perdeu territrios e foi
desmilitarizada.
- A primeira Guerra Mundial trouxe o emprego de armas qumicas
(proibidas pela Conveno de Genebra).Os alemes foram os
primeiros a utilizar gs cloro contra tropas inimigas.
A guerra provocou a morte de mais de 8 milhes de pessoas e
deixou 21 milhes de feridos.

- Para uma boa parte dos historiadores, a Segunda Guerra Mundial


tem sua origens ainda com o final da Primeira. Como explicar essa
relao ?

2) Dentre as causas da Primeira Grande Guerra, destaca-se a


questo balcnica, que pode ser associada:
a) formao de novas nacionalidades, como a Iugoslava sob a
tutela da Alemanha.
b) s disputas coloniais na sia e na frica entre a Frana e a
Inglaterra.
c) ao interesse russo em abrir os estreitos de Bsforo e Dardanelos,
o nacionalismo eslavo e ao temor austraco quanto formao da
Grande Srvia.
d) s desavenas entre o Imprio Austro-Hngaro e a Inglaterra
ligadas anexao da Bsnia-Herzegovina.
e) ao assassinato do Prncipe Herdeiro, Francisco Ferdinando, e as
questes pendentes relacionadas ao Tratado de Brest-Litowsky e o
desmembramento da ustria-Hungria.

_______________________________________________________
_______________________________________________________
_______________________________________________________
_______________________________________________________
_______________________________________________________

SECRETARIA DE ESTADO DA EDUCAO

81

QUMICA
HISTRIA

AULA 11 : A REPBLICA VELHA

EXERCCIOS DE FIXAO

- Poltica dos Governadores (1898,Campos Sales):


* Manipulao do voto (voto de cabresto e currais eleitorais);
* Recurso fraude e violncia;
* Domnio das oligarquias nos Estados;
* Hegemonia poltica de So Paulo e Minas (caf com leite).
* Comisso Verificadora de Poderes (prtica da degola ou seja no
diplomao dos candidatos eleitos sob a alegao de cometimento
de fraude);
* Auge e decadncia da cafeicultura: Convnio de Taubat (
1906, Poltica de valorizao do caf);
Industrializao; Crise
de 1929;

1) A identificao dos governos da Repblica Velha com os


interesses da economia cafeeira pode ser expressa pelo(a):
a) financiamento, atravs do Banco do Brasil, para o plantio de novas
lavouras, no Encilhamento.
b) estatizao das exportaes, com o objetivo de garantir os preos,
durante a Primeira Guerra Mundial.
c) adoo de uma poltica de valorizao, reduzindo a oferta do
produto, a partir do Convnio de Taubat.
d) controle da mo de obra camponesa e apoio imigrao, com a
Lei Adolfo Gordo.
e) iseno de tributos assegurada no programa de estabilizao de
Campos Sales.

* Movimentos sociais :
- Coronelismo
- Cangao (banditismo social)
* Guerra de Canudos (BA,1893-1897): Revolta sertaneja liderada
pelo beato Antonio Conselheiro contra a excluso social;
* Revolta da Vacina (RJ,1904) : Contra a poltica de urbanizao e
higienizao da capital federal;
* Revolta da Chibata( RJ,1910): Contra os castigos fsicos na
Marinha. Liderana do marinheiro Joo Cndido, o Almirante
Negro;
* Guerra do Contestado(limite entre o Paran e Santa Catarina,
1912-1916): Movimento liderado pelo monge Jos Maria contra a
expulso de camponeses na regio.
* Tenentismo: Movimento de jovens oficiais contra a corrupo do
governo, as fraudes eleitorais, o predomnio poltico das oligarquias e
as dificuldades enfrentadas pelo exrcito.
As principais revoltas tenentistas foram:
* Revolta dos 18 do Forte de Copacabana (1922);
*Revolta Paulista de 1924;
* Coluna Prestes (1925-1927)
* Revoluo de 1930: Fim do caf com leite e ascenso de Getlio
Vargas.

2) O perodo de 1900 a 1930, identificado no processo histrico


brasileiro como Repblica Velha, teve por trao marcante:
a) o fortalecimento da burguesia mercantil, que se utilizou do Estado
como instrumento coordenador do desenvolvimento.
b) a abertura para o capital estrangeiro, principal alavanca do rpido
desenvolvimento da regio amaznica.
c) a modificao da composio social dos grandes centros urbanos,
com a transferncia de mo de obra do Centro-Sul para reas do
Nordeste.
d) o pleno enquadramento do Brasil s exigncias do capitalismo
ingls, ao qual o pas se mantinha cada vez mais atrelado.
e) o predomnio das oligarquias dos grandes Estados, que
procuravam assegurar a supremacia do setor agrrio-exportador.

- Diante do meu charuto muito doutor de lei ficou menor que ano
de circo de cavalinho. (Ponciano de Azeredo Furtado, personagem
criado por Jos Cndido de Carvalho, em O coronel e o lobisomem.)
Tomando como referncia o texto, identifique o fenmeno nele
retratado e explique suas razes e permanncias.
_______________________________________________________
_______________________________________________________
_______________________________________________________
_______________________________________________________
_______________________________________________________
_______________________________________________________
______________________________________________________

SECRETARIA DE ESTADO DA EDUCAO

82

QUMICA
HISTRIA

Aula 12 : OS TOTALITARISMOS

EXERCCIOS DE FIXAO

a) Causas :
1) Entre Mussolini e Hitler, h em seus programas, pontos em
comum, como a:
a) mobilizao contnua das massas atravs de apelos nacionalistas
e a manuteno de uma poltica de apoio aos socialistas.
b) ideia de centralizao administrativa e o fortalecimento dos
mercados de troca, principalmente ingleses.
c) organizao militar da juventude e a no interveno do Estado na
vida econmica e poltica.
d) necessidade de fortalecimento do Estado e a adoo do
corporativismo como base da reestruturao das relaes sociais.
e) produo de um ideal blico que acentuasse o gnio militar dos
fascistas e a incorporao das minorias tnicas ao Estado com plena
liberdade.

* Crise da democracia liberal;


* Efeitos da Grande Depresso;
* Intensificao dos protestos operrios.
- Caractersticas gerais:
* Unipartidarismo;
* Militarismo;
* Anticomunismo;
* Propaganda ideolgica;
* Censura;
* Represso poltica.
b) Denominaes:
* Fascismo (Itlia, sob a liderana de Benito Mussolin):
-

Criao dos Esquadres de Combate;


Surgimento do Partido Fascista;
Marcha sobre Roma(1922);
Mussolini torna-se Primeiro-Ministro;
A ditadura fascista implantada.

* Nazismo (Alemanha,Adolf Hitler):


-

Fundao do Partido Nazista (1921);


Hitler torna-se chanceler (1933);
Hitler declara-se Fuhrer (1934);
Implantao da ditadura nazista.

* Franquismo (Espanha, General Francisco Franco):


- Guerra Civil Espanhola (1936-1939): Republicanos (Frente Popular)
X Nacionalistas de extrema-direita (Falange);
- Apoio dos nazistas direita espanhola: bombardeio de Guernica
(Provncia Basca,1939).
*Salazarismo (Portugal, Oliveira Salazar) :
- Estado Novo o nome do regime poltico autoritrio, autocrata e
corporativista de Estado que vigorou em Portugal durante 41 anos
sem interrupo, desde a aprovao da Constituio de 1933 at
sua queda pela Revoluo de 25 de Abril de 1974 (Revoluo dos
Cravos).
c) Caractersticas especficas:

2) A soluo americana para a crise de 1929 caracteriza-se como:


a) o processo de busca de alternativas socialistas para a crise do
capitalismo com a mudana de regime poltico.
b) o resultado das presses comunistas sobre o governo americano,
que acaba assumindo, como poltica, a eliminao dos interesses
privados na economia.
c) o resultado da insatisfao da sociedade americana com relao
aos princpios liberais assumidos pelos partidos de esquerda que se
vinculavam ao governo.
d) a introduo, na cultura americana, de valores europeus atravs
da incorporao de tecnologia economia americana e de
alternativas de seguridade total.
e) uma sada nacional que acentua o papel dirigente do Estado em
determinados setores econmicos, conhecida como "New Deal".

*Fascismo: corporativismo (controle dos sindicatos pelo Estado);


*Nazismo: antissemitismo ( perseguio aos judeus) e arianismo
(ideia de raa pura).

Um dos fatores que contribuiu para a ascenso dos regimes


totalitrios de direita na Europa foi a crise de 1929. O que provocou a
crise de 1929 e como os EUA, epicentro da crise, conseguiram
super-la ?
_______________________________________________________
_______________________________________________________
_______________________________________________________
_______________________________________________________
_______________________________________________________
_______________________________________________________
_______________________________________________________
_______________________________________________________

SECRETARIA DE ESTADO DA EDUCAO

83

QUMICA
HISTRIA

EXERCCIOS DE FIXAO
AULA 13 : ERA VARGAS
- Governo Provisrio (1930-34):
* Dissoluo do Congresso Nacional, das assembleias legislativas
estaduais e municipais e nomeao de interventores em substituio
aos presidentes de estado;
* Lei de Sindicalizao,1931 : controle dos sindicatos pelo Estado;
* Revoluo Constitucionalista,SP,1932 : Exigncia de convocao
da Assembleia Constituinte;
* Assembleia Constituinte (1933).
- Governo Constitucional ( 1934-37) :
* Constituio de 1934 : Inspirada na Constituio social
democrata da Repblica de Weimar (Alemanha); estabeleceu o voto
secreto ,idade mnima de 18 anos para o exerccio do voto de todo
cidado ,independente do gnero; primeiras leis trabalhistas.
* Polarizao ideolgica: AIB (Ao Integralista Brasileira, fundada
por Plnio Salgado em 1932) X ANL ( Aliana Nacional Libertadora,
fundada em 1935 e presidida por Lus Carlos Prestes);
* Intentona Comunista (1935) : Tentativa fracassada da ANL de
promover a derrubada do Governo;
* Golpe de Estado: Getlio Vargas outorga a Constituio de 1937
(polaca) e impe a ditadura do Estado Novo.

1) O regime poltico conhecido como Estado Novo implantado por


golpe do prprio Presidente Getlio Vargas, em 1937, pode ser
associado (ao):
a) radicalizao poltica do perodo representada pela Aliana
Nacional Libertadora, de orientao comunista e a Ao Integralista
Brasileira, de orientao fascista.
b) modernizao econmica do pas e seu conflito com as principais
potncias capitalistas do mundo, que tentavam lhe barrar o
desenvolvimento.
c) ascenso dos militares direo dos principais rgos pblicos,
porque j se delineava o quadro da Segunda Guerra Mundial.
d) democratizao da sociedade brasileira em decorrncia da
ascenso de novos grupos sociais como os operrios.
e) retorno das oligarquias agrrias ao poder, restaurando-se a
Federao nos mesmos moldes da Repblica Velha.

- Estado Novo (1937-45) :


* Excessiva centralizao poltica ( nomeao de interventores; uso
sistemtico do decreto lei; fim dos partidos polticos e ausncia de
eleies);
*
Intervencionismo estatal na esfera econmica (criao da
Companhia Siderrgica Nacional, Companhia Vale do Rio Doce e
Companhia Siderrgica de Volta Redonda);
*
decretada, em 1943, a CLT (Consolidao das Leis do
Trabalho): Sistematizao das leis trabalhistas;
* Forte censura : criao do DIP (Departamento de Imprensa e
Propaganda);
* Criao do DASP (Departamento Administrativo do Servio
Pblico) e dos Daspinhos (a nvel estadual);
* Criao do programa radiofnico Hora do Brasil com o objetivo de
promover o Governo.
- A crise do Estado Novo :
* Entrada do Brasil na Segunda Grande Guerra ;
* Presses da sociedade civil (Manifesto dos Mineiros);
*
Abertura poltica ( Anistia dos presos polticos,volta do
pluripartidarismo;convocao de eleies gerais, marcadas para
dezembro de 1945);
*
Queremismo :
Movimento organizado pelo PTB (Partido
Trabalhista Brasileiro) e simpatizantes de Getlio Vargas que
pretendia sua manuteno no poder.
* Getlio Vargas deposto pelos militares. O General Eurico Gaspar
Dutra,representando a coligao PSD-PTB,foi eleito Presidente da
Repblica.

2)"Batemo-nos pelo Estado Integralista. Queremos a reabilitao do


princpio de autoridade, que esta se respeite e faa respeitar-se.
Defendemos a famlia, a instituio fundamental cujos direitos mais
sagrados so proscritos pela burguesia e pelo comunismo."
Este texto, pelas ideias que defende, provvel que tenha sido
escrito por:
a) Jorge Amado
b) Carlos Drummond de Andrade
c) Mrio de Andrade
d) Oswald de Andrade
e) Plnio Salgado

- Como definir populismo ? Quais as suas maiores expresses dentro


do cenrio poltico da Amrica Latina ?
_______________________________________________________
_______________________________________________________
_______________________________________________________
_______________________________________________________
_______________________________________________________
_______________________________________________________
______________________________________________________

SECRETARIA DE ESTADO DA EDUCAO

84

QUMICA
HISTRIA

AULA 14 : A SEGUNDA GUERRA MUNDIAL

EXERCCIOS DE FIXAO

- Antecedentes :
*Anexao da ustria por Hitler (Anschluss,maro de 1938);
*Conferncia de Munique(1938): Hitler incorpora o norte da
Thecoslovquia (Sudetos) Alemanha. Pouco tempo depois, ocupa
todo o restante do territrio daTchecoslovquia;
*Pacto Nazi-Sovitico de No-Agresso (1939);
*A Polnia invadida pelas tropas nazistas e soviticas (1 de
setembro de 1939). Comea a Segunda Guerra Mundial.
-Blocos militares:
*Eixo: Alemanha,Itlia e Japo;
*Aliados: Inglaterra,Frana e EUA.
-Fatos importantes:
* Ocupao da Frana (Junho de1940) : Repblica de Vichy;
* Batalha da Inglaterra (julho a outubro de 1940): A aviao inglesa
provoca muitas baixas na Lutfwaffe(aviao alem) e na marinha
alem;

1) A ocupao da Polnia marca o incio da Segunda Guerra


Mundial. A tentativa de manter a paz a qualquer custo, como foi feito
em Munique, se revelou impossvel. Hitler no se dava por satisfeito
com a reconquista do "espao vital", queria mais e mais. Sobre a
Segunda Guerra, correto afirmar:
a) A Itlia, aliada da Alemanha desde a assinatura do Pacto de Ao,
declarou guerra Inglaterra e Frana em junho de 1940. Em
setembro do mesmo ano, a Itlia atacou o Egito e a Turquia.
b) Em 1941, tropas alems invadiram o territrio sovitico e
dominaram definitivamente Leningrado e Moscou.
c) A partir dos sucessos na frente ocidental, da invaso e conquista
da Blgica, Holanda e Frana e do recuo ingls para o outro lado do
canal, Hitler voltou sua ateno para a Polnia.
d) O sucesso definitivo alemo deveu-se sua ttica militar,
conhecida como "guerra relmpago"; essa consistia no uso de foras
motorizadas, tanques e aviao, conjugados e combinados entre si,
em uma ao defensiva.
e) A partir da declarao de guerra, feita por Inglaterra e Frana
contra a Alemanha, outros pases foram entrando no conflito, de
ambos os lados. A cada novo beligerante, a relao de foras se
alterava, e a guerra entrava em uma nova fase. Inicialmente uma
guerra europeia, estendeu-se paulatinamente sia e a frica.

*Ocupao do norte da frica pelos exrcitos nazifascistas (1941);


*Operao Barbarossa: Hitler invade a Unio Sovitica,rompendo o
Pacto de No-Agresso;
*Escalada expansionista do Japo no Pacfico: ocupao da
Manchria e leste da China,em 1931; da Indochina,em 1941;
* Ataque japons base norte-americana de Peal Harbor,no Hava;
*Batalha de Stalingrado (1943):
ofensiva sovitica contra os
nazistas; A Europa do Leste comea a ser desnazificada;
*Dia D (6 de junho de 1944): Desembarque dos aliados na costa
francesa (Normandia). A Frana libertada da ocupao nazista.
*A Alemanha assina sua rendio incondicional (abril de 1945).
*Hiroshima e Nagasaki (agosto de 1945): Os EUA lanam as bombas
atmicas sobre as duas cidades japonesas e encerram a guerra no
Pacfico.
- Consequncias:
* Holocausto: Entre os judeus,foram aproximadamente 6 milhes de
vtimas;
* Criao da ONU (Organizao das Naes Unidas);
* Diviso da Alemanha (RFA e RDA);
* Bipolaridade ideolgica (EUA X URSS) : Guerra Fria;
* Processo de descolonizao afro-asitica.

Qual era a situao da Alemanha na dcada de 20 e segundo


Hitler,o que era preciso fazer para reverter essa situao ?
_______________________________________________________
_______________________________________________________
_______________________________________________________
_______________________________________________________
______________________________________________________

SECRETARIA DE ESTADO DA EDUCAO

85

QUMICA
HISTRIA

AULA 15 : A DITADURA MILITAR


- Antecedentes:
* Crise poltica gerada com a renncia de Jnio Quadros (1961);
* Implantao do Parlamentarismo ( Ato Adicional de 1961);
Tancredo Neves eleito Primeiro Ministro;
*Plebiscito(1963): vitria do presidencialismo;
* Reformas de Base: anunciadas no comcio da Central do
Brasil(RJ), em 13 de maro de 1964;
* Marcha da Famlia com Deus pela Liberdade (SP,19 de maro):
resposta da direita contra as reformas;
* Golpe militar derruba o Presidente Joo Goulart(31 de maro de
1964). Jango parte para o exlio no Uruguai.
- Aspectos importantes :
* Atos Institucionais:
a) AI-1 (1964) : O Marechal Castelo Branco assume o comando do
Pas; o governo militar poderia cassar mandatos legislativos,
suspender os direitos polticos (por dez anos) ou afastar do servio
pblico todo aquele que pudesse ameaar a segurana nacional.
Alm disso, convocou eleies indiretas para presidente e a
extenso do mesmo cargo at o ano de 1966. Em abril daquele
mesmo ano, o novo governo divulgou uma lista com 102 polticos e
funcionrios que tiveram seus postos e direitos anulados.
b) AI-2(1965): Institui o bipartidarismo ( ARENA X MDB);
c)AI-3 (1966):Estabelece eleies indiretas para os governadores de
estados e prefeitos das capitais;
d) AI-4(1967): Convocou o Congresso Nacional para votar o projeto
da nova Constituio e estabeleceu o Colgio Eleitoral para
instrumentalizar a eleio indireta para Presidente da Repblica.
e)AI-5(1968): Fechou o Congresso Nacional; suspendeu os direitos
e garantias individuais e promoveu uma nova onda de cassaes.
* Os anos de chumbo (Governo Mdici,1969-1974):
- Institucionalizao da OBAN (Operao Bandeirantes): Tinha como
objetivo aumentar a represso e eliminar os focos guerrilheiros;
- Criados o Destacamento de Operaes e Informaes (DOI) e o
Centro de Operaes de Defesa Interna(CODI): prtica sistemtica
de tortura e assassinatos de presos polticos;
- Milagre Econmico: Crescimento econmico, endividamento
externo e concentrao de renda;
- Propaganda ufanista: Brasil, ame-o ou deixe-o .
- A distenso lenta, gradual e segura:
- Governo Geisel (1974-1979):
* Lei Falco (1976): regulamentava a propaganda eleitoral na
televiso;
* Pacote de Abril (1977): O Congresso foi fechado por 14 dias,o
mandato do presidente aumentou de 5 para 6 anos e um tero dos
senadores passaram a ser escolhidos pelo voto indireto (binicos);

* Campanha das Diretas J (1984): derrota da Emenda Dante de


Oliveira que pretendia restabelecer eleio direta j para o sucesso
de Figueiredo;
* Nas eleies indiretas de 1985, o Colgio Eleitoral elege o
candidato da Aliana Democrtica (oposio ao governo militar)
Tancredo Neves que morre antes mesmo de tomar posse na
Presidncia da Repblica.
- A Nova Repblica (Governo Sarney,1985-1989): A transio
democrtica
- Revogao da legislao autoritria;
- Legalizao dos partidos comunistas;
- Extenso do voto aos analfabetos;
- Convocao da Assembleia Constituinte;
- Promulgao da nova Constituio (5 de outubro de 1988);
- Plano Cruzado: combate inflao com congelamento de preos e
salrios;
- Realizao das primeiras eleies diretas desde o incio do regime
militar: Collor de Melo eleito Presidente;
- Em 1992, Collor sofre impeachment Itamar Franco,seu vice,
assume a Presidncia da Repblica.
- FHC ,Lula e Dilma Rousseff (dois mandatos consecutivos de
quatro anos cada um) assumiram a Presidncia da Repblica no
perodo compreendido entre 1995 a 2018.
QUAIS AS PRINCIPAIS CARACTERSTICAS DA ATUAL
CONSTITUIO BRASILEIRA NO QUE SE REFERE AOS
DIREITOS DO CIDADO ?
_______________________________________________________
_______________________________________________________
_______________________________________________________
_______________________________________________________
_______________________________________________________
EXERCCIO DE FIXAO
1) O Movimento "Diretas J", que promoveu em 1984 uma intensa
mobilizao popular a favor da eleio direta para Presidente da
Repblica, teve como resultado imediato:
a) a eleio de um governo popular e democrtico chefiado por Jos
Sarney;
b) a eleio do candidato da oposio, Tancredo Neves, pela via
indireta;
c) a primeira eleio direta do Presidente da Repblica, a primeira
em quase trinta anos, com a vitria de Fernando Collor de Mello;
d) a anticandidatura de Ulysses Guimares e a convocao da
Assembleia Nacional Constituinte;
e) a revogao dos Atos Institucionais, apesar da derrota da emenda
das Diretas.

* Revogao do AI-5 (1978);


* Mortes sob tortura do jornalista Vladimir Herzog (1975) e do
operrio Manoel Fiel Filho (1976) nas dependncias do DOI-CODI
paulista. O episdio resulta nas exoneraes do General
Ednardo Dvila Mello ( ento Comandante do II Exrcito) e do
General Sylvio Frota, Ministro do Exrcito.
- A abertura poltica :
- Governo Figueiredo (1979-1985):
a) Lei da Anistia (1979);
b) Retorno do pluripartidarismo;
* Eleies diretas para prefeitos, deputados, senadores e a primeira
eleio direta para o governo dos estados desde o incio do regime
militar;
* Atentados de direita: Riocentro (1980) e OAB (1981);

SECRETARIA DE ESTADO DA EDUCAO

86

QUMICA
HISTRIA

EXERCCIO DE FIXAO
AULA 16 : A GUERRA FRIA
1) " lgico que os EUA devem fazer o que lhes for possvel para
ajudar a promover o retorno ao poder econmico normal do mundo,
sem o que no pode haver estabilidade poltica nem garantia de
Paz."

- Antecedentes:

- As Conferncias do ps-guerra:
* Yalta (fevereiro,1945) : Diviso do mundo em reas de influncia
das superpotncias (EUA e URSS);
* Potsdam (abril,1945): Diviso da Alemanha e administrao do
territrio por um Conselho Interaliado;
* Sovietizao do leste europeu : imposio, por parte da URSS, de
regimes comunistas aos pases da Europa Oriental (formao da
denominada Cortina de Ferro);
* Doutrina Truman (1947) :criada pelos EUA para conter o avano
comunista na Europa;
* Plano Marshall (1947) :
programa de recuperao
da Europa empreendido pelos Estados Unidos aps a Segunda
Guerra Mundial. Um dos seus objetivos era conter o avano do
comunismo e estabelecer um plano de cooperao militar com a
Europa Ocidental.
- Alianas Militares:
* OTAN (Organizao do Tratado do Atlntico Norte,1949) : Aliana
militar do bloco capitalista(EUA e Europa Ocidental);
* Pacto de Varsvia (1955): Aliana militar do bloco comunista
(URSS e Europa do leste);

O Plano Marshall se constituiu


a) na principal meta da poltica externa norte-americana, que era
pacificar o Extremo Oriente.
b) num projeto de ajuda industrial aos pases da Amrica Latina.
c) num importante instrumento de expanso do comunismo na
Europa.
d) na definio da poltica externa isolacionista dos EUA, paralela
montagem do complexo industrial militar.
e) num dos meios de penetrao dos capitais norte-americanos nas
economias europeias.
2) Com o final da Segunda Guerra Mundial, os pases vitoriosos
procuraram criar vrios mecanismos internacionais que buscassem o
desenvolvimento do planeta de forma mais harmnica. dessa
poca a criao do seguinte organismo:
a) ONU- para a constituio de um exrcito internacional para pr fim
s guerras.
b) OTAN - para a desmilitarizao dos pases ocidentais e a
diminuio das zonas de conflito.
c) GATT - para a implantao de uma tarifa nica sobre os produtos
e servios internacionais.
d) UNESCO - para a melhoria da qualidade alimentar das
populaes miserveis do Terceiro Mundo.
e) FMI - para ajudar financeiramente aos pases membros, quando
em dificuldades.

- As crises :
* Bloqueio de Berlim (1949) : resultou na diviso territorial e poltica
da Alemanha (Repblica Federal da Alemanha,capitalista
e
Repblica Democrtica da Alemanha,comunista);
* Guerra da Coreia (1950) : diviso territorial e poltica da Coreia (
Coreia do Norte,comunista e Coreia do Sul,capitalista);
* Tropas russas invadem a Hungria (1956): com
sufocar um levante anticomunista;

o objetivo de

* O Muro de Berlim:
smbolo da Guerra Fria, comeou a ser
construdo em 13 de agosto de 1961,por deciso do governo da
Alemanha Oriental,para evitar a migrao em massa da populao
para o lado ocidental,capitalista.
- Crise dos msseis (1962) : Os graves conflitos entre Cuba e EUA
conduziram Fidel Castro a buscar uma progressiva aproximao com
a URSS, objetivando garantir a estabilidade do socialismo cubano.
Essa aproximao levou o governo de Cuba a permitir a instalao
de msseis nucleares soviticos na ilha ,em 1962
O Presidente Kennedy (Aliana para o Progresso) reagiu
ao fato e imps um poderoso bloqueio naval ilha de Cuba, forando
os soviticos a recuarem em seus planos. No mesmo ano da crise
dos msseis, como foi denominado o episdio, Cuba foi expulsa da
OEA (Organizao dos Estados Americanos),por influncia dos EUA,
impondo

A Guerra do Vietn (1955-1975) foi um dos desdobramentos da


Guerra Fria. Quais as razes desse conflito ocorrido no Sudeste
asitico ?
_______________________________________________________
_______________________________________________________
_______________________________________________________
_______________________________________________________
_______________________________________________________
_______________________________________________________

SECRETARIA DE ESTADO DA EDUCAO

87

QUMICA
HISTRIA

EXERCCIOS DE FIXAO
AULA 01: Meio Fsico da Terra
Hidrosfera: a esfera de gua contida na superfcie terrestre. A
gua pode estar em diferentes estados: gasoso, na atmosfera;
lquido, nos mares, rios, lagos e subterrnea; slido, nas geleiras.
Com isso, as guas que fazem parte da hidrosfera pode ser
continentais (3%) ou marinhas (97%)

Ciclo Hidrolgico: A hidrosfera apresenta um volume total


de gua constante e em contnua renovao, por meio de
um movimento cclico
3-

01. Na terra a gua existe em diferentes estados: gasoso, na


atmosfera; lquido, nos mares, rios, lagos e no subterrneo; e slido,
nas geleiras. A esse conjunto denomina-se hidrosfera, que a esfera
de gua contida na superfcie terrestre. Assim a gua apresenta um
volume total de gua constante e em contnua renovao, por meio
de um movimento cclico, chamado:
a) Ciclo das rochas
b) Ciclagem de nutrientes
c) Ciclo Hidrolgico

d) Renovao Solar
e) Renovao da gua

02. "Nos grandes centros urbanos a venda de geladeiras dobrou.


Esse o principal motivo para o aumento de consumo de
clorofluorcarbono (CFCs) no Brasil, onde o produto usado como
resfriante".
A emisso de poluentes como os CFCs est diretamente ligada a um
srio desequilbrio ecolgico que compromete a (o):
a) Camada de oznio.
b) Aumento das chuvas cidas.
c) Inverso trmica.

d) Capa de gs carbnico.
e) Efeito estufa.

03. O conjunto de processos que ocasionam a desagregao e a


decomposio das rochas e dos minerais denominado de :
Atmosfera: A atmosfera do nosso planeta uma mistura de
nitrognio (78%), oxignio (21%) e pequenas propores de outros
gases, como o dixido de carbono. Entre as suas funes esto:
proteo da Terra contra asteroides, filtrar raios solares (camada de
oznio), manter a temperatura da terra para proporcionar a vida e
fornecer os gases necessrios para o desenvolvimento da vida.

a) Tectonismo
b) Corroso
c) Diastrofismo

d) Intemperismo
e) Eroso

04. O relevo terrestre encontra-se em contnuo processo de evoluo


por causa da ao dos diferentes agentes modeladores, de acordo
com o meio ambiente. Considerando essa afirmao, analise as
sentenas a seguir:
I) Nos desertos, sobressai a ao da gua
II) Nas regies equatoriais, tm atuao predominante as chuvas
III) Nas regies de elevada altitude e latitude, o trabalho do gelo o
mais relevante
IV) Nas reas tropicais, a gua e o gelo so os principais agentes
modeladores.
a)I

b) II

c)II e III

d)I e III

e)II e IV

05. A parte da superfcie terrestre coberta por geleiras e que est


permanentemente congelada denominada de
Litosfera: uma camada slida da terra composta de rochas e
fragmentos de rochas que formam os mais variados tipos de relevo.
As rochas so divididas em sedimentares (que se formam pelo
processo de sedimentao), gneas ou magmticas (que se originam
da solidificao da lava ou do magma) e metamrficas (que surgem
da transformao fsico-qumica de rochas preexistentes).

Fatores
endgenos: Essa camada se divide em vrias placas
tectnicas que se movimentam e interagem entre si,
proporcionando transformaes no relevo

Fatores
exgenos: O clima, a gua e os seres vivos tambm
modificam os aspectos externos da litosfera atravs do
intemperismo e eroso.
Obs.: Intemperismo:______________________________________

06. A biodiversidade constitui uma das propriedades fundamentais e


tem sido muito debatida em reunies e fruns internacionais para sua
proteo e preservao. Portanto podemos considerar como
biodiversidade
a) conjunto de um nico tipo de espcie em determinado ecossistema.
b) quantidade de pessoas que existe distribuda no planeta.
c) a quantidade de biomassa de uma determinada espcie.
d) a variedade de espcies animais e vegetais nos ecossistemas.
e) a quantidade de plantas e animais da mesma espcie.
07. A regio do nosso planeta ocupada pelos seres vivos constitui
a(o):

______________________________________________________
Eroso:____________________________________________
_______________________________________________________
Biosfera: : A palavra formada por Bio= vida e Esfera= camada,
espao; sendo assim a biosfera o espao que possui vida na Terra.
Esse termo est relacionado com os fatores abiticos (no possui
vida) do nosso planeta, como a Hidrosfera, Litosfera e Atmosfera. A
biosfera se estende desde as profundezas dos oceanos at a
troposfera, camada inferior da atmosfera.

SECRETARIA DE ESTADO DA EDUCAO

a) Litosfera
b) Criosfera
c) Biosfera
d) Hidrosfera
e) Atmosfera

88

a) Biosfera
b) Biomassa
c) Biologia
d) Bioma
e) Ecossistema

QUMICA
GEOGRAFIA

AULA 02: Litosfera: Movimentos Tectnicos

EXERCCIOS DE FIXAO

O grande avano no conhecimento sobre o interior da Terra


surgiu com a sismologia a cincia que estuda os terremotos -, com
a descoberta do ncleo da Terra pelo gelogo irlands Richard D.
Oldham.

01. O tectonismo, tambm conhecido por diastrofismo, consiste em


movimentos decorrentes de presses vindas do interior da Terra,
agindo na crosta terrestre. Quando as presses so verticais, os
blocos continentais sofrem levantamentos, abaixamentos ou sofrem
fraturas ou falhas. Quando as presses so horizontais, so
formados dobramentos ou enrugamentos que do origem s
montanhas. As consequncias do tectonismo podem ser vrias,
como por exemplo a formao de bacias ocenicas, continentes,
plats e cadeias de montanhas.
Qual alternativa melhor contempla o texto acima?
a) Teoria das mltiplas faces do relevo
b) Teoria da Deriva continental
c) Teoria dos mundos
d) Fenmenos atribudos a evoluo social do homem
e) Fenmenos naturais que transformam os ambientes geofsicos do
planeta.

A partir dos avanos nos estudos sobre o interior do nosso


planeta, ficou claro que o a Terra dinmica e est em continua
transformao, resultado de processos que atuam em escala temporal
de milhares, milhes e bilhes de anos e envolve continentes, crosta e
manto. As mudanas podem ser internas (endgenas) e externas
(exgenas), em relao as mudanas internas, os movimentos
tectnicos so os mais relevantes e dividem-se em:

__________________________________________

__________________________________________

__________________________________________

02. Atualmente existem seis continentes, sendo eles: Amrica, frica,


sia, Oceania, Europa e Antrtica. H cerca de 200 milhoes de anos
atrs, todos esses continentes estavam agrupados em uma nica
massa continental chamada de Pangeia. Com base nessas
informaes podemos dizer que esta teoria de :
a) Yves Lacoste
b) Galileu Galilei
c) Alfred Wegner

d) Isaac Newton
e) Milton Santos

03. A figura a seguir representa diversos aspectos da crosta


terrestre, segundo a tica da Tectnica de Placas. A anlise dessa
figura e os conhecimentos sobre esse assunto permitem concluir que:

Sempre que houver contato entre as placas poder ocorrer


abalos ssmicos
Abalos Sismico:__________________________________________
_______________________________________________________
Terremoto:______________________________________________
_______________________________________________________
Tsunami:________________________________________________
_______________________________________________________
Portanto, nos limites entre as placas que se encontra a mais
intensa atividade geolgica do planeta vulces ativos, falhas e
abalos ssmicos frequentes, soerguimento de cadeias montanhosas e
formao e destruio de placas e crosta.
Obs.: O motor que move as placas tectnicas a conveco do
manto. A astenosfera e a litosfera esto mecanicamente relacionadas
de tal modo que, quando a astenosfera se move, a litosfera acoplada a
ela se move tambm.
Teoria da Deriva Continental
A teoria surge a partir das ideias do alemo Alfred Wegener, onde
ele buscava explicar o encaixe das linhas das costas da Amrica do
Sul e da frica. Segundo Wegener todos os continentes poderiam ter
estado juntos, no passado, como num quebra-cabea gigante,
formando um nico supercontinente, que ele denominou de Pangea.
Posteriormente a Pangea teria se fragmentado, dando origem aos
continentes e oceanos que conhecemos hoje. Atualmente, devido aos
avanos cientificos, sabemos que este processo de fragmentao e
deslocamento que continua ativo, iniciou-se h cerca de 230 milhes
de anos, qando os dinossauros comearam seu reino.

SECRETARIA DE ESTADO DA EDUCAO

89

( ) a separao de placas pode ocasionar a formao de grandes


rupturas na crosta, quando na placa ocenica pode originar as
dorsais meso-ocenicas
( ) o mecanismo fsico das correntes de conveco do manto uma
das causas mais importantes da separao das placas litosfricas.
(
) a formao de cadeias orogenticas se d exatamente nas
reas em que duas placas litosfricas se afastam.
(
) as reas ssmicas acontecem no ponto de divergncia das
placas tectnicas
04. Com o desenvolvimento da Teoria da Tectnica de Placas,
fenmenos como a formao das cadeias montanhosas e das fossas
submarinas foram melhor compreendidos.
Com isso, sabe-se que a Cordilheira dos Andes se encontra em uma
regio da crosta terrestre que:
a) apresenta uma rea de coliso de placas tectnicas.
b) no h o contato da placas tectnicas.
c) se situa em uma rea de expanso do assoalho ocenico.
d) apresenta uma rea falhada pela formao de uma dorsal
ocenica.
e) coincide com limites divergentes de placas tectnicas.

QUMICA
GEOGRAFIA

AULA 03: Tipos de vegetao, ecossistema e biomas

EXERCCIOS DE FIXAO

Ecossistema: o conjunto de interaes dinmicas que


envolvem os seres vivos e os no vivos de um determinado
ambiente.
Bioma: ____________________________________________

_______________________________________________________
_______________________________________________________
Espcies vegetais dos mais diferentes tamanhos e
caractersticas, desde microscpicas, como algas e musgos, at as
imensas rvores, habitam os diversos ambientes terrestres. A
associao de plantas em um determinado ambiente, com clima,
relevo e solo especfico, forma as paisagens naturais.
O domnio das florestas
So reas recobertas por florestas no planeta e podem ser
encontradas tanto em climas quentes como em climas frios.
Floresta boreal de confera: Predomina espcies aciculifoliadas
(folhas em forma de agulha) e com copa em forma de cone, como o
pinheiro. Desenvolve-se em clima frio e altas latitudes, onde o
inverno rigoroso e a elevada amplitude trmica limitam a variedade
de espcies.
Floresta temperada: Localiza-se em clima temperado com
precipitao abundante. Suas rvores, como o carvalho, o cedro e as
sequoias, so caduciflias, ou seja, perdem as folhas no outono e no
inverno.
Florestas equatoriais e tropicais: Na regio equatorial, com alta
precipitao, surge uma vegetao perene, exuberante e latifoliada
que se adapta muito bem umidade (espcies hidrfilas). Essa
vegetao abriga milhares de espcies que coexistem no mesmo
espao e dependem umas das outras.

01. Que bioma brasileiro se assemelha com a Savana?


a) Floresta Amaznica
b) Cerrado
c) Pampas
d) Mata de Araucrias
e) Vegetao Litornea
02. Vegetao tpica de regies costeiras, sendo uma rea de
encontro das guas do mar com as guas doces dos rios. A principal
espcie encontrada nesse bioma o caranguejo. Essas
caractersticas so do:
a) Cerrado
b) Mata de Cocais
c) Mangue
d) Caatinga
e) Pantanal
03. As formaes vegetais, observadas na superfcie terrestre,
sofrem influncias diretas e indiretas de numerosos fatores
estticos e dinmicos, demonstrando a interdependncia entre os
elementos biolgicos, climticos e geomorfolgicos. Com relao a
esse tema, julgue as alternativas a seguir:
I ( ) a altitude e a latitude so dois fatores estticos que influenciam
consideravelmente a distribuio das espcies vegetais sobre a
superfcie terrestre, propiciando uma zonao fitogeogrfica.
II. ( ) nas zonas frias, a vegetao de tundra grandemente
submetida influncia da luz; as plantas tm que se reproduzir muito
rapidamente, produzindo, portanto, um grande nmero de sementes.
III. ( ) as florestas pluviais so tpicas de ambientes quentes e
midos, geralmente sem expressivas variaes de umidade; os solos
nesses ambientes so repletos de hmus.

Obs.: Amplitude trmica: ___________________________________


_______________________________________________________
rvores latifoliada: ________________________________________

IV. ( ) as pradarias, que dominam em ambientes temperados, de


precipitaes reduzidas, caracterizam-se pelas gramneas altas, que
morrem na superfcie no perodo de inverno.

_______________________________________________________

04. Leia o texto a seguir com ateno:

O domnio da vegetao arbustiva e herbcea

Trata-se de um dos mais esplendorosos ecossistemas do planeta.


Desenvolvendo-se na zona intertropical, prximo ao equador,
apresenta elevadas e constantes taxas de umidade, sendo muito rica
e variada a sua flora e a sua fauna.
O texto trata do domnio natural da:

Em reas com menor precipitao, em diversas latitudes do


globo, aparece uma vegetao onde predomina arbustos e
vegetao rasteira.
Tundra: uma vegetao rasteira, formada de liquens e musgos,
que aparece em latitudes superiores a 60, prximo ao Crculo Polar
rtico, nos continentes americano, europeu e asitico.
Pradarias: So compostas de vegetao herbcea, com gramneas e
capim. Predomina em reas de clima temperado.
Estepes: So constitudas de arbustos espinhentos ou tufos isolados,
que deixam grande parte dos solos recobertos.
Savanas: So plantas herbceas como o capim e os arbustos.
Aparecem tambm rvores esparsas e retorcidas. A vegetao se
adapta a uma estao seca e outra mida. No Brasil recebem o
nome de cerrados.
Vegetao mediterrnea: Se adapta ao clima caracterizado por
veres quentes e secos, a vegetao arbustiva apresenta espcies
xerfilas (adaptadas a aridez).

a) Mata Atlntica
b) Savana Africana
c) Florestas Temperadas
d) Floresta Amaznica ou equatorial
e) Floresta boreal de confera
05. Nas regies de altas latitudes e prximas ao Crculo Polar rtico,
os horizontes superficiais do solo permanecem gelados. comum,
durante poucos meses do ano, a ocorrncia de um tipo de vegetao
de pequeno porte e que constitudo, basicamente, por musgos e
liquens. Assinale a alternativa que contm essa vegetao.
a) Estepe
b) Savanas
c) Tundra
d) Vegetao desrtica
e) Vegetao mediterrnea

Vegetao desrtica: As espcies xerfilas se adaptam s baixas


precipitaes, caractersticas das reas desrticas. Predomina
plantas como cactos e arbustos espinhosos com razes profundas.

SECRETARIA DE ESTADO DA EDUCAO

90

QUMICA
GEOGRAFIA

Aula 4: Tipos de clima e dinmica climtica

EXERCCIOS DE FIXAO

Dinmica climtica

01. Considere as frases abaixo:

O conjunto de caractersticas do estado da atmosfera em um


determinado momento chamado de tempo. J o clima de um local
ou regio resulta dos tipos de tempo mais comuns que ocorrem
durante anos seguidos (para classificao no mnimo 30 anos).
Quase toda a energia que atinge a Terra provm do Sol. O
recebimento de energia solar, chamado de insolao, a causa de
diversos movimentos na atmosfera.
Boa parte da energia recebia pelo Sol refletida ou irradiada
novamente para o espao. Nesse processo, ocorre o aquecimento
das terras emersas, dos oceanos e da atmosfera, alm da
evaporao de gua.
O estado trmico em que se encontra o ar (quente ou frio)
chamado de temperatura atmosfrica, que varia durante o dia num
mesmo lugar e de um lugar para o outro da Terra.

I. Far muito calor hoje em So Paulo. Em Goinia, as temperaturas

Obs.: Devido a forma esfrica da Terra, as regies prximas ao


Equador recebem os raios solares quase verticalmente durante o ano
todo. Com maior quantidade de energia (maior insolao), essa
regio tende a se aquecer mais e apresentar climas mais quentes se
no houver a influncia de outros fatores. O contrrio ocorre quanto
mais nos aproximamos dos polos.

c) II e III fazem referncia ao clima e I e IV fazem referncia ao

Solstcio: _______________________________________________
_______________________________________________________

sero mais amenas.


II. As temperaturas em todo o mundo esto cada vez maiores e vm
causando preocupaes entre os cientistas.
III. Costuma chover muito em Salvador nessa poca do ano,
melhor estarmos preparados!
IV. Li no jornal que essa semana ser chuvosa em Belm.
Com base nas afirmaes acima, possvel afirmar que:
a) Todas fazem referncia ao clima
b) I, II e III fazem referncia ao clima e IV faz referncia ao tempo.
tempo.
d) II faz referncia ao clima e I, III e IV fazem referncia ao tempo.
e) Todas fazem referncia ao tempo.
02. Nos Alpes europeus, os centros de esqui funcionam no inverno,
enquanto o vero a estao do alpinismo. Nas zonas temperadas,
o turismo de praia se restringe aos meses de vero.
MAGNOLI, D. Geografia para o Ensino Mdio. So Paulo: Atual, 2008.p.49.

Equincio:______________________________________________

As transformaes nos hbitos e prticas humanas, em parte,

______________________________________________________

devem-se ao clima, que condicionado a partir de inmeros fatores.


Tipos de clima

Assinale a alternativa que indica o fator responsvel pelas

Clima equatorial: Ocorre em reas prximas ao Equador com baixas


altitudes, apresenta temperaturas elevadas e pouca amplitude
trmica anual.

transformaes climticas indicadas no trecho acima:

Clima semirido: Situado entre os trpicos de cncer e capricrnio,


com chuvas escassas e mal distribudas e temperaturas elevadas.

c) a altitude e as consequentes diferenas de presso atmosfrica.

Clima desrtico: Tanto o quente como o frio apresenta as menores


precipitaes e grandes amplitudes trmicas diria.
Clima temperado: Apresenta inverno rigoroso e grande amplitude
trmica anual.

a) as zonas trmicas da Terra.


b) as diferenas de latitude.
d) o movimento de translao terrestre.
e) a maritimidade e a continentalidade.
03. Baseado no climograma e nas afirmativas abaixo, responda a
questo:

Clima mediterrneo: Tem veres quentes e secos e invernos amenos


e chuvosos.
Clima subtropical: Tem veres moderadamente quentes e invernos
amenos, com maiores precipitaes.
Clima polar: As temperaturas podem chegar a -35C no inverno, e no
vero no ultrapassam os 10C
Clima frio: Aparece em latitudes superiores a 45C, o clima frio de
montanha varia bastante, de acordo com a altitude: quanto mais alto,
mais frio.

I. Trata-se de um clima subtropical mido com maiores


concentraes pluviomtricas no vero, superando os mil milmetros
anuais.
II. Refere-se a um clima de baixa amplitude trmica, com grande
pluviosidade anual.
III. Nas reas em que predomina esse clima, produzido trigo, com
excelente produtividade.
IV. O clima representado denominado de Equatorial, existindo
nestas reas uma vegetao do tipo hidrfila e latifoliada,
combinando-se para a formao da Floresta Equatorial.
Esto corretas as afirmativas
A) I e III.
B) II e IV.
C) III e IV.
D) I, II e III.
E) I, II e IV.

SECRETARIA DE ESTADO DA EDUCAO

91

QUMICA
GEOGRAFIA

AULA 05: Hidrosfera e os recursos hdricos

EXERCCIOS DE FIXAO

O conjunto de todos os reservatrios de gua que existem no


planeta forma a hidrosfera est associada condensao do vapor
dagua que existia na atmosfera primitiva quando ocorreu o
esfriamento da superfcie da Terra, nos primrdios de sua histria
geolgica. Entretanto, ainda no existe uma teoria cientifica
conclusiva sobre a origem desse vapor primordial, sem o qual no
existiria nem gua nem vida no planeta.
Supostamente a hidrosfera um sistema fechado. Isso significa
que, ao longo das eras geolgicas, a quantidade total de gua do
planeta continua a mesma: ela no criada nem destruda, apenas
migra de um tipo de reservatrio como oceanos, rios e lagos,
aqufero, calotas polares, solos saturados e nuvens para outro.

01. Sobre a presena da gua no sistema terrestre, julgue as


afirmativas a seguir:
I. A disponibilidade de gua no planeta cada vez menor, o que vem
gerando profundas preocupaes para a humanidade.
II. A maior parte das guas no mundo concentra-se nos oceanos.
III. A gua doce do mundo est em maior parte presente em seu
estado slido.
IV. Os rios sempre surgem da formao de nascentes do solo, que
nada mais so do que afloramentos do lenol fretico.
V. Graas ao ciclo da gua, podemos dizer que as guas potveis
so um recurso natural renovvel e de uso contnuo.
Esto corretas as afirmativas:

Conceitos
Oceano:________________________________________________
_______________________________________________________

a) I e IV
b) II e III
c) I, II e IV
d) II, III e V
e) I, III, IV e V

Rio:____________________________________________________
Lago::__________________________________________________
Aqufero:________________________________________________
_______________________________________________________
Calotas polares: __________________________________________
Esse processo contnuo de transferncia, movido pela fora da
gravidade ou pela energia solar, chamado de ciclo hidrolgico,
podendo ocorrer em perodos que variam de alguns poucos minutos
a milhares de anos.
O tempo de residncia da gua em cada tipo de reservatrio
tambm varia enormemente: em mdia, a gua permanece dez dias
na atmosfera, sob forma de nuvens ou de chuva, enquanto nos
oceanos ela permanece em mdia 37 mil anos.
De toda a gua que existe no planeta, cerca de 97% salgada,
ou seja, est nos oceanos, e s pode ser aproveitada para consumo
humano se passar por um processo caro e complexo de
dessalinizao. As guas doces representam cerca de 2,5% do total,
e esto armazenadas essencialmente nas calotas polares, geleiras e
depsitos subterrneos. Os lagos de gua doce e os rios, de onde
provm a maior parte da gua utilizada para consumo humano,
representa nfimos 0,3% do estoque global de gua do planeta.
Crise da gua
As alteraes no ciclo hidrolgico no implicam a diminuio do
estoque total dos recursos hdricos. A crise da gua diz respeito
escassez de gua potvel, principalmente em razo da ampliao do
consumo e da poluio dos mananciais disponveis.

O consumo mundial de gua dobrou nos ltimos 50 anos, em


grande parte devido ao crescimento da populao e a urbanizao
acelerada. Por ano, so consumidos no planeta cerca de 4.500 km
de gua doce, o que corresponde a metade da disponibilidade anual
efetiva de todos os recursos hdricos acessveis.
Entre os diversos usos da gua, a irrigao o que apresenta
maiores taxas de desperdcio, com isso, a irrigao, na maior parte
do mundo, baseia-se em tcnicas pouco eficazes e de baixo
rendimento, onde h um grande desperdcio.

SECRETARIA DE ESTADO DA EDUCAO

92

02. A bacia Amaznica, considerada a maior bacia hidrogrfica do


mundo, irriga uma rea superior metade do territrio brasileiro (6,5
milhes de km), abrangendo outros seis pases sul-americanos. De
toda a gua fluvial lanada nos oceanos, 20% vm da foz do rio
Amazonas.
(LUCCI, E. A. et. al. Territrio e sociedade no mundo globalizado: Geografia
Geral e do Brasil. Ensino Mdio. Editora Saraiva, 2005. p.547).

Conforme podemos notar na descrio acima da Bacia Amaznica,


uma bacia hidrogrfica :
a) a rea ocupada por um rio e suas margens, a exemplo da rea
acima citada referente ao rio Amazonas.
b) a rea drenada por um rio e seus afluentes. No caso, a rea da
Bacia Amaznica drenada pelo rio Amazonas e sua rede
hidrogrfica.
c) o conjunto de rios e afluentes, tal qual ocorre com o Rio Amazonas
e os rios que o abastecem.
d) uma rea onde as guas das chuvas so drenadas para um rio
principal, tal qual o rio Amazonas que desgua no oceano Atlntico.
e) um conjunto de rios onde eles correm em direes diferentes e
todos eles possuem a mesma importncia.
03. O ciclo hidrolgico a circulao contnua da gua entre a terra,
o mar e a atmosfera. Nesse ciclo, inclui-se a gua potvel,
essencial vida. Entre os condicionantes e as interferncias da
existncia de gua potvel no planeta, incorreto afirmar que:
a) A gua potvel cada vez mais escassa no globo, em razo do
uso crescente para irrigao, atividade industrial e abastecimento
urbano;
b) As guas subterrneas constituem os lenis que alimentam os
cursos dgua, impedindo o escoamento superficial;
c) A contaminao da gua por substncias txicas, como resduos
industriais ou agrotxicos, agrava o quadro de escassez de gua
continental na superfcie;
d) A emisso de resduos poluentes prejudicial capacidade de
regenerao da gua, interferindo no controle de qualidade
ambiental.
e) A alterao no ciclo hidrolgico implica na diminuio da
quantidade de gua existente no mundo.
04. Segundo uma organizao mundial de estudos ambientais, em
2025, duas de cada trs pessoas vivero situaes de carncia de
gua, caso no haja mudanas no padro atual de consumo do
produto. Uma alternativa adequada e vivel para prevenir a
escassez, considerando-se a disponibilidade global, seria:
a) desenvolver processos de reutilizao da gua.
b) explorar leitos de gua subterrnea.
c) ampliar a oferta de gua, captando-a em outros rios.
d) captar guas pluviais.
e) importar gua doce de outros estados.

QUMICA
GEOGRAFIA

AULA 06: Problemas ambientais

EXERCCIOS DE FIXAO

Efeito Estuda: um fenmeno natural, porm com a inerncia das


atividades humanas (emisses de dixido de carbono, metano e
outros gases), h uma intensificao desse fenmeno. Com isso, a
excessiva concentrao desses gases na atmosfera provoca um
aumento da reteno de calor e, portanto, elevaria as mdias
trmicas globais.

01. Um fenmeno natural onde os gases que compem a atmosfera


retm parte do calor recebido do sol. O aumento anormal na
concentrao de gases CO2 , liberado pelas indstrias, pelos
veculos e pelo desmatamento, potencializa o efeito estufa,
provocando:
a) a diminuio de temperatura global
b) o aquecimento global
c) as chuvas torrenciais cidas
d) o alagamento de grandes metrpoles
e) o aumento das geleiras nos polos da terra
02. Segundo o cientista da NASA James Hansen, a temperatura da
Terra alcanou, nos ltimos 30 anos, uma rpida ascenso de cerca
de 0,2 graus Celsius, fenmeno este que jamais havia ocorrido dede
que acabou a era glacial, h 12 mil anos. Tal aquecimento se explica,
conforme o cientista, pelo aumento de emisso de gases estufa.
So consequncias do fenmeno de aquecimento global:

A - ____________________________________________________

I Derretimento das calotas glaciais.


II Interferncia na biodiversidade
III Aumento da emisso dos gases do efeito estufa
IV- Melhor qualidade de vida
V Reduo e at destruio de alguns biomas
A resposta que apresenta apenas as consequncias do fenmeno :

_______________________________________________________
B - ____________________________________________________

a) I e II
b) IV e V
c) I, II, III e V

d) I, II e V
e) I, IV e V

_______________________________________________________
C - ____________________________________________________
_______________________________________________________
A emisso de gases do efeito estufa (GEEs) reflete, essencialmente,
o nvel de industrializao e o modelo energtico das economias
nacionais. Os pases mais pobres apresentam baixo consumo per
capita de energia e relativamente pouca atividade industrial e, por
isso tm menor emisso dos GEEs.
Obs.: Principais GEEs:_____________________________________
_______________________________________________________

03. "Nos grandes centros urbanos a venda de geladeiras dobrou.


Esse o principal motivo para o aumento de consumo de
clorofluorcarbono (CFCs) no Brasil, onde o produto usado como
resfriante".
A emisso de poluentes como os CFCs est diretamente ligada a um
srio desequilbrio ecolgico que compromete a (o):
a) Camada de oznio.
b) Aumento das chuvas cidas.
c) Inverso trmica.
d) Capa de gs carbnico.
e) Efeito estufa.
04. Sobre o tema abordado pelo chargista foram feitas as seguintes
proposies:

Camada de oznio: Em volta da Terra h uma frgil camada de um


gs chamado oznio (O3), que protege animais, plantas e seres
humanos dos raios ultravioleta emitidos pelo Sol. Na superfcie
terrestre, o oznio contribui para agravar a poluio do ar das
cidades. Mas, nas alturas da estratosfera (entre 25 e 30 km acima da
superfcie), um filtro a favor da vida. Sem ele, os raios ultravioleta
poderiam aniquilar todas as formas de vida no planeta.
Obs.: Os clorofluorcarbonos (CFCs) usados como propelentes em
aerossis, como isolantes em equipamentos de refrigerao e para
produzir materiais plsticos, destroem a camada de oznio, expondo
a superfcie terrestre a grande quantidade de radiao ultravioleta.
I - A camada de oznio (O3) age como um verdadeiro "filtro solar",
protegendo a superfcie do
planeta da radiao ultravioleta emitida pelo sol.
II - A destruio da camada de oznio consequncia da liberao
de gases do tipo CFCs
(clorofluorcarbonos) para a atmosfera, o que facilita a passagem da
radiao infravermelha.
III - A alta incidncia de radiao ultravioleta, atravs dos "buracos"
na camada de oznio, leva ao
aumento dos casos de cncer de pele e cegueira em pessoas e
animais.
Dessas proposies, somente so CORRETAS
(A) I e III.
(B) II e IV.
(C) I, II e III

SECRETARIA DE ESTADO DA EDUCAO

93

(D) I, III e IV.


(E) II, III e IV.

QUMICA
GEOGRAFIA

EXERCCIOS DE FIXAO
AULA 07: Problemas ambientais nos centros urbanos e no
meio rural
Com o advento da Revoluo Industrial, especialmente a partir do
sculo XIX, os problemas ambientais comearam a agravar-se cada
vez mais, primeiro nos atuais pases desenvolvidos e depois no
restante do mundo, embora possa ocorrer em pequena escala nas
sociedades tradicionais, decorrncia tpica da chamada
modernizao, constituda sobretudo pela industrializao e pela
urbanizao.

01. Por cerca de 4 bilhes de anos o balano ecolgico do planeta


esteve protegido. Com o surgimento do homem, meros 100 mil anos,
o processo degradativo do meio ambiente tem sido proporcional
sua evoluo. Podemos afirmar que no Brasil, o incio da influncia
do homem sobre o meio ambiente pode ser notada a partir da
chegada dos portugueses. Antes da ocupao do territrio brasileiro,
os indgenas que aqui habitavam (estimados em 8 milhes)
sobreviviam basicamente da explorao de recursos naturais, por
isso, utilizavam-nos de forma sustentvel.

O meio ambiente humano costuma ser dividido em:

A degradao ambiental decorrncia

Meio cultural:_________________________________________

A exclusivamente do avano rural que a cada dia destri mais


mais florestas nativas.
B apenas da urbanizao, pois as pessoas precisam destruir
meio ambiente para construir suas casas.
C do desequilbrio ambiental entre os animais silvestres devido
falta de reas verdes.
D da interferncia negativa do homem no meio ambiente.
E das mudanas climticas que esto ocorrendo e afetando
fauna e flora do planeta

____________________________________________________
Meio natural:__________________________________________
____________________________________________________
Problemas ambientais dos centros urbanos
As grandes e mdias cidades geralmente so mais poludas do
que as pequenas cidades e o meio rural. Isso porque nelas se
concentram maior nmero de indstrias, veculos e pessoas,
agravando o acumulo de lixo, de resduos, as emisses industriais, o
congestionamento, a poluio atmosfrica entre outros. Entre os
principais problemas ambientais dos centros urbanos esto:

e
o
a

02. A soja foi o cultivo que mais cresceu no Brasil nos ltimos vinte
anos e, atualmente o ramo mais importante do setor oleaginoso.
Os principais produtores nacionais so, pela ordem: Mato Grosso,
Paran, Rio Grande do Sul, Gois e Mato Grosso do Sul.

Lixes:__________________________________________________
________________________________________________________
Ilhas de calor:_____________________________________________
________________________________________________________
_______________________________________________________

A produo de soja atualmente causa srios problemas ambientais


dentre eles podemos destacar

Inverso Trmica:_________________________________________
A a destruio da Mata Atlntica.
B os danos biodiversidade dos Pampas.
C a degradao do Cerrado.
D as inundaes decorrentes da alta produo.
E o fenmeno da chuva cida.

_______________________________________________________
_______________________________________________________
Problemas ambientais do meio rural
O campo tambm enfrenta problemas ambientais, especialmente
as reas que passam por um processo de modernizao agrria, com
a mecanizao e o uso intensivo de adubos qumicos e agrotxicos ou
defensivos agrcolas.

Derrubada da vegetao original da rea a ser cultivada;


Extermino da fauna local;
Contaminao do solo, rios e lagos pelo uso de agrotxicos.

Com a modernizao da sociedade, o meio cultural passa a


predominar sobre o natural, que se transforma e acaba dependendo
cada vez mais da ao humana, das modificaes impostas pela
sociedade. Dizemos, ento, que a natureza se humaniza, deixa de ser
primeira natureza para se transformar em uma segunda natureza.
Primeira natureza:____________________________________
____________________________________________________
____________________________________________________

03. A questo ambiental


nos dias de hoje um processo
contraditrio que gera, ao mesmo tempo, mudanas positivas e
negativas na regio em que ocorre. De um lado diminui as taxas de
mortalidade, ampliando a expectativa de vida das pessoas. De outro
lado, muitas vezes agrava as desigualdades sociais, causando, por
exemplo, enormes poluio do ar e das guas.
Podemos afirmas que a degradao do meio ambiente nos dias de
hoje est ligada a
A conservao do meio ambiente por rgo pblicos como o
IBAMA.
B modernizao, constituda pela industrializao e pela
urbanizao.
C biodiversidade, pois as espcies de animas selvagens esto
destruindo os seus habitats naturais.
D crescente expanso das reas verdes nas cidades e a constante
preocupao em conservao do meio ambiente.
E enorme quantidade de espcies animais e vegetais que existem
em nosso planeta, buscando, cada vez mais, reas disponveis para
habitarem e se reproduzirem.

Segunda natureza:_____________________________________
____________________________________________________
_____________________________________________________

SECRETARIA DE ESTADO DA EDUCAO

94

QUMICA
GEOGRAFIA

EXERCCIOS DE FIXAO
AULA 08: Fontes de energia e convenes sobre mudanas
climticas
At o sculo XVIII, a evoluo do consumo e o aprimoramento de
novas tecnologias de gerao de energia foram lentos e
descontnuos. A Revoluo Industrial alterou substancialmente esse
panorama. Os ciclos iniciais de inovao tecnolgica da economia
industrial foram marcados pela incorporao de novas fontes de
energia. Assim, o pioneiro ciclo hidrulico, no qual o giro de turbinas
hidrulicas ou de moinhos de gua permitia acionar equipamentos,
foi sucedido pelo ciclo do carvo, que por sua vez cedeu lugar ao
ciclo do petrleo.
O mundo utiliza principalmente trs fontes de energia que libera
gases poluentes na atmosfera e provocam o aquecimento global.
So elas: _____________________, ________________________ e
_____________________________________.
Energia renovvel: toda energia produzida com o uso de
recursos naturais que se renovam ou podem ser renovados.
Ex.__________________________________________________
Energia no renovvel: tm recursos teoricamente limitados,
sendo que esse limite depende dos recursos existentes no nosso
planeta, como o exemplo dos combustveis fsseis.
Ex.____________________________________________________
Energia limpa: aquela que no polui, ou que polui menos que
as tradicionais.
Ex.____________________________________________________
Principais convenes sobre mudanas climticas

Estocolmo 1972: Essa Conferncia chamou a ateno das


naes para o fato de que a ao humana estava causando
sria degradao da natureza e criando severos riscos para o
bem estar e para a prpria sobrevivncia da humanidade. Foi
marcada por uma viso antropocntrica de mundo, em que o
homem era tido como o centro de toda a atividade realizada no
planeta, desconsiderando o fato de a espcie humana ser parte
da grande cadeia ecolgica que rege a vida na Terra.

Rio de Janeiro 92: Em 1988 a Assembleia Geral das


Naes Unidas aprovou uma Resoluo determinando
realizao, at 1992, de uma Conferncia sobre o meio
ambiente e desenvolvimento que pudesse avaliar como os
pases haviam promovido a Proteo ambiental desde a
Conferncia de Estocolmo de 1972. Na sesso que aprovou
essa resoluo o Brasil ofereceu-se para sediar o encontro em
1992.

Rio + 20: A Rio+20 foi assim conhecida porque marcou os


vinte anos de realizao da Conferncia das Naes Unidas
sobre Meio Ambiente e Desenvolvimento (Rio-92) e contribuiu
para definir a agenda do desenvolvimento sustentvel para as
prximas dcadas. O objetivo da Conferncia foi a renovao
do compromisso poltico com o desenvolvimento sustentvel,
por meio da avaliao do progresso e das lacunas na
implementao das decises adotadas pelas principais cpulas
sobre o assunto e do tratamento de temas novos e
emergentes.
Protocolo de Kyoto:____________________________________
______________________________________________________
______________________________________________________

01. No presente, observa-se crescente ateno aos efeitos da


atividade humana, em diferentes reas, sobre o meio ambiente,
sendo constante, nos fruns internacionais e
nas instncias
nacionais, a referncia sustentabilidade como princpio orientador
de aes e propostas que deles emanam. A sustentabilidade
explica-se pela
a) incapacidade de se manter uma atividade econmica ao longo do
tempo sem causar danos ao meio ambiente.
b) incompatibilidade entre crescimento econmico acelerado e
preservao de recursos naturais e de fontes no renovveis de
energia.
c) interao de todas as dimenses do bem-estar humano com o
crescimento econmico, sem a preocupao com a conservao
dos recursos naturais que estivera presente desde a Antiguidade.
d) proteo da biodiversidade em face das ameaas de destruio
que sofrem as florestas tropicais devido ao avano de atividades
como a minerao, a monocultura, o trfico de madeira e de
espcies selvagens.
e) necessidade de se satisfazer as demandas atuais colocadas pelo
desenvolvimento sem comprometer a capacidade de as geraes
futuras atenderem suas
prprias necessidades nos campos
econmico, social e ambiental.
02. Sobre a globalizao dos problemas ambientais correto afirmar:
I - Aps a Revoluo Industrial, a Natureza passou a ser vista como
uma fonte de recursos econmicos a ser explorada por meio de
instrumentos cada vez mais sofisticados, criados pela cincia e pela
tecnologia. Nesse processo, o meio ambiente foi submetido a uma
contnua devastao, pondo em risco o equilbrio do planeta e
afetando a vida de toda a humanidade.
II - Nas ltimas dcadas do seculo XX, com o agravamento dos
problemas ambientais, a sociedade se mobilizou para deter os
efeitos nocivos das atividades econmicas, predatrias e poluentes.
III - Os grupos ecolgicos se multiplicaram e a presso social
resultou na aprovao pelos poderes pblicos de leis de proteo ao
meio ambiente.
IV - No mbito internacional, a preservao do meio ambiente
passou a constituir elemento importante de um pas para negociar a
comercializao de seus produtos e recebimento de emprstimos.
Est(o) correta(s)
a) Apenas a proposio I
b) Todas as proposies
c) Apenas as proposies II e IV

d) Apenas as proposies I e II
e) Apenas as proposies I e III

03. Na contradio entre a sociedade industrial e o meio ambiente,


surge a necessidade de acordos mundiais, sempre polmicos, pois
denotam a desacelerao do crescimento econmico para preservar
o meio ambiente.
Um dos acordos que alertou o mundo e representou um momento
chave da agenda ambiental foi a ECO 92, no Rio de Janeiro. Entre
as preocupaes desse encontro, estava
a) a ideia de que o controle da natalidade seria uma sada para evitar
desgastes ambientais, pois quanto mais gente maior o consumo
industrial.
b) a convico de que os pases pobres precisavam de ajuda para
gerir suas florestas, pois no tinham condies de evitar o
desmatamento e as queimadas.
c) o conceito de desenvolvimento sustentvel, que compreendia o
uso de elementos naturais somente em determinados lugares do
globo, pois isso representaria um equilbrio ambiental.
d) a proposio de acordos de controle entre a produo e o
comrcio, envolvendo pases ricos e pobres, com isso, os que mais
comercializam poderiam desgastar mais o meio ambiente.
e) a gerao de um tratado global referente aos Princpios para a
Administrao Sustentvel das Florestas, pois, se este princpio
fosse seguido, seria alcanado o consenso entre conservao,
manejo e desenvolvimento sustentvel de todos os biomas florestais.

______________________________________________________
______________________________________________________

SECRETARIA DE ESTADO DA EDUCAO

95

QUMICA

GEOGRAFIA

EXERCCIOS DE FIXAO
AULA 09: Revoluo industrial, classificao da indstria
moderna e cenrios regionais
A Revoluo Industrial, teve como ponto principal o
desenvolvimento no mbito tecnolgico, por isso o advento da
indstria e da produo mecanizada, ocorrido na Inglaterra do sculo
XVIII principalmente a partir da inveno da mquina a vapor em
1760, caracterizou-se como revoluo. A principal caracterstica da
Revoluo Industrial foi a criao do sistema fabril mecanizado, isto
, as fbricas passaram da simples produo manufaturada para a
complexa substituio do trabalho manual por mquinas.
Essa substituio implicou na acelerao da produo de
mercadorias, que passaram a ser produzidas em larga escala. Essa
produo em larga escala, por sua vez, exigiu uma demanda cada
vez mais alta por matria-prima, mo de obra especializada para as
fbricas e mercado consumidor. Tal exigncia implicou, por sua vez,
tambm na acelerao dos meios de transporte de pessoas e
mercadorias. Era necessrio o encurtamento do tempo que se
percorria de uma regio outra para escoar os produtos.
Nesse sentido, a Revoluo Industrial estimulou o
desenvolvimento das cidades que tiveram que se adaptar ao
grande contingente de pessoas que migrava do meio rural em busca
de emprego nas fbricas , bem como a criao de transportes,
como a locomotiva a vapor (ou trem de ferro), que exigia uma
malha ferroviria, isto , linhas de trem feitas de ferro para
estabelecer a ligao entre as regies.

01. A indstria moderna resultou da Revoluo Industrial, que


ocorreu inicialmente no Reino Unido (Inglaterra) em meados do
sculo XVIII e depois se espalhou para praticamente todo o mundo.
Essa atividade industrial tornou a fabricao de bens e servios ou
objetos materiais a atividade econmica mais importante da
sociedade.
O emprego de mquinas modernas permite que a indstria possa
produzir em larga escala que seria a produo
A de poucos produtos e privilegiando os detalhes das mercadorias.
B uniformizada, ou seja, de produtos do mesmo tipo e em
mquinas simples.
C sem o uso de mquinas, apenas de ferramentas antigas.
D artesanal, onde o arteso privilegia o detalhe de cada pea.
E em enormes quantidades, em nveis jamais atingidos pelo
artesanato ou manufatura.
02.

Produo:
Artesanal:______________________________________________
_______________________________________________________
Manufaturada:____________________________________________
_______________________________________________________
Mecanizada:_____________________________________________
_______________________________________________________
Classificao da indstria moderna
Industria de transformao: Foi com esse tipo de indstria que se
iniciou a Revoluo Industrial. Esse tipo de indstria transforma
produtos naturais (matria-prima) e produtos industrializados.
Indstria extrativa: Utiliza mquinas de grande porte para extrair a
matria-prima da natureza e enviar para que outras industrias
transforme essa matria-prima em bens de consumo.
Indstria de construo: Destaca-se na construes de instalaes
de grande porte, como portos, rodovias e pontos, bem como a de
edifcios e casas.
Portanto, o emprego de mquinas modernas permite que a
indstria possa produzir em larga escala e em srie. Assim, podemos
afirmas que as caractersticas bsicas da indstria moderna so: a
mecanizao (uso intenso de mquinas) e a produo em srie.
Larga escala: ____________________________________________
Em srie:________________________________________________
Cenrios regionais
Os Estados Unidos, o Japo e a Unio Europeia so as
principais potncias indstrias da atualidade. Em cada uma delas, a
herana das velhas aglomeraes industriais e o impacto das
mudanas
tecnolgicas
produziram
paisagens
industrias
caractersticas. Existem tambm pases com realidade diferente dos
pases desenvolvidos como China e Rssia que vem se destacando
como grandes exportadores industrializados e integram-se ao
mercado mundial.

SECRETARIA DE ESTADO DA EDUCAO

96

A industrializao aconteceu em diferentes momentos da histria,


mas o seu primeiro passo ocorreu no sculo XVIII, com a Revoluo
Industrial. Assim, as indstrias
A concentram-se principalmente no continente africano, pois l a
rea mais desenvolvida do globo.
B ainda esto em processo de construo e consolidao no
continente europeu.
C esto em massa na Amrica do Sul, tornando essa regio, a
mais industrial do mundo.
D possuem grande desenvolvimento tecnolgico e esto em
grande quantidade no continente europeu.
E passa por um processo de decadncia, pois, a maioria dos
produtos, so feitos de forma artesanal.
03. O processo de fabricao de bens necessrios a vida teve inicio
desde que o ser humano comeou a transformar qualquer elementos
da natureza para fazer artefatos. Mas essa fabricao de bens de
consumo a partir da extrao de elementos da natureza passou por
diferentes momentos da histria. As principais etapas desse
processo foram o artesanato, a manufatura e a indstria moderna.
Quais so as caractersticas da fabricao atravs da manufatura?
A Produo manual e individualizada.
B Fabricao de bens em larga escala e em srie.
C O predomnio das mquinas modernas.
D Utilizao de ferramentas antigas.
E Uso de mquinas simples.
04. De acordo com a natureza dos bens que produzem podemos
classificar a indstria de transformao em
A indstria extrativa, indstria de bens intermedirios e indstria do
algodo.
B indstria de construo, indstria de transformao e indstria de
bens de produo.
C indstrias de bens de produo, indstrias de bens
intermedirios e indstrias de bens de consumo.
D indstria artesanal, indstria extrativa e indstria de construo.
E indstria de bens de consumo, indstria de bens intermedirios e
indstria extrativa.

QUMICA
GEOGRAFIA

AULA 10: Meio Urbano

EXERCCIOS DE FIXAO

Em 2008, pela primeira vez na histria, o nmero de pessoas


vivendo em cidades superou o nmero de pessoas vivendo no
campo. As cidades resultam de um processo de ocupao e
organizao do espao com algumas caractersticas comuns. Em
primeiro lugar so locais de aglomeraes de pessoas, onde
desenvolve-se o comrcio, as indstrias e a administrao.
A partir do sculo XVIII, com a revoluo Industrial, as atividades
industriais e de servios expandiram-se enormemente nas cidades,
intensificando ainda mais a concentrao da populao nesses
espaos e consolidando seu papel centralizador.

01. Uma cidade pode crescer de duas formas: pela construo de


elevados edifcios ou galerias subterrneas (crescimento vertical) e
pela expanso de seu espao urbano (crescimento horizontal).

Conceitos
Urbanizao:_____________________________________________
_______________________________________________________
Nvel de urbanizao:______________________________________
_______________________________________________________
Taxa de urbanizao:______________________________________
_______________________________________________________
Metrpoles e megalpoles
A concentrao da populao nas cidades provocou dois
processos interligados: a verticalizao das reas mais centrais e o
surgimento de novos loteamentos na periferia urbana (Peri
urbanizao).
Fatores responsveis pela:
Verticalizao das cidades:_________________________________
_______________________________________________________
Peri urbanizao:_________________________________________
_______________________________________________________
O processo de expanso territorial das cidades muitas vezes
resulta na formao de grandes manchas urbanas, que atravessam
as fronteiras municipais. Esse fenmeno chamado de conurbao.
As cidades mais importantes de um pas, ou a cidade de uma rede
urbana conturbada, recebem o nome de metrpole que podem ser
divididas
em
metrpoles
___________________________,
_____________________
ou
________________________
dependendo do seu grau de influencia.

A imagem acima mostra duas cidades de Minas Gerais,


evidenciando um fenmeno chamado conurbao, que significa:
A Uma rea onde se concentra mais pessoas e onde h mais
comrcio e outros servios.
B Bairros mais distantes do centro, com pouca infraestrutura.
C Subrbios, onde residem populaes de alta renda, que se
deslocaram das reas centrais.
D A juno de dois espaos urbanos, geralmente causado pela
expanso das cidades.
E O terreno sobre o qual a cidade se desenvolveu ao longo do
tempo.
02. A intensa urbanizao que vem ocorrendo no Brasil,
especialmente a partir de 1950, tem sido acompanhada por um
processo de metropolizao, isto , concentrao demogrfica nas
metrpoles
A lei federal aprovada em 1973 entende como regio metropolitana:
A Um crescimento do meio urbano proporcionalmente maior que o
do meio rural.
B A separao de dois espaos vizinhos, devido ao seu rpido
crescimento econmico.
C Conjunto de municpios contguos e integrados
socioeconomicamente a uma cidade central.
D O processo de migrao das pessoas do campo para as cidades.
E Aspectos da passagem de uma economia agrrio-exportadora
para uma urbano-industrial.

Questes ambientais urbanas

03. A urbanizao significa um crescimento do meio urbano


proporcionalmente maior que o do meio rural, ou seja, representa
uma migrao gradativa da populao do campo para a cidade.
Um dos principais fatores que contribuiu para o aumento da
populao nas reas urbanas foi:

A maioria das cidades apresenta altos ndices de poluio, e


muitos dos problemas ambientais so gerados nela.
A medida que as cidades se expandem e novas reas so
ocupadas, ocorrem mudanas ao seu redor, como a ocupao de
terras agricultveis, perda de reas de vegetao e o consequente
aumento da eroso dos solos, obstruo de leitos de rios,
contaminao de fontes de gua etc.

A Os problemas urbanos encontrado na maioria das cidades.


B A boa oferta de emprego oferecida na zona rural.
C O aumento na oferta de emprego nas cidades devido
industrializao.
D A diminuio do emprego no meio rural devido ao retrocesso
tecnolgico.
E As pragas que destruam toda a produo do campo.

Obs.: Na maioria das cidades do mundo menos desenvolvido as


montanhas de lixo geradas diariamente so depositadas em lixes a
cu aberto. Esse modo de destinao final do lixo causa graves
problemas ambientais: a deteriorao dos materiais exala odores
fortes, contamina as guas superficiais e subterrneas pela
infiltrao do chorume (lquido proveniente de decomposio do lixo)
e foco de transmisso de doenas populao e do seu entorno. O
modo adequado de descarte do lixo serio nos:
Aterros sanitrios:________________________________________
_______________________________________________________
_______________________________________________________

SECRETARIA DE ESTADO DA EDUCAO

97

04. A megalpole, centro econmico do Brasil, uma rea que


abrange desde a Grande So Paulo at a Grande Rio de Janeiro,
nessa rea:
A Predomina a atividade industrial, com os estabelecimentos fabris
mais modernos do pas.
B Destaca-se o desenvolvimento de atividades ligadas
exclusivamente a agropecuria.
C O gado de corte tem maior fora na atividade econmica junto
com a produo agrcola.
D Observa-se uma grande produo agrcola com destaque para o
cultivo do arroz, soja e trigo.
E Ocorre a maior extrao de minrio de ferro do Brasil, alm de
apresentar poucas indstrias.
AULA 11: Meio rural

QUMICA
GEOGRAFIA

EXERCCIOS DE FIXAO
A agropecuria sempre ocupou lugar de destaque nas atividades
humanas, pois necessitamos dos recursos produzidos no campo, tais
como alimentos e matrias-primas. Na Antiguidade, o meio rural era
o espao de produo mais importante, no qual a maior parte das
riquezas eram produzidas.
A agricultura tradicional, ou arcaica, empregava tcnicas e
instrumentos rudimentares de cultivo, como enxadas, arados e foices
e utilizava a energia humana e a de animais. At hoje esse tipo de
agricultura praticado em vastas extenses do mundo em
desenvolvimento.
A agropecuria e o desenvolvimento tecnolgico
A partir do sculo XVIII, no perodo da Revoluo Industrial, o
aperfeioamento dos instrumentos e tcnicas de cultivo permitiu o
aumento da produtividade agrcola, originando a agricultura moderna.
Os espaos agrrios continuaram a sofrer profundas mudanas
advindas das transformaes tecnolgicas durante os sculos XIX e
XX. Na segunda metade do sculo XX, com a agricultura
contempornea, iniciou-se uma fase de emprego de tcnicas
diversificadas na agricultura e na pecuria.
A partir da dcad de 1960, o perodo conhecido como revoluo
verde, caracterizou-se pelo:
_______________________________________________________
_______________________________________________________
_______________________________________________________
_______________________________________________________

01. O agronegcio refere-se especificamente s indstrias cujos


produtos tm por base um produto agrcola, como a indstria de:
A Cigarros, onde o fumo bastante utilizado.
B Plstico baseado em polmeros sintticos.
C Ligas de ao onde o ferro sua principal matria-prima.
D Calados que usam elementos qumicos na sua fabricao.
E Cermica, que utiliza basicamente a argila na sua produo.
02. Considere as afirmaes sobre a implantao da agricultura
moderna ou intensiva que encontrada em larga escala nos pases
desenvolvidos.
I Utilizao da pesquisa agronmica com o objetivo de
aperfeioamento gentico das espcies.
II Predomnio de grandes propriedades rurais, s quais se aplica a
especulao imobiliria, objetivando a valorizao da terra.
III Intensa utilizao de fertilizantes, corretivos e defensivos
agrcolas.
IV Desenvolvimento de uma rede de transportes estruturada,
permitindo rpido acesso entre as reas de produo e as reas de
consumo.
So verdadeiras:
a) apenas I, II e IV.
b) apenas I, III e IV.
c) apenas II, III e IV.
d) apenas I e II.
e) todas
03. Antes, eram apenas as grandes cidades que apresentavam
como o imprio da tcnica, objeto de modificaes, suspenses,
acrscimos, cada vez mais sofisticadas e carregadas de artifcio.
Esse mundo artificial inclui, hoje, o mundo rural.

Portanto a revoluo verde est baseada no trip:

__________________________

(Milton Santos, A Natureza do Espao. S. Paulo, Hucitec: 1994).

Essa sofisticao tcnica no meio rural pode ser identificada no


seguinte exemplo:
_______________________

________________________

A agropecuria nos pases desenvolvidos e subdesenvolvidos


Subdesenvolvidos: A agricultura de plantation foi introduzida pelos
europeus em suas colnias tropicais a partir do sculo XVI e
praticada at hoje em terras da frica, da Amrica Latina e da sia
meridional. Sem capital e tecnologia, em geral os pases
subdesenvolvidos aumentam sua produtividade rural por meio da
incorporao de novas reas para o cultivo. Constituem sociedades
rurais com baixo poder aquisitivo, de baixo consumo interno e
submisso ao mercado mundial de commodities.
Plantation: ______________________________________________
_______________________________________________________
Commodities: ____________________________________________
_______________________________________________________
Desenvolvidos: Nos pases desenvolvidos, a agropecuria se insere
na economia mundializada de mercado, sendo bastante integrada
aos setores secundrios e tercirios. Ao mesmo tempo que atende
s necessidades urbano-industriais de alimentos e matrias-primas,
representa um mercado de consumo de importantes produtos
industrializados como mquinas (tratores, semeadeiras, colhedeiras),
implementos e insumos industrias. Alm dessa industrializao do
meio rural, emprega-se tambm a agropecuria cientfica.
Agropecuria cientfica:____________________________________
_______________________________________________________
_______________________________________________________

SECRETARIA DE ESTADO DA EDUCAO

98

a) a presena de complexos agroindustriais restrita s grandes


propriedades agrcolas dos pases latino-americanos.
b) expanso da mecanizao na agricultura de jardinagem praticada
nas mdias propriedades do Sudeste Asitico.
c) produtividade da agricultura empresarial norte-americana
impulsionada pela fora dos mercados interno e externo.
d) atuao de uma poltica agrcola comum nos pases europeus
consolidada desde a formao do Mercado Comum Europeu.
e) ocorrncia de migraes internas (cidade-cidade; campo-cidade)
em busca de melhores condies de vida.
04. Geralmente a indstria que financia os produtores, e eles, em
contrapartida, vendem toda a sua produo para a indstria, assim
podemos afirmar que o agronegcio consiste
a) Em reas perifricas no campo onde residem populaes de baixa
renda.
b) Na criao de aves como: galinhas, galos, frangos, frangas, pintos
e codornas.
c) Na plantao de soja, planta leguminosa com alto valor proteico,
que se expandiu bastante nas reas do cerrado do Brasil central.
d) Na elevao das mdias trmicas dos centros urbanos devido
poluio da produo agropecuria.
e) Uma integrao entre as atividades primrias e o setor industrial.
05. Um dos fatores que contribuem para o processo de urbanizao
o xodo rural que consiste
a) Na migrao de pessoas entre as cidades.
b) No nmero de nascimentos de pessoas nas reas urbanas.
c) Na migrao gradativa da populao do campo para as cidades.
d) Em aumentar a populao do campo em relao s zonas
urbanas.
e) No aumento do nmero de nascimentos nas reas rurais.

QUMICA
GEOGRAFIA

AULA 12: A formao territorial do Brasil


O territrio atribudo a Portugal pelo tratado de Tordesilhas,
assinado com a espanha em 1494, abrangia cerca de 2.800.000 km
do trecho oriental da Amrica do Sul. O Brasil atual, herdeiro da
Amrica Portuguesa, ocupa 8.514.876,5 km, equivalentes a 47,3%
da superfcie sul-americana. Suas fronteiras estendem-se por 23.086
km lineares: uma seo martima de 7.367 km e outra terrestre, de
15.719 km. Na Amrica do Sul, apenas o Chile e o Equador no so
limtrofes ao Brasil.
Tratado de Tordesilhas: ___________________________________
______________________________________________________
A expanso territorial e a construo do Brasil
Nos primeiros tempos da colonizao, a economia voltava-se
para o mercado externo, com a produo de acar nas reas
prximas ao litoral. No interior, a presena europeia praticamente se
limitava vila de So Paulo e a um punhado de ncleos vizinhos.
Dali partiram os bandeirantes, que desde o inicio do sculo XVII
deixaram para trs a linha de Tordesilhas e percorreram reas ento
pertencentes Espanha.
Bandeirantes: ____________________________________________
_______________________________________________________
Com a independncia de 1822, nasceu um imenso imprio nos
trpicos, comandado por D. Pedro I. A constituio de 1824, imposta
pelo imperador, consolidou o carter hereditrio e escravista desse
imprio, cuja capital era o Rio de Janeiro. Assim, aps a
independncia, outras reas se incorporaram ao territrio do Brasil.
O imprio foi responsvel pela fixao de mais de metade da
extenso total das fronteiras terrestres.
Diviso poltica atual
A Republica Federativa do Brasil formada por 26 estados e
pelo Distrito Federal. Os estados, por sua vez, dividem-se em
municpios. Os estados so unidades de maior hierarquia na
organizao poltico-administrativa do pas; a localidade que abriga a
sede do governo chamada capital. Ao longo de sua histria, o
Brasil j teve tambm unidades administrativas chamadas territrios
federais.
Territrios federais:_______________________________________
_______________________________________________________
Os municpios so as menores unidades polticas autnomas na
federao brasileira. Na maioria dos casos apresentam reas rurais e
urbanas.
O Distrito Federal uma unidade federativa autnoma, que sedia
o governo federal. Braslia a capital federal do Brasil. O Distrito
Federal no se divide em municpios e sim em regies
administrativas (RAs), muitas das quais correspondem a cidadessatlites.

EXERCCIOS DE FIXAO
01. Esse principio foi uma soluo diplomtica que conferia a um
Estado o direito de se apropriar de um novo territrio com base na
ocupao, na posse efetiva da rea, e no em ttulos anteriores de
propriedade. Esse principio foi utilizado entre Portugal e Espanha
sem nunca levar em conta a posse das diversas tribos indgenas.
Assinale a alternativa que o texto faz referncia:
a) O principio do uti possidetis
b) O principio da apropriao
c) Principio da lei regente
d) Acordo entre Portugal e Espanha
e) O principio da colonizao
02. Marque a alternativa que marca o inicio da colonizao em 1534,
onde o rei de Portugal dividiu o Brasil em faixas de terra que seriam
administradas cada uma por um donatrio, um nobre portugus rico.
a) Expanso para o Oeste
b) Provncias
c) Estados divididos
d) Sistema colonial
e) Capitanias Hereditrias
03. Marque a alternativa que indica as duas nicas naes sulamericanas
que
no
fazem
fronteira
com
o
Brasil.
a) Argentina e Uruguai
b) Equador e Chile
c) Colmbia e Suriname
d) Chile e Panam
e) Paraguai e Venezuela
04. Para se compreender a diviso do territrio brasileiro em estados
e, consequentemente, a existncia dos estados federados e a
desigualdade de seu desenvolvimento, torna-se necessrio
compreender tambm o processo de transformao do espao
brasileiro em territrio, o processo de povoamento, as motivaes
que o provocaram e os percalos encontrados durante cinco sculos
de povoamento.
(Fonte: ANDRADE, M. C. de. A Federao brasileira uma anlise geopoltica
e geossocial. So Paulo: Contexto, 1999.)

Com base nesse texto, assinale a alternativa correta.


a) Mesmo aps cinco sculos de ocupao e povoamento, a diviso
dos estados brasileiros e sua configurao atual resultam da
implantao das capitanias hereditrias.
b) As motivaes para o povoamento do territrio estiveram ligadas
existncia dos estados federados e desigualdade de
desenvolvimento existente entre eles.
c) Alguns estados brasileiros tm maior populao e so
considerados mais desenvolvidos pela forma como ocorreu sua
diviso.
d) A diviso do territrio brasileiro e suas caractersticas podem ser
compreendidas pela forma histrica como ocorreu a ocupao e o
povoamento do espao.
e) A forma como foram criados os estados federados gerou um pas
com distribuio populacional e desenvolvimento desiguais.

Sistema de governo

05. Sobre o Tratado de Tordesilhas, assinado em 7 de junho de


1494, pode-se afirmar que objetivava:

O Brasil uma Repblica federativa presidencialista. A


Repblica, proclamada em nosso pas em 1889, uma forma de
governo na qual representantes eleitos pelo povo governam por
tempo determinado. O presidencialismo um regime poltico
chefiado por um presidente da Repblica, que acumula as funes
de chefe de Estado e chefe de governo. O termo federativa indica
que os estados esto unidos numa federao, mas mantm relativa
autonomia. No Brasil, o presidente da Repblica eleito por voto
direto para um perodo de quatro anos, podendo ser reeleito para
mais quatro anos. O mesmo acontece com os governadores dos
estados e os prefeitos dos municpios.
A Constituio a Lei Maior que rege a vida de um pas,
determinando, entre outros aspectos, a organizao do Estado. De
acordo com a constituio existem trs poderes da Unio,
independentes: o Executivo, o Legislativo e o Judicirio.

a) demarcar os direitos de explorao dos pases ibricos, tendo


como elemento propulsor o desenvolvimento da expanso comercial
martima.
b) estimular a consolidao do reino portugus, por meio da
explorao das especiarias africanas e da formao do exrcito
nacional.
c) reconhecer a transferncia do eixo do comrcio mundial do
Mediterrneo para o Atlntico, depois das expedies de Vasco da
Gama s ndias.
d) reconhecer a hegemonia anglo-francesa sobre a explorao
colonial, aps a destruio da Invencvel Armada de Felipe II, da
Espanha.
e) acabar com o mercantilismo, por meio da criao de um sistema
de monoplios que atingia todas as riquezas coloniais.

SECRETARIA DE ESTADO DA EDUCAO

99

QUMICA
GEOGRAFIA

EXERCCIOS DE FIXAO
AULA 13: Hidrografia, climas e biomas do Brasil
Hidrografia
Os rios so um dos principais agentes modeladores do relevo e
realizam tanto o transporte dos sedimentos erodidos dos planaltos e
depresses como os processos de sedimentao nas plannices. Os
compartimentos do relevo, por sua vez estabelecem os vetores do
escoamento das chuvas, distribuindo as guas superficiais para as
calhas dos rios. esse conjunto de cursos de gua que circula
vertente abaixo, convergindo para um curso principal, e suas terras
drenadas que se denominam bacia hidrogrfica.
Pelo
territrio
brasileiro
ocorre
o
escoamento
de
aproximadamente 15% da gua superficial existente na terra. Todo
esse volume de gua foi subdividido pelo Conselho Nacional de
Recursos Hdricos (CNRH) em 12 regies hidrogrficas: Amaznica,
Tocantins-Araguaia, Paran, Paraguai, Uruguai (Juntas, as bacias
hidrogrficas Paraguai, Paran e Uruguai formam a regio
hidrogrfica do Prata), Parnaba, So Francisco, Nordeste Ocidental,
Nordeste Oriental, Atlntico Leste, Atlntico Sul e Atlntico Sudeste.
Clima
A localizao da maior parte do extenso territrio brasileiro em
reas de baixas latitudes, entre o Equador e o Trpico de
Capricrnio, a inexistncia de altas cadeias montanhosas e a
dinmica das massas de ar so fatores que explicam a configurao
de seis principais tipos climticos no Brasil.

01. A Caatinga uma vegetao caracterstica do clima semirido


do Serto nordestino. A sua fauna abundante em rpteis, entre os
quais se destacam os lagartos e cobras. Existem alguns roedores e
muitos insetos e aracndeos. A dificuldade de obter gua um
obstculo para a existncia de grandes mamferos na regio.
Alm dessas caractersticas o Bioma da Caatinga
A) possui plantas xerfilas, adaptadas aridez.
B) apresenta uma floresta com folhas largas.
C) h o predomnio de plantas hidrfilas.
D) est adaptado a muita gua.
E) concentra uma grande variedade de rvores altas.
02. A floresta Amaznica costuma se dividida em trs tipos de mata,
uma delas seria a mata de vrzea
A) recobre os baixos planaltos sedimentares e nunca inundada.
B) localizada ao longo dos rios permanentemente inundada.
C) sujeita a inundaes peridicas ao longo dos rios.
D) suas plantas so de menor porte, como a vitria-rgia.
E) possui plantas de grande porte como a castanheira.

03.

De acordo com as massas de ar, verificamos a existncia de


seis tipos de clima no Brasil

Tropical:________________________________________________
Litorneo mido:__________________________________________
Equatorial mido:_________________________________________
Semirido:_______________________________________________
Tropical de altitude:_______________________________________
Subtropical:_____________________________________________
Biomas
Amaznico: Possui um tero do nmero de espcies do planeta,
grande quantidade de rvores e abrange uma rea de oito milhes
de quilmetros quadrados, sendo metade dessa rea em territrio
brasileiro.
Mata Atlntica: Localiza-se na poro leste do Brasil, principalmente
no litoral. Atualmente s existe aproximadamente 7% da sua rea
original. Possui um a floresta densa com uma grande variedade de
espcies.
Cerrado: constitudo por dois estratos: um superior, composto de
arbustos e de pequenas rvores retorcidas e dispersas, e um inferior,
formado de gramneas. Predomina na parte central do Brasil.
Caatinga: Predomina em clima semirido e abrange todos os estados
nordestinos. A vegetao do tipo xerfita, com folhas pequenas e
algumas com espinhos alm de possurem caules que armazenam
gua.
Pantanal: um complexo heterogneo composto de espcies
vegetais do cerrado, pampas, Amaznia, e mata atlntica. Situa-se
no sul do Mato Grosso e no noroeste do Mato Grosso do Sul possui
perodos de seca e de inundaes, com isso, desenvolve-se
espcies xerfilas e higrfilas.
Pampa: Predomina no sul do Rio Grande do Sul, em clima
subtropical, o relevo baixo, com suaves ondulaes e coberto por
vegetao herbcea.
Xerfila:_________________________________________________
Higrfila:________________________________________________

SECRETARIA DE ESTADO DA EDUCAO

100

De acordo com a classificao climtica exposta quais tipos de clima


predominam na Paraba?
A) Equatorial e Tropical
B) Semirido e Tropical de Altitude
C) Subtropical e Semirido
D) Tropical e Semirido
E) Tropical Atlntico e Semirido
04. O comportamento mdio do clima equatorial mido
A possui as quatro estaes bem definidas.
B quente e mido com grandes ndices pluviomtricos.
C as temperaturas so baixssimas podendo at precipitar granizo.
D a pluviosidade muito baixa podendo passar meses sem chover.
E apresenta temperaturas elevadssimas e pouca chuva
05. Assinale a alternativa que apresenta a maior bacia hidrogrfica
localizada em territrio totalmente brasileiro.
a) Bacia do Parnaba
b) Bacia do Tocantins-Araguaia
c) Bacia Amaznica
d) Bacia do So Francisco
e) Bacia do Paran
06. A bacia hidrogrfica brasileira com maior possibilidade de
navegao :
a) Bacia do So Francisco;
b) Bacia do Paran;
c) Bacia do Uruguai;
d) Bacia Amaznica;
e) Bacia do Paraba do Sul.

QUMICA
GEOGRAFIA

EXERCCIOS DE FIXAO
AULA 14: Estrutura fundiria brasileira, reforma agrria e
desenvolvimento sustentvel

01. Sobre a estrutura fundiria e as relaes de trabalho no campo


brasileiro, assinale a alternativa correta:

Para entender a estrutura fundiria de um pas preciso saber


como a terra dividida e utilizada, quem so os proprietrios e qual o
tipo de mo de obra empregada no cultivo da terra. Para
compreender a estrutura fundiria do Brasil, preciso se reportar
herana colonial, mas tambm como as polticas territoriais do
governo brasileiro influenciaram na composio dessa estrutura e
suas caractersticas.

a) a estrutura fundiria apresenta acentuada concentrao da


propriedade decorrente das formas de apropriao das terras, desde
o perodo colonial.
b) a partir de 1850, com a Lei de Terras, todos os trabalhadores
rurais passaram a ter acesso terra.
c) a modernizao do campo proporcionou a extino dos contratos
de parceria em todas as regies brasileiras.
d) nas reas de fronteiras agrcolas, todos os trabalhadores rurais
possuem ttulos de propriedade da terra.
e) os boias-frias so assalariados que trabalham nas propriedades
de forma permanente e com vnculo empregatcio.
02.

Fonte: Incra, 1998

Em 1822, o fim do regime de sesmarias criou condies para que


as unidades familiares de subsistncia pudessem ocupar
regularmente a terra por meio da posse. No entanto, os latifundirios
fizeram valer apenas os prprios interesses; com isso, a terra
continuou a ser um privilgio das elites, e no um direito dos que
trabalhavam nela.
Reforma agrria
Com os srios problemas de concentrao de terras ainda
existentes no pas, novos movimentos de trabalhadores rurais tm
surgido para organizar a presso poltica sobre os governantes,
visando resoluo do problema.
Uma das principais estratgias do movimento de trabalhadores
sem-terra a ocupao de grandes propriedades improdutivas ou
sem registro legal, forando o Instituto Nacional de Reforma Agrria
(Incra) a desapropriar a rea para o assentamento de famlias de
agricultores.
Reforma Agrria: _________________________________________
_______________________________________________________
Desenvolvimento sustentvel
A agroecologia rene um conjunto de prticas agrcolas que
visam a tirar proveito de processos naturais de fertilidade do solo e
de controle biolgico de pragas, buscando-se o equilbrio entre as
condies ambientais e a interveno humana, alm de um alimento
mais saudvel.
A agroecologia engloba uma variedade muito grande de
experincias que buscam a sustentabilidade da agricultura familiar,
estimulando o agricultor a produzir sem depender de insumos
industriais, como os agrotxicos, e valorizando os conhecimentos
tradicionais dos pequenos agricultores.

O grfico representa a relao entre o tamanho e a totalidade dos


imveis rurais no Brasil. Que caracterstica da estrutura fundiria
brasileira est evidenciada no grfico apresentado?
a) A concentrao de terras nas mos de poucos.
b) A existncia de poucas terras agricultveis.
c) O domnio territorial dos minifndios.
d) A primazia da agricultura familiar.
e) A debilidade dos plantations modernos.
03. A maioria das pessoas daqui era do campo. Vila Maria hoje
exportadora de trabalhadores. Empresrios de Primavera do
Leste, Estado de Mato Grosso, procuram o bairro de Vila Maria para
conseguir mo de obra. gente indo distante daqui 300, 400 quilmetros
para ir trabalhar, para ganhar sete conto por dia. (Carlito, 43 anos,
maranhense, entrevistado em 22/03/98).
O texto retrata um fenmeno vivenciado pela agricultura brasileira
nas ltimas dcadas do sculo XX, consequncia
a) dos impactos sociais da modernizao da agricultura.
b) da recomposio dos salrios do trabalhador rural.
c) da exigncia de qualificao do trabalhador rural.
d) da diminuio da importncia da agricultura.
e) dos processos de desvalorizao de reas rurais.
04. Entre os princpios orientadores do processo de converso em
Agroecologia, pode-se destacar todos os indicados abaixo, exceto:

Experincias em agroecologia
Agricultura biodinmica: utiliza preparados que incrementam e
dinamizam a capacidade intrnseca da planta a ser produtora de
nutrientes.
Agricultura orgnica:_______________________________________
_______________________________________________________

a) Controlar pragas, doenas e ervas invasoras.


b) Enfatizar a conservao do solo, gua, energia e recursos
biolgicos.
c) Restabelecer as relaes biolgicas que podem ocorrer
naturalmente na unidade produtiva.
d) Incorporar a ideia de sustentabilidade em longo prazo no desenho
e manejo geral do agro ecossistema.
e) Mover-se de um manejo de nutrientes, cujo fluxo passa atravs do
sistema, para um manejo baseado na reciclagem de nutrientes.

Permacultura: Busca a harmonia entre os elementos existentes no


assentamento rural, trabalhando principalmente com culturas
perenes e utilizando as culturas anuais em sistema de rotao nos
espaos das culturas perenes em crescimento.

SECRETARIA DE ESTADO DA EDUCAO

101

QUMICA
GEOGRAFIA

AULA 15: Dinmicas demogrficas

EXERCCIOS DE FIXAO

Entender o perfil da populao de uma pas fundamental para o


delineamento de polticas pblicas nas reas de sade, educao,
habitao e sistema previdencirio, bem como para a elaborao de
anlises, estudos e prognsticos sobre o desenvolvimento
demogrfico e socioeconmico da nao.
Um dos aspectos primordiais no estudo das populaes o
estudo das dinmicas demogrficas. Para isso preciso considerar o
crescimento vegetativo que a diferena entre as taxas de
____________________ e ______________________.
A dinmica demogrfica tambm considera a taxa de
fecundidade que _________________________________________

01. Embora o Brasil esteja colocado entre os pases mais populosos


do mundo, quando se relaciona sua populao total com a rea do
pas, obtm-se um nmero relativamente baixo. A essa relao de
populao x rea, damos o nome de:

_______________________________________________________
Se a taxa de mdia de fertilidade for igual a 2,1, considera-se
que houve reposio populacional; se for superior a esse nmero, a
populao cresceu; e, se for inferior, a populao diminuiu.
Transio demogrfica
A transio demogrfica divide-se em trs fases. Na primeira, as
taxas de mortalidade diminuem, mas o nmero de nascimentos
continua elevado. O resultado o aumento significativo da taxa de
crescimento da populao. A segunda fase caracteriza-se pela
queda simultnea das taxas de natalidade e de mortalidade. Na
terceira fase, a queda da natalidade mais acentuada do que a
queda na mortalidade. Ou seja, a populao envelhece e, apesar do
maior acesso aos cuidados com a sade, os bitos so iguais ou
superiores ao nmero de nascimentos.
O modelo de transio demogrfica apenas uma referncia
terica, mas as transformaes demogrficas ocorridas ao longo dos
sculos XIX e XX reforam os princpios que sustentam o modelo:
nesse perodo, a maior parte dos pases conheceu uma fase mxima
de incremento populacional, seguida por um movimento de declnio
ou de estabilizao.

a) Taxa de crescimento.
b) ndice de desenvolvimento.
c) Densidade demogrfica.
d) Taxa de natalidade.
e) Taxa de fertilidade.
02. Considerando-se a distribuio da populao mundial por
atividades econmicas, incorreto afirmar que
a) a repartio da PEA pelos setores de atividades reflete o grau de
desenvolvimento econmico
b) o setor tercirio apresenta-se em expanso em quase todos os
pases do mundo;
c) em diversos pases subdesenvolvidos, o nmero de pessoas
empregadas no setor secundrio vem aumentando devido
existncia de um processo de industrializao;
d) os pases subdesenvolvidos apresentam geralmente um setor
tercirio hipertrofiado;
e) em todos os pases subdesenvolvidos, de economia capitalista, o
predomnio dos setores primrio e secundrio reflete o elevado poder
aquisitivo da sociedade.
.
03. Leia com ateno a notcia que se segue:

Estrutura etria da populao

Frana pagar 750 euros mensais por terceiro filho


O governo francs ir pagar uma licena de 750 euros (cerca de R$
2.050,00) por ms durante um ano a famlias que decidirem ter um
terceiro filho, anunciou ontem o primeiro ministro do pas, Dominique
Villepin.
Folha de S. Paulo, 23/09/2005. Folha mundo, p. A-16.
A medida anunciada pelo governo francs est diretamente
relacionada:

A estrutura etria da populao pode ser retratada por meio de


grficos em forma de pirmides. Nas ordenadas, so colocadas os
grupos de idade; nas abscissas, o contingente populacional (em
nmeros absolutos ou em percentuais) enquadrado em cada um
dos grupos a idade.
A forma da pirmide estaria de um pas constantemente
associada ao seu grau de desenvolvimento. As pirmides etrias
referentes a pases subdesenvolvidos costumam apresentar uma
base larga ou seja, tm um grande nmero de crianas e jovens,
resultante das altas taxas de natalidade e um topo estreito, em
virtude da baixa expectativa de vida da populao.

a) poltica anti-imigrao (xenfoba) e de purificao racial adotada


pela Frana nas ltimas dcadas.
b) s elevadas taxas de natalidade verificadas no pas e em toda a
Europa.
c) sobrecarga no sistema de previdncia social francs, em que um
nmero cada vez menor de jovens precisa sustentar um nmero
cada vez maior de aposentados.
d) aproximao do governo francs com as ideias da Igreja
Catlica, que probe o uso de mtodos contraceptivos no naturais.
e) ideia imperialista de que o poderio econmico de uma nao
est diretamente ligado ao tamanho de sua populao.

04. Existem duas formas principais de se abordar o quantitativo


populacional em um espao. De um lado temos as taxas de
______________________,

que

representam

nmero

de

habitantes por quilmetro quadrado; de outro, temos as taxas de


______________________, que esto relacionais ao nmero de
habitantes independente do tamanho do territrio.
A alternativa que completa corretamente as lacunas acima :
a) densidade demogrfica e superpovoamento
b) crescimento vegetativo e populao absoluta
A estrutura etria da populao tem reflexos importantes na
economia de um pas. A populao economicamente ativa (PEA)
responsvel pelo recolhimento de impostos que ajuda o Estado a
sustenta a economia nacional. Uma defasagem muito grande no
nmero de ativos em relao aos inativos desequilibra essa
equao.

c) populao local e populao geral


d) densidade demogrfica e populao absoluta
e) crescimento vegetativo e populao geral.

PEA: ___________________________________________________

SECRETARIA DE ESTADO DA EDUCAO

102

QUMICA

GEOGRAFIA

AULA 16: Blocos regionais

EXERCCIOS DE FIXAO

O mundo globalizado necessita de pases com economias


baseadas no mercado, com uma trama de redes que cooperem entre
si para a abertura dos mercados. Porm, os pases no se inseriram
da mesma forma na economia mundial. O atraso econmico de
muitos pases resultado de um processo histrico.
A economia capitalista desenvolveu-se concentrando riqueza e
poder nas mos das elites, principalmente das potncias dominantes,
criando, em
contrapartida, regies pouco desenvolvidas
economicamente e pouco industrializadas, chamadas a partir da
segunda metade do sculo XX de subdesenvolvidas.

01. Os blocos econmicos tm por objetivo a reduo e/ou


eliminao das tarifas alfandegrias. Sua formao proporciona
maior dinamismo nas relaes comerciais entre os pases
integrantes. Nesse sentido, os pases da Amrica Platina, juntamente
com o Brasil, integram um bloco econmico visando maior
flexibilidade nas importaes e exportaes de produtos. Esse bloco
:

Principais blocos econmicos regionais


A disputa pelo mercado global regionalizou-se a partir da
formao de grandes blocos econmicos ou megablocos, liderados
pelos principais polos capitalistas.
Entre os principais esto: Acordo Norte-Americano de LivreComrcio (NAFTA), liderado pelos Estados Unidos; os pases
europeus reuniram-se na Unio Europeia (UE); e o Japo exerce
grande influncia sobre a Cooperao Econmica da sia e do
Pacfico (Apec).
UE: ____________________________________________________
_______________________________________________________
NAFTA:_________________________________________________
_______________________________________________________
APEC:__________________________________________________
_______________________________________________________
Tipos de acordos
Mercado comum: Alm da eliminao ou reduo de taxas
alfandegrias entre os pases, da rea de livre-comrcio e da
abertura de mercados internos, prope regras comuns de comrcio
com pases exteriores ao bloco e o fluxo comercial, de capitais, de
mo de obra e de servios entre os pases do bloco.
Unio monetria: Inclui a coordenao de polticas econmicas e de
defesa.
Zona Franca: Regio dentro de um pas que recebe estmulos ao
comrcio e indstria, como iseno ou reduo de impostos,
facilidades de importao e exportao.
Zona de livre-comrcio: rea que abrange diversos pases cujo
objetivo a eliminao ou reduo de taxas alfandegrias entre os
pases.
BRICS
At 2006, os BRICS no estavam reunidos em mecanismo que
permitisse a articulao entre eles. O conceito expressava a
existncia de quatro pases que individualmente tinham
caractersticas que lhes permitiam ser considerados em conjunto,
mas no como um mecanismo. Isso mudou a partir da Reunio de
Chanceleres dos quatro pases organizada margem da 61.
Assembleia Geral das Naes Unidas, em 23 de setembro de 2006.
Este constituiu o primeiro passo para que Brasil, Rssia, ndia e
China comeassem a trabalhar coletivamente. Pode-se dizer que,
ento, em paralelo ao conceito BRICs passou a existir um grupo
que passava a atuar no cenrio internacional, o BRIC. Em 2011,
aps o ingresso da frica do Sul, o mecanismo tornou-se o BRICS.
PIB dos BRICS: Brasil:_____________ ; Rssia:________________

a) APEC Cooperao Econmica da sia e do Pacfico.


b) CAN Comunidade Andina.
c) Mercosul Mercado Comum do Sul.
d) Nafta Acordo de Livre Comrcio da Amrica do Norte.
e) Caricom Mercado Comum e Comunidade do Caribe
02. A tentativa de criar uma imensa rea de comrcio entre as
Amricas um sonho antigo dos Estados Unidos, desde 1823 os
Norte Americanos tentam incansavelmente consolidar seu controle
nas Amricas, uma das formas foi a constituio de um bloco unindo
as trs Amricas que seria a formao do (da):
(A) ALCA
(B) MERCOSUL
(C) UE
(D) APEC
(E) NAFTA
03. A Argentina, o Brasil, o Paraguai e o Uruguai formam o Mercosul
(Mercado Comum do Sul), o organismo que estabelece as regras e
os procedimentos para a integrao econmica entre os quatro
pases. Sobre esse bloco econmico, correto afirmar que:
a) integra pases com povoamento, dinmica econmica e nvel de
renda iguais.
b) estabelece fronteiras abertas para o livre deslocamento de
pessoas, produtos e capitais.
c) No permite a livre circulao dos bens industriais, ou seja, no h
relaes comerciais.
d) restringe os fluxos migratrios devido s rivalidades histricas
existentes dentro do bloco.
e) apresenta uma moeda nica com parte de unio econmica entre
os pases pertencentes ao bloco.
04. Os blocos econmicos tm por objetivo a reduo e/ou
eliminao das tarifas alfandegrias. Sua formao proporciona
maior dinamismo nas relaes comerciais entre os pases
integrantes. Nesse sentido, o Mxico se aproximou dos Estado
Unidos e Canad, atravs do bloco:
a) U. E Unio Europeia
b) APEC Cooperao Econmica da sia e do Pacfico.
c) CAN Comunidade Andina.
d) Mercosul Mercado Comum do Sul.
e) Nafta Acordo de Livre Comrcio da Amrica do Norte.
05. Trata-se de um pas que faz parte dos chamados BRICS e que
sofreu com uma profunda crise ao longo dos anos 1990, superando
as suas dificuldades econmicas a partir das exportaes de
combustveis fsseis, sobretudo o petrleo. Apesar de no ser uma
nao desenvolvida, respeitada e temida por muitos pases em
razo de seu poderio militar.
O pas ao qual o texto faz referncia :
a) Brasil
b) Rssia
c) ndia
d) China
e) frica do Sul

ndia:________________ ; China:__________________ ; frica do


Sul:________________; Total: ____________________.

SECRETARIA DE ESTADO DA EDUCAO

103

QUMICA
GEOGRAFIA

AULA 01: INTRODUO FILOSOFIA

EXERCCIOS DE FIXAO
1) (Dinho Zambia - 2015) A filosofia comea dizendo no s crenas
e aos preconceitos do dia a dia para que possam ser avaliados
racional e criticamente. Por isso diz que no sabemos o que
imaginvamos saber ou, como dizia Scrates, comeamos a buscar
o conhecimento quando somos capazes de dizer:
a) Quanto maior sua Dvida maior ser sua Certeza.
b) S quem se humilha est apto a receber o ensinamento.
c) "S sei que nada sei".
d) Tens que esvaziar teu prato para receber novo alimento.
e) S a Cincia nos leva a certeza, o resto mero mito.
2) (UEPE) A questo do conhecimento esteve na pauta dos filsofos
desde o comeo da Filosofia. Esse interesse deve-se ao fato de que
foi sempre presente a discusso em torno da questo da verdade.
Nesse sentido, ao tratar do conhecimento, a filosofia pretende:

Ao analisar a charge acima responda uma questo vital para o


incio do nosso estudo: Por que estudar Filosofia?
_______________________________________________________
_______________________________________________________
_______________________________________________________
A Atitude Filosfica positiva, isto , uma interrogao sobre o que
so as coisas, as ideias, os fatos, as situaes, os comportamentos,
os valores, ns mesmos. tambm uma interrogao sobre porque
e como disso tudo e de ns prprios. "O que e?", "Por que ?",
"Corno ?". Essas so as indagaes fundamentais da Atitude
Filosfica.

I. afirmar que nosso pensamento parece seguir certas leis ou regras


para conhecer as coisas.
II. defender que h uma diferena entre perceber e pensar.
III. indagar a capacidade humana de conhecer.
IV. dizer que no existe conhecimento, se o sujeito s consegue
represent-lo mentalmente.
V. defender que o conhecimento se confunde com a relao entre o
pensamento e as coisas.
Somente est CORRETO o que se afirma em:
a) II e III.
b) II, IV e V.
c) I e III.
d) I, III e IV.
e) I, II, III e V.
3) (UPE) A atitude filosfica inicia-se indagando "O que ?", "Como
?", "Por que ?", "Para que ?", dirigindo-se ao mundo que nos
rodeia e aos seres humanos que nele vivem e com ele se
relacionam. Estas so perguntas sobre:

Qual a indagao filosfica provacada pela personagem


Mafalada?

a) A essncia, a significao, a origem e a finalidade de todas as


coisas;
b) O conhecer, o falar, o pensar e o agir, prprios dos seres
humanos;
c) A capacidade, a finalidade, o conceito e a origem do mundo;
d) Os motivos, as razes, as causas e os interesses para
pensarmos;
e) O contedo, o sentido, a inteno e a finalidade do que
pensamos.

_______________________________________________________
_______________________________________________________
_______________________________________________________
Definio de Filosofia
A filosofia surge enquanto cincia na Grcia Antiga, como uma
atividade especial do homem sbio, o amigo do saber (filo + sophia =
amor sabedoria). Desde ento inmeras foram as tentativas de
definir exatamente o que procura e o que faz um filsofo. Todos
reconhecem a sua importncia e a imensa utilidade, so porm
imprecisos e divergem em relao a determinar qual a sua
verdadeira cincia. Aristteles, discpulo de Plato e fundador do
Liceu, uma escola voltada para o saber e a cincia que ele instalou
em Atenas no sculo IV a.C., fez uma das mais claras exposies
sobre as qualidades da filosofia.
O filsofo um amante do conhecimento isso significa que ele
no dono do saber, nem uma espcie de guru ou sbio dogmtico.
O filsofo apenas um amigo e amante da verdade que est fora de
seu domnio, seu papel buscar a sabedoria, relacionar-se com ela e
dividi-la com os outros.
Qual a diferena entre Conhecimento e Sabedoria?
_______________________________________________________
_______________________________________________________
_______________________________________________________

SECRETARIA DE ESTADO DA EDUCAO

104

QUMICA
FILOSOFIA

AULA 02: FILOSOFIA GREGA

EXERCCIOS DE FIXAO

1) Para Aristteles, o ser a substncia, cada indivduo


particular constitudo por uma forma, que responde pela
essncia imutvel do ser, unida matria, que garante o
movimento, a diversidade, a transformao. Assim sendo:
a) o ser existe primeiramente como a essncia expressa em
sua forma;
b) matria e forma determinam a existncia do ser como um
composto;
c) o que define o ser exclusivamente sua essncia;
d) o ser depende de uma causa final e uma causa formal
excludentes
e) a substncia causa do ser enquanto ser, e exclui o dever.

Por que, mesmo sabendo que haviam povos contemporneos, ou


at anteriores aos gregos, atribui-se Filosofia, uma criao da
Grcia Antiga?
______________________________________________________
______________________________________________________
______________________________________________________
Filsofos
Tales
Anaximandro
Anaxmenes
Xenfanes
Pitgoricos
Herclito

Parmnides

Empdocles

Anaxgoras

Demcrito

OS PR-SOCRTICOS
Physis
gua
Aperon (infinito, indeterminado, ilimitado
Ar
Deus
Nmero
A prpria mudana e a prpria pluralidade
enfim, o prprio devir, cujo smbolo seria o
fogo.
O Ser, nica realidade de fato existente
(eterno, indestrutvel, inabalvel, uno,
idntico, indivisvel, etc). Toda mudana e
toda pluralidade (No Ser) seriam ilusrias.
Quatro Elementos (gua, fogo, terra e ar)
movidos por duas foras (dio: fora de
repulsa e Amor: fora de atrao)
Homeomerias (partculas fundamentais da
realidade: cada uma seria uma mistura
minscula de todos os elementos que
compem o cosmo
tomos
OS FILSOFOS SOCRTICOS

2) "Tem presente, portanto, que concordaste que tambm


justo cometer atos prejudiciais aos governantes e aos mais
poderosos, quando os governantes, involuntariamente,
tomam determinaes inconvenientes para eles - uma vez
que declaras ser justo que os sditos executem o que
prescreveram os governantes." (Plato, Repblica, 339e)
Em meio discusso sobre a justia, Scrates sintetiza do
modo acima citado a posio de Trasmaco.
As afirmaes de Trasmaco que do base a essa sntese de
Scrates so:
a) Justia fazer o mal para os mais fracos; os que
obedecem devem, s vezes, recusar-se a obedecer aos mais
fortes.
b) Justia fingir que se faz o bem geral, visando ao proveito
prprio; deve-se fazer os fracos pensarem que as decises
dos fortes os favorecem.
c) justo obedecer queles que governam; os governantes
so passveis de cair em erro em suas decises.
d) Justia aquilo que convm ao mais forte; os governantes
nunca se enganam ao intentar tomar decises que lhes
favorecem.
3) Na gerao do cosmo, a preponderncia terra, ao fogo,
gua e ao ar costuma ser atribuda, respectivamente, aos
seguintes pensadores pr-socrticos:
a) Parmnides, Herclito, Tales e Pitgoras.
b) Xenfanes, Herclito, Tales e Anaxmenes.
c) Pitgoras, Anaxgoras, Tales e Anaximandro.
d) Emp

SCRATES
_______________________________________________________
_______________________________________________________
_______________________________________________________
PLATO
_______________________________________________________
_______________________________________________________
_______________________________________________________
ARISTTELES
_______________________________________________________
_______________________________________________________
_______________________________________________________
EPICURO
_______________________________________________________
_______________________________________________________
_______________________________________________________

SECRETARIA DE ESTADO DA EDUCAO

105

QUMICA
FILOSOFIA

AULA 03: FILOSOFIA ROMANA E MEDIEVAL

AULA 04: FILOSOFIA MODERNA

_______________________________________________________
_______________________________________________________
_______________________________________________________

PLOTINO - Trs coisas conduzem a Deus: A msica, o amor e a

filosofia.
_______________________________________________________
_______________________________________________________
_______________________________________________________

Frequentemente, os historiadores da filosofia designam como


filosofia moderna aquele saber que se desenvolve na Europa
durante o sculo XVII tendo como referncias principais o
cartesianismo isto , a filosofia de Ren Descartes , a cincia
da Natureza galilaica isto , a mecnica de Galileu Galilei , a
nova idia do conhecimento como sntese entre observao,
experimentao e razo terica baconiana isto , a filosofia de
Francis Bacon e as elaboraes acerca da origem e das formas
da soberania poltica a partir das idias de direito natural e direito
civil hobbesianas isto , do filsofo Thomas Hobbes.

EPTETO - impossvel para um homem aprender aquilo que ele

acha que j sabe.

FILOSOFIA ABSOLUTISTA

_______________________________________________________
_______________________________________________________
_______________________________________________________

MARCO AURLIO Compete a vec!


_______________________________________________________
_______________________________________________________
_______________________________________________________

A vida de um homem o que seus pensamentos fazem dela.


Nunca discuta com um superior, voc corre o risco de ter razo.

Concentrao de todo poder e autoridade na pessoa do Rei;


Poder da Igreja;
Mercantilismo;
REI + BURGUESIA = ABSOLUITUSMO
MAQUIAVEL

Uma das alternativas para construir um governo forte seria a


separao entre moral e politica, uma vez que as razes do
Estado deveriam ser superiores a quaisquer valores culturais
e sociais da nao.
_______________________________________________________

JEAN BODIN
O Poder Supremo sobre os cidados e sditos, sem
restries determinadas pelas leis.

FILOSOFIA MEDIEVAL

As caractersticas e principais questes debatidas e


analisadas pelos filsofos medievais so:
Relao entre razo e f; Existncia e natureza de Deus;
Fronteiras entre o conhecimento e a liberdade humana;
Escolstica:
_______________________________________________________
_______________________________________________________
_______________________________________________________

Santo Agostinho:
_______________________________________________________
_______________________________________________________
_______________________________________________________

So Toms de Aquino:
_______________________________________________________
_______________________________________________________
_______________________________________________________

THOMAS HOBBES
No estado de natureza, imperava originalmente a guerra de
todos contra todos; que, para por fim a essa situao de
violncia e anarquia, os homens firmaram um pacto
(CONTRATO SOCIAL), renunciando Liberdade em troca da
Segurana oferecida pelo Estado, cuja soberania sobre os
sditos tornou-se absoluta.
_______________________________________________________

HUGO GROTIUS
Um dos fundadores do Direito Internacional, Do direito da
Paz e da Guerra
A ordem interna da sociedade s poderia ser preservada pelo
Estado por meio da autoridade ilimitada do soberano.
JACQUES BOSUET
O rei era o representante de Deus e, como tal, responsvel
apenas perante Ele por seus atos de governo.
Cardeal e orador da corte de Lus XIV, escreveu A Politica
inspirada nas Sagradas Escrituras.
Formulou a doutrina:
_______________________________________________________
_______________________________________________________
_______________________________________________________
_______________________________________________________

Como podemos explicar o questionamento do Sagrada


na charge acima?
_______________________________________________________
_______________________________________________________
_______________________________________________________

SECRETARIA DE ESTADO DA EDUCAO

106

QUMICA
FILOSOFIA

FILOSOFIA ILUMINISTA
A filosofia iluminista centrava-se em dois aspectos para

explicar todas as coisas: razo e cincia


REN DESCARTES
Fundador da Filosofia Moderna
Discurso sobre o Mtodo, 1637: fundamentos do
racionalismo.
preciso partir de Axiomas = verdades indiscutveis que no
precisam ser demonstradas
_________________________________________________

4) (ENEM 2013) Nasce daqui uma questo: se vale mais ser


amado que temido ou temido que amado. Responde-se que
ambas as coisas seriam de desejar; mas porque difcil
junt-las, muito mais seguro ser temido que amado,
quando haja de faltar uma das duas. Porque dos homens
que se pode dizer, duma maneira geral, que so ingratos,
volveis, simuladores, covardes e vidos de lucro, e
enquanto lhes fazes bem so inteiramente teus, oferecem-te
o sangue, os bens, a vida e os filhos, quando, como acima
disse, o perigo est longe; mas quando ele chega, revoltamse.
MAQUIAVEL, N.

Com o mtodo dedutivo matemtico se deve alcanar


verdades mais amplas.
JOHN LOCKE
Acreditava que o homem adquiria conhecimento com o
passar do tempo atravs do EMPIRISMO
VOLTAIRE

Defendia a Liberdade de Pensamento e no poupava crtica


a intolerncia religiosa.
JEAN-JACQUES ROUSSEAU
Defendia a ideia de um Estado Democrtico que garantia a
IGUALDADE para todos.
MONTESQUIEU
A Tripartio do Poder:
_________________________________________________
DENIS DIDEROT / JEAN DALEMBERT:
_________________________________________________
BENTO ESPINOSA
Defendeu principalmente a TICA e o Pensamento Lgico
ADAM SMITH
Economista e Filosofo ingls. O Grande Defensor do
Liberalismo Econmico
_________________________________________________
IMMANUEL KANT
Epistemologia, tica e Metafsica.
_______________________________________________________
_______________________________________________________
______________________________________________________

EXERCCIOS DE FIXAO
1) Os governantes absolutistas de alguns pases europeus
adotaram certos princpios do iluminismo, promovendo em
seus estados uma srie de reformas nos campos social e
econmico. Esses governantes ficaram conhecidos como:
a) Iluministas
b) Senhores Feudais
c) Dspotas esclarecidos
d) fisiocratas
e) conservadores
2) O termo vem do grego que significa natureza e poder.
Designa o governo comandado pelas leis da natureza:
a) Fisiocracia
b) Lei iluminista c) Iluminismo
d) feudalismo
e) filosofia
3) Assinale a alternativa que apresenta uma caracterstica
que o iluminismo no defendia:
a) Igualdade
b) Escravido
c) Tolerncia religiosa
d) Liberdade social e pessoal
e) Propriedade privada

SECRETARIA DE ESTADO DA EDUCAO

107

rncipe Rio de Janeiro: Bertrand, 1991.

A partir da analise histrica do comportamento humano em


suas relaes sociais e politicas, Maquiavel define o homem
como um ser
a) munido de virtude, com disposio nata a praticar o bem a
si e aos outros.
b) possuidor de fortuna, valendo-se de riquezas para
alcanar xito na politica.
c) guiado por interesses, de modo que suas aes so
imprevisveis e inconstantes.
d) naturalmente racional, vivendo em um estado pr-social e
portando seus direitos naturais.
e) socivel por natureza, mantendo relaes pacficas com
seus pares.
5) (ENEM 2012)
TEXTO I
Experimentei algumas vezes que os sentidos eram
enganosos, e de prudncia nunca se fiar inteiramente em
quem j nos enganou uma vez.
DESCARTES, R.

edita es

etafsicas.

o Paulo: Abril Cultural,1979

TEXTO II
Sempre que alimentarmos alguma suspeita de que uma ideia
esteja sendo empregada sem nenhum significado,
precisaremos apenas indagar: de que impresso deriva esta
suposta ideia? E se for impossvel atribuir-lhe qualquer
impresso sensorial, isso servir para confirmar nossa
suspeita.
HUME, D. ma in esti ao sobre o entendimento o aulo
Unesp, 2004 (adaptado).

Nos textos, ambos os autores se posicionam sobre a


natureza do conhecimento humano. A comparao dos
excertos permite assumir que Descartes e Hume
a) defendem os sentidos como critrio originrio para
considerar um conhecimento legitimo.
b) entendem que desnecessrio suspeitar do significado de
uma ideia na reflexo filosfica e critica.
c) so legtimos representantes do criticismo quanto
gnese do conhecimento.
d) concordam que conhecimento humano impossvel em
relao as ideias e aos sentidos.
e) atribuem diferentes lugares ao papel dos sentidos no
processo de obteno do conhecimento.
6) (UFSM-PEIES) A tica normativa de Kant prope como
fundamento ltimo, o imperativo categrico que afirma, numa
das suas formulaes:
Procede apenas segundo aquela mxima, em virtude da
qual podes querer ao mesmo tempo que ela se torne em lei
universal.
O imperativo pretende garantir
I. a moralidade do agir.
III. a heteronomia do agir
II. a autonomia do agir.
Est(o) correta(s) a(s) alternativa(s)
a) I apenas.
d) I e II apenas
b) II apenas.
e) I e III apenas
c) III apenas.

QUMICA
FILOSOFIA

AULA 05: FILOSOFIA CONTEMPORNEA I

Algumas perguntas que so tpicas:


- Ser que a cincia poder resolver todos os problemas da
humanidade?
- O homem deve confiar apenas na razo?
- A tecnologia impedir o fim da humanidade?
_______________________________________________________
_______________________________________________________
_______________________________________________________

HORKHEIMER
Parece que enquanto o conhecimento tcnico expande o
horizonte da atividade e do pensamento humanos, a
autonomia do homem enquanto indivduo, a sua capacidade
de opor resistncia ao crescente mecanismo de manipulao
de massas, o seu poder de imaginao e o seu juzo
independente sofreram uma reduo. O avano dos recursos
tcnicos de informao se acompanha de um processo de
desumanizao. (O Eclipse da Razo)
Horkheimer ope o conhecimento tcnico e autonomia do
homem enquanto indivduo. Parece que a tecnologia tem
diminudo a capacidade do ser humano em se opor aos
mecanismos de manipulao do sistema capitalista
Voc concorda com o filsofo? Ser que a tecnologia
desumaniza o homem?
_________________________________________________
_________________________________________________
_________________________________________________
AUGUSTE COMTE
foi um dos principais tericos a pensar a idia de progresso.
Tanto a razo quanto o saber cientfico caminham na direo
do desenvolvimento do homem (o lema da bandeira
brasileira, ordem e progresso, inspirado nas idias de
Comte).
O positivismo uma doutrina filosfica, sociolgica e
poltica. Surgiu como desenvolvimento sociolgico do
iluminismo, das crises social e moral do fim da Idade Mdia e
do nascimento da sociedade industrial - processos que
tiveram como grande marco a Revoluo Francesa (17891799). Em linhas gerais, ele prope existncia humana
valores completamente humanos, afastando radicalmente a
teologia e a metafsica (embora incorporando-as em uma
filosofia da histria).
Quais as caractersticas da Filosofia Positivista?
_________________________________________________
_________________________________________________
_________________________________________________
JEAN-PAUL SARTRE
Tambm pensou as questes do homem frente liberdade e
ao seu compromisso com a histria. Utilizando tambm as
contribuies do marxismo e da psicanlise, o filsofo
elaborou um pensamento sistemtico que pe em relevo a
noo de existncia em lugar da essncia, o
existencialismo.
HEGEL
apontado como o ponto culminante do racionalismo, da
crena que a razo o elemento solucionador dos problemas
humanos. Hegel integra o grupo de pensadores que
defendiam o idealismo.
O que a realidade?

SECRETARIA DE ESTADO DA EDUCAO

108

_________________________________________________
_________________________________________________
_________________________________________________
Esta uma pergunta que Hegel responde com clareza.
Hegel via a realidade como uma unidade orgnica, uma
unidade que no estava numa condio estvel mas num
constante processo de desenvolvimento.
A dialtica no uma forma de pensar a realidade, mas
sim o movimento real da realidade. Por isso, para
acompanhar a realidade, o pensamento tambm deve ser
dialtico. A realidade um contnuo devir, vir a ser. Um
momento prepara outro momento mas, para que esse outro
momento acontea, o anterior tem de ser negado. Esses trs
momentos so comumente chamados de ______,
__________ e ___________.
MARX
Foi um homem e um filsofo preocupado com a explorao
dos trabalhadores.
No famoso Manifesto Comunista, Marx afirma que toda a
histria humana pode ser entendida como uma srie de lutas
de classes.
A LUTA DE CLA E ___________________________
O grande trabalho filosfico de Marx foi o livro O Capital.
Descrito com a bblia da classe operria numa resoluo da
Associao Operria Internacional.
AULA 06: FILOSOFIA CONTEMPORNEA II
O que voc responderia se algum perguntasse: a vida
essencialmente um sofrimento?
_________________________________________________
_________________________________________________
_________________________________________________
SCHOPENHAUER
No sistema de Schopenhauer, a vontade a raiz metafsica
do mundo e da conduta humana; ao mesmo tempo, a fonte
de todos os sofrimentos.
_______________________. Schopenhauer influenciou
uma gerao de artistas, como o cantor e compositor
brasileiro, Renato Russo. O lder do Legio Urbana revelava
em vrias de suas msicas, o que podemos chamar de dor
de viver. Em discos aclamados pela crtica musical, como o
famoso Cinco e A tempestade as composies de Renato
Russo, Dado Villa Lobos e Marcelo Bonf so permeadas de
um pessemismo arrebatador.
Na msica Quando o sol bater na janela do teu quarto,
Renato Russo cita Schopenhauer literalmente:
tudo dor e toda dor vem do
_________________________________.
NIETZCHE

Nos desafia, ele critica a moral crist, analisa a ideia de bem e mal
imposta pelo pensamento religioso na Europa de sua poca.
No sou um homem, sou _______________________.
Como podemos viver em um mundo sem algo (um Deus) que
garanta que a vida tenha sentido?

QUMICA
FILOSOFIA

Niilismo a crena de que nada tem sentido


Torna-te quem tu s!
O padre est mentindo.
Deus est morto mas o seu cadver permanece insepulto.
HEIDEGGER

A destruio do eu: tentando realizar seu projeto, o homem sofre


a interferncia de uma srie de fatores adversos que o desviam de
seu caminho existencial. Trata-se do confronto do eu com os
outros. Um confronto no qual o homem comum , geralmente,
derrotado. O seu eu destrudo, arruinado, dissolve-se na massa
humana. Em vez de tornar-se si mesmo, o homem torna-se aquilo
que os outros desejam.
PAULO FREIRE
Para ele a dialogicidade a essncia da educao como
prtica da liberdade. Por isso afirma que: Se ao dizer suas
palavras, ao chamar o mundo, os homens o transformam, o
dilogo impe-se como o caminho pelo qual os homens
encontram seu significado enquanto homens; o dilogo ,
pois, uma necessidade existencial.
Esse mtodo involudo, antidialgico por essncia,
isto, no comunicativo, onde o educador deposita
educando o contedo programtico da educao, que
mesmo elabora ou elaboram para ele, chamado
__________________________.

por
no
ele
de

EXERCCIOS DE FIXAO
1) Husserl considerava a si mesmo e a Heidegger como os
pensadores que desenvolviam e aplicavam a fenomenologia
na reflexo filosfica, mas o estudo dos pensadores indica
que Heidegger se distanciava do fundador da fenomenologia.
Assinale a alternativa que demonstra esse distanciamento.
a) Heidegger, ao contrrio de Husserl, assumia o conceito de
ser como evidente.
b) Heidegger considerava a fenomenologia como a prpria
forma de se filosofar, enquanto que Husserl a via como
mtodo.
c) Heidegger evidencia uma preocupao com a metafsica,
ao passo que Husserl construiu uma fenomenologia
antimetafsica.
d) Heidegger, ao contrrio de Husserl, buscou resolver o
problema do ser recorrendo conscincia.
e) A reduo em Heidegger se refere a dados existenciais da
conscincia, e em Husserl, ao homem concreto.
2) Em seu livro "Dialtica do Esclarecimento", Adorno e
Horkheimer estudam o conceito de publicidade e seu papel
na sociedade.
Qual das seguintes alternativas no se relaciona com os
efeitos da publicidade na sociedade?
a) A publicidade conduz mimese entre os indivduos.
b) A publicidade e a indstria cultural confundem-se
econmica e tecnicamente.
c) A publicidade serve diretamente venda de mercadorias.
d) A publicidade aprisiona o indivduo junto s grandes
corporaes.
e) A publicidade, ao funcionar pela repetio, assemelha-se
s "palavras de ordem".

SECRETARIA DE ESTADO DA EDUCAO

109

3) No sistema de pensamento cartesiano, como se pode


compreender a Natureza?
a) A Natureza possui um dinamismo prprio, independente
de Deus.
b) A Natureza foi criada por Deus seguindo leis
incompreensveis ao homem.
c) A Natureza consiste numa criao da razo humana, e no
se pode provar sua existncia.
d) A Natureza pode ser compreendida como uma mquina
regida por leis matemticas.
e) A Natureza absolutamente subjetiva, sendo impossvel
que todos a compreendam da mesma forma.
4) Descartes empregou um mtodo universal para o
conhecimento.
Qual das seguintes alternativas no est de acordo com o
mtodo cartesiano?
a) Nada pode ser aceito como verdade mesmo quando
reconhecido como tal.
b) Deve-se dividir os problemas em tantas partes quanto
possvel.
c) A reflexo deve seguir uma ordem definida, comeando
com o que for mais simples.
d) Deve-se ter certeza de que tudo foi examinado, sem
omisses.
e) A ordem da reflexo pode ser inteiramente fictcia.
5) Sobre o funcionamento da indstria cultural em Adorno e
Horkheimer, correto afirmar:
a) Todo o mundo passa pela indstria cultural.
b) Os produtos da indstria cultural exigem mxima ateno.
c) Cada manifestao da indstria cultural reproduz os
homens como algo novo.
d) No se pode violar o sistema da indstria cultural em
hiptese alguma.
e) No h lugar para os mais capazes, somente para os
medocres.
6) Sobre o conceito de liberdade em Sartre, pode-se afirmar
que sua tese central a de que ela deve ser absoluta.
Assinale a alternativa que se coaduna com esta tese.
a) Os valores permitem definir a liberdade para os homens e
suas sociedades.
b) No existe angstia no homem ao se defrontar com a
liberdade.
c) O simples fato da liberdade impe uma forma materialista
de determinismo, em que se abandona a ideia de
conscincia.
d) Os atos livres do ser humano possuem uma essncia
psicolgica que os define e possibilita.
e) preciso excluir a possibilidade da existncia de Deus,
pois sua onipotncia no permite a liberdade humana.

7) O pensamento de Marx pode ser considerado como uma


crtica aos sistemas de pensamento tanto de autores
anteriores quanto contemporneos, embora incorpore
diversos conceitos utilizados por eles.
Qual dos conceitos abaixo no essencial para a
compreenso do materialismo histrico?
a) A dialtica.
b) A escassez.
c) A alienao.
d) O valor-trabalho.
e) O modo de produo.

QUMICA

AULA 01: SOCIOLOGIA

Os diferentes grupos tribais do Brasil se caracterizavam


pela utilizao de uma tcnica rudimentar na obteno dos
meios de subsistncia. Isto se refletia na explorao dos
recursos naturais, bastante limitada, e na maior ocupao do
tempo nas tarefas que garantissem a sobrevivncia. Embora
a caa, a pesca e a coleta fossem atividades comuns a todas
as tribos, assumiam maior importncia para as nmades que
desconheciam a agricultura. A atividade agrcola era
realizada quase que somente em terrenos florestais, dada a
sua fertilidade.

Por que estudar Sociologia?


_________________________________________________
_________________________________________________
_________________________________________________
Os resultados da pesquisa sociolgica no so de interesse
apenas de socilogos. Cobrindo todas as reas do convvio
humano desde as relaes na famlia at a organizao das
grandes empresas, o papel da poltica na sociedade ou o
comportamento religioso, a Sociologia pode vir a interessar,
em diferentes graus de intensidade, a diversas outras reas
do saber.
Qual a principal funo da Sociologia nos dias de hoje?
- Compreender as diversas sociedades e culturas que
existem no mundo;
- Desenvolver as relaes harmoniosas entre os seres
humanos sem nenhum tipo de distino.
O que Sociologia?
Cincia Humana que tem como objetos de estudo a
sociedade, a sua organizao social e os processos que
interligam os indivduos em grupos, instituies e
associaes.

A sociologia a busca pelo conhecimento dos fenmenos


sociais, do comportamento das instituies e das relaes de
convivncia entre os seres humanos.
Quais as sociedades que formamos na nossa vida?
_________________________________________________
_________________________________________________
_________________________________________________
Quais as primeiras sociedades na Histria da
Humanidade?
_________________________________________________
_________________________________________________
_________________________________________________
SOCIEDADES INDGENAS NO BRASIL
Cerca de 3,5 milhes de ndios habitavam o Brasil na poca
do Descobrimento. Dividiam-se em quatro grupos lingusticoculturais: Tupi, J, Aruaque e Caraba. Naquela ocasio, os
Tupis acabavam de ocupar o litoral, expulsando para o
interior as tribos que no fossem Tupis. Portanto, manter
relaes de amizade e aliana com o grupo dominante
passou a ser fundamental para os conquistadores europeus.

SECRETARIA DE ESTADO DA EDUCAO

110

EXERCCIOS DE FIXAO
1) (ENEM - 2013) O socilogo espanhol Manuel Castells
sustenta que a comunicao de valores e a mobilizao em
torno do sentido so fundamentais. Os movimentos culturais
(entendidos como movimentos que tm como objetivo
defender ou propor modos prprios de vida e sentido)
constroem-se em torno de sistemas de comunicao
essencialmente a internet e os meios de comunicao
porque esta a principal via que esses movimentos
encontram para chegar quelas pessoas que podem
eventualmente partilhar os seus valores, e a partir daqui atuar
na conscincia da sociedade no seu conjunto.
Disponvel em: www.compolitica.org. Acesso em: 2 mar. 2012 (adaptado).

Em 2011, aps uma forte mobilizao popular via redes


sociais, houve a queda do governo de Hosni Mubarak, no
Egito. Esse evento ratifica o argumento de que
a) a internet atribui verdadeiros valores culturais aos seus
usurios. s
b) a conscincia das sociedades foi estabelecida com o
advento da internet.
c) a revoluo tecnolgica tem como principal objetivo a
deposio de governantes antidemocrticos.
d) os recursos tecnolgicos esto a servio dos opressores e
do fortalecimento de suas prticas polticas.
e) os sistemas de comunicao so mecanismos importantes
de adeso e compartilhamento de valores sociais.

2) (ENEM - 2013) Mesmo tendo a trajetria do movimento


interrompida com a priso de seus dois lderes, o tropicalismo
no deixou de cumprir seu papel de vanguarda na msica
popular brasileira. A partir da dcada de 70 do sculo
passado, em lugar do produto musical de exportao de nvel
internacional prometido pelos baianos com a retomada da
linha evolutria, instituiu-se nos meios de comunicao e na
indstria do lazer uma nova era musical.
TINHORO, J. R. Pequena histria da msica popular: da modinha
ao tropicalismo. So Paulo: Art, 1986 (adaptado).

A nova era musical mencionada no texto evidencia um


gnero que incorporou a cultura de massa e se adequou
realidade brasileira. Esse gnero est representado pela obra
cujo trecho da letra :
a) A estrela dalva / No cu desponta / E a lua anda tonta /
Com tamanho esplendor. (As pastorinhas, Noel Rosa e Joo
de Barro)
b) Hoje / Eu quero a rosa mais linda que houver / Quero a
primeira estrela que vier / Para enfeitar a noite do meu bem.
(A noite do meu bem, Dolores Duran)
c) No rancho fundo / Bem pra l do fim do mundo / Onde a
dor e a saudade / Contam coisas da cidade. (No rancho
fundo, Ary Barroso e Lamartine Babo)
d) Baby Baby / No adianta chamar / Quando algum est
perdido / Procurando se encontrar. (Ovelha negra, Rita Lee)
e) Pois h menos peixinhos a nadar no mar / Do que os
beijinhos que eu darei / Na sua boca. (Chega de saudade,
Tom Jobim e Vinicius de Moraes)

QUMICA
SOCIOLOGIA

AULA 2 ORIGEM DA SOCIOLOGIA


Socilogos no s esperavam entender o que unia os grupos
sociais, mas tambm desenvolver um "antidoto" para a
desintegrao social.

A Revoluo Industrial
Foi ela que transformou a sociedade rural e agrcola do
mundo ocidental em uma sociedade basicamente urbana e
industrial.
A Revoluo Industrial provocou grandes transformaes
no mundo: a economia transformou-se, pois a atividade
industrial passou a ocupar o centro da vida econmica;
formaram-se grandes empresas industriais e o trabalho
assalariado passou a predominar em toda parte; em outras
palavras, imps-se o capitalismo industrial. A sociedade foi
profundamente afetada pelo xodo rural e pelo crescimento
da vida urbana;
_________________________________________________
_________________________________________________
_________________________________________________
Resuma como se deu as formas de trabalho que variaram
conforme o momento histrico.
Pr-Histria:

_________________________________________________
_________________________________________________
_________________________________________________
Antiguidade Oriental

_________________________________________________
_________________________________________________
_________________________________________________
Antiguidade Ocidental

_________________________________________________
_________________________________________________
_________________________________________________
Sociedade Medieval

_________________________________________________
_________________________________________________
_________________________________________________
Sociedade Moderna

Movimentos Sociais do sculo XIX


O ludismo foi uma das primeiras formas de luta dos
trabalhadores. Eles formavam grupos que invadiam as fbricas e
destruam as mquinas.
Os sindicatos da poca, semelhantes aos de hoje, procuravam
atrair outros trabalhadores e organizar as lutas econmicas contra a
burguesia e a mais influente forma de luta era a greve. A maioria
dos trabalhadores de fbrica cruzava os braos e se recusava a
trabalhar enquanto os patres no atendessem s suas
reivindicaes (op. cit.). As principais exigncias dos trabalhadores
eram aumento de salrio, diminuio da jornada de trabalho e a
proibio do trabalho infantil.
A partir de 1830 formou-se na Inglaterra o movimento cartista. O
cartismo juntava operrios, artesos e at gente da pequena
burguesia. Os cartistas redigiram um documento chamado Carta do
Povo e o enviaram ao Parlamento ingls. A principal reivindicao do
documento era o sufrgio universal masculino. O cartismo organizou
gigantescos comcios em Londres. Mas o Parlamento permaneceu
insensvel. Somente em 1867, os operrios especializados e a
pequena burguesia conquistariam o direito de voto. Apesar disso, o
cartismo foi importante para que o proletariado ingls adquirisse
conscincia poltica.
O termo Sociologia foi criado em 1838 (sc. XIX) por Auguste
Comte, que pretendia unificar todos os estudos relativos ao homem
como a Historia, a Psicologia e a Economia.
Isidore Auguste Marie Franois Xavier (1798-1857) Comte foi um
filsofo francs, fundador da Sociologia e do Positivismo.
EXERCCIOS DE FIXAO
1) (ENEM - 2010) A evoluo do processo de transformao de
matrias-primas em produtos acabados
ocorreu em trs estgios: artesanato, manufatura e maquino fatura.
Um desses estgios foi o artesanato, em que se.
a) trabalhava conforme o ritmo das mquinas e de maneira
padronizada.
b) trabalhava geralmente sem o uso de mquinas e de modo
diferente do modelo de produo em srie.
c) empregavam fontes de energia abundantes para o funcionamento
das mquinas.
d) realizava parte da produo por cada operrio, com uso de
mquinas e trabalho assalariado.
e) faziam interferncias do processo produtivo por tcnicos e
gerentes com vistas a determinar o ritmo de produo.

3) (ENEM - 2013) Na produo social que os homens realizam, eles


entram em determinadas relaes indispensveis e independentes
de sua vontade; tais relaes de produo correspondem a um
estgio definido de desenvolvimento das suas foras materiais de
produo. A totalidade dessas relaes constitui a estrutura
econmica da sociedade fundamento real, sobre o qual se erguem
as superestruturas poltica e jurdica, e ao qual correspondem
determinadas formas de conscincia social.
MARX, K. Prefcio Crtica da economia poltica. In: MARX, K.; ENGELS, F.
Textos 3. So Paulo: Edies Sociais, 1977 (adaptado).
Para o autor, a relao entre economia e poltica estabelecida no
sistema capitalista faz com que
a) o proletariado seja contemplado pelo processo de mais-valia.
b) o trabalho se constitua como o fundamento real da produo
material.
c) a consolidao das foras produtivas seja compatvel com o
progresso humano.
d) a autonomia da sociedade civil seja proporcional ao
desenvolvimento econmico.
e) a burguesia revolucione o processo social de formao da
conscincia de classe.

_________________________________________________
_________________________________________________
_________________________________________________
Os trabalhadores logo perceberam a necessidade de se unir e
lutar por seus direitos. Assim, o comeo da Revoluo Industrial
representou tambm o incio das lutas operrias. Por meio dessas
lutas, os operrios formavam a conscincia de que pertenciam a uma
mesma classe social: o proletariado.

SECRETARIA DE ESTADO DA EDUCAO

111

QUMICA
SOCIOLOGIA

AULA 3 - CULTURA

EXERCCIOS DE FIXAO
1) A ideia principal da charge abaixo :

O antroplogo Claude Lvi-Strauss (1908-2009) diz que a


CULTURA deve ser considerada como um conjunto de sistemas
simblicos, entre os quais se incluem a arte, a cincia, a religio e as
normas econmicas.
DIVERSIDADE o movimento que vai na contracorrente da
monocultura ou cultura nica.
ETNOCENTRISMO uma viso do mundo onde o nosso prprio
grupo tomado como centro de tudo e todos os outros so pensados
e sentidos atravs dos nossos valores, nossos modelos, nossas
definies do que a existncia.

a) a PEC regularizou a profisso;


b) as domsticas esto satisfeitas com seu emprego;
c) as domsticas esto satisfeitas com seu trabalho em casa;
d) as domsticas esto animadas com seu emprego, mas infelizes
em casa;
e) a PEC tambm condicionou as regras da prpria casa.
Como podemos dizer que a charge acima apresenta um
problema social?
_______________________________________________________
_______________________________________________________
_______________________________________________________

2) Um professor do Estado da Paraba que leciona 30 horas/aula por


semana recebe um salrio lquido de R$1.522,12. Um pedreiro em
Joo pessoa, num edifcio sendo construdo no bairro do Altiplano
recebe, como diria, R$80,00, o que equivale a um salrio mensal de
R$2060,00.

CULTURA ERUDITA
_______________________________________________________
_______________________________________________________
_______________________________________________________
CULTURA POPULAR
_______________________________________________________
_______________________________________________________
_______________________________________________________
CULTURA DE MASSA
_______________________________________________________
_______________________________________________________
_______________________________________________________
MACHISMO - consiste num determinado conjunto de atitudes e
ideias que coloca o sexo masculino em um patamar elevado na
sociedade, subjugando o sexo feminino e no admitindo a igualdade
de direitos para o homem e a mulher.1 muito identificado com o
patriarcado, sendo este o nome dado a estrutura que relega
privilgios aos homens.
FEMINISMO - um movimento social, filosfico e poltico que tem
como meta direitos equnimes (iguais) e uma vivncia humana por
meio do empoderamento feminino e da libertao de padres
opressores patriarcais, baseados em normas de gnero. Envolve
diversos movimentos, teorias e filosofias que advogam pela
igualdade entre homens e mulheres, alm de promover os direitos
das mulheres e seus interesses
Voc conhece algum movimento feminista do nosso estado?
_______________________________________________________
_______________________________________________________
_______________________________________________________

SECRETARIA DE ESTADO DA EDUCAO

112

Sabendo disso, podemos dizer que o governo Dilma promete que os


Royalties do Petrleo dirijam-se para a Educao.
O maior problema desse anncio da presidenta :
a) que a desvalorizao do professor continuaria pelas outras
classes;
b) que os salrios daqueles que no precisam necessariamente de
um estudo formal, como os pedreiros, cairiam em relao ao dos
docentes;
c) que s serviu para acalmar os nimos das agitaes sociais, nada
foi providenciado ainda;
d) no adianta investir na estrutura fsica da Educao se o salrio
do professor defasado;
e) os Royalties do Petrleo no so suficientes para atender a
Educao de todo o Brasil.
3) (ENEM - 2011) Na dcada de 1990, os movimentos sociais
camponeses e as ONGs tiveram destaque, ao lado de outros sujeitos
coletivos. Na sociedade brasileira, a ao dos movimentos sociais
vem construindo lentamente um conjunto de prticas democrticas
no interior das escolas, das comunidades, dos grupos organizados e
na interface da sociedade civil com o Estado. O dilogo, o confronto
e o conflito tm sido os motores no processo de construo
democrtica.
SOUZA, M. A. Movimentos sociais no Brasil contemporneo: participao
possibilidades das prticas democrticas. Disponvel em:
http://www.ces.uc.pt. Acesso em: 30 abr. 2010 (adaptado)

Segundo o texto, os movimentos sociais contribuem para o processo


de construo democrtica, porque
a)
determinam
o
papel
do
Estado
nas
transformaes socioeconmicas.
b) aumentam o clima de tenso social na sociedade civil.
c) pressionam o Estado para o atendimento das demandas da
sociedade.
d) privilegiam determinadas parcelas da sociedade em detrimento
das demais.
e) propiciam a adoo de valores ticos pelos rgos do Estado.

QUMICA
SOCIOLOGIA

AULA 4 : OS PRINCIPAIS SOCILOGOS

EXERCCIOS DE FIXAO

AUGUSTO COMTE (1798-1857)


Sociologia, que a princpio Comte denominou "Fsica Social", um
vocbulo criado por ele no seu Curso de Filosofia Positiva. Para
Comte, a Sociologia procura estudar e compreender a sociedade,
para organiz-la e reform-la depois. Acreditava que os estudos das
sociedades deveriam ser feitos com verdadeiro esprito cientfico e
objetividade.

1) (ENEM - 2005)

O pensamento de Comte provocou polmicas no mundo


todo e reformulaes de teorias at ento incontestveis. Sua
influncia foi imensa, quer como filsofo social, quer como
reformador social, principalmente sobre os republicanos brasileiros.
O lema da Bandeira Nacional "___________________________",
criador por Benjamin Constant, de inspirao contista.Suas
principais obras so: Curso de Filosofia Positiva (1830-1842) e
Sistema de Poltica Positiva (1851-1854)..Morreu em Paris a 5
setembro de 1857.
KARL MARX (1818-1883)
Em 1867 publicou o primeiro volume de sua obra mais
importante. ________________ , em que fez uma crtica ao
capitalismo e sociedade burguesa. Marx o principal idealizador
do socialismo do comunismo revolucionrio. O marxismo conjunto
de ideias poltico-filosficas de Marx propunha a derrubada da
classe dominante, a burguesia, atravs de uma revoluo do
proletariado. Marx criticava o capitalismo e seu sistema de livre
empresa que, segundo ele, pelas contradies econmicas internas,
levaria a classe operria misria.
DURKHEM (1858-1917)
_______________________________________________________
_______________________________________________________
_______________________________________________________
- A sociologia uma cincia independente das demais Cincias
Sociais e da Filosofia.
- A realidade social formada pelos fenmenos coletivos,
considerados como "coisa".
- A causa de cada fato social deve ser preocupada entre os
fenmenos sociais que o antecedem. Para explicar um fenmeno
social, deve-se procurar sua casa.
- Todos os fatos sociais so exteriores aos indivduos, formando uma
realidade especfica.
Segundo Durkeim, o homem um animal que s se humaniza pela
socializao.

As tiras ironizam uma celebre fbula e a conduta dos


governantes. Tendo como referncia o estado atual dos
pases perifricos, pode-se afirmar que nessas histrias est
contida a seguinte ideia:
a) Crtica precria situao dos trabalhadores ativos e
aposentados.
b) Necessidade de atualizao crtica de clssicos da
literatura.
c) Menosprezo governamental com relao a questes
ecologicamente corretas.
d) Exigncia da insero adequada da mulher no mercado de
trabalho.
e) Aprofundamento do problema social do desemprego e do
subemprego.

MAX WEBER (1864-1920)


So famosas suas teses a respeito das relaes do capitalismo como
protestantismo. Weber procurou investigar a influncia das doutrinas
religiosas no campo econmico e, em particular, na formao do
esprito capitalista.

2) (ENEM - 2010) A poltica foi, inicialmente, a arte de impedir


as pessoas de se ocuparem do que lhes diz respeito.
Posteriormente, passou a ser a arte de compelir as pessoas
a decidirem sobre aquilo de que nada entendem.

_______________________________________________________
_______________________________________________________
_______________________________________________________
MANNHEIM (1893-1947)
elaborou as primeiras teses sobre a Sociologia do Conhecimento
uma nova disciplina cientfica, cujas bases esto lanadas em
Ideologia e utopia, sua obra principal. Mannheim afirmava que todas
as ideias polticas e sociais so inspiradas pela situao social dos
pensadores na sociedade. Segundo ele, cada fase humanista
dominada por um estilo de pensamento. em cada fase surgem
tendncias para a conservao ou para a mudana. A conservao
produz ideologia, e a mudana leva a utopias. Para ele, portanto, as
ideologias se destinam a justificar a situao social existentes,
enquanto as utopias pretendem, ao contrrio, justificar uma desejada
modificao da estrutura social.

SECRETARIA DE ESTADO DA EDUCAO

113

VALRY, P. Cadernos. Apud BENEVIDES, M. V. M. A cidadania


ativa. So Paulo: tica, 1996.

Nessa definio, o autor entende que a histria da poltica


est dividida em dois momentos principais: um primeiro,
marcado pelo autoritarismo excludente, e um segundo,
caracterizado por uma democracia incompleta.
Considerando o texto, qual o elemento comum a esses dois
momentos da histria poltica?
a) A distribuio equilibrada do poder.
b) O impedimento da participao popular.
c) O controle das decises por uma minoria.
d) A valorizao das opinies mais competentes.
e) A sistematizao dos processos decisrios.

QUMICA
SOCIOLOGIA

SECRETARIA DE ESTADO DA EDUCAO

114

QUMICA

SECRETARIA DE ESTADO DA EDUCAO

115

QUMICA

AULA 1 : VARIAES LINGUSTICAS

EXERCCIOS DE FIXAO

As variaes lingusticas demonstram o carter dinmico da


linguagem, inserindo no vernculo um pouco de nossa histria e
cultura
A linguagem a caracterstica que nos difere dos demais seres,
permitindo-nos a oportunidade de expressar sentimentos, revelar
conhecimentos, expor nossa opinio frente aos assuntos
relacionados ao nosso cotidiano, e, sobretudo, promovendo nossa
insero ao convvio social.
E dentre os fatores que a ela se relacionam destacam-se os nveis
da fala, que so basicamente dois: O nvel de formalidade e o de
informalidade.
O padro formal est diretamente ligado linguagem escrita,
restringindo-se s normas gramaticais de um modo geral. Razo pela
qual nunca escrevemos da mesma maneira que falamos. Este fator
foi determinante para a que a mesma pudesse exercer total
soberania sobre as demais.
Quanto ao nvel informal, este por sua vez representa o estilo
considerado de menor prestgio, e isto tem gerado controvrsias
entre os estudos da lngua, uma vez que para a sociedade, aquela
pessoa que fala ou escreve de maneira errnea considerada
inculta, tornando-se desta forma um estigma.
Compondo o quadro do padro informal da linguagem, esto as
chamadas variedades lingusticas, as quais representam as
variaes de acordo com as condies sociais, culturais, regionais e
histricas em que utilizada. Dentre elas destacam-se:
Variaes histricas:
Dado o dinamismo que a lngua apresenta, a mesma sofre
transformaes ao longo do tempo. Um exemplo bastante
representativo a questo da ortografia, se levarmos em
considerao a palavra farmcia, uma vez que a mesma era grafada
com ph, contrapondo-se linguagem dos internautas, a qual
fundamenta-se pela supresso do vocbulos.
Analisemos, pois, o fragmento exposto:
Antigamente
Antigamente, as moas chamavam-se mademoiselles e eram todas
mimosas e muito prendadas. No faziam anos: completavam
primaveras, em geral dezoito. Os janotas, mesmo sendo rapages,
faziam-lhes p-de-alferes, arrastando a asa, mas ficavam longos
meses debaixo do balaio."
Carlos Drummond de Andrade
Comparando-o modernidade, percebemos um vocabulrio
antiquado.
Variaes regionais:
So os chamados dialetos, que so as marcas determinantes
referentes a diferentes regies. Como exemplo, citamos a palavra
mandioca que, em certos lugares, recebe outras nomenclaturas, tais
como: macaxeira e aipim. Figurando tambm esta modalidade esto
os sotaques, ligados s caractersticas orais da linguagem.
Variaes sociais ou culturais:
Esto diretamente ligadas aos grupos sociais de uma maneira geral
e tambm ao grau de instruo de uma determinada pessoa. Como
exemplo, citamos as grias, os jarges e o linguajar caipira.
As grias pertencem ao vocabulrio especfico de certos grupos,
como os surfistas, cantores de rap, tatuadores, entre outros.
Os jarges esto relacionados ao profissionalismo, caracterizando
um linguajar tcnico. Representando a classe, podemos citar os
mdicos, advogados, profissionais da rea de informtica, dentre
outros.
Vejamos um poema e o trecho de uma msica para entendermos
melhor sobre o assunto:
VCIOS DE FALA
Para dizerem
Dizem
milho
mio
melhor
mi
pior
pi
telha
teia
a gente fomos no shopping
eu di um beijo nela
esse tal Chopis Centis muito legalzinho.
pra levar a namorada e dar uns rolezinho,

1)Tendo em vista que as grias compem o quadro de variantes


lingusticas ligadas ao aspecto sociocultural, analise os excertos a
seguir, indicando o significado de cada termo destacado de acordo
com o contexto:

SECRETARIA DE ESTADO DA EDUCAO

116

a) Possivelmente no iremos festa. L, todos os convidados so


patricinhas e mauricinhos!
b) Nossa! Como meu pai careta! No permitiu que eu assistisse
quele filme.
c) Os namoros resultantes da modernidade baseiam-se somente
no ficar.
d) E a mano? Ests a fim de encontrar com uma mina hoje? A
parada vai bombar!
e) Aquela aula de matemtica foi pssima, no saquei nada daquilo
que o professor falou.

2)Os enunciados lingusticos em evidncia encontram-se grafados na


linguagem coloquial. Reescreva-os de acordo com o padro culto da
linguagem.
a)Os livros esto sobre a mesa. Por favor, devolve eles na biblioteca.
b) Falar no celular uma falha grave. A consequncia deste ato pode
ser cara.
c) Me diga se voc gostou da surpresa, pois levei muito para
preparar ela.
d) No aviso havia o seguinte comentrio: No aproxime-se do
alambrado. Perigo constante.
e) Durante a reunio houveram reclamaes contra o atraso do
pagamento dos funcionrios.

3) (Enem 2013)
At quando?
No adianta olhar pro cu
Com muita f e pouca luta
Levanta a que voc tem muito protesto pra fazer
E muita greve, voc pode, voc deve, pode crer
No adianta olhar pro cho
Virar a cara pra no ver
Se liga a que te botaram numa cruz e s porque Jesus
Sofreu no quer dizer que voc tenha que sofrer!
GABRIEL, O PENSADOR. Seja voc mesmo (mas no seja sempre o
mesmo).

Rio de Janeiro: Sony Music, 2001 (fragmento).


As escolhas lingusticas feitas pelo autor conferem ao texto
a) carter atual, pelo uso de linguagem prpria da internet.
b) cunho apelativo, pela predominncia de imagens metafricas.
c) tom de dilogo, pela recorrncia de grias.
d) espontaneidade, pelo uso da linguagem coloquial.
e) originalidade, pela conciso da linguagem.

QUMICA
GRAMTICA

AULA 2 : FUNES DA LINGUAGEM


As funes da linguagem servem para expressar exatamente
nossas intenes no momento da fala. A linguagem, uma eficiente
forma de comunicao, elemento fundamental para
estabelecermos comunicao com outras pessoas. Por ser mltipla e
apresentar peculiaridades de acordo com a inteno do falante,
divide-se em seis funes:

Funo referencial ou denotativa: transmite uma


informao objetiva, expe dados da realidade de modo
objetivo, no faz comentrios, nem avaliao. Geralmente,
o texto apresenta-se na terceira pessoa do singular ou
plural, pois transmite impessoalidade. A linguagem
denotativa, ou seja, no h possibilidades de outra
interpretao alm da que est exposta.
Em alguns textos mais predominante essa funo, como:
cientficos, jornalsticos, tcnicos, didticos ou em correspondncias
comerciais, como por exemplo:
Bancos tero novas regras para acesso de deficientes.
O Popular, 16 out. 2008.

Funo emotiva ou expressiva: o objetivo do emissor


transmitir suas emoes e anseios. A realidade
transmitida sob o ponto de vista do emissor, a mensagem
subjetiva e centrada no emitente e, portanto, apresentase na primeira pessoa. A pontuao (ponto de exclamao,
interrogao e reticncias) uma caracterstica da funo
emotiva, pois transmite a subjetividade da mensagem e
refora a entonao emotiva. Essa funo comum em
poemas ou narrativas de teor dramtico ou romntico.
Por exemplo:
Porm meus olhos no perguntam nada./ O homem atrs do
bigode srio, simples e forte./Quase no conversa./Tem poucos,
raros amigos/o homem atrs dos culos e do bigode.
(Poema de sete faces, Carlos Drummond de Andrade)

Funo conativa ou apelativa: O objetivo de influenciar,


convencer o receptor de alguma coisa por meio de uma
ordem (uso de vocativos), sugesto, convite ou apelo (da
o nome da funo). Os verbos costumam estar no
imperativo (Compre! Faa!) ou conjugados na 2 ou 3
pessoa (Voc no pode perder! Ele vai melhorar seu
desempenho!). Esse tipo de funo muito comum em
textos publicitrios, em discursos polticos ou de
autoridade.
Por exemplo: No perca a chance de ir ao cinema pagando menos!

Funo metalingustica: Essa funo refere-se


metalinguagem, que quando o emissor explica um cdigo
usando o prprio cdigo. Quando um poema fala da
prpria ao de se fazer um poema, por exemplo. Veja:
Pegue um jornal
Pegue a tesoura.
Escolha no jornal um artigo do tamanho que voc deseja dar a seu
poema.
Recorte o artigo.
Este trecho da poesia, intitulada Para fazer um poema dadasta
utiliza o cdigo (poema) para explicar o prprio ato de fazer um
poema.

Funo ftica: O objetivo dessa funo estabelecer uma


relao com o emissor, um contato para verificar se a
mensagem est sendo transmitida ou para dilatar a
conversa.
Quando estamos em um dilogo, por exemplo, e dizemos ao nosso
receptor Est entendendo?, estamos utilizando este tipo de funo
ou quando atendemos o celular e dizemos Oi ou Al.

Na poesia acima Epitfio para um banqueiro, Jos de Paulo Paes


faz uma combinao de palavras que passa a ideia do dia a dia de
um banqueiro, de acordo com o poeta
EXERCCIOS PROPOSTOS
1)Assinale a alternativa que contenha a sequncia correta sobre as
funes da linguagem, importantes elementos da comunicao:
1. nfase no emissor (l pessoa) e na expresso direta de suas
emoes e atitudes.
2. Evidencia o assunto, o objeto, os fatos, os juzos. a linguagem
da comunicao.
3. Busca mobilizar a ateno do receptor, produzindo um apelo ou
uma ordem.
4. nfase no canal para checar sua recepo ou para manter a
conexo entre os falantes.
5. Visa traduo do cdigo ou elaborao do discurso, seja ele
lingustico ou extralingustico.
6. Voltada para o processo de estruturao da mensagem e para
seus prprios constituintes, tendo em vista produzir um efeito
esttico.
( ) funo metalingustica.
( ) funo potica.
( ) funo referencial.
( ) funo ftica.
( ) funo conativa.
( ) funo emotiva.
a) 1, 2, 4, 3, 6, 5.
b) 5, 2, 6, 4, 3, 1.
c) 5, 6, 2, 4, 3, 1.
d) 6, 5, 2, 4, 3, 1.
e) 3, 5, 2, 4, 6, 1

2)(ENEM)
Aula de Portugus
A linguagem
na ponta da lngua
to fcil de falar
e de entender.
A linguagem
na superfcie estrelada de letras,
sabe l o que quer dizer?
Professor Carlos Gis, ele quem sabe,
e vai desmatando
o amazonas de minha ignorncia.
Figuras de gramtica, esquipticas,
atropelam-me, aturdem-me, sequestram-me.
J esqueci a lngua em que comia,
em que pedia para ir l fora,
em que levava e dava pontap,
a lngua, breve lngua entrecortada
do namoro com a priminha.
O portugus so dois; o outro, mistrio.
Carlos Drummond de Andrade. Esquecer para lembrar. Rio de
Janeiro: Jos Olympio, 1979.
Explorando a funo emotiva da linguagem, o poeta expressa o
contraste entre marcas de variao de usos da linguagem em
a) situaes formais e informais.
b) diferentes regies dos pais.
c) escolas literrias distintas.
d) textos tcnicos e poticos.
e) diferentes pocas.

Funo potica: O objetivo do emissor expressar seus


sentimentos atravs de textos que podem ser enfatizados
por meio das formas das palavras, da sonoridade, do ritmo,
alm de elaborar novas possibilidades de combinaes
dos signos lingsticos. presente em textos literrios,
publicitrios e em letras de msica.
Por exemplo: negcio/ego/cio/cio/0

SECRETARIA DE ESTADO DA EDUCAO

117

QUMICA
GRAMTICA

AULA 3 : PROCESSO DE CORREO DAS REDAES NO ENEM

O processo de correo
Como sabemos, a redao passa por trs avaliadores diferentes.
Cada um atribuir uma nota entre 0 (zero) e 200 (duzentos) pontos
para cada uma das cinco competncias e a soma desses pontos
resulta na nota total de cada avaliador, que pode chegar a 1000
pontos. A nota final ser a mdia aritmtica das notas totais
atribudas pelos dois avaliadores.
A discrepncia
considerado que h discrepncia quando a divergncia de notas
atribudas pelos avaliadores ultrapassar 200 (duzentos) pontos ou for
superior a 80 (oitenta) pontos em qualquer uma das competncias.
Caso haja essas diferenas, a redao passa por um terceiro
avaliador para, posteriormente, haver um consenso. (maiores
detalhes no guia em anexo)
Vamos s competncias:
Competncia 1: Demonstrar domnio da norma padro da lngua
escrita:
Aqui sero avaliados critrios, como:
ausncia de marcas de oralidade e de registro informal;
preciso vocabular;
obedincia s regras gramaticais de:
concordncia nominal e verbal;
regncia nominal e verbal;
pontuao;
flexo de nomes e verbos;
colocao de pronomes tonos;
grafia das palavras;
acentuao grfica;
emprego de letras maisculas e minsculas;
diviso silbica na mudana de linha.

Competncia 4: Demonstrar conhecimento dos mecanismos


lingusticos necessrios para a construo da argumentao:
Aqui so avaliados os mecanismos lingusticos utilizados para
encadear as ideias na redao coeso textual.
Competncia 5: Elaborar proposta de interveno para o problema
abordado, respeitando os direitos humanos:
Nessa ltima e importante competncia, so analisados os seguintes
itens:
presena de proposta x ausncia de proposta;
proposta explcita x proposta implcita;
proposta com detalhamento dos meios para sua realizao x
proposta sem o detalhamento dos meios para sua realizao.
CUIDADO!
A redao no ENEM recebe nota 0 (zero) caso vocs:
fujam totalmente do tema;
no obedeam estrutura dissertativo-argumentativa;
faam um texto com at 7 (sete) linhas;
redijam improprios, desenhos ou outras formas propositais de
anulao;
desrespeitem os direitos humanos (desconsiderao da
Competncia 5);
deixem a folha de redao em branco, mesmo que tenha sido escrita
no rascunho.
IMPORTANTE!
No copiem informaes dos textos motivadores ou de questes
objetivas do caderno de prova. Isso acarretar a desconsiderao do
nmero de linhas copiadas. Alm disso, isso prejudica bastante a
originalidade do seu texto. Sejam criativos e originais em suas ideias.
A banca corretora valoriza isso.
Alm disso, procurem escrever sua redao com letra legvel, para
evitar dvidas na hora da correo.
O ttulo um elemento opcional na produo da sua redao,
entretanto um ttulo interessante e coerente pode enriquecer seu
texto, causando uma boa impresso no corretor.

Competncia 2: Compreender a proposta de redao e aplicar


conceitos das vrias reas de conhecimento, para desenvolver o
tema dentro dos limites estruturais do texto dissertativoargumentativo:
Nessa competncia, avaliada a sua capacidade de conectar as
informaes do mundo real com o seu texto de forma pertinente,
demonstrando que voc est atualizado e bem informado. Isso deve
ser feito, sem deixar de respeitar a estrutura dissertativa
argumentativa - introduo, desenvolvimento e concluso.
Segue um esquema divulgado no Guia:

A seguir, temos um esquema bsico, retirado do Guia do


participante:

(Fonte: Guia do Participante. A redao no ENEM 2012)


Competncia 3: Selecionar, relacionar, organizar e interpretar
informaes, fatos, opinies e argumentos em defesa de um ponto
de vista:
Basicamente, nesta competncia leva-se em considerao
a argumentao do seu texto. Analisa-se a criticidade, ou seja, a sua
capacidade de argumentar de forma clara e objetiva. O texto precisa
tercoerncia do incio ao fim. As ideias devem seguir uma sequncia
lgica de raciocnio. Lembrem-se de que as informaes
da introduo devem ser coerentes com a concluso da redao. O
seu texto uma unidade de sentido.

SECRETARIA DE ESTADO DA EDUCAO

118

(Fonte: Guia do Participante. A redao no ENEM 2012)

QUMICA
GRAMTICA

AULA 5 : COESO e COERNCIA


No basta somente ter ideias, elas precisam estar bem articuladas,
para que assim se materialize a textualidade

Clareza da palavras Tal requisito se mostra como algo

Textualidade... Ao nos depararmos com essa palavra, logo nos

delas implicar na qualidade da mensagem que produz. Dessa

remetemos escrita que, por sua vez, requer determinadas

forma, no busque palavras difceis, muitas vezes por voc e pelo

habilidades por parte do emissor. Nesse sentido e, sobretudo,

seu interlocutor desconhecidas. Tal atitude s comprometer aquilo

partindo do pressuposto de que independentemente de qualquer que

que pretende conquistar.

seja a finalidade discursiva a que se presta um determinado texto,

Expressividade Todos os aspectos aqui firmados parecem se

esse deve estar claro, preciso e objetivo para quem o l isso a

complementar mutuamente, pois a expressividade resulta na escolha

verdadeira textualidade.

bem feita que fizer das palavras. Sempre tendo em vista os critrios

Mas, ateno! Um texto se constitui de ideias, de argumentos

de coeso e coerncia, os quais incidiro de forma direta na

firmados por parte de quem o escreve, concorda? Sim, fato

qualidade da mensagem que construir.

indiscutvel, contudo, somente elenc-las no o suficiente, haja

Ordem direta das palavras Procure sempre fazer uso delas, ou

vista que precisam, antes de tudo, estar bem ordenadas, articuladas.

seja,

Fato esse que somente se materializa se os pargrafos se

complemento, quando houver, pois assim voc ser mais bem

apresentarem bem construdos, ordenados. Quando assim se

entendido (a).

apresentam, logo afirmamos que se trata de um texto que possui

Originalidade Seja original o quanto puder, por isso nada de optar

coeso, e se ela se faz evidente, afirmamos tambm que ele est

por

coerente, pois ambos os aspectos trilham (ou pelo menos devem

erudies que voc mesmo (a) desconhece. Procure revelar sua

trilhar) juntos o mesmo caminho.

mensagem de forma clara e objetiva, de modo a fazer com que o

Partindo ento de tais princpios, cabe ressaltar que essa articulao,

interlocutor compreenda perfeitamente aquilo que pretende dizer.

uma vez manifestada, pode se dar tanto no nvel das frases quanto

Para tanto, utilize-se de uma linguagem simples, porm adequada ao

no nvel do prprio texto, por meio dos articuladores lgicos do

padro formal requisito tambm essencial modalidade escrita. O

texto e dos prprios conectivos.

uso de termos tcnicos, bem como de chaves e modismos, uma

Quando no nvel das frases, a articulao se d mediante o uso de

atitude inadequada a um bom redator.

pronomes,

os

quais

fazem

referncia

elementos

indispensvel a qualquer discurso, haja vista que a escolha que fizer

antes

optando

frases

pelo

desgastadas,

uso

do sujeito

rebuscamentos,

predicado

pedantismos,

falsas

Exerccios de Fixao

proferidos; bem como das conjunes, uma vez que essas


estabelecem distintas relaes entre as oraes, podendo ser
de causalidade,

temporalidade,

oposio,

consequncia,

condio, concluso, entre outros aspectos.

1. As frases a seguir se encontram em ordem inversa. Sua tarefa


consistir em coloc-las na ordem direta, apontar o sujeito e destacar
o ncleo do sujeito.
a)
No beco escuro explode a violncia. ( Hebert Vianna )

Manifestando-se no nvel do texto, a articulao se caracteriza


pela relao que se estabelece entre as partes maiores desse, como
o caso da introduo, desenvolvimento e concluso. Dessa
forma, atuando como casos representativos desse aspecto, eis
algumas expresses notadamente expressas por dessa forma,

b) Apareceu no jardim da casa de Ana Maria um ramalhete de


flores.

por outro lado, por exemplo; sequncias numricas, tais como


primeiro, segundo, primeiramente, em segundo plano, entre
outras; conjunes de oposio, como, por exemplo, no
obstante, apesar de, ente outras.
Os elementos da textualidade representam um conjunto de
aspectos que, uma vez manifestados, conferem clareza ao discurso

c)

Chegou ontem a So Paulo o presidente da FIFA.

d)

Aconteceram, naquela cidade, muitos fenmenos inexplicveis.

Caro (a) usurio (a), gostaramos de fazer um questionamento a


voc: a palavra textualidade o (a) remete a alguma ideia?
Independentemente

da

resposta

conferida

tal

indagao,

elucidemos acerca desse importante aspecto indispensvel a


qualquer produo textual.
Compreenda que o sentido dessa palavra se encontra intimamente
relacionado

caractersticas

das

quais

necessita

qualquer

enunciao, haja vista que a mensagem, para ser materializada de

e)Ouviram do Ipiranga as margens plcidas de um povo heroico o


brado retumbante

forma plausvel, precisa, antes de tudo, estar decifrvel mediante os


olhos do interlocutor. Partindo de tal princpio, eis que estudaremos
esses elementos, to importantes quanto necessrios, de forma que
a textualidade se faa presente, sempre. Assim, analisemos cada
um deles de forma particular:

SECRETARIA DE ESTADO DA EDUCAO

119

QUMICA
GRAMTICA

AULA 6: OS MECANISMOS DE COESO E COERNCIA


TEXTUAIS
O texto no simplesmente um conjunto de palavras; pois se o
fosse, bastaria agrup-las de qualquer forma e teramos um:
"O ontem lanche menino comeu"
Veja que neste caso no h um texto, h somente um grupo de
palavras dispostas em uma ordem qualquer. Mesmo que
colocssemos estas palavras em uma ordem gramatical correta:
sujeito-verbo-complemento, precisaramos ainda organizar o nvel
semntico do texto, deixando-o inteligvel.
"O lanche comeu o menino ontem"
O nvel sinttico est perfeito:
sujeito = o lanche
verbo = comeu
complementos = o menino ontem
Mas o nvel semntico apresenta problemas, pois no possvel que
o lanche coma o menino, pelo menos neste contexto. Caso a frase
estivesse empregada num sentido figurado e em outro contexto, isto
seria possvel.
Pedrinho saiu da lanchonete todo lambuzado de maionese, mostarda
e catchup, o lanche era enorme, parecia que "o lanche tinha comido
o menino".
A coeso e a coerncia garantem ao texto uma unidade de
significados encadeados.
A COESO
H, na lngua, muitos recursos que garantem o mecanismo de
coeso:
* por referncia: Os pronomes, advrbios e os artigos so os
elementos de coeso que proporcionam a unidade do texto.
"O Presidente foi a Portugal em visita. Em Portugal o presidente
recebeu vrias homenagens."

O que ser que havia dentro do pacote? Veja como o narrador


acabou com a histria na escolha infeliz do ttulo.
A incoerncia est presente, tambm, em textos dissertativos que
apresentam defeitos de argumentao.
Em muitas redaes observamos afirmaes falsas e inconsistentes.
Observe:
No fundo nenhuma escola est realmente preocupada com a
qualidade de ensino.
Estava assistindo ao debate na televiso dos candidatos ao governo
de So Paulo, eles mais se acusavam moralmente do que
mostravam suas propostas de governo, em um certo momento do
debate dois candidatos quase partem para a agresso fsica. Dessa
forma, isso nos leva a concluir que o homem no consegue conciliar
ideias opostas por isso que o mundo vive em guerras
frequentemente.
Note que nos dois primeiros exemplos as informaes so amplas
demais e sem nenhum fundamento. J no terceiro, a concluso
apresentada no tem ligao nenhuma com o exemplo argumentado.
Esses exemplos caracterizam a falta de coerncia do texto.
EXERCCIOS DE FIXAO
Apresenta-se a seguir uma anedota de Ziraldo. Analise-a
e, em seguida, responda:
A me chega na varanda e encontra o maluquinho
ensinando palavres pro papagaio:
- Maluquinho, que que voc est fazendo, menino?
- Ora, me, tou ensinando pro papagaio as palavras que
ele no deve dizer.
(O livro do riso do Menino maluquinho. 2.ed.
So Paulo: Melhoramentos,2000.p.74)

Esse texto repetitivo torna-se desagradvel e sem coeso. Observe a


atuao do advrbio e do pronome no processo de e elaborao do
texto.
"O Presidente foi a Portugal. L, ele foi homenageado."
Veja que o texto ganhou agilidade e estilo. Os termos L e ele
referem-se a Portugal e Presidente, foram usados a fim de tornar o
texto coeso.
* por elipse: Quando se omite um termo a fim de evitar sua
repetio.
"O Presidente foi a Portugal. L, foi homenageado."
Veja que neste caso omitiu-se a palavra Presidente, pois
subentendida no contexto.

Atenha-se ao enunciado lingustico ora expresso,


identificando os elementos coesivos presentes neste:
O lide da secretria
Uma das grandes dificuldades que o reprter tem, ao
apurar uma notcia pelo telefone, conseguir passar pela
telefonista/secretria. Invariavelmente, elas fazem trs
perguntas ao interlocutor, que funcionam como uma
espcie de lide: quem deseja? de onde?, pode
adiantar o assunto?, com frequncia completadas pelo
fatal no se encontra, que at hoje no descobri de que
lngua .

A COERNCIA
muito confusa a distino entre coeso e coerncia, aqui
entenderemos como coerncia a ligao das partes do texto com o
seu todo. Ao elaborar o texto, temos que criar condies para que
haja uma unidade de coerncia, dando ao texto mais fidelidade.
Estava andando sozinho na rua, ouvi passos atrs de mim,
assustado nem olhei, sa correndo, era um homem alto, estranho,
tinha em suas mos uma arma...
Se o narrador no olhou, como soube descrever a personagem?
A falta de coerncia se d normalmente: Na inverossimilhana, falta
de concatenao e argumentao falsa.
Observe outra situao:
Estava voltando para casa, quando vi na calada algo que parecia
um saco de lixo, cheguei mais perto para ver o que acontecia...
Ocorre neste trecho uma incoerncia pois se era realmente um saco
de lixo, com certeza no iria acontecer coisa alguma.
Outro tipo de incoerncia: Ao tentar elaborar uma histria de
suspense, o narrador escolhe um ttulo que j leva o leitor a concluir
o final da histria.
Um milho de dlares
Estava voltando para casa, quando vi na calada algo que parecia
um saco de lixo, ao me aproximar percebi que era um pacote...

SECRETARIA DE ESTADO DA EDUCAO

120

QUMICA
GRAMTICA

AULA 7 :CONJUNES COORDENATIVAS

EXERCCIOS DE FIXAO

So aquelas que ligam oraes de sentido completo e


independente ou termos da orao que tm a mesma funo
gramatical. Subdividem-se em:

1)(UFSM) Assinale a sequncia de conjunes que estabelecem,


entre as oraes de cada item, uma correta relao de sentido.
1. Correu demais, ... caiu.
2. Dormiu mal, ... os sonhos no o deixaram em paz.
3. A matria perece, ... a alma imortal.
4. Leu o livro, ... capaz de descrever as personagens com detalhes.
5. Guarde seus pertences, ... podem servir mais tarde.
a) porque, todavia, portanto, logo, entretanto
b) por isso, porque, mas, portanto, que
c) logo, porm, pois, porque, mas
d) porm, pois, logo, todavia, porque
e) entretanto, que, porque, pois, portanto

1) Aditivas: ligam oraes ou palavras, expressando ideia de


acrescentamento ou adio. So elas: e, nem (= e no), no s...
mas tambm, no s... como tambm, bem como, no s... mas
ainda , como exemplo,
A sua pesquisa clara e objetiva.
Ela no s dirigiu a pesquisa como tambm escreveu o relatrio.
2) Adversativas: ligam duas oraes ou palavras, expressando ideia
de contraste ou compensao. So elas: mas, porm, contudo,
todavia, entretanto, no entanto, no obstante , como exemplo,

2)(ENEM)

Tentei chegar mais cedo, porm no consegui.


3) Alternativas: ligam oraes ou palavras, expressando ideia de
alternncia ou escolha, indicando fatos que se realizam
separadamente. So elas: ou, ou... ou, ora... ora, j... j, quer...
quer, seja... seja, talvez... talvez, como exemplo,
Ou escolho agora, ou fico sem presente de aniversrio.
4) Conclusivas: ligam a orao anterior a uma orao que expressa
ideia de concluso ou consequncia. So elas: logo, pois (depois do
verbo), portanto, por conseguinte, por isso, assim, como exemplo,
Marta estava bem preparada para o teste, portanto no ficou
nervosa.

Nessa charge, o recurso morfossinttico que colabora para o efeito


de humor est indicado pelo(a)

5) Explicativas: ligam a orao anterior a uma orao que a explica,


que justifica a ideia nela contida. So elas: que, porque, pois (antes
do verbo), porquanto, como exemplo,

A emprego de uma orao adversativa, que orienta a quebra da


expectativa ao final.

No demore, que o filme j vai comear.

b) uso de conjuno aditiva, que cria uma relao de causa e efeito


entre as aes

OBSERVAES
a) As conjunes "e"," antes", "agora"," quando" so adversativas
quando equivalem a "mas".
Exemplos:
Carlos fala, e no faz.
O bom educador no probe, antes orienta.
Sou muito bom; agora, bobo no sou.
Foram mal na prova, quando poderiam ter ido muito bem.

c) retomada do substantivo me, que desfaz a ambiguidade dos


sentidos a ele atribudos.
d))utilizao da forma pronominal la, que reflete um tratamento
formal do filho em relao me.
e) repetio da forma verbal , que refora a relao de adio
existente entre as oraes.

b) "Seno" conjuno adversativa quando equivale a "mas sim".


Exemplo:
Conseguimos vencer no por protecionismo, seno por capacidade.
c) Das conjunes adversativas, "mas" deve ser empregada sempre
no incio da orao: as outras (porm, todavia, contudo, etc.) podem
vir no incio ou no meio.
Exemplo:
Ningum respondeu a pergunta, mas os alunos sabiam a resposta.
Ningum respondeu a pergunta; os alunos, porm, sabiam a
resposta.
d) A palavra "pois", quando conjuno conclusiva, vem geralmente
aps um ou mais termos da orao a que pertence.
Exemplo:
Voc o provocou com essas palavras; no se queixe, pois, de seus
ataques.
e)Quando conjuno explicativa," pois" vem, geralmente, aps um
verbo no imperativo e sempre no incio da orao a que pertence.
Exemplo:
No tenha receio, pois eu a protegerei.

SECRETARIA DE ESTADO DA EDUCAO

121

3)Sobre as conjunes, correto afirmar:


a) As conjunes so unidades lingusticas desprovidas de
independncia, ou seja, no aparecem sozinhas no enunciado
porque apenas introduzem a ideia de funo gramatical de um
elemento.
b) As conjunes so expresses modificadoras do verbo que
apresentam uma ideia de circunstncia e atuam como adjunto
adverbial em uma orao.
c) As conjunes so palavras que podem ser utilizadas no lugar de
um nome, referir-se a ele ou acompanhar o nome com a inteno de
qualific-lo.
d) As conjunes tm por finalidade traduzir nossos estados
emotivos. Apresentam independncia e podem constituir, por si,
verdadeiras oraes.
e) As conjunes so unidades da lngua que tm por finalidade
reunir oraes em um mesmo enunciado, classificando-se em
conjunes coordenativas e subordinativas.

QUMICA
GRAMTICA

AULA 8: Conjunes Subordinativas

EXERCCIOS DE FIXAO

O
Flamengo
comeou
a
partida no
ataque, enquanto o Botafogo procurava fazer uma forte
marcao nomeio campo e tentar lanamentos para Victor
Simes, isolado entre os zagueiros rubro-negros. Mesmo
com mais posse de bola, o time dirigido por Cuca
tinha grande dificuldade de chegar rea alvinegra por
causa do bloqueio montado pelo Botafogo na frente da sua
rea.
No entanto, na primeira chance rubro-negra, saiu o
gol. Aps cruzamento da direita de Ibson, a zaga alvinegra
rebateu a bola de cabea para o meio da rea. Klberson
apareceu na jogada e cabeceou por cima do goleiro Renan.
Ronaldo Angelim apareceu nas costas da defesa e
empurrou para o fundo da rede quase que em cima da linha:
Flamengo 1 a 0.
1)(ENEM)

A conjuno a palavra que une oraes ou termos de uma orao


que possuem mesma funo. Essa relao estabelecida entre as
oraes pode ser independente, quando se trata de oraes
coordenadas; ou dependente, quando se trata de subordinadas.
Quando a conjuno exerce seu papel de ligar as oraes, e
estabelece entre elas uma relao de dependncia sinttica, temos
subordinao. Veja o exemplo:
Maria confirmou que no esteve em casa hoje.
As duas oraes esto ligadas pela conjuno que e tm relao de
dependncia entre si, uma vez que na primeira orao (Maria
confirmou) o verbo confirmou no tem por si s sentido completo e
depende da segunda (que no esteve em casa hoje) para t-lo. No
entanto, a segunda orao a subordinada, j que est sujeita
primeira, com funo de complemento do verbo. Assim temos a
orao principal: Maria confirmou, e a orao subordinada: que no
esteve em casa hoje, ligadas pela conjuno integrante que, a qual
subordinativa, pelo fato de estar unindo a orao subordinada
orao principal.
Podemos classificar as conjunes subordinativas em:
integrantes so introdutrias de oraes subordinadas
substantivas: que, se, como, etc.;
Exemplo: No sei dizer se ele chegou.
causais exprimem causa: porque, como, uma vez que, j que,
etc.;
Exemplo: Eu sou feliz porque tenho uma famlia.
concessivas exprimem concesso: embora, ainda que, mesmo
que, apesar de que, etc.;
Exemplos: Quando fui dormir ainda estava claro, ainda que passasse
das sete da noite.
Apesar de estarmos refletindo mais sobre a economia do pas, os
juros s tem aumentado.
condicionais exprimem condio ou hiptese: se, desde que,
contanto que, caso, se, etc.;
Exemplo: Avise-me caso eles j saibam da nova lei.
conformativas exprimem conformidade: conforme, segundo,
como, consoante.;
Exemplo: Conforme ia passando o tempo, meu corpo cansava cada
vez mais.

Disponvel em: http://momentodofutebol.blogspot.com


(adaptado).
O texto, que narra uma parte do jogo final do Campeonato
Carioca de futebol, realizado em 2009, contm vrios
conectivos, sendo que
a) aps conectivo de causa, j que apresenta o motivo de
a zaga alvinegra ter rebatido a bola de cabea.
b) enquanto tem um significado alternativo, porque conecta
duas opes possveis para serem aplicadas no jogo.
c) no entanto tem significado de tempo, porque ordena os
fatos observados no jogo em ordem cronolgica de
ocorrncia.
d) mesmo traz ideia de concesso, j que com mais posse
de bola, ter dificuldade no algo naturalmente esperado.
e) por causa de indica consequncia, porque as tentativas
de ataque do Flamengo motivaram o Botafogo a fazer um
bloqueio.
2)Levando em considerao os aspectos inerentes dependncia
sinttica entre os elementos de uma orao, indique em cada
orao abaixo, o tipo de conjuno subordinativa.

Como no havia recursos financeiros suficientes, as obras


ficaram paralisadas.(.........................................................)

Fizemos a pesquisa como o professor indicou.


(.........................................................)

Voc meiga como uma flor.


(.........................................................)

consecutivas exprimem consequncia: de forma que, de sorte


que, que, etc.;
Exemplo: Estudou tanto que adormeceu.

finais exprimem finalidade: a fim de que, que, porque, para que,


etc.;
Exemplo: Vamos embora a fim de que possamos assistir ao filme.

Desde que voc apresente justificativa poder faltar


reunio.
(.........................................................)

Estamos morando aqui desde que a cidade foi fundada.


(.........................................................)

comparativas estabelecem comparao: como, mais...do que,


menos...do que, etc.;
Exemplo: Estou mais feliz hoje do que ontem.
Ele chorou como quem tivesse perdido algo de muito valor
sentimental.

proporcionais - estabelecem proporo: medida que,


proporo que, ao passo que, etc.;
Exemplo: medida que estudo todos os dias, minha memria se
torna melhor.
temporais indicam tempo: quando, depois que, desde que, logo
que, assim que, etc..
Exemplo: Desde que voc foi embora, meu corao gerou
expectativa para que voltasse.

SECRETARIA DE ESTADO DA EDUCAO

122

QUMICA

GRAMTICA

AULA 9: ESQUEMAS PARA DISSERTAO

Quando estamos fazendo o esquema do desenvolvimento (letra b),


comum surgirem inmeras ideias. Registre-as todas, mesmo que
mais tarde voc no venha a utiliz-las. Essas ideias normalmente
vm sem ordem alguma; por isso, mais tarde preciso orden-las,
selecionando as melhores e colocando-as em ordem de importncia.
A esse processo damos o nome de hierarquizao das ideias.
Para no se perder tempo elaborando um outro esquema, a
hierarquizao das ideias pode ser feita por meio de nmeros
atribudos s palavras que aparecem no esquema, seguindo a ordem
em
que
sero
utilizadas
na
produo
do
texto.
Apresentamos, agora, um exemplo do esquema com as ideias j
hierarquizadas:

Dissertao: Esquema 1
primeiro pargrafo:
Frase inicial + objetivo + citao dos argumentos 1,2 e 3.
segundo pargrafo: desenvolva o argumento 1
terceiro pargrafo: desenvolva o argumento 2
quarto pargrafo : desenvolva o argumento 3
quinto pargrafo :
Concluso : frase conclusiva citando o tema da redao +
observao final (impessoal, positiva, otimista, que solucione o
problema e a apresente uma feio humanstica)

Tema: A pena de morte: contra ou a favor?

Exemplo:
Vamos supor que voc j tenha lido com bastante ateno os textos
base da prova e concluiu que o tema :
A exploso do Crack nas cidades brasileiras.
objetivo: mostrar a real e trgica situao e apresentar solues
argumento1: A infncia e a adolescncia sendo destrudas pelo
consumo do crack
argumento 2: a demora do governo em agir;
argumento 3: Governo Federal, Estados, prefeituras e a televiso
devem unir esforos.

Seguindo o esquema construdo acima, a introduo ficaria assim:


O consumo do crack uma febre em todas as cidades brasileiras
(frase inicial contendo o tema). E preciso tomar medidas rpidas e
eficazes contra essa epidemia (objetivo). Ela est tornando vrios
adolescentes e crianas verdadeiros "zumbis" (argumento1).
Sabendo disso, o Governo federal j criou vrios projetos, mas
efetivamente ainda no agiu. Mas essa ao deve ter a ajuda das
prefeituras e estados, atuando em conjunto. E como complemento, a
televiso deve contribuir , expondo o problema em novelas e
seriados (argumento 3).

a) contra, no resolve.
b) 1. direito vida religio
2. outros pases EUA
3. erro judicirio
4. classes baixas
5. tradio.
c) ineficaz: soluo: erradicao da pobreza.
Feito o esquema, segui-lo passo a passo, transformando as
palavras em frases, dando forma redao.
No exemplo dado, na introduo voc se declararia contrrio (a)
pena de morte porque ela no resolve o problema do crescente
aumento da criminalidade no pas.
No desenvolvimento, voc utilizaria os argumentos de que todas as
pessoas tm direito vida, consagrado pelas religies; de que nos
pases em que ela existe, citando os Estados Unidos como exemplo,
no fez baixar a criminalidade; de que sempre possvel haver um
erro judicial que leve a matar um inocente; de que, no caso brasileiro,
ela seria aplicada somente s classes mais baixas; que no podem
pagar bons advogados; e, finalmente, de que a tradio jurdica
brasileira consagra o direito vida e repudia a pena de morte.
Como concluso, retomaria a tese insistindo na ineficcia desse tipo
de pena e indicando outras solues para resolver o problema da
criminalidade, como a erradicao da misria

Repare que o esquema foi seguido, mas no copiado, pois o


escrevi com outras palavras (isso fundamental). Nos prximos trs
pargrafos, desenvolva seus trs argumentos. Para tal, siga a ordem
de sua introduo, abordando cada argumento em um pargrafo.
Nesse momento que voc ir convencer o leitor, portanto d
bastantes explicaes e exemplos sobre cada argumento.
O Esquema-Padro
Inicialmente, preciso no confundir esquema com rascunho.
importante atentar para um fato: cada dissertao, dependendo do
tema e da argumentao, pede um esquema. Uma dissertao
subjetiva, por exemplo, permite ao produtor do texto utilizar certos
recursos que seriam descabidos numa dissertao objetiva.
Esquema um guia que estabelecemos para ser seguido, no qual
colocamos em frases sucintas (ou mesmo em simples palavras) o
roteiro para a elaborao do texto. No rascunho, vamos dando forma
redao, porque nele as ideias colocadas no esquema passam a
ser redigidas, tomando a forma de frases at chegar a um texto
coerente.
O primeiro passo para a elaborao de um esquema ter entendido
o tema proposto, pois de nada adiantar um timo esquema se ele
no estiver adequado ao tema.
Por ser um roteiro a seguir, deve-se dividir o esquema nas partes
de que se compe a redao. Se formos escrever uma redao
dissertativa, o esquema j dever apresentar as trs partes da
dissertao: introduo, desenvolvimento e concluso, que
podem vir representadas pelas letras a, b e c, respectivamente.
Na letra a, voc dever colocar a tese que vai defender;
Na letra b, palavras que resumam os argumentos que voc
apresentar para sustentar a tese;
Na letra c, uma palavra que represente a concluso a ser dada.

SECRETARIA DE ESTADO DA EDUCAO

123

QUMICA
GRAMTICA

AULA 10 :INTERPRETAO

EXERCCIOS PROPOSTOS
1)(ENEM)

Interpretao de texto o "terror" para muitos candidatos


que disputam vagas em concursos e vestibulares.
No de se estranhar, afinal cada vez mais se l menos
neste pas.
Ento, esta a primeira dica para quem quer melhorar a
interpretao de texto: leia, e no para ler pouco, para ler
muito e de forma diversificada: literatura, jornais, revistas,
sites, gibis...
Afora a leitura, existem alguns "macetes" que ajudam na hora
de interpretar um texto:
1. Leia o texto no mnimo duas vezes. Na primeira leitura,
atenha-se ao contedo, na segunda, veja como o texto est
articulado, desenvolvido.
2. Observe as relaes Inter pargrafos. Um pargrafo
geralmente mantm com outro uma relao de continuao,
concluso ou falsa oposio. Identifique muito bem essas
relaes.
3. Sublinhe, em cada pargrafo, o tpico frasal, ou seja, a
ideia mais importante.
4. Leia com muito cuidado os enunciados das questes, sem
pressa, para entender bem a pergunta.
5. Grife palavras como correto ou incorreto, evitando, assim,
uma confuso na hora da resposta.
6. No d muito valor ao que o autor quis dizer, mas sim ao
que ele disse, expressou no texto.

Mafalda uma garotinha de seis anos que sempre lana perguntas


desconcertantes para os adultos
QUINO, J. L Mafalda. Traduo de Mnica S. M. da Silva, So
Paulo: Martins Fontes, 1988.

O efeito de humor foi um recurso utilizado pelo autor da


tirinha para mostrar que o pai de Mafalda:
a) revelou desinteresse na leitura do dicionrio.
b) tentava ler um dicionrio, que uma obra muito extensa.
c) causou surpresa em sua filha, ao se dedicar leitura de
um livro to grande.
d) queria consultar o dicionrio para tirar uma dvida, e no
ler o livro, como sua filha pensava.
e) demonstrou que a leitura do dicionrio o desagradou
bastante, fato que decepcionou muito sua
filha.
2)(ENEM)

7. Identifique os personagens principais e secundrios e


centre-se nas caractersticas fsicas e psicolgicas deles.
8. Se o foco do enunciado for o tema ou a ideia principal, leia
com ateno a introduo e/ou a concluso.
9. Se o enunciado focar o item argumentao, concentre-se
no desenvolvimento.
10. Olhe com especial ateno os pronomes relativos,
pronomes pessoais, pronomes demonstrativos, etc.,
chamados vocbulos relatores, porque remetem a outros
vocbulos do texto.

SECRETARIA DE ESTADO DA EDUCAO

124

A publicidade, de uma forma geral, alia elementos verbais e


imagticos na constituio de seus textos. Nessa pea
publicitria, cujo tema a sustentabilidade, o autor procura
convencer o leitor a
a) assumir uma atitude reflexiva diante dos fenmenos
naturais.
b) evitar o consumo excessivo de produtos reutilizveis.
c) aderir onda sustentvel, evitando o consumo excessivo.
c) abraar a campanha, desenvolvendo projetos
sustentveis.
e) consumir produtos de modo responsvel e ecolgico

QUMICA
GRAMTICA

04) Por que s vezes o silncio muito importante em nossas vidas?


Use bons argumentos

AULA 11 :Interpretao de letras de msicas

O SILNCIO
antes de existir computador existia tev
antes de existir tev existia luz eltrica

05) O poeta faz uma comparao no ltimo verso do texto. A que o

antes de existir luz eltrica existia bicicleta

termo "tambor" est sendo comparado?

antes de existir bicicleta existia enciclopdia


antes de existir enciclopdia existia alfabeto
antes de existir alfabeto existia a voz
antes de existir a voz existia o silncio

06) O que, de acordo com a letra, foi a primeira coisa que existiu?

o silncio
foi a primeira coisa que existiu
um silncio que ningum ouviu
astro pelo cu em movimento

07) Qual o processo de formao da palavra tev?

e o som do gelo derretendo


o barulho do cabelo em crescimento
e a msica do vento
e a matria em decomposio

08) Qual o tema dessa cano? Explique sua resposta:

a barriga digerindo o po
exploso de semente sob o cho
diamante nascendo do carvo
homem pedra planta bicho flor
luz eltrica tev computador

09) O que voc entende do verso Antes de existir a luz eltrica

batedeira, liquidificador

existia bicicleta?

vamos ouvir esse silncio meu amor


amplificado no amplificador
do estetoscpio do doutor
10) Que outro ttulo voc daria cano?

no lado esquerdo do peito, esse tambor


(Arnaldo Antunes)

01) O texto apresenta uma gradao de invenes humanas.


a)

Enumere-as na ordem em que teriam surgido.


11) No verso E o som do gelo derretendo, temos:

b)

Releia o texto identificando a ltima inveno. J houve outras

aps a citada? Justifique sua resposta.

a) metonmia;

d) sinestesia;

b) comparao;

e) pleonasmo;

c) hiprbole;
Justifique sua resposta

02) Observe o verso "o silncio que ningum ouviu". Neste verso h:
a) paradoxo
d) metfora
b) pleonasmo
e) anttese
c) personificao
Justifique a sua resposta.

12) Copie da cano alguns verbos no gerndio e explique o efeito


disso para o texto.

03) Cite dois pontos negativos e dois positivos em relao


tecnologia:

SECRETARIA DE ESTADO DA EDUCAO

125

QUMICA
GRAMTICA

AULA 12 :A pontuao na redao

EXERCCIOS PROPOSTOS

Os sinais de pontuao so recursos destinados escrita, dotados


de finalidades especficas
1. Vrgula (,)
usada para:
a) separar termos que possuem mesma funo sinttica na orao:
O menino berrou, chorou, esperneou e, enfim, dormiu.
Nessa orao, a vrgula separa os verbos.

1) Assinale a opo em que a supresso das vrgulas alteraria o


sentido
do
anunciado:
a) os pases menos desenvolvidos vm buscando, ultimamente,
solues para seus problemas no acervo cultural dos mais
avanados;
b) alguns pesquisadores, que se encontram comprometidos com as
culturas dos pases avanados, acabam se tornando menos criativos;
c) torna-se, portanto, imperativa uma reviso modelo presente do
processo
de
desenvolvimento
tecnolgico;
d) a atividade cientfica, nos pases desenvolvidos, to natural
quanto
qualquer
outra
atividade
econmica;
e) por duas razes diferentes podem surgir, da interao de uma
comunidade
com
outra,
mecanismos
de
dependncia.

b) isolar o vocativo: Ento, minha cara, no h mais o que se dizer!


c) isolar o aposto: O Joo, ex-integrante da comisso, veio assistir
reunio.
d) isolar termos antecipados, como complemento ou adjunto:
1. Uma vontade indescritvel de beber gua, eu senti quando olhei
para aquele copo suado! (antecipao de complemento verbal)
2. Nada se fez, naquele momento, para que pudssemos sair!
(antecipao de adjunto adverbial)

2. Assinale a opo em que est corretamente indicada a ordem dos


sinais de pontuao que devem preencher as lacunas da frase
abaixo:

e) separar expresses explicativas, conjunes e conectivos: isto ,


ou seja, por exemplo, alm disso, pois, porm, mas, no entanto,
assim, etc.

a) dois pontos, ponto e vrgula, ponto e vrgula


b) dois pontos, vrgula, ponto e vrgula;
c) vrgula, dois pontos, ponto e vrgula;
d) pontos vrgula, dois pontos, ponto e vrgula;
e) ponto e vrgula, vrgula, vrgula.

f) separar os nomes dos locais de datas: Braslia, 30 de janeiro de


2009.
g) isolar oraes adjetivas explicativas: O filme, que voc indicou
para mim, muito mais do que esperava.

3. Assinale o exemplo em que h emprego incorreto da vrgula:


a) como est chovendo, transferi o passeio;
b) no sabia, por que todos lhe viravam o rosto;
c) ele, caso queira, poder vir hoje;
d) no sabia, por que no estudou;
e) o livro, comprei-o por conselho do professor.

2. Ponto e vrgula (;)


usado para:
a) separar itens enumerados:
A Matemtica se divide em:
- geometria;
- lgebra;
- trigonometria;
- financeira.

4. Assinale o trecho sem erro de pontuao,

b) separar um perodo que j se encontra dividido por vrgulas: Ele


no disse nada, apenas olhou ao longe, sentou por cima da grama;
queria ficar sozinho com seu co.
3. Dois-pontos (:)
usado quando:
a) se vai fazer uma citao ou introduzir uma fala:
Ele respondeu: no, muito obrigado!
b) se quer indicar uma enumerao:
Quero lhe dizer algumas coisas: no converse com pessoas
estranhas, no brigue com seus colegas e no responda
professora.

a) vimos pela presente solicitar de V.Sas., que nos informe a


situao econmica da firma em questo;
b) cientificamo-lo de que na marcha do processo de restituio de
suas contribuies, verificou-se a ausncia da declarao de
beneficirios;
c) o Instituto de Previdncia do Estado, vem solicitar de V.Sa. o
preenchimento da declarao;
d) encaminhamos a V.Sa., para o devido preenchimento, o formulrio
em anexo;
e) estamos remetendo em anexo, o formulrio
2. Observe as oraes abaixo ,

4. Aspas ()
So usadas para indicar:
a) citao de algum: A ordem para fechar a priso de
Guantnamo mostra um incio firme. Ainda na edio, os 25 anos
do MST e o bloqueio de 2 bilhes de dlares do Oportunity no
exterior (Carta Capital on-line, 30/01/09)

1) depois de muito pedir ( ) obteve o que desejava;


2) se fosse em outras circunstncias ( ) teria dado tudo certo;
3) exigiam-me o que eu nunca tivera ( ) uma boa educao;
4) fez primeiramente seus deveres ( ) depois foi brincar;
Assinale a alternativa que preencha mais adequadamente os
parnteses:

b) expresses estrangeiras, neologismos, grias: Nada pode com a


propaganda de outdoor.

a) (;) (,) (:) (;);

5. Travesso ()
O travesso indicado para:
a) Indicar a mudana de interlocutor em um dilogo:
- Quais ideias voc tem para revelar?
- No sei se sero bem-vindas.
b) Separar oraes intercaladas, desempenhando as funes da
vrgula e dos parnteses:
Precisamos acreditar sempre disse o aluno confiante que tudo
ir dar certo.
c) Colocar em evidncia uma frase, expresso ou palavra:
O prmio foi destinado ao melhor aluno da classe uma pessoa
bastante esforada.

SECRETARIA DE ESTADO DA EDUCAO

Quando se trata de trabalho cientfico ___ duas coisas devem ser


consideradas ____ uma a contribuio terica que o trabalho
oferece ___ a outra o valor prtico que possa ter.

b) (,) (;) (:) (;);


c) (,) (,) (:) (;);
d) (?) (,) (,) (:);
e) (,) (;) (.) (;).

126

QUMICA
GRAMTICA

AULA 13 :A concordncia na Redao

HAVER FAZER
"Haver" no sentido de existir, indicando tempo ou no sentido de

SUJEITO CONSTITUDO PELOS PRONOMES QUE & QUEM

ocorrer ficar na terceira pessoa do singular. impessoal, ou seja,

QUE: se o sujeito for o pronome relativo que, o verbo concorda com


o antecedente do pronome relativo.

no admite sujeito.
"Fazer" quando indica tempo ou fenmenos da natureza,

- Fui eu que falei. (eu falei)

tambm impessoal e dever ficar na terceira pessoa do singular.

- Fomos ns que falamos. (ns falamos)


- Nesta sala h bons e maus alunos. (= existe)
QUEM: se o sujeito for o pronome relativo quem, o verbo ficar na

- J houve muitos acidentes aqui. (= ocorrer)

terceira pessoa do singular ou concordar com o antecedente do

- Faz 10 anos que me formei. (= tempo decorrido)

pronome (pouco usado).


- Fui eu quem falou. (ele (3 pessoa) falou)
Observao: nas expresses um dos que, uma das que, o verbo
deve ir para o plural. Porm, alguns estudiosos e escritores aceitam
ou usam a concordncia no singular.
- Joo foi um dos que saram.
SUJEITO COLETIVO (SUJEITO SIMPLES)
- O cardume escapou da rede.
- Os cardumes escaparam da rede.
Nesses dois exemplos o verbo concordou com o coletivo (sujeito
simples).
Quando o sujeito formado de um coletivo singular seguido de
complemento no plural, admitem-se duas concordncias:
1) verbo no singular.
- O bando de passarinhos cantava no jardim.
- Um grupo de professores acompanhou os estudantes.
2) o verbo pode ficar no plural, nesse caso o verbo no plural dar
nfase ao complemento.
- O bando de passarinhos cantavam no jardim.
- Um grupo de professores acompanharam os estudantes
SE
Verbos transitivos diretos e verbos transitivos diretos e indiretos + se:

NOMES PRPRIOS NO PLURAL


Se o nome vier antecedido de artigo no plural, o verbo dever
concordar no plural.
- Os Andes ficam na Amrica do Sul.
Se no houver artigo no plural, o verbo dever concordar no
singular.
- Santos fica em So Paulo.
- Memrias Pstumas de Brs Cubas consagrou Machado de
Assis.
Obs 1: Com nome de obras artsticas, admite-se a concordncia
ideolgica com a palavra obra, que est implcita na frase.
- Os Lusadas imortalizou Cames.
Obs 2: Com o verbo ser e o predicativo no singular, o verbo fica no
singular.
Os Lusadas a maior obra da Literatura Portuguesa.
- Os EUA j foi o primeiro mercado consumidor.
SER
O verbo ser concordar com o predicativo quando o sujeito for
o pronome interrogativo que ou quem.
- Quem so os eleitos?
- Que seriam aqueles rudos estranhos?
- Que so dois meses?
- Que so clulas?
- Quem foram os responsveis?
Quando o verbo ser indicar tempo, data, dias ou distncia, deve
concordar com a apalavra seguinte.
- uma hora.
- So duas horas.
EXERCCIO DE FIXAO

Se o termo que recebe a ao estiver no plural, o verbo deve ir para


o plural, se estiver no singular, o verbo deve ir para o singular.
- Alugam-se cavalos.
Alugar verbo transitivo direto.
Cavalos recebe a ao e est no plural, logo o verbo vai para o
plural.
Aqui o se chamado de partcula apassivadora (Cavalos so
alugados).

A frase em que a concordncia est de acordo com a norma culta


,
a) Tudo indica que, se houvessem sido feitas consultas aos
especialistas, no teriam surgido tantas crticas.
b) Alterao de rituais so sempre passveis de controvrsia, por
isso muitas vezes evitado.
c) Feito repentinamente, muitas mudanas acabam por exigirem
revises posteriores.
d) No mesmo dia foi publicado, anonimamente, as mensagens
curtas a favor da quebra do formalismo processual.
e) Nota-se que desse autor que vir, por seu alto conhecimento
jurdico, as orientaes mais confiveis sobre o assunto

Outros exemplos:
- Vendem-se casas.
- Alugam-se apartamentos.
- Exigem-se referncias.
- Consertam-se pianos.
- Plastificam-se documentos.
Qualquer outro tipo de verbo (transitivo indireto ou intransitivo) fica
no singular.
- Precisa-se de professores. (Precisar verbo transitivo indireto)
- Trabalha-se muito aqui. (trabalhar verbo intransitivo)
Nesse caso, o se chamado de ndice de indeterminao do
sujeito ou partcula indeterminadora do sujeito.

SECRETARIA DE ESTADO DA EDUCAO

127

QUMICA
GRAMTICA

6) METFORA COMPARAO
1-Aquele homem um leo.

AULA 14 : FIGURAS DE LINGUAGEM I


So recursos usados pelo falante para realar a sua mensagem.

Estamos comparando um homem com um leo, pois esse homem


forte e corajoso como um leo.

1) ELIPSE ZEUGMA
Veja os exemplos:

2-A vida vem em ondas como o mar.

1-Na estante, livros e mais livros.


2-Ele prefere um passeio pela praia; eu, cinema.

Aqui tambm existe uma comparao, s que desta vez usado o


conectivo comparativo: como.

No 1 exemplo temos uma elipse, j no 2, a figura que aparece o


zeugma.

O exemplo 1 uma metfora e o exemplo 2 uma comparao.

A elipse consiste na omisso de um termo que facilmente


identificado.

Ele um anjo.
Ela uma flor.

No exemplo 1, percebemos claramente que o verbo haver foi


omitido.
No exemplo 2, ocorre zeugma, que a omisso de um termo que j
fora expresso anteriormente.
Ele prefere um passeio pela praia;eu, (prefiro) cinema.(No houve
necessidade de repetir o verbo, pois entendemos o recado).
2) PLEONASMO
Na orao: Ela cantou uma cano linda!, houve o emprego de um
termo desnecessrio, pois quem canta, s pode cantar uma cano.
Na famosa frase: Vi com meus prprios olhos., tambm ocorre o
mesmo.
Pleonasmo a repetio de idias

Exemplos de metfora.

Exemplos de comparao.
A chuva cai como lgrimas.
A mocidade como uma flor.
Metfora: sem o conectivo comparativo.
Comparao: com o conectivo (como, tal como, assim como)
A velhice deve ser respeitada.
Po para quem tem fome.(Po no lugar de alimento)
No tinha teto em que se abrigasse.(Teto em lugar de casa)
EXERCCIOS DE FIXAO

3) HIPRBATO
Exemplos:
Correm pelo parque as crianas da rua.
Na escada subiu o pintor.
As duas oraes esto na ordem inversa.
O hiprbato consiste na inverso dos termos da orao.
Na ordem direta ficaria:
As crianas da rua correm pelo parque.
O pintor subiu na escada.

1)(ENEM)As figuras de linguagem so comumente encontradas nos


textos literrios, bem como em charges e tirinhas.
Nesta tirinha, a personagem faz referncia a uma das mais
conhecidas figuras de linguagem para
a) condenar a prtica de exerccios fsicos.
b) desestimular o uso das bicicletas.
c) caracterizar o dilogo entre geraes.
d) criticar a falta de perspectiva do pai.
e) valorizar aspectos da vida moderna

4) ANACOLUTO
a falta de nexo que existe entre o incio e o fim de uma frase.
Dois gatinhos miando no muro, conversvamos sobre como
complicada a vida dos animais.
5) SILEPSE
a concordncia com a ideia e no com a palavra dita.
Pode ser: de gnero, nmero ou pessoa.
SILEPSE DE GNERO (masc./fem.)Vossa Excelncia est
admirado do fato?

2) Nos trechos:
"...nem um dos autores nacionais ou nacionalizados de oitenta pra
l faltava nas estantes do major" e "...o essencial achar-se as
palavras que o violo pede e deseja"
encontramos, respectivamente, as seguintes figuras de linguagem:
a) prosopopeia e hiprbole;
b) hiprbole e metonmia;
c) perfrase e hiprbole;
d) metonmia e eufemismo;
e) metonmia e prosopopeia.

O pronome de tratamento Vossa Excelncia feminino, mas o


adjetivo admirado est no masculino. Ou seja, concordou com a
pessoa a quem se referia (no caso, um homem).
Aqui temos o feminino e o masculino, logo, silepse de gnero.
SILEPSE DE NMERO (singular/plural)
Aquela multido gritavam diante do dolo.
Multido est no singular, mas o verbo est no plural.
Gritavam concorda com a ideia de plural que est em multido.
SILEPSE DE PESSOA
Todos estvamos nervosos.
Esta frase levaria o verbo normalmente para a 3 pessoa (estavam eles) mas a concordncia foi feita com a 1 pessoa(ns).
Temos aqui 2 pessoas ( eles e ns ) logo, silepse de pessoa.
Mais exemplos:
As duas comemos muita pizza.(elas ns)
Todos compramos chocolates e balas.(eles ns)
Os brasileiros sois um povo solidrio. (eles vs)
Os cariocas somos muito solidrios.(eles ns)

SECRETARIA DE ESTADO DA EDUCAO

128

3)Analisando os fragmentos de textos abaixo,


I. "No te esqueas daquele amor ardente que j nos olhos meus
to puro viste."
II. "A moral legisla para o homem; o direito para o cidado."
III. "A maioria concordava nos pontos essenciais; nos pormenores
porm, discordavam."
IV. "Isaac a vinte passos, divisando o vulto de um, para, ergues a
mo em viseira, firma os olhos."
encontramos, na ordem dada as figuras de linguagem,
a) anacoluto, hiprbato, hiplage, pleonasmo;
b) hiprbato, zeugma, silepse, assndeto;
c) anfora, polissndeto, elipse, hiprbato;
d) pleonasmo, anacoluto, catacrese, eufemismo;
e) hiplage, silepse, polissndeto, zeugma.

QUMICA
GRAMTICA

AULA 16 : FIGURAS DE LINGUAGEM II


1) METONMIA

7) HIPRBOLE
o exagero na afirmao.

Aqui tambm existe a comparao, s que desta vez ela mais

J lhe disse isso um milho de vezes.

objetiva.

Quando o filme comeou, voei para casa.

Ele gosta de ler Agatha Christie.


Ele comeu uma caixa de chocolate.
(Ele comeu o que estava dentro da caixa)
A velhice deve ser respeitada.
Po para quem tem fome.(Po no lugar de alimento)
No tinha teto em que se abrigasse.(Teto em lugar de casa)

8) PROSOPOPIA
Atribuio de qualidades e sentimentos humanos a seres irracionais
e inanimados.
A formiga disse para a cigarra: Cantou...agora dana!
EXERCCIOS DE FIXAO

2) PERFRASE - ANTONOMSIA
A Cidade Maravilhosa recebe muitos turistas durante o carnaval.

1)(Mack) Nos versos abaixo uma figura se ergue garas ao conflito


de duas vises antagnicas:

O Rei das Selvas est bravo.


A Dama do Suspense escreveu livros timos.

Saio do hotel com quatro olhos,


Dois do presente,
Dois do passado.

O Mestre do Suspense dirigiu grandes clssicos do cinema.


Nos exemplos acima notamos que usamos expresses especiais
para falar de algum ou de algum lugar.

Essa figura de linguagem recebe o nome de:


a) metonmia
b) catacrese
c) hiprbole
d ) anttese
e ) hiprbato

Cidade Maravilhosa: Rio de Janeiro


Rei das Selvas: Leo
A Dama do Suspense: Agatha Christie
O Mestre do Suspense: Alfred Hitchcock

2)Analise os exemplos que seguem evidenciando a figura de


linguagem caracterizada pelos mesmos:

Quando usamos esse recurso estamos empregando a perfrase ou


antonomsia.

I. Amor fogo que arde sem se v;


ferida que di e no se sente;
um contentamento descontente;
[...]

Perfrase, quando se tratar de lugares ou animais.


Antonomsia, quando forem pessoas
3) CATACRESE
A catacrese o emprego imprprio de uma palavra ou expresso
por esquecimento ou ignorncia do seu real sentido.
Sentou-se no brao da poltrona para descansar.
A asa da xcara quebrou-se.

II . nunca contentar-se de contente;


um cuidar que ganha em se perder;
[...]

O p da mesa estava quebrado.


Vou colocar um fio de azeite na sopa.
4) ANTTESE
Emprego de termos com sentidos opostos.
Ela se preocupa tanto com o passado que esquece o presente.
A guerra no leva a nada, devemos buscar a paz.

III .Proferiu um milho de palavras tentando convencer-me de que


tinha razo.

5) EUFEMISMO
Aquele rapaz no legal, ele subtraiu dinheiro.
Acho que no fui feliz nos exames.
O intuito dessas oraes foi abrandar a mensagem, ou seja, ser
mais educado.
No exemplo 1 o verbo roubar foi substitudo por uma expresso

IV . A excelente Dona Incia era mestra na arte de judiar de


crianas.

mais leve.
O mesmo ocorre com o exemplo 2 , reprovado tambm foi
substitudo por uma expresso mais leve.
6) IRONIA
Que homem lindo! (quando se trata, na verdade, de um homem
feio.)
Como voc escreve bem, meu vizinho de 5 anos teria feito uma

V.

Faria isso mil vezes se fosse necessrio

redao melhor!
Que bolsa barata, custou s mil reais!

SECRETARIA DE ESTADO DA EDUCAO

129

QUMICA
GRAMTICA

AULA 16 : DICAS PARA MELHORAR SUA REDAO


Dez erros que podem diminuir sua nota na redao do Enem
Separar sujeito e verbo por vrgulas;
Colocar crase antes de verbos;
Usar grias e expresses da internet;
Ler o tema da redao com pressa;
Fugir da proposta de redao;
Estabelecer um ponto de vista vago;
No colocar causas e consequncias da situao proposta pela
prova;
No separar os argumentos que vo construir o texto;
Propor solues inviveis, difceis de serem concretizadas;
Desrespeitar os direitos humanos

O que no usar na redao


longo o caminho para se escrever bem, mas h um conjunto de
regrinhas simples que ajudam a errar menos nessa importante
atividade do vestibular. Vejamos algumas das coisas que voc pode
evitar.
"No sei bem o que dizer da especializao precoce dos
jovens, mas a eliminao do juvenil do Corinthians foi tambm
muito precoce no campeonato!"
Est a um bom exemplo de bola fora. Comece sua redao lendo
com ateno todos os elementos que o examinador apresentou para
voc utilizar. Procure ver se realmente entendeu o que se pede.
Pense num caminho, faa um esquema se preferir, e veja se voc
no est forando, inventando algo fora do contexto. Sim, porque
conta demais o fato de voc escrever dentro ou fora do tema pedido.
J pensou se acharem que voc fugiu do assunto? ... zzzzzzzero!

As 5 competncias avaliadas na redao do Enem


Domnio da norma padro da lngua escrita
O ttulo da primeira competncia avaliada pelos examinadores
pode parecer complexo primeira vista. Trata-se, na verdade, de um
conceito simples. O que o Enem busca avaliar aqui a capacidade
dos estudantes de diferenciar os registros oral e escrito da lngua.
Um exemplo simples: no cotidiano, usamos a expresso "pra"
(contrao da preposio "para" e do artigo "a"). Ela pode se
adequar perfeitamente a nossas conversas dirias, mas no fica bem
quando precisamos fazer um discurso na formatura do colgio ou
ainda ao escrever uma carta para a direo da empresa na qual
trabalhamos. Nessas situaes, deve-se primar pela clareza e pela
preciso, possveis graas norma culta da lngua. O examinador do
Enem quer saber se o candidato conhece essas diferenas e se
sabe escrever usando o portugus correto.

"Esse
tipo
de
gente
merece
ser
exterminado"
Radical demais, no parece? at grosseiro. Nas dissertaes,
como recomendao geral, evite os riscos desnecessrios de
posies extremistas. Vale a pena procurar desenvolver um
texto equilibrado, mostrando as coisas de forma ampla e
assumindo
posies
sem
partidarismos
extremos.
"De uma perspectiva pedaggica e heuristicamente viesada o
numerrio foi nfimo".
Pois . Veja a que escrever com clareza muito importante. Os
outros devem entender o que voc escreveu (especialmente os
examinadores). No use palavras que voc acha bonitas mas no
entende o que significam. Alis, a frase acima no apenas
complexa - apesar de usar termos que existem, ela desprovida de
sentido. Um pequeno truque: procure colocar-se no lugar do leitor ele entenderia seu argumento?

Compreenso da proposta
Um dos erros mais frequentes e graves em redaes de
vestibulares e do Enem a inadequao ao tema proposto. o que
acontece quando o candidato "foge do tema", como se costuma
dizer. Trata-se de uma falta grave porque sinaliza que o estudante
sequer conseguiu entender a proposta da prova (na verdade, o erro
fatal: quem no demonstra essa competncia ganha nota zero na
redao).

Evite erros bsicos (um exerccio: corrija os erros que esto em


destaque vermelho nas frases entre aspas.). Fuja de palavras de
grafia duvidosa para voc. Use s termos que voc conhece.
Respeite a gramtica e as regras de grafia. Isso pega bem.
"Ele deu um pum fedido pacas. Foi aquele au!"

Capacidade de organizar e relacionar informaes


Alm de apresentar o tema de redao, o Enem oferece aos
candidatos textos de apoio, que podem servir de subsdio reflexo
a ser desenvolvida. Esses textos ajudam o exame a avaliar a
capacidade do estudante de selecionar e interpretar essas
informaes e as relacionar com outras, previamente conhecidas por
ele. avaliada ainda a capacidade de organizar todo esse
conhecimento em defesa de um ponto de vista pessoal.
Construo da argumentao
Os aspectos avaliados nessa competncia dizem respeito
estruturao do texto e apresentao da argumentao. O estudante
deve demonstrar que sabe usar o idioma para desenvolver suas
ideias sobre o tema proposto de maneira clara e lgica. Dessa forma,
ser bem-sucedido na tarefa de comunicar a mensagem pretendida.
Elaborar proposta de interveno ao problema exposto
A ltima competncia busca avaliar se o candidato tem condies
de propor alguma ideia para solucionar um problema. fundamental
detalhar os meios que seriam utilizados para a soluo do problema.
O prprio MEC ressalta que as propostas devem ser feitas
respeitando-se os direitos humanos, o que implica no romper com
valores como cidadania, liberdade, solidariedade e diversidade
cultural.

SECRETARIA DE ESTADO DA EDUCAO

"Fazem muitos anos que eu queria isso/ Se ela quizer deixar


isso para mim fazer/ A muito tempo estou afins".

130

Gostou disso? E o examinador tambm iria gostar? Hoje, a


linguagem parece que "liberou geral" -- como costuma-se dizer. Ser
que entramos na era do vale-tudo? Cuidado! Esse territrio
perigoso. Sugerimos que deixe a "franqueza" vocabular para lanar
alguma nova verso de "sabo cr-cr". A, pode deitar e rolar -- o
pblico ir agradecer, se voc for bem-sucedido. Agora, no
vestibular, cuide-se. Evite grosserias na sua redao. No s voc
que tem me, irm etc. Os examinadores tambm tm e nem todos
so apaixonados pelo "extico".
Ahxxvdfjkkl...gfgdfg34tggg
Nem precisava ser dito, mas limpeza conta. Procure manter uma
letra razovel. Nada de emporcalhar a folha. o mnimo para seu
cuidado com o contedo e para a qualidade do texto no
desaparecer no meio de rabiscos.

Afastar pedras do caminho no constri a boa redao, apenas


evita que voc comprometa seu trabalho. O trabalho de formular um
bom texto tambm tem suas regras, mas esse outro problema...

QUMICA
GRAMTICA

AULA 01: A LINGUAGEM LITERRIA E O CONCEITO DE


ESTILOS DE POCA NA LITERATURA
I)A LINGUAGEM LITERRIA
Sabemos que o reino das palavras farto. Elas brotam de nosso
pensamento de maneira natural, no temos a preocupao de
elaborar o que dizemos ou at mesmo escrevemos. As palavras,
contudo, podem ultrapassar seus limites de significao. Podendo
assim, conquistar novos espaos e passar novas possibilidades de
perceber a realidade. O caminho que a literatura percorre este. O
artista sente, escolhe e manipula as palavras, as organiza para que
produzam um efeito que v alm da sua significao objetiva,
procurando aproxima-las do imaginrio. A obra do escritor fruto de
sua imaginao, embora seja baseado em elementos reais. Da
concretizao desse trabalho surge ento a obra literria. Dotado de
uma percepo aguada, o escritor capta a realidade atravs de
seus sentimentos. Explora as possibilidades lingusticas e as
manipula no nvel semntico, fontico e sinttico.
II)LINGUAGEM CONOTATIVA, a linguagem da literatura
Em semntica, a denotao de um termo o objeto ao qual o
mesmo se refere. A palavra tem valor referencial ou denotativo
quando tomada no seu sentido usual ou literal, isto , naquele
que lhe atribuem os dicionrios; seu sentido objetivo, explcito,
constante.
a forma de linguagem que lemos em jornais, bulas de
remdios, em um manual de instrues etc. Conotao o
conjunto de caracteres compreendidos na significao de um
dado termo, conceito, etc. Alm do sentido referencial, literal,
cada palavra remete a inmeros outros sentidos, virtuais,
conotativos, que so apenas sugeridos, evocando outras ideias
associadas, de ordem abstrata, subjetiva.
Veja no quadro abaixo as diferenas entre linguagem denotativa
e linguagem conotativa:
DENOTAO
CONOTAO
A palavra tem significado restrito. A palavra tem significado amplo,
criado pelo contexto.
O sentido da palavra comum, o O sentido das palavras
do dicionrio.
carregado de contexto emotivo,
social, ideolgico, etc.
A palavra utilizada de modo
Utiliza-se a palavra de modo
objetivo: diz-se o que se quer
criativo, artstico.
dizer.
A linguagem exata e precisa: o A linguagem expressiva, possui
objetivo deixar claro o que est mais de um significado.
sendo dito.
Logia e mitologia
Meu corao
de mil e novecentos e setenta e dois
j no palpita fagueiro
sabe que h morcegos de pesadas olheiras
que h cabras malignas que h
cardumes de hienas infiltradas
no vo da unha na alma
um porco belicoso de radar
e que sangra e ri
e que sangra e ri
a vida anoitece provisria
centuries sentinelas
do Oiapoque ao Chu.
CACASO. Lero-lero. Rio de Janeiro: 7 Letras; So
Paulo: Cosac & Naify, 2002

O ttulo do poema explora a expressividade de termos que


representam o conflito do momento histrico vivido pelo poeta na
dcada de 1970. Nesse contexto, correto afirmar que
a) o poeta utiliza uma srie de metforas zoolgicas com
significado impreciso.
b) morcegos, cabras e hienas metaforizam as vtimas do
regime militar vigente.
c) o porco, animal difcil de domesticar, representa
os movimentos de resistncia.
d) o poeta caracteriza o momento de opresso atravs de
alegorias de forte poder de impacto.
e) centuries e sentinelas simbolizam os agentes que garantem
a paz social experimentada.

SECRETARIA DE ESTADO DA EDUCAO

131

III)ESTILOS DE POCA NA LITERATURA: as manifestaes


artsticas demarcadas nos estilos de poca foram produto de
um contexto histrico-social dominante.
As manifestaes literrias integram os chamados estilos de
poca ou estticas literrias, escolas literrias, ou
movimentos literrios.
Os autores que vivem em um mesmo tempo, geralmente,
apresentam semelhanas em assuntos abordados, linguagem,
estrutura textual e vises de mundo pautadas em princpios
parecidos.
Estilo de poca, portanto, a maneira de ver e de expressar o
mundo, refletindo caractersticas prprias de uma poca
histrica. Por ser arte, a Literatura um meio pelo qual os
escritores e poetas mostram suas vises de mundo, tanto
relacionado ao individual, quanto da realidade histrica. Assim
ele focaliza a sociedade; o cotidiano; valores polticos,
sociais e religiosos.
Veja como Jos de Alencar, escritor da primeira metade do sculo
XIX, descreve a personagem Iracema, de livro homnimo.
"Alm, muito alm daquela serra, que ainda azula no horizonte,
nasceu Iracema. Iracema, a virgem dos lbios de mel, que tinha os
cabelos mais negros que a asa da grana, e mais longos que seu
talhe de palmeira.
O favo da jati no era doce como seu sorriso (...)
Mais rpida que a ema selvagem, a morena virgem corria o serto e
as matas do Ipu..."
Iracema, JOS DE ALENCAR
Agora veja, como na letra da msica de Chico, Iracema tem outra
perspectiva: uma imigrante nos Estados Unidos:
"Iracema voou
Para a Amrica
............................................
V um filme de quando em vez
No domina o idioma ingls
Lava cho numa casa de ch
...............................................
No d mole pra polcia
Se puder, vai ficando por l
Tem saudade do Cear"
Iracema, CHICO BUARQUE
EXERCCIO DE FIXAO
No trecho abaixo, o narrador, ao descrever a personagem, critica
sutilmente um outro estilo de poca: o romantismo.
Naquele tempo contava apenas uns quinze ou dezesseis anos; era
talvez a mais atrevida criatura da nossa raa, e, com certeza, a mais
voluntariosa. No digo que j lhe coubesse a primazia da beleza,
entre as mocinhas do tempo, porque isto no romance, em que o
autor sobredoura a realidade e fecha os olhos s sardas e espinhas;
mas tambm no digo que lhe maculasse o rosto nenhuma sarda ou
espinha, no. Era bonita, fresca, saa das mos da natureza, cheia
daquele feitio, precrio e eterno, que o indivduo passa a outro
indivduo, para os fins secretos da criao.
ASSIS, Machado de. Memrias Pstumas de Brs Cubas.
Rio de Janeiro: Jackson,1957.

A frase do texto em que se percebe a crtica do narrador ao


romantismo est transcrita na alternativa
a)
b)
c)
d)
e)

... o autor sobredoura a realidade e fecha os olhos s sardas e


espinhas ...
... era talvez a mais atrevida criatura da nossa raa ...
Era bonita, fresca, saa das mos da natureza, cheia daquele
feitio, precrio e eterno, ...
Naquele tempo contava apenas uns quinze ou dezesseis anos
...
... o indivduo passa a outro indivduo, para os fins secretos da
criao.

QUMICA
LITERATURA

AULA 02: PERODO COLONIAL DA LITERATURA BRASILEIRA:


A Literatura de Informao e o Barroco.
A nossa literatura colonial manteve aqui to viva quanto lhe
era possvel a tradio literria portuguesa. Submissa a esta e
repetindo-lhe as manifestaes, embora sem nenhuma excelncia e
antes inferiormente, animou-a todavia desde o princpio o nativo
sentimento de apego terra e afeto s suas cousas. Ainda sem
propsito acabaria este sentimento por determinar manifestaes
literrias que em estilo diverso do da metrpole viessem a exprimir
um gnio nacional que paulatinamente se diferenava.
Necessariamente nasceu e desenvolveu-se a literatura no Brasil
como rebento da portuguesa e seu reflexo. Somente para o fim do
sculo XVIII que entramos a sentir nos poetas brasileiros algo que
os comea a distinguir.
No era de se admirar que, durante esse perodo, que corresponde
do Descobrimento ao processo de emancipao poltica,
ocorressem vrias manifestaes literrias de um grupo composto
por alguns escritores que copiavam os padres e tendncias de
Portugal
OS
ESTILOS
DE
POCA
DO
BRASIL
COLONIAL:
contextualizao histrica geral.
QUINHENTISMO (1500-1601): a chegada dos colonizadores e
cronistas estrangeiros e a literatura sobre o Brasil.
Diversos viajantes europeus que estiveram no pas durante a
colonizao do Brasil, no sculo XVI, registraram suas observaes
sobre a terra. Fizeram-no por obrigao profissional ou motivos
pessoais. Assim, seus textos so basicamente depoimentos e relatos
de viagem, com a finalidade de apresentar aos compatriotas um
panorama do Novo Mundo.
Sob forma de cartas, dirios, tratados ou crnicas esses textos
informativos foram escritos principalmente por portugueses e
compem o chamado Perodo de Informao, Literatura de
Informao, ou Literatura Colonial.
O primeiro texto da poca a clebre "Carta de achamento do
Brasil", de Pero Vaz de Caminha ao rei de Portugal, D. Manuel 1.,
escrita entre abril e maio de 1500, quando a frota de Cabral se
preparava para deixar o Brasil, seguindo em direo ndia. Nela, o
escrivo da armada d conta do descobrimento da terra,
descrevendo seus aspectos fsicos e os contatos com os nativos.
Europeus de outras nacionalidades que aqui estiveram tambm
deixaram documentos importantes sobre o Brasil de ento. o caso
de "Duas Viagens ao Brasil" (1557), do alemo Hans Staden, que
descreve pormenorizadamente o modo de vida dos tupinambs, dos
quais o autor foi prisioneiro em 1554.
Tambm se destacam "Viagem Terra do Brasil", de Jean de
Lry e "As Singularidades da Frana Antrtica", de Andr Thevet,
que documentam a tentativa de colonizao francesa na baa da
Guanabara comandada por Villegaignon.
Missionrios jesutas tambm estiveram no Brasil, a partir do
primeiro Governo-geral. Seu objetivo principal era catequizar os
ndios, convertendo-os ao cristianismo, mas seu trabalho acabou
ultrapassando os limites religiosos e interferiu em diversos aspectos
da vida colonial, particularmente com a criao de escolas.
Os jesutas tambm nos legaram obras sobre o perodo, como as
"Cartas", de Manuel da Nbrega e Jos de Anchieta, os fundadores
da cidade de So Paulo.

EXERCCIO DE FIXAO
A CARTA DE CAMINHA: a certido de nascimento do Brasil Pero Vaz de Caminha, referindo-se aos indgenas escreveu:
E naquilo sempre mais me conveno que so como aves
ou animais montesinhos, aos quais faz o ar melhor pena e melhor
cabelo que aos mansos, porque os seus corpos so to limpos, to
gordos
e
formosos,
a
no
mais
poder.
[]
Parece-me gente de tal inocncia que, se ns entendssemos a sua
fala e eles a nossa, seriam logo cristos, visto que no tm nem
entendem crena alguma, segundo as aparncias. E, portanto, se os
degredados que aqui ho de ficar aprenderem bem a sua fala e eles
a nossa, no duvido que eles, segundo a santa teno de Vossa
Alteza, se faro cristos e ho de crer na nossa santa f, qual
praza a Nosso Senhor que os traga, porque certamente esta gente
boa e de bela simplicidade. E imprimir-se- facilmente neles todo e
qualquer cunho que lhes quiserem dar, uma vez que Nosso Senhor
lhes deu bons corpos e bons rostos, como a homens bons. E o fato
de Ele nos haver at aqui trazido, creio que no o foi sem causa. E
portanto, Vossa Alteza, que tanto deseja acrescentar santa f
catlica, deve cuidar da salvao deles. E aprazer Deus que com
pouco trabalho seja assim.[]
Eles no lavram nem criam. No h aqui boi ou vaca,
cabra, ovelha ou galinha, ou qualquer outro animal que esteja
acostumado ao convvio com o homem. E no comem seno deste
inhame, de que aqui h muito, e dessas sementes e frutos que a
terra e as rvores de si deitam. E com isto andam tais e to rijos e
to ndios que o no somos ns tanto, com quanto trigo e legumes
comemos.
CASTRO, Silvio. A carta de Pero Vaz de Caminha. Porto Alegre:
L&PM, 1996.

01) De acordo com o texto e seus conhecimentos, marque a


alternativa correta:
a)

b)

c)

d)

e)

Caminha, numa viso eurocentrista, exalta a cultura do


descobridor, menosprezando todos os aspectos referentes ao
modo de vida dos nativos, por exemplo, a no explorao
daqueles mamferos placentrios exticos, citados na carta,
introduzidos no Brasil quando da colonizao.
Caminha realiza, atravs de farta adjetivao, descries
botnicas minuciosas acerca da flora da nova terra, destacando
o tipo de alimentao do europeu rica em vitaminas e sais
minerais em contraposio indgena, que rica em lipdios.
A religiosidade est presente ao longo do texto, quando se
constata que o emissor, tendo em mente a converso dos
ndios santa f catlica pretenso dos europeus
conquistadores , ressalta positivamente a existncia de
crenas animistas entre os nativos.
Na carta de Pero Vaz de Caminha, que apresenta linguagem
formal, por ser o rei portugus o destinatrio, h forte
preocupao com aspectos da necessria converso dos ndios
ao catolicismo, no contexto de crise religiosa na Europa.
Ao realizar concomitantemente a narrao e a descrio dos
hbitos dos nativos, o remetente destaca informaes no s do
habitat como dos usos e costumes indgenas, exaltando o
cultivo das plantas de lavouras e dos pomares.

Textos que apresentam: aspectos literrios e informativos,


deslumbramento e fantasia em relao terra, viso
etnocntrica do colonizador, perplexidade e choque cultural,
sentimento nativista, influncia em estilos posteriores.

SECRETARIA DE ESTADO DA EDUCAO

132

QUMICA
LITERATURA

AULA 03: PERODO COLONIAL DA LITERATURA BRASILEIRA


o Arcadismo O Iluminismo e o Sculo das Luzes
O Arcadismo, tambm conhecido como Setecentismo ou
Neoclassicismo, o movimento que compreende a produo literria
brasileira na segunda metade do sculo XVIII. O nome faz referncia
Arcdia, regio do sul da Grcia que, por sua vez, foi nomeada em
referncia ao semideus Arcas (filho de Zeus e Calisto). Denota-se,
logo de incio, as referncias mitologia grega que perpassa o
movimento. Profundas mudanas no contexto histrico mundial
caracterizam o perodo, tais como a ascenso do Iluminismo, que
pressupunha o racionalismo, o progresso e as cincias. Na Amrica
do Norte, ocorre a Independncia dos Estados Unidos, em 1776,
abrindo caminho para vrios movimentos de independncia ao longo
de toda a Amrica, como foi o caso do Brasil, que presenciou
inmeras revolues e inconfidncias at a chegada da Famlia Real
em 1808.
O Arcadismo se inicia no incio do ano de 1700 e por isso
recebe o nome tambm de Setecentismo, ou ainda neoclassicismo.
Esta ltima denominao surgiu do fato dos autores do perodo
imitarem, no de uma forma pura, mas alguns aspectos da
antiguidade greco-romana ou o chamado Classicismo, e tambm os
escritores do Renascimento, os quais vieram logo aps a idade
clssica. O classicismo compreende a poca literria do
Renascimento, no qual o homem tem a viso antropocntrica do
mundo, ou seja, o homem como centro de todas as coisas. Os
renascentistas prezavam as obras clssicas, j que tinham a
convico de que a arte tinha alcanado sua perfeio. Assim como
os renascentistas, os escritores rcades pretendiam retomar o estilo
clssico, contudo com uma nova maneira, denominada de
Neoclssica, de observar as consideraes artsticas abordadas
naquele perodo, como a razo e a cincia, conceitos oriundos do
Iluminismo.
O Iluminismo determinado pela revoluo intelectual
ocasionada por volta dos sculos XVII e XVIII, o qual trazia como
lema: liberdade, igualdade e fraternidade, o que influenciou os
pensamentos artsticos da poca na Europa, e principalmente a
Revoluo Francesa, a independncia das colnias inglesas da
Amrica Anglo-Saxnica e no Brasil, a Inconfidncia Mineira.
O novo modo de analisar a cientificidade e a racionalidade
da poca rcade fugia das convenes artsticas da poca, j que os
escritores retomam as caractersticas clssicas, como: bucolismo
(busca de uma vida simples, pastoril), exaltao da natureza (refgio
potico, em oposio vida urbana), pacificidade amorosa
(relacionamentos tranquilos), a mitologia pag, clareza na escrita
com utilizao de perodos curtos e versos sem rima. Os poetas
rcades so frequentemente citados como fingidores poticos, pois
escrevem sobre temas que no correspondem com a realidade do
perodo histrico, visto anteriormente.
Um dos principais escritores rcades foi o poeta latino
Horcio, que viveu entre 68 a.C. e 8 a.C., e foi influenciador do
pensamento do carpe diem, viver agora, desfrutar do presente,
adotado pelo Arcadismo e permanente at os dias de hoje.
Os principais autores do Arcadismo brasileiro so: Toms
Antnio Gonzaga, Cludio Manuel da Costa e Santa Rita Duro.
SNTESE HISTRICA

O sculo XVIII, tambm referido como Sculo das Luzes,


representa uma fase de importantes transformaes no campo
da cultura europeia.
O ILUMINISMO e as ideias libertrias: em toda a Europa a
influncia do pensamento Iluminista burgus se alastra.
As ideias dos filsofos iluministas: Descartes, Voltaire, Diderot,
Montesquieu.
Publicao da Enciclopdia.
Crescente urbanizao e fortalecimento poltico da burguesia.
Perodo de renovao cultural que se caracteriza, em linhas
gerais, pela valorizao da Cincia e do esprito racionalista.
A Inconfidncia Mineira e os ideais nativistas e sentimentos
separatistas

CARACTERSTICAS DO ARCADISMO OU NEOCLASSICISMO.


O arcadismo constitui-se numa forma de literatura mais simples,
opondo-se aos exageros e rebuscamentos do Barroco, expresso pela
expresso latina "inutilia truncat" ("cortar o intil").

SECRETARIA DE ESTADO DA EDUCAO

133

As situaes mais frequentes apresentam um pastor abandonado


pela amada, triste e queixoso. a "aurea mediocritas"
("mediocridade urea"), que simboliza a valorizao das coisas
cotidianas, focalizadas pela razo.
Os seus autores acreditavam que a Arte era uma cpia da
natureza, refletida atravs da tradio clssica. Por isso a presena
da mitologia pag, alm do recurso a frases latinas.
Inspirados na frase do escritor latino Horcio "fugere urbem" ("fugir
da cidade"), e imbudos da teoria do "bom selvagem" de JeanJacques Rousseau, os autores rcades voltam-se para a natureza
em busca de uma vida simples, buclica, pastoril, do "locus
amoenus", do refgio ameno em oposio aos centros urbanos
dominados pelo Antigo Regime, pelo absolutismo monrquico.
Utilizavam tambm o "fingimento potico" fato que transparece no
uso dos pseudnimos pastoris, como forma de criar um quadro
idealizado da vida simples e aprazvel.
Diante da efemeridade da vida, defendem o "carpe diem", pelo qual
o pastor, ciente da brevidade do tempo, convida a sua pastora a
gozar o momento presente.
Valorizao da vida no campo (bucolismo)
Crtica a vida nos centros urbanos
Objetividade
Idealizao da mulher amada
Cabe ressaltar, no entanto, que os membros da Arcdia eram
todos burgueses e habitantes dos centros urbanos. Por isso a eles
so atribudos um fingimento potico, isto , a simulao de
sentimentos fictcios
EXERCCIO DE FIXAO
Torno a ver-vos, montes; o destino
Aqui me torna a pr nestes outeiros,
Onde um tempo os gabes deixei grosseiros
Pelo traje da Corte, rico e fino.
Aqui estou entre Almendro, entre Corino,
Os meus fiis, meus doces companheiros,
Vendo correr os mseros vaqueiros
Atrs de seu cansado desatino.
Se o bem desta choupana pode tanto,
Que chega a ter mais preo, e mais valia
Que, da Cidade, o lisonjeiro encanto,
Aqui descanse a louca fantasia,
E o que at agora se tornava em pranto
Se converta em afetos de alegria.
Cludio Manoel da Costa. In: Domcio Proena Filho. A poesia dos
inconfidentes. Rio de Janeiro: Nova Aguilar, 2002, p. 78-9.

(ENEM 2010)- Considerando o soneto de Cludio Manoel da Costa e


os elementos constitutivos do Arcadismo brasileiro, assinale a opo
correta acerca da relao entre o poema e o momento histrico de
sua produo.
a) Os montes e outeiros, mencionados na primeira estrofe, so
imagens relacionadas Metrpole, ou seja, ao lugar onde o poeta se
vestiu com traje rico e fino.
b) A oposio entre a Colnia e a Metrpole, como ncleo do poema,
revela uma contradio vivenciada pelo poeta, dividido entre a
civilidade do mundo urbano da Metrpole e a rusticidade da terra da
Colnia.
c) O bucolismo presente nas imagens do poema elemento esttico
do Arcadismo que evidencia a preocupao do poeta rcade em
realizar uma representao literria realista da vida nacional.
d) A relao de vantagem da choupana sobre a Cidade, na
terceira estrofe, formulao literria que reproduz a condio
histrica paradoxalmente vantajosa da Colnia sobre a Metrpole.
e) A realidade de atraso social, poltico e econmico do Brasil
Colnia est representada esteticamente no poema pela referncia,
na ltima estrofe, transformao do pranto em alegria.

QUMICA
LITERATURA

A LITERATURA DA ERA NACIONAL I contexto histrico e


caractersticas gerais do Romantismo
Gerado sob o impacto da Revoluo Industrial e da
Revoluo Francesa, de fins do sculo 18, o Romantismo surgiu no
incio do sculo XIX, na Alemanha, Frana e Inglaterra, num
momento histrico em que as classes sociais, como as conhecemos
hoje, se definiam. Na ocasio, a sociedade se reorganizava e as
classes sociais criavam ou redefiniam suas vises da existncia e do
mundo.
Das classes sociais desse perodo, a nobreza e a
pequena burguesia so as classes que vo atuar essencialmente
no movimento romntico. Assim, o romantismo expressa, nas
palavras de Karl Mannheim, os sentimentos dos descontentes com a
nova ordem socioeconmica, isto , com o capitalismo industrial.
Recm-afastada do poder pelas revolues, a nobreza s
podia amargar uma nostalgia do Antigo regime. Ao contrrio, a
pequena burguesia expressava espanto e insegurana, vendo
barrados pela grande burguesia, pelos verdadeiros capitalistas, seus
projetos de ascenso social, desenvolvidos durante a luta contra a
nobreza
INSATISFEITOS E INCONFORMADOS
Em comum, essas duas classes sociais tm a
insatisfao e o inconformismo com a realidade, o que permite
compreender muitos traos subjacentes ao movimento romntico.
o caso do escapismo ou evasionismo, a necessidade de escapar
ao mundo objetivo, sociedade, ao tempo presente, em busca de
refgio no mundo subjetivo, no indivduo, no tempo passado.
A viso de mundo que privilegia o sujeito ou o
subjetivismo, elemento essencial ao pensamento romntico, uma
manifestao de amor liberdade do prprio romantismo, na
medida em que constitui uma afirmao dos valores individuais em
oposio s normas sociais. tambm uma forma de oposio aos
valores neoclssicos dos sculos anteriores, cujo racionalismo
artstico passou a ser desprezado em favor de um emocionalismo
e de um misticismo (este ltimo, por sua vez, ostenta a religio
crist em oposio mitolologia clssica, que tinha sido
revalorizada durante sculo XVIII).
AMORES E AVENTURAS
De qualquer modo, ponto pacfico que a forma romance
se propaga e consolida no sculo 19, tendo se tornado o grande
veculo de difuso de ideias, sentimentos e emoes, e inclusive
crtica social da poca. Dando vazo ao registro dos costumes,
fico histrica, narrativa de amores e de aventuras, o romance foi
a forma que melhor se adaptou s necessidades expressivas dos
autores daquela poca.
Da mesma maneira, foi a que melhor serviu ao
entretenimento do pblico leitor de ento. Nos centros urbanos, que
conheciam um perodo de franca expanso com a implantao da
indstria e dos servios, a classe mdia crescia e se consolidava,
descobrindo na leitura uma acessvel forma de lazer (hoje
encontrado na televiso).
Assim, eram os jovens e as mulheres das cidades, com
alguns recursos e instruo, que compunham basicamente o pblico
leitor de romances, onde encontravam, em forma narrativa, uma
projeo de suas prprias emoes, expectativas, busca de amor e
felicidade, e ainda identificava suas desiluses.

O gosto pela cultura clssica, que se inicia no sculo XVI,


com o Renascimento, perdura at o sculo XVIII. Com o tempo, ele
foi se restringindo ao pblico aristocrata, formado pela nobreza e
pelo alto clero. As revolues burguesas mudam o perfil da
sociedade europeia no incio do Sculo XIX. Com a ascenso da
burguesia ao poder na Frana, em 1789, surge a necessidade de
uma arte sintonizada com aquele contexto social e com a
sensibilidade do novo pblico que se formava. Liderando
transformaes sociais e econmicas, a burguesia ganha poder
poltica e passa a buscar uma arte na qual possa reconhecer-se.
Quem vai suprir este anseio deste novo estrato social o
Romantismo. O Romantismo renega as formas rgidas da literatura,
como versos de mtrica exata. O romance torna-se o gnero
narrativo preferencial, em oposio a epopeia. Dois dos aspectos
fundamentais da temtica romntica so o historicismo e o
individualismo.
CARACTERSTICAS DO ROMANTISMO

O PAPEL DA MDIA
Alm disso, o desenvolvimento do jornalismo no sculo
XIX, com o surgimento de jornais e revistas regulares (dirios e
semanrios) gerou um suporte material que, alm de barato e de fcil
acesso ao pblico em geral, se revelou intrinsecamente propcio ao
romance. Afinal, por apresentar uma narrativa longa, o romance se
subdivide em unidades menores, os captulos.
Assim, o romance romntico do sculo XIX era publicado,
captulo por captulo, numa parte dos jornais, no chamado folhetim,
espao cuja funo era amenizar o peso e a gravidade das leituras
polticas, econmicas, do noticirio em geral, caracterizando-se pela
diverso e o entretenimento.

SECRETARIA DE ESTADO DA EDUCAO

134

Individualismo e o subjetivismo: em oposio ao objetivismo


dos clssicos, o que faz com que os romnticos vejam o mundo
unicamente por meio de seu mundo interior. A 1 pessoa (eu)
constante na poesia romntica.
O nacionalismo: ao qual se relacionam: o culto a da idade me
dia pelos europeus, na qual se encontravam elementos
formadores da nacionalidade de cada povo: os heris das
cruzadas, as damas, os monges, as lendas, as crenas e
tradies.
O indianismo: que no Brasil, devido ausncia de um passado
medieval, foi um dos elementos da sustentao do sentimento
nacionalista, acentuado com a proximidade com a
Independncia.
O culto da natureza: que se supervalorizava, no s constitui
um refgio no contaminado pela sociedade, como tambm
uma forma de exaltao da terra brasileira. Ao contrrio do
Arcadismo, em que a natureza apenas decorativa, para os
romnticos ela fonte de inspirao e est intimamente
vinculada aos sentimentos do poeta.
Evaso ou escapismo: resultante do conflito do eu com a
realidade, o que leva o romntico a evadir-se na aspirao de
um outro mundo-situado no tempo e no espao ( paisagens
novas, primitivas ou exticas) -, onde ele no encontre asa
dificuldades da realidade a que est vinculado. Resultam da:
saudosismo (da infncia, do passado, da ptria, dos entes
queridos);
o sonho (que permite a criao de um mundo pessoal e
idealizado);
a conscincia da solido (resultante de uma inadaptao ao
mundo e da crena de que um incompreendido);
o exagero, isto , apelo aos extremos e excesso de figuras de
linguagem. Egocentrismo.
O exagero, somado instabilidade e nsia de evaso,
provocou o chamado mal-do-sculo, expresso que
caracterizava o estado de esprito de vrios romnticos que se
entregavam a uma excessiva melancolia e solido e a um
grande pessimismo que conduziu exaltao da morte.
O sobrenatural, o mrbido, a noite e o mistrio da
existncia esto presentes nos textos dos poetas da gerao
do mal-do-sculo.
A idealizao da mulher e do amor: somando espiritualismo e
temperamento sonhador, o poeta romntico reveste a mulher de
uma aura angelical, retratando-a como figura poderosa e
inacessvel. Porm, apesar do espiritualismo, a poesia
romntica reflete muitas vezes o sensualismo bem material na
descrio feminina.
Erotismo, lutas sociais e desejo de liberdade

QUMICA
LITERATURA

AULA 05: A LITERATURA DA ERA NACIONAL II a produo


literria do Romantismo no Brasil.
INTRODUO: Contexto Histrico do Romantismo no Brasil
Considera-se que o perodo romntico no Brasil inicia em
1836, com a publicao da obra Suspiros Poticos e Saudades, do
poeta Gonalves de Magalhes e vai at o ano de 1881, com a
publicao do romance realista Memrias Pstumas de Brs
Cubas de Machado de Assis.
Como dito anteriormente, o desenvolvimento da
literatura brasileira propriamente dita aconteceu a partir da
vinda da Famlia Real para o Rio de Janeiro que gerou um forte
desenvolvimento artstico e cultural na colnia, agora afinado
com a produo literria europeia. Porm, a insatisfao das
classes dominantes com o Imprio fez com que surgissem tentativas
de independncia da metrpole, produzindo um sentimento de
nacionalismo que culminaria com a Declarao da Independncia,
em 1822, por Dom Pedro I.
Outro aspecto importante com relao escravido dos
negros: o Brasil era uma das poucas colnias americanas que ainda
sustentava o sistema econmico baseado do trabalho escravo, o que
gerou opinies controversas por parte dos autores daquela poca.
Temos expresses literrias abolicionistas (p. ex.: o poeta Gonalves
de Magalhes) e outras que tratavam do tema superficialmente (p.
ex.: o romancista Bernardo Guimares) ou sequer tocavam na
questo.
A
independncia
das
colnias
latino-americanas
impulsionou um sentimento de nacionalidade diretamente
refletida pela literatura. A formao dessas literaturas esteve a
cargo de autores que projetavam os ideais de uma nao em
crescimento e desenvolvimento e que at hoje so considerados
constitutivos da histria da nao. No entanto, essa literatura
fundacional e cannica da Amrica Latina revista por muitos
professores, crticos literrios e historiadores pois apresentam
apenas uma viso referente formao das naes latinoamericanas. Como assinala o professor e crtico literrio Eduardo F.
Coutinho:
"Na Amrica Latina, durante o sculo XIX, o sujeito
encunciador do discurso fundador do estado-nao tomou
como base um projeto patriarcal e elistista, que excluiu no s
a mulher, mas ndios, negros, analfabetos e, em muitos casos,
aqueles que no possuam nenhum tipo de propriedade. A
preocupao dominante era marcar a diferena da nova nao
com relao matriz colonizadora, mas o modelo era bvia e
paradoxalmente a metrpole; da a necessidade de forjar-se
uma homogeneidade que exclusse todas as diferenas."
O que causa uma sensao de estranhamento o
paradoxo observado no perodo: ao mesmo tempo em que ideias
sobre o sentimento de nacionalidade aflorava nos coraes dos
brasileiros (e demais latino-americanos), parte da populao
permanecia na misria e/ou em situaes de escravido, sem acesso
emancipao e aos direitos humanos bsicos.
A POESIA ROMNTICA NO BRASIL: as trs geraes o
nacionalismo, o pessimismo e o desejo de liberdade
Na poesia, distinguem-se trs fases, as chamadas Geraes
Romnticas:

A PROSA DO ROMANTISMO NO BRASIL: o surgimento do


folhetim e do romance.
No perodo romntico, o termo FOLHETIM servia para
designar dois tipos de texto, tendo ambos como suporte a imprensa
peridica: o folhetim-crnica e o romance folhetinesco.
Na primeira aceo, o folhetim constitua uma seco de
crnicas da atualidade, a qual, segundo o tipo de publicao em que
surgia, podia abrigar matrias variadas: notcias nacionais e
estrangeiras, notas de viagem, recenses, crticas literrias e
teatrais, etc. Conhecidos folhetinistas foram Lopes de Mendona
(1827-1865), Incio Pizarro de Morais Sarmento (1807-1870),
Ernesto Biester (1829-1880), Ricardo Guimares (1830-1889) e Jlio
Csar Machado (1835-1890), entre outros.
Na segunda aceo, o termo folhetim aplicava-se a
narrativas literrias publicadas em srie nos jornais e revistas. Neste
caso, a publicao de um romance ou novela em folhetim servia,
muitas vezes, como meio de divulgao e de lanamento para a
publicao em livro. Assim aconteceu, por exemplo, com as Viagens
na Minha Terra, de Almeida Garrett (1799-1854), inicialmente
publicadas em srie na Revista Universal Lisbonense em 1843 e
1845 e depois publicadas em volume em 1846.
TIPOS DE FOLHETIM: a tendncia do romance romntico
O primeiro romance brasileiro surgiu durante o romantismo
em 1851, e a Moreninha, de Joaquim Manoel de Macedo. No
comeo os romances era, publicados em capitulos dirios assim
como as novelas e seriados, s que naquele tempo, era em jornais
em forma de folhetins, levava muito tempo para ler a histria toda.
Mas se parar para pensar, as pessoas naquela poca dormiam e
acordavam muito cedo, no tinha muitas formas de entreterimento
como hoje em dia, ler esses romances devia ser u do borogod!.
a) Romances romnticos urbanos ou de costumes : so
aqueles que narram uma histria que se passa na cidade do Rio de
Janeiro, retratando os costumes da vida na corte.
Exemplos: A Moreninha, Cinco Minutos, A viuvinha, Sonhos d'ouro,
Lucola, Diva, Senhora.
b) Romances regionalistas sertanejos
So aqueles que retratam os costumes, o modo de falar, pensar e
agir do homem interiorano.
Exemplos: Inocncia, O gacho, O sertanejo, Escrava Isaura, O
tronco do ip.
c) Romances regionalistas indianistas: o romance que traz o
indgena como protagonista, como heri.
Exemplos: Iracema, O Guarani, Ubirajara.
d) Romances regionalistas histricos: um romance onde o autos
mistura fico com fatos reais atravs de uma pesquisa histrica do
Brasil.
Exemplos: A guerra dos mascates, As minas de prata
Ele era o inimigo do rei, nas palavras de seu bigrafo, Lira Neto.
Ou, ainda, um romancista que colecionava desafetos, azucrinava D.
Pedro II e acabou inventando o Brasil. Assim era Jos de Alencar
(1829-1877), o conhecido autor de O guarani e Iracema, tido como o
pai do romance no Brasil. Alm de criar clssicos da literatura
brasileira com temas nativistas, indianistas e histricos, ele foi
tambm folhetinista, diretor de jornal, autor de peas de teatro,
advogado, deputado federal e at ministro da Justia. Para ajudar na
descoberta das mltiplas facetas desse personagem do sculo XIX,
parte de seu acervo indito ser digitalizada.
Histria Viva, n. 99, 2011.

EXERCCIO DE FIXAO
01) (ENEM 2012)- Com base no texto, que trata do papel do escritor
Jos de Alencar e da futura digitalizao de sua
obra, depreende-se que
a) a digitalizao dos textos importante para que os leitores
possam compreender seus romances.
b) o conhecido autor de O guarani e Iracema foi importante porque
deixou uma vasta obra literria com temtica atemporal.
c) a divulgao das obras de Jos de Alencar, por meio da
digitalizao, demonstra sua importncia para a histria do
Brasil Imperial.
d) a digitalizao dos textos de Jos de Alencar ter importante
papel na preservao da memria lingustica e da identidade
nacional.
e) o grande romancista Jos de Alencar importante porque se
destacou por sua temtica indianista.

SECRETARIA DE ESTADO DA EDUCAO

135

QUMICA
LITERATURA

AULA 06:A LITERATURA DA ERA NACIONAL E O REALISMO I


contextualizao histrica do Realismo e caractersticas gerais.
I)CONTEXTUALIZAO HISTRICA: a nova fisionomia do
sculo XIX materialismo e cientificismo
Na segunda metade do sculo XIX, a consolidao do
poder burgus na Europa se traduz na implementao acelerada do
sistema capitalista, que esboa a mecanizao do mundo e da vida,
tpica da sociedade moderna. Dentre outros fatores de
modernizao, destacam-se o crescente uso da eletricidade e a
maior eficincia das comunicaes, garantidas por importantes
descobertas, como as mquinas a vapor que sofisticam o
sistema de locomoo, e as novas tcnicas de impresso, que
agilizam a circulao de livros e jornais.
Simultaneamente intensificam-se contradies e tenses
poltico-sociais, pois o poder burgus marginalizava a maior parte da
populao e, em especial, o proletariado urbano, que no tem
acesso aos benefcios do progresso.
A Coluna de Paris, de 1871, tentativa da classe operria
de tomar poder, constitui o ponto culminante de uma srie de
movimentos e crises que se alastram por toda a Europa desde 1860
e colocam em xeque o modelo burgus-capitalista nela
predominante.
Nesse contesto, o grande desenvolvimento cientfico e
filosfico fortemente racionalista, provocado pela UTILIZAO DA
CINCIA como suporte do desenvolvimento econmico-industrial, d
origem conhecida gerao materialista ou gerao de 70. Essa
gerao dominar o cenrio intelectual da segunda metade do sculo
XIX, em todos os campos do conhecimento.
Em consonncia com essas ideias surge, no cenrio
artstico literrio: o Realismo-Naturalismo, ou melhor, dizendo
surgem as Escolas Realistas, que compreendem trs estilos: O
Realismo, o Naturalismo e o Parnasianismo. Esses estilos
apresentam traos especficos que os diferenciam, embora sejam
todos uma reao contra o Romantismo, cujas frmulas se
encontravam envelhecidas.
Essa evoluo da cincia permitiu explicar fenmenos at
ento inexplicveis ou alterou a maneira de explicar outros,
considerados como j desvendados. O CIENTIFICISMO substituiu
progressivamente o subjetivismo romntico, que marcara o incio do
sculo. Atribuiu-se cincia o poder de dar a conhecer as coisas
como elas efetivamente eram e a capacidade de resolver os
problemas da humanidade. Por isso, considerou-se que os mtodos
da cincia deviam ser estendidos a todos os domnios da vida
humana. Esse ESPRITO CIENTFICO gerou novas maneiras de
pensar, de viver, de produzir arte. Podemos destacar como fatores
fundamentais desse perodo:
a publicao da obra A origem das espcies, de Charles
Darwin
na anlise e compreenso da cultura a pequena burguesia
passou a ocupar o primeiro plano no cenrio histrico, em
detrimento da aristocracia e mesmo do clero. Unida de incio ao
proletariado, essa classe social ope-se aristocracia, seguindo
mais tarde rumo diferente.
no campo da economia, o acentuado interesse pelo liberalismo
herana do perodo anterior fazia prever a vitria do
capitalismo industrial e, com ele, a ideia do lucro. Como diz o
historiador Arnold Hauser: O dinheiro a fora que domina
toda a vida pblica e privada e[...] todos os direitos passam
a exprimir-se atravs dele.
os trabalhadores assalariados vivem em condies miserveis e
excludos das vantagens decorrentes do avano cientfico e
industrial.
tentou-se aplicar filosofia os mesmos critrios empregados
nas cincias naturais, e surgiu a corrente de pensamento
denominada positivismo, que sustentava a tese de que a
realidade s poderia ser analisada a partir da observao dos
fatos, rejeitando qualquer explicao metafsica para a origem
do homem e do mundo. Todos os fenmenos se resumiriam,
portanto, ao seu aspecto material. Essa concepo filosfica era
bastante adequada a uma sociedade que valorizava os bens
materiais acima de qualquer coisa. O materialismo outro
trao dominante no perodo.
a REVOLUO INDUSTRIAL muda a face da Europa, gerando
um paradoxo capitalista: desenvolvimento e misria.
novas doutrinas sociais. Marx e a luta de classes

SECRETARIA DE ESTADO DA EDUCAO

136

II)A LITERATURA E OS ESCRITORES DO REALISMO: a denncia


da realidade e a mostragem do comportamento humano
caractersticas gerais.
Cientificismo: objetividade e percepo realista dos fatos
narrados.
Veracidade: despreza a imaginao romntica.
Contemporaneidade: descreve a realidade, para falar sobre o
que est acontecendo de verdade. H, portanto, a reproduo
da realidade observada;
A objetividade no compromisso com a verdade (o autor
imparcial),
Retrato fiel das personagens: carter, aspectos negativos da
natureza humana. Personagens baseadas em indivduos
comuns (no h idealizao da figura humana, tendo suas
condies sociais e culturais das expostas; uma espcie de
anatomia do carter humano.
Gosto pelos detalhes: lentido na narrativa.
Materialismo do amor: a mulher objeto de prazer/adultrio.
Denncia das injustias sociais: mostra para todos a
realidade dos fatos.
Determinismo e relao entre causa e efeito: o realista
procurava uma explicao lgica para as atitudes das
personagens, considerando a soma de fatores que justificasse
suas aes. Na literatura naturalista, dava-se nfase ao instinto,
ao meio ambiente e hereditariedade como foras
determinantes do comportamento dos indivduos. a chamada
lei da causalidade (toda ao tem uma reao);
Linguagem prxima realidade: simples, natural, clara e
equilibrada.
Surgimento do romance de tese: os autores do Realismo
buscam criar histrias mais prximas da analise cientificista, da
os romances adquirirem um carter de tese, de libelo contra os
fatos sociais.
EXERCCIO DE FIXAO
01) Leia o fragmento com ateno.
[...] Jamais o problema da vida e da morte me oprimira o
crebro; nunca at esse dia me debruara sobre o abismo do
Inexplicvel; faltava-me o essencial, que o estmulo, a vertigem...
Para lhes dizer a verdade toda, eu refletia as opinies de um
cabeleireiro, que achei em Mdena, e que se distinguia por no as
ter absolutamente. Era a flor dos cabeleireiros; por mais demorada
que fosse a operao do toucado, no enfadava nunca; ele
intercalava as penteadelas com muitos motes e pulhas, cheios de um
pico, de um sabor... No tinha outra filosofia. Nem eu. No digo que
a Universidade me no tivesse ensinado alguma; mas eu decorei-lhe
s as frmulas, o vocabulrio, o esqueleto. Tratei-a como tratei o
latim: embolsei trs versos de Virglio, dois de Horcio, uma dzia de
locues morais e polticas, para as despesas da conversao.
Tratei-os como tratei a histria e a jurisprudncia. Colhi de todas as
coisas a fraseologia, a casca, a ornamentao...
ASSIS, Machado de. Memrias pstumas de Brs Cubas. 22. ed. So
Paulo: tica, 1997, p. 55.

O narrador-personagem:
a) revela-se vaidoso ao evidenciar os seus dotes intelectuais.
b) ressalta o papel decisivo da universidade na sua formao
cultural.
c) pe em destaque a superficialidade e mediania de sua postura em
face do conhecimento.
d) mostra-se preocupado com questes que problematizam a vida
humana.
e) confessa o seu desejo de libertar-se das limitaes da vida
acadmica.

QUMICA

LITERATURA

AULA 07: A LITERATURA DA ERA NACIONAL E O REALISMO II


a diferena entre a prosa realista e naturalista.
O Naturalismo, assim como o Realismo, volta-se para o retrato
objetivo da realidade. Entretanto, observa-a, documenta-a, analisaa, disseca-a sob uma tica rigorosamente cientfica. Os escritores
naturalistas, valendo-se de temas inovadores, mostram a
decadncia das instituies, denunciam a hipocrisia social,
falam da fragilidade do indivduo perante as foras da
hereditariedade e do meio e vem as lutas sociais com simpatia.
Nas ltimas dcadas do sculo XIX, como consequncia
do esprito racionalista que se disseminava pelo mundo ocidental e
dos investimentos da burguesia no campo da cincia e da tecnologia,
surgiram novas concepes a respeito do homem, da vida em
sociedade, das relaes trabalho, etc. Sistematizavam-se estudos
da Biologia, da Psicologia e da Sociologia, que punham em xeque
a cultura impregnada de idealismo e religiosidade crist ainda
dominante. Os escritores naturalistas, sintonizados com o
esprito cientfico orientado pela concepo de que o mundo pode
ser estudado em partes e explicado a partir de leis objetivas,
assumiram uma nova postura diante do trabalho artstico. Tal qual
estivessem em laboratrios, sua misso passou a ser documentar,
dissecar e analisar o comportamento humano e social, quase sempre
apontando sadas e solues da o carter reformista dessa
literatura.
No Brasil, contudo, com raras excees, em vez de se
dedicarem ao estudo de grupos humanos, detiveram-se em casos
individuais, ocupando-se com temperamentos patolgicos. Desse
modo, fatos histricos e sociais de grande relevncia como a
Abolio, a Repblica, a Revolta da Armada e o Encilhamento foram
subestimados, dando lugar a pequenos casos que no so
representativos da realidade brasileira.
A LINGUAGEM DA PROSA NATURALISTA
De acordo com a concepo naturalista, o homem no
passa de um animal cujo destino determinado pelo meio social e
pela hereditariedade, desprovido de livre-arbtrio e sempre merc
de foras que fogem ao seu controle.
A literatura naturalista constri sua fico sob o regime
das leis cientficas: o homem um caso, um objeto a ser
cientificamente estudado. O romancista assume uma atitude de
no-envolvimento com as personagens, pois ele um observador,
que deve registrar a realidade impessoalmente, com preciso e
objetividade cientfica. A linguagem naturalista caracteriza-se ainda
pela exatido das descries, pelo apelo mincia e pela linguagem
simples, coloquial.
O Naturalismo introduziu na literatura todos os assuntos
ligados ao homem, inclusive aqueles repulsivos e bestiais, e deu
voz s camadas desfavorecidas da sociedade.
MINAS DE CARVO FRANCESAS X PEDREIRA CARIOCA
Germinal, o romance de mile Zola que deu origem ao
Naturalismo, um retrato comovente das condies subumanas em
que viviam os trabalhadores das minas de carvo na Frana, no
sculo XIX. Morando e se alimentando de forma precria e tendo
contato com as idias socialistas que ento circulavam pela Europa,
os trabalhadores desafiam a prepotncia da burguesia e fazem uma
greve. O movimento reprimido e neutralizado, mas fica viva a idia
de luta e conquista social.
Brasil, Alusio Azevedo mostra em O cortio uma situao
at certo ponto semelhante por Zola: os trabalhadores da pedreira
de Joo Romo vivem precariamente num quarto alugado do cortio
do patro e gastam o resto do dinheiro na venda do prprio Joo
Romo, fechando o crculo de quase escravido. As idias polticas,
nessa obra, transformam-se em crtica burguesia e em referncias
vagas aos movimentos da Repblica e da Abolio.
CARACTERSTICAS DA LINGUAGEM NATURALISTA
Alm das caractersticas da linguagem naturalista
observadas, outras merecem destaque. Segundo Afrnio Coutinho, o
Naturalismo amplia as caractersticas do Realismo, acentuando-se
acrescentando a elas certos elementos que tornam inconfundvel sua
fisionomia em relao do Realismo. So estes os elementos:

SECRETARIA DE ESTADO DA EDUCAO

137

Determinismo:
Para os naturalistas, o homem e uma mquina guiada pela
ao de leis fsicas e qumicas, pela hereditariedade e pelo meio fsico e
social. Os seres aparecem, ento, como produtos, como consequncias
de foras preexistentes que lhes roubam o livre-arbtrio, que limitam sua
responsabilidade e os tornam, em casos extremos, verdadeiros joguetes
nas mos do destino.
No seria filho de Pedro Ribeiro de Morais, o devasso
fazendeiro do Igarap-Mirim se o seu crebro no fosse dominado por
instintos egosticos, que a privao dos prazeres aulava e que uma
educao superficial no soubera subjugar. E como os senhores padres
do Seminrio haviam pretendido destruir ou, ao menos, regular e conter a
ao dominante da hereditariedade psicofisiolgica sobre o crebro do
seminarista?
Ingls
de
Sousa,
O
missionrio
Preferncia por temas de patologia social:
Baseando-se nos problemas da hereditariedade (ainda pouco
conhecidos na poca), os escritores naturalistas no hesitaram em
ressaltar o efeito das taras, das doenas e dos vcios na formao do
carter, juntando-lhes ainda os efeitos complementares da formao
familiar, da educao e do nvel cultural. Interessam-se por temas como
misria, adultrio, criminalidade, desequilbrio psquico, problemas ligados
ao sexo, etc. Observe:
Ele ali se achava tambm, no seu posto, espera de um sinal
para descarregar a chibata, implacavelmente sobre a vtima. Sentia um
prazer especial naquilo, que diabo! Cada qual tem a sua mania...
Adolfo Caminha, O bom
crioulo
Objetivismo cientfico e impessoalidade:
Das teorias cientficas em voga, amplamente divulgadas na
poca, o Naturalismo, toma-lhes a preciso e a objetividade, descrevendo
com impessoalidade, exatido e mincia, e atendo-se somente aos fatos.
Literatura engajada:
O Naturalismo implica uma posio de combate, de anlise dos
problemas evidenciados pela decadncia social, fazendo da obra de arte
uma verdadeira tese com inteno cientfica. Apresenta a mesma
inteno realista de reformar a sociedade.

EXERCCIO DE FIXAO
(ENEM 2005)Entre a morte do Quincas Borba e a minha,
mediram os sucessos narrados na primeira parte do livro. O principal
deles foi a inveno do emplasto Brs Cubas, que morreu comigo,
por causa da molstia que apanhei. Divino emplasto, tu me darias o
primeiro lugar entre s homens, acima da cincia e da riqueza, porque
eras a genuna e direta inspirao do cu. O acaso determinou o
contrrio; e a vos ficais eternamente hipocondracos. Este ltimo
captulo todo de negativas. No alcancei a celebridade do
emplasto, no fui ministro, no fui califa, no conheci o casamento.
Verdade que, ao lado dessas faltas, coube-me a boa fortuna de
no comprar o po com o suor do meu rosto. Mais; no padeci a
morte de Dona Plcida, nem a semidemncia do Quincas Borba.
Somadas umas coisas e outras, qualquer pessoa imaginar que no
houve mngua nem sobra, e conseguintemente que sa quite com a
vida. E imaginar mal; porque ao chegar a este outro lado do
mistrio, achei-me com um pequeno saldo, que a derradeira
negativa deste captulo de negativas: - No tive filhos, no transmiti a
nenhuma criatura o legado da nossa misria.

01)

Machado de Assis, in Memrias Pstumas de Brs Cubas.

Neste captulo, o narrador apresenta ao leitor um balano de sua


vida. Pela leitura do texto, NO correto afirmar que:
a) O narrador chama a ateno para os aspectos negativos do
homem e da vida, a face m da natureza humana, onde a bondade e
a grandeza so atitudes relativas.
b) O narrador manifesta sua viso pessimista da vida, diante da qual
o homem aparece como um ser sem sada diante das foras que
comandam seu destino.
c) A leitura do captulo leva-nos a concluir que o ambiente em que
viveu Brs Cubas foi o das elites dos Oitocentos caracterizado pelo
cio e pela hipocrisia.
d) A viso pessimista do mundo que o narrador apresenta no texto
provm, principalmente, do fato de no ter gerado filhos, o que faz
dele um grande perdedor no jogo da vida.
e) Essa viso amarga e pessimista mostrada pelo narrador no trecho
acima demonstra a maneira de narrar de Machado de Assis, em que
a metalinguagem se junta a uma viso negativa e depurada da
natureza humana e da sociedade.

QUMICA
LITERATURA

AULA 08: A LITERATURA DA ERA NACIONAL A POESIA


PARNASIANA: linguagem e temticas.
Valorizando o emprego da palavra rara, do vocabulrio
precioso, da frase rebuscada, a poesia parnasiana teve, na
preocupao com a perfeio da forma, a sua caracterstica bsica,
ainda que em prejuzo da qualidade de sua expresso potica.
O estilo se define, portanto, pelo culto da forma e foi,
sobretudo uma renovao potica. Esta renovao teve sua origem
na Frana. Em l886, foi editada uma antologia, Le Parnasse
Contemporin, que reunia composies de diversas tendncias, com
uma linha comum: reagir contra o romantismo. Seus principais
colaboradores, Leconte de Lisle, Thephile Gautier, Thodore
Banville, Jos Maria Herdia (de nacionalidade cubana), Baudelaire,
Sully Prudhomme (ganhador do prmio Nobel em 1901), Verlaine,
Mallarm, obedeciam a uma nova esttica que pregava o princpio da
Arte pela Arte. Defendiam em ltima anlise, uma arte que no
servisse a nada, nem a difuso qualquer ideologia, nem a ningum;
uma arte voltada para si mesma em suma. O objetivo da arte pela
arte o Belo, a criao da beleza pelo uso perfeito dos recursos
artsticos. Neste sentido, levam ao exagero o culto da rima, do ritmo,
do vocabulrio, do verso longo. Para o Parnasiano, a poesia deveria
ser trabalhada at que resultasse perfeita.
Caracteriza-se pela sacralidade da forma, pelo respeito
s regras de versificao, pelo preciosismo rtmico e vocabular,
pela rima rica e pela preferncia por estruturas fixas, como os
sonetos. O emprego da linguagem figurada reduzido, com a
valorizao do exotismo e da mitologia. Os temas preferidos so os
fatos histricos, objetos e paisagens. A descrio visual o forte
da poesia parnasiana, assim como para os romnticos so a
sonoridade das palavras e dos versos. Os autores parnasianos
faziam uma "arte pela arte", pois acreditavam que a arte devia existir
por si s, e no por subterfgios, como o amor, por exemplo. A
seguir, selecionamos um comentrio sobre as propostas estticas e
lingusticas dessa corrente literria no Brasil.
"A necessidade de objetivar ou despersonalizar a poesia
tomou vulto em Frana nos meados do sc. XIX. sempre algo
fictcia a tarefa de rotular poetas e delimitar o mbito de uma escola
literria, at porque as revolues do pensamento e do gosto
germinam j em pocas anteriores quela em que se declaram. Foi a
saturao das indecorosas carpiduras romnticas, o pudor do
egolatrismo, que, at certo ponto, determinou o movimento
parnasiano, pois se afirmara uma reao antirromntica , na rejeio
da confidncia, na transposio dramtica da experincia ntima; e
Vtor Hugo tentara operar a transio do individual para o geral. Com
toda a complexidade que os seus mltiplos expoentes lhe trazem, o
Parnasianismo francs, que ao mundo ditou os moldes de uma nova
esttica, concentra-se, como teoria, em torno do ideal da Arte pela
Arte, renovado programaticamente (pois se trata de uma posio
eterna do esprito) por volta de 1830. Prende-se esta atitude com o
repdio da tendncia para tornar a arte til, para colocar ao servio
da sociedade.
O nome de parnasianos foi dado ao grupo de poetas
cujos versos o editor Lemerre publicou numa colectnea - sucedneo
da revista Art - intitulada Parnasse Contemporain (1866-1871-1876).
O elo entre esses poetas (e alguns prosadores) de diferentes origens
e com diferentes propsitos, era o respeito pela arte e pelo ofcio,
pelo artifcio. [...]
Rodrigues, Urbano Tavares, DICIONRIO DE LITERATURA, 3 edio, 3
volume, Porto, Figueirinhas, 1979

CARACTERSTICAS DO PARNASIANISMO
Objetividade e descritivismo
Reagindo contra o sentimentalismo e o subjetivismo
romnticos, a poesia parnasiana era comedida, objetiva: fugia das
manifestaes sentimentais. Buscando esta impassibilidade ( frieza),
empenhava-se em descrever mincias, na fixao de cenas,
personagens histricos e figuras mitolgicas.
Rigor formal
Opondo-se simplicidade formal romntica, que de certa
forma popularizou a poesia os parnasianos eram rigorosos quanto
mtrica em rimas e tambm quanto riqueza e raridade do
vocabulrio. por isso que so frequentes, nos textos parnasianos,
os hiprbatos( ordem indireta), as palavras eruditas e difceis, as
rimas foradas.

SECRETARIA DE ESTADO DA EDUCAO

138

Retorno ao Classicismo
Abordando temas mitolgicos e da antigidade grecolatina, os poetas parnasianos valorizavam as normas e tcnicas de
composio e, regra geral, exploravam o soneto (poema de forma
fixa).
Arte pela arte
A poesia parnasiana atribua a arte o papel de voltar-se somente
para o ato de sua prpria criao. Assim, o poeta deveria se
preocupar exclusivamente com o fazer potico
Alheiamento a problemas sociais
Os parnasianos, preocupados que estavam em valorizar a
forma potica, deixavam de lado tudo aquilo que no fosse
relacionado ao fazer potico. Da, sua poesia se estender somente
para a preocupao com a forma, ficando completamente desprovida
de preocupaes sociais, polticas o quaisquer outras que no
interesse poesia.
Outras caractersticas:
- culto da forma
- forte descritivismo
- necessidade de aproximar a poesia das artes plsticas, em especial
a escultura
- valorizao de versos longos.
- sensualismo
- conteno emocional, atravs da impassibilidade
- comparao da poesia com as artes plsticas
- descrio de objetos exticos
- seleo vocabular e valorizao do vernculo
- arte pela arte
- alheamento a problemas sociais
- esvaziamento do contedo
- volta ao passado histrico
EXERCCIOS DE FIXAO
Texto para as questes 01 E 02
Remorso
s vezes, uma dor me desespera. . .
Nestas nsias e dvidas em que ando,
Cismo e padeo, neste outono, quando
Calculo o que perdi na primavera.
Versos e amores sufoquei calando,
Sem os gozar numa exploso sincera. . .
Ah! mais cem vidas! com que ardor quisera
Mais viver, mais penar e amar cantando!
Sinto o que esperdicei na juventude;
Choro, neste comeo de velhice,
Mrtir da hipocrisia ou da virtude,
Os beijos que no tive por tolice,
Por timidez o que sofrer no pude,
E por pudor os versos que no disse!
BILAC, Olavo. Melhores poemas de Olavo Bilac /Seleo de Marisa Lajolo.
4 ed. So Paulo: Global, 2003, p.106. (Melhores poemas: 16)

01) No poema, o eu lrico


a) revela a frustrao de no ter vivido plenamente a sua
mocidade.
b) sente-se infeliz por ter chegado velhice.
c) considera-se mrtir da hipocrisia, por ter revelado sentimentos
no verdadeiros.
d) mostra-se fracassado por no ter amado ningum.
e) lamenta no ter sido um poeta, por lhe faltar inspirao.
02) Considerando os recursos estilsticos, correto afirmar que
esse poema
a) enquadra-se tipicamente no estilo parnasiano, apresentando
uma descrio objetiva da realidade vivida pelo eu lrico.
b) aproxima-se da esttica romntica, uma vez que o eu lrico faz
confisses de seu estado de alma.
c) afasta-se radicalmente do estilo romntico, traduzindo uma
preocupao excessiva com o rigor formal.
d) classifica-se como simbolista, por apresentar preocupaes
espirituais e religiosas do eu lrico diante da vida.
e) traduz uma caracterstica da poesia simbolista, voltada para a
supervalorizao da forma em detrimento do contedo do texto.

QUMICA
LITERATURA

AULA 09: A LITERATURA DA ERA NACIONAL A POESIA


SIMBOLISTA: contexto histrico e caractersticas da linguagem
e da temtica
1. A VIRADA DO SCULO E A GRANDE DEPRESSO
O SIMBOLISMO, movimento essencialmente potico, do
fim do sculo XIX, representa uma ruptura artstica radical com a
mentalidade
cultural
do
Realismo-Naturalismo,
buscando
fundamentalmente retomar o primado das dimenses no racionais
da existncia.
Reagindo contra o pensamento cientfico e filosfico
dominante na segunda metade do sculo XIX, as manifestaes
artsticas do Simbolismo pe em xeque as certezas doutrinrias do
positivismo e do determinismo, que j comeam a ruir.
De fato, nas duas ltimas dcadas do sculo XIX, j
percebemos em boa parte dos autores do Realismo e at mesmo
dos parnasianos, como o caso de Alberto de Oliveira, uma postura
de desiluso, e mesmo de frustrao, em consequncia das
infrutferas tentativas de transformar a sociedade burguesa industrial.
O crtico Alfredo Bossi sintetizou inteligentemente esse perodo:
Do mago da inteligncia europeia surge uma oposio
vigorosa ao triunfo da coisa e do fato sobre o sujeito aquele sujeito
a quem o otimismo do sculo prometera o paraso mas no dera
seno um purgatrio de contrastes e frustraes.
(Histria concisa da Literatura Brasileira)

Numa viso mais ampla, tanto no campo da filosofia e das


cincias da natureza quanto no campo das cincias humanas, a
desconstruo das teorias racionalistas faz-se notar, seja por meio
da fsica relativista de Einstein, da psicologia do inconsciente de
Freud, ou das teorias filosficas de Schopenhauer e de Friedrich
Nietzsche.
No campo filosfico, Henry Bergson prope a busca do eu
profundo, ou seja, das realidades interiores, verdadeira essncia
do ser humano. Este eu-profundo, porm, no pode ser explicado,
traduzido em palavras, apenas evocado e sugerido.
Em termos sociais, polticos e econmicos, a burguesia
industrial, aps algum tempo de progresso avassalador, desgasta-se
com as disputas colonialistas, que evoluem em direo Primeira
Guerra Mundial (1914-1918), quando atingem um momento de
culminncia.
O Simbolismo vai refletir um momento bastante complexo
da histria europeia, que marcaria a transio para o sculo XX e a
definio de um novo mundo, o qual se consolidaria a partir da
segunda metade deste sculo.
As correntes materialistas e racionalistas que dominaram
na metade do sculo XIX no mais respondiam s exigncias de
uma nova realidade, j que o processo burgus industrial evolua a
passos largos, gerando, inclusive, a luta das grandes potncias pelos
mercados consumidores e fornecedores de matria prima. Por esses
motivos, fragmenta-se a frica e ampliam-se as influncias sobres os
territrios asiticos. Tomava corpo, assim, o fantasma da guerra.
Sempre que se torna difcil analisar o mundo exterior e entend-lo
racionalmente, a tendncia natural neg-lo, voltando para a
realidade subjetiva; as tendncias espiritualistas renascem; o
subconsciente e o inconsciente so valorizados, segundo a lio
freudiana.
O Simbolismo nasceu na Frana e teve como os mais
autnticos representantes Mallarm (o gosto pelo obscurantismo,
pelo hermetismo, por insinuar e sugerir) e Verlaine (teoria da
musicalidade do verso, associando poesia e msica). Esses poetas
abandonam os princpios da escola realista e tambm parnasiana e
dedicam-se ao culto do etreo, do subjetivismo, do vago e do
sugestivo, para eles, a palavra deve antes de mais nada sugerir e
no determinar.
No Brasil, o Simbolismo comea em 1893 com a
publicao de duas importantes obras: Missal (prosa) e Broquis
(poesia), ambos de Cruz e Sousa. Estende-se at o ano de 1922,
data da Semana de Arte Moderna. Entre ns, o movimento simbolista
no conseguiu suplantar o perodo parnasiano, j que no teve tanta
influncia esse movimento aqui no Brasil. O Simbolismo, enquanto
esttica de arte, somente repercutiu em pases que sentiam na pele
o FANTASMA DA GUERRA, que no era o caso do Brasil. Se no
fosse CRUZ E SOUSA, esse movimento nem teria acontecido entre
ns.
interessante notar que as origens do Simbolismo em
territrio brasileiro se encontram numa regio marginalizada pela
elite cultural e poltica de ento: O sul, que mais sofreria com a

SECRETARIA DE ESTADO DA EDUCAO

139

oposio jovem Repblica vestida com a farda do exrcito. O Rio


Grande do Sul foi palco de intensas lutas iniciadas em 1893. A
Revoluo Federalista (1893/1895), a princpio uma disputa regional,
logo ganha dimenso nacional ao opor-se ao governo de Floriano
Peixoto, gerando cenas de extrema violncia e crueldade no Rio
Grande do Sul, Santa Catarina e Paran; ao mesmo tempo temos a
Revolta da Armada (1893/1894), com os navios da Marinha
mobilizados na baa da Guanabara, apontando seus canhes para o
palcio do governo e exigindo a renncia de Floriano.

SINTETIZANDO AS CARACTERSTICAS

sugere em vez de descrever; simboliza em vez de nomear


redescobre a subjetividade, o sentimento, a imaginao, a
espiritualidade
explora o subconsciente e o inconsciente
predominncia de imagens sensoriais e metafsicas
priorizao da musicalidade, com aliteraes, assonncias e
onomatopeias
prefere as sinestesias, as metforas, prosopopeias e analogias
como figuras de linguagem
utilizao de maisculas em substantivos comuns, para tornlos absolutos
religiosidade no convencional; pelo desregramento dos
sentidos, da sexualidade espiritual, emoes de delrios e
alucinaes
busca do misterioso, o oculto, o vago, o catico, o indefinvel e o
inexprimvel
temas transcendentalistas
sublimao espiritual
linguagem evocativa subjetivismo aprofundado
EXERCCIO DE FIXAO

(ENEM 2014)
Vida obscura
Ningum sentiu o teu espasmo obscuro,
ser humilde entre os humildes seres,
embriagado, tonto de prazeres,
o mundo para ti foi negro e duro.
Atravessaste no silncio escuro
a vida presa a trgicos deveres
e chegaste ao saber de altos saberes
tornando-te mais simples e mais puro.
Ningum te viu o sentimento inquieto,
magoado, oculto e aterrador, secreto,
que o corao te apunhalou no mundo,
Mas eu que sempre te segui os passos
sei que cruz infernal prendeu-te os braos
e o teu suspiro como foi profundo!
SOUSA, C. Obra completa. Rio de Janeiro: Nova Aguilar, 1961.

Com uma obra densa e expressiva no Simbolismo brasileiro, Cruz e


Sousa transps para seu lirismo uma sensibilidade em conflito com a
realidade vivenciada. No soneto, essa percepo traduz-se em
a) sofrimento tcito diante dos limites impostos pela discriminao.
b) tendncia latente ao vcio como resposta ao isolamento social.
c) extenuao condicionada a uma rotina de tarefas degradantes.
d) frustrao amorosa canalizada para as atividades intelectuais.
e) vocao religiosa manifesta na aproximao com a f crist.

QUMICA
LITERATURA

AULA 10: A LITERATURA DA ERA MODERNA PrModernismo, a realidade do Brasil republicano, conflitos e
contrastes.
1.
A VIRADA PARA O SCULO XX: O PR-MODERNISMO
E A REDESCOBERTA DO BRASIL o regionalismo, a denncia
social e a poetizao do hediondo
O avano cientfico e tecnolgico no incio do sculo XX traz novas
perspectivas humanidade. As invenes contribuem para um clima
de conforto e praticidade. Afinal, o telefone, a lmpada eltrica, o
automvel e o telgrafo comeam a influenciar, definitivamente, a
vida das pessoas. Alm dessas, a arte mostrou um inovado meio de
comunicao, diverso e entretenimento: o cinema.
em meio a tanto progresso que a 1 Guerra Mundial eclode. Em
meio a tantos acontecimentos, havia muito que se dizer, e por isso,
a literatura vasta nos primeiros anos do sculo XX. Logo, os estilos
literrios vo desde os poetas parnasianos e simbolistas (que ainda
produziam) at os que se concentravam na poltica e nas
peculiaridades de sua regio.
Chamamos de Pr-Modernismo a essa fase de transio literria
entre as escolas anteriores e a ruptura dos novos escritores com as
mesmas. Enquanto a Europa preparava-se para a guerra, o Brasil
vivia a chamada poltica do caf-com-leite, onde os grandes
latifundirios do caf dominavam a economia. Ao passo que esta
classe dominante e consumista seguia a moda europeia, as
agitaes sociais aconteciam, principalmente no Nordeste.
Na Bahia, ocorre a famosa Revolta de Canudos, que inspirava a
obra Os Sertes do escritor Euclides da Cunha. Em 1910, a
rebelio Revolta da chibata era liderada por Joo Cndido, o
Almirante Negro, contra os maltratos vividos na Marinha.
Aos poucos, a Repblica caf-com-leite ficava em crise e em 1920
comeam os burburinhos da Semana de Arte Moderna, que
marcaria o incio do Modernismo no Brasil. Os principais escritores
pr-modernistas so: Euclides da Cunha, Lima Barreto, Monteiro
Lobato, Graa Aranha e Augusto dos Anjos.
Os marcos literrios so, especialmente, o j citado Os Sertes,
de Euclides da Cunha, e Cana, de Graa Aranha. O PrModernismo no chega a ser considerado uma escola literria,
pois no h um grupo de escritores que seguem a mesma linha
temtica ou os mesmos traos literrios.
CONTEXTO HISTRICO E SOCIAL DO PR-MODERNISMO
O final do sculo XIX e o incio do sculo XX foi marcado
pela ascenso do regime republicano e por diversas manifestaes
populares que se opunham ao sistema poltico, bem como
combatiam as injustias sociais, que foram deixadas de lado pelas
oligarquias cafeeiras que controlavam a mquina eleitoral. Nesse
contexto marcado por modificaes nas cidades e, ao mesmo tempo,
abatidas por crises sociais que se estenderam tambm ao campo,
resqucios de um processo de abolio tardio e mal executado,
nasce o Pr-Modernismo. Um perodo transitrio na produo
literria, no qual alguns autores, ainda no modernos, buscavam
inovar e romper com o passado. Nesse momento efervescente de
ideias, com estilos variados, destacam-se alguns autores que,
embora com individualidades marcantes, mantm alguns pontos em
comum com relao s obras. Os Sertes e Cana, publicados em
1902, marcam o incio do perodo pr-modernista.
SNTESE DOS FATOS HISTRICOS
Uma srie de revoltas eclodiu em todo o pas. Os motivos eram
diversos, as situaes eram bem diferentes, as reivindicaes
eram vrias: a transio da Repblica da Espada para a
Repblica do caf-com-leite (predominantemente civil);
enfraquecimento da Repblica Velha (1889-1930 ).
Em 1893, tem lugar no Rio de Janeiro a Revolta da Armada,
levante de uma faco monarquista da Marinha brasileira, que,
insatisfeita, com a Repblica recm-proclamada (1889), exigia a
renncia do presidente Floriano Peixoto. O restante das Foras
Armadas, contudo, colocou-se ao lado do governante, que
contava anda com forte apoio civil, graas sua imensa
popularidade. A revolta foi sufocada aps seis meses.
Em 1896 , estourou na Bahia a revolta de sertanejos que ficou
conhecida como Guerra de Canudos. As causas mais
profundas da Guerra eram outras: a misria, o
subdesenvolvimento, a opresso, o abandono a que a
populao da regio estava relegada. Depois de um ano de
renhida resistncia, Canudos caiu, mas os problemas sociais
no foram resolvidos, e o misticismo e o cangao continuaram
sendo respostas populares opresso e misria.

SECRETARIA DE ESTADO DA EDUCAO

140

A urbanizao do RJ, propondo um processo de modernizao


da cidade que era a capital do pas, muito populosa, e que
contava com pssimas condies de higiene. O novo
planejamento urbano, contudo, previa uma recolocao da
moradia dos mais pobres, excluindo-os das benesses da
modernizao e desamparando grande parte da populao, j
massacrada pelo desemprego e pela carestia. A insatisfao
popular explodiu quando o governo lanou a campanha de
vacinao obrigatria. A verdadeira batalha que se travou no
Rio de Janeiro, em 1904, opondo policiais aos pobres, recebeu
o nome de Revolta da Vacina, tendo sido violentamente
reprimida.
Em 1910, os marinheiros de dois navios de guerra, liderados
por Joo Cndido (o "Almirante Negro"), manifestaram-se
contrrios aos castigos corporais ainda costumeiramente
aplicados a eles na Marinha: era a Revolta da Chibata. O
prprio governo reconheceu a pertinncia das reivindicaes,
pressionando a Marinha para que as atendesse. Os revoltosos
foram anistiados, mas logo depois perseguidos, presos e
torturados.
A greve geral de 1917, ocorrida em So Paulo, organizada por
trabalhadores anarco-sindicalista, reivindicando melhoria nos
salrios e nas condies de trabalho (reduo da jornada,
segurana, etc.).
CARACTERSTICAS GERAIS DO PR-MODERNISMO: o ataque
ao academicismo e os autores em busca de um pas.

Retomada da literatura social, a partir de uma redescoberta do Brasil


pela literatura; um Brasil que, na verdade, sempre tinha existido,
mas que fora at ento presena excessivamente reduzida na
literatura.

Renovao temtica que se opera no mbito literrio, mostrando a


preocupao com a realidade nacional atravs da crtica s
instituies, tomadas como elementos de cristalizao e
acomodao de uma estrutura de poder que resultava na cegueira
s reivindicaes de vastas camadas da populao brasileira.

A focalizao do homem brasileiro, atravs da mostragem dos


grupos humanos marginalizados das diversas regies do pas: o
serto, o interior, os subrbios, que j apareciam antes em
romances, porm de forma a focalizar uma comunho com a
natureza; agora so retratados como representaes do atraso
brasileiro. Da mesma forma, os personagens que figuraram nessas
produes esto muito distantes dos modelos assumidos em
estticas anteriores: o sertanejo no mais servil e ordeiro; o
suburbano no mais alegre e expansivo; o caipira no mais
saudvel e trabalhador. A imagem que essas personagens passam
a representar liga-as decadncia, ao desmazelo, ignorncia.

Surgimento de um novo regionalismo, dessa vez, bastante diferente,


por exemplo, da idealizao romntica.

A literatura como misso.

A ruptura com o passado, com o academicismo, atravs da defesa


de um estilo prprio, da linguagem coloquial e desleixada e do
questionamento do que seja boa poesia

Formalmente, os pr-modernistas caracterizar-se-o por uma


linguagem
oscilante:
expresses
eruditas,
francamente
influenciadas pelo Parnasianismo, convivem com um
vocabulrio mais livre e popular, que tenta funcionar como
registro de expresses regionais. Pode-se entender essa prtica
como uma ponte de ligao entre a linguagem dicionaresca e
cientificista do sculo XIX e aquela, mais despojada, que
caracterizaria a arte moderna do sculo XX. De certa forma, essa
oscilao reflete ainda um contato crescente entre as culturas
popular e erudita.

Poetizao da linguagem cientfica, representada pelo poeta


Augusto dos Anjos. SUA POESIA ATACA VEEMENTEMENTE O
ARISTOCRACISMO PARNASIANO, AO QUESTIONAR O
CONCEITO DE BOA POESIA. Materialismo, viso pessimista da
vida, vocabulrio agressivo e de fortes marcas cientificistas;
utilizao de termos considerados no poticos, como escarro,
cuspe, menstruao, sangue podre, casca de ovo, bicharia,
dentre outros.
A prtica literria desse perodo no chegou a se constituir em escola,
estruturada e organizada em um programa esttico definido. Assim, o
Pr-Modernismo , mais que um fato artstico, um momento importante
do desenvolvimento das letras brasileiras. Seus autores mais
significativos so: Euclides da Cunha, Lima Barreto, Monteiro Lobato,
Graa Aranha, Augusto dos Anjos, entre outros

QUMICA
LITERATURA

AULA 11: A LITERATURA DA ERA MODERNA


Modernismo: a contribuio dos autores.

- O Pr-

EUCLIDES DA CUNHA: Os sertes, a bblia da


nacionalidade brasileira
Nasceu em Cantagalo (RJ), no dia 20 de janeiro de 1866.
Foi escritor, professor, socilogo, reprter jornalstico e engenheiro,
tendo se tornado famoso internacionalmente por sua obra-prima, Os
Sertes, que retrata a Guerra dos Canudos. Sua obra mais
conhecida OS SERTES, onde ele narra a guerra de Canudos.
Sua narrativa apresenta estilo literrio, de fundo histrico (apesar do
fato recente) e de rigor cientfico. Seu modelo determinista,
segundo o qual o meio determina o homem. A obra Os sertes
Trata da Guerra de Canudos (1896-1897), no interior da Bahia.
Euclides da Cunha presenciou uma parte desta guerra como
correspondente do jornal O Estado de S. Paulo, e ao retornar
escreveu um dos maiores livros j escritos por um brasileiro.
Pertence, ao mesmo tempo, prosa cientfica e prosa artstica.
Pode ser entendido como um obra de Sociologia, Geografia,
Histria ou crtica humana. Mas no errado l-lo como uma
epopeia da vida sertaneja em sua luta diria contra a paisagem e
a incompreenso das elites governamentais. Contedo/partes do
livro
O livro OS SERTES divide-se em trs partes: A TERRA,
O HOMEM E A LUTA.
A terra: Na primeira parte so estudados o relevo, o solo,
a fauna, a flora e o clima da regio nordestina. Euclides da Cunha
revelou que nada supera a principal calamidade do serto: a seca.
Registrou, ainda, que as grandes secas do Nordeste brasileiro
obedecem a um ciclo de nove a doze anos, desde o sculo XVIII,
numa ordem cabalstica.
O homem: O determinismo julgava que o homem
produto do meio (geografia), da raa (hereditariedade) e do momento
histrico (cultura). O autor faz uma anlise da psicologia do sertanejo
e de seus costumes.
A luta: Fala sobre o que foi a Guerra de Canudos e explica
com riqueza de detalhes os fatos dessa guerra que dizimou a
populao de Canudos.
1.

LIMA BARRETO: a vida nos subrbios cariocas a


denncia do preconceito e o nacionalismo quixotesco.
Afonso Henriques de Lima Barreto (Rio de Janeiro, 13
de maio de 1881 - So Paulo, 1 de novembro de 1922), melhor
conhecido como Lima Barreto, foi um jornalista e um dos mais
importantes escritores libertrios[1] brasileiros. Lima Barreto foi o
crtico mais agudo da poca da Repblica Velha no Brasil, rompendo
com o nacionalismo ufanista e pondo a nu a roupagem da Repblica,
que manteve os privilgios de famlias aristocrticas e dos militares.
Em sua obra, de temtica social, privilegiou os pobres, os
bomios e os arruinados. Foi severamente criticado pelos seus
contemporneos parnasianos por seu estilo despojado, fluente e
coloquial, que acabou influenciando os escritores modernistas. um
escritor de transio: fiel ao modelo do romance realista e naturalista
do final do sculo XIX, procurou, entretanto desenvolv-lo,
resgatando as tradies cmicas, carnavalescas e picarescas da
cultura popular, ao mesmo tempo em que manteve "uma viso
neo-romntica e elegaca da natureza, da cidade e do ser
humano. Tambm queria que a sua literatura fosse militante.
Escrever tinha finalidade de criticar o mundo circundante para
despertar alternativas renovadoras dos costumes e de prticas que,
na sociedade, privilegiavam pessoas e grupos. Para ele, o escritor
tinha uma funo social.
2.

3.

MONTEIRO LOBATO: a decadncia do caf e as cidades


mortas do interior paulista

Jos Bento Renato Monteiro Lobato (Taubat, 18 de


abril de 1882 So Paulo, 4 de julho de 1948)[1] foi um dos mais
influentes escritores brasileiros do sculo XX. Foi um importante
editor de livros inditos e autor de importantes tradues. Seguido a
seu precursor Figueiredo Pimentel ("Contos da Carochinha") da
literatura infantil brasileira, ficou popularmente conhecido pelo
conjunto educativo de sua obra de livros infantis, que constitui
aproximadamente a metade da sua produo literria. A outra
metade, consistindo de contos (geralmente sobre temas brasileiros),
artigos, crticas, crnicas, prefcios, cartas, um livro sobre a

SECRETARIA DE ESTADO DA EDUCAO

141

importncia do petrleo e do ferro, e um nico romance, O


Presidente Negro, o qual no alcanou a mesma popularidade que
suas obras para crianas, que entre as mais famosas destaca-se
Reinaes de Narizinho (1931), Caadas de Pedrinho (1933) e O
Picapau Amarelo (1939). Alm de literatura infantil, Lobato se
destacou por abordar a problemtica do caboclo interiorano.
Publicado em 1918, Urups basicamente uma srie de 14
contos, tendo como nfase a vida quotidiana e mundana do
caboclo, atravs de seus costumes, crenas e tradies.
Monteiro Lobato reuniu na obra alguns contos que a experincia de
fazendeiro do Vale do Paraba lhe proporcionou. a obra de estria
de Monteiro Lobato. Urups no contm uma nica histria, mas
vrios contos e um artigo, quase todos passados na cidadezinha de
Itaoca, no interior de SP, com vrias histrias, geralmente de final
trgico e algum elemento cmico. O ltimo conto, Urups,
apresenta a figura de Jeca Tatu, o caboclo tpico e preguioso,
no seu comportamento tpico. No mais, as histrias contam de
pessoas tpicas da regio, suas venturas e desventuras, com seu
linguajar e costumes. Alm dos traos expressionistas (na
descrio
das
personagens
Lobato
utiliza
tcnicas
expressionistas que as deformam, quando se dedica a
caracterizar a natureza passa a vazar metforas de bela
plasticidade que em vrios pontos lembra a idealizao
romntica. Afasta-se, no entanto, dessa escola literria por utilizar
uma linguagem mais simples, arejada, moderna) usados nas
descries das personagens, Lobato utiliza constantemente a
ironia, o que revela uma emotividade extremamente carregada,
fruto de um misto de indignao, impacincia e at intolerncia
ao enxergar os problemas brasileiros e como eles so
provocados pela lassido, fraqueza e indolncia do carter de
nosso povo. Derrubando uma tradio cara, inaugurada por Jos de
Alencar, que apontava como a mestiagem do ndio com o branco
como geradora de uma nao forte, Lobato cr no contrrio. Sua
teoria institui a tese do caboclismo, ou seja, a mistura de raas
gera um tipo fraco, indolente, preguioso, passivo. Sua religio
manifesta-se por meio das mais primitivas formas de
superstio e magia. Sua medicina mais rala ainda. Sua poltica
inexistente, j que vota sem conscincia, conduzido pelo maioral das
terras em que mora. Seu mobilirio o mais escasso possvel,
havendo, no mximo, apenas um banquinho (de trs pernas, o que
poupa o trabalho de nivelamento) para as visitas. No tem sequer
senso esttico, coisa que at o homem das cavernas possua. E
quanto produo, dedica-se apenas a colher o que a natureza
oferece. , portanto, o prottipo de tudo quanto h de atrasado
no pas.
4.

AUGUSTO DOS ANJOS: A poetizao do feio e do hediondo


cientificismo e viso trgica da existncia humana.
A marca da angstia e do pessimismo. essa a principal
caracterstica de se autor. Paraibano, nascido em 1884, Augusto dos
A marca da angstia e do pessimismo. essa a principal
caracterstica de se autor. Paraibano, nascido em 1884, Augusto dos
Anjos, apesar de ter se formado em Direito, elegeu como profisso
apenas o magistrio. Como o prprio poeta reconheceu, seu livro
causou um verdadeiro choque aos padres literrios da poca. Entre
elogios e improprios, havia, no entanto, unanimidade quanto
originalidade da sua obra: com sua linguagem tcnica-cientfica e
grotesca, contrariava a ideologia vigente da 'belle poque' carioca.
Em linhas gerais, 'Eu e Outras Poesias' representa a soma de todas
as tendncias e estilos dominantes desde o final do sculo XIX at o
incio do sculo XX. Em outras palavras, sua obra recebe influncia
do Parnasianismo, do decadentismo, do Simbolismo e ainda
antecipa uma srie de caractersticas modernistas. Em face
disso, podemos dizer que, na realidade, Augusto dos Anjos no se
filiou, com exatido, a nenhuma escola em particular, produzindo,
desse modo, uma obra mltipla e personalssima, at mesmo com
um vocabulrio naturalista. Entre as suas principais caractersticas,
temos, alm da linguagem cientfica e extravagante, a temtica
do vazio das coisas, o nada e a morte, finitude da vida em seus
estgios mais degradados: a putrefao, a decomposio da
matria. Simultaneamente, reflete em seus versos a profunda
melancolia, a descrena e o pessimismo frente ao ser e
sociedade, elaborando, assim, uma poesia de negao: nega as
falsas ideologias, a corrupo, os amores fteis e as paixes
transitrias.

QUMICA
LITERATURA

AULA 12: A LITERATURA DA ERA MODERNA as Vanguardas


Europeias do incio do sculo XX e as ideias radicais do
Modernismo
AS VANGUARDAS EUROPEIAS: a ruptura com o passado, com
o academicismo e as propostas inovadoras para a arte no
sculo XX
Vanguarda (deriva do francs avant-garde) em sentido
literal faz referncia ao batalho militar que precede as tropas em
ataque durante uma batalha. Da deduz-se que vanguarda aquilo
que "est frente". Desta forma, todo aquele que est frente de
algo e, portanto aquele que est frente do seu tempo em uma
atitude poderia se intitular como pertencente a uma vanguarda.
Desta deduo surge a definio adotada por uma srie de
movimentos artsticos e polticos do fim do sculo XIX e incio do
sculo XX. Os movimentos europeus de vanguarda eram
aqueles que, segundo seus prprios autores, guiavam a cultura de
seus tempos, estando de certa forma frente deles. Muitos destes
movimentos acabaram por assumir um comportamento prximo ao
dos partidos polticos: possuam militantes, lanavam manifestos e
acreditavam que a verdade encontrava-se com eles. Muitos outros
artistas e movimentos artsticos, posteriores, por sua atitude
semelhante a das vanguardas europeias cannicas, poderiam ser
referidos pelo termo vanguarda, sendo usual, porm, utilizarmos o
termo somente para os artistas participantes daquelas,
especialmente para fins didticos. Octavio Paz utiliza o termo para
definir toda esttica considerada "fundadora", que representa uma
ruptura nos padres artsticos de sua poca.
Os movimentos de vanguarda emergiram na Europa nas
duas primeiras dcadas do sculo 20 e provocaram ruptura com a
tradio cultural do sculo 19. Foram extremamente radicais e
influenciaram manifestaes artsticas em todo o mundo.
As principais correntes de vanguardas que vo, de modo
bastante forte, influenciar a arte e a literatura do Modernismo so:
1. O CUBISMO
Tratava as formas da natureza por meio de figuras geomtricas,
representando todas as partes de um objeto no mesmo plano. A
representao do mundo passava a no ter nenhum compromisso
com a aparncia real das coisas. Geometrizao das formas e
volumes, renncia perspectiva, o claro-escuro perde sua funo,
representao do volume colorido sobre superfcies planas,
sensao de pintura escultrica, cores austeras, do branco ao negro
passando pelo cinza, por um ocre apagado ou um castanho suave

Les demoiselles d'Avignon Picasso 1907


Esta obra representa, para alm de uma obra-prima do
cubismo mundial, a violao de todas as tradies e convenes
visuais naturalistas ocidentais, ao apresentar cinco aleivosas
(prostitutas), representadas de forma cubista, como se nota na
mulher nua sentada direita, vista simultaneamente de frente e de
costas. Os rostos das personagens refletem o incio do "Perodo
Negro" na obra de Pablo Picasso, quando este sofre uma forte
influncia da primitivismo assemelhando-se a mscaras e
esculturas africanas. Ao exibirem corpos e rostos definidos por
formas geomtricas, traz a impresso de que foram talhadas a
golpes de machado. Braos disformes, cotovelos pontiagudos,
acentuando os ngulos. Com essa obra, Picasso destri o mito da
beleza feminina, ao mesmo tempo em que alude adversidade
tnica deixando evidente nos rostos a inspirao das mscaras
africanas.

2. O FUTURISMO
Foi um movimento artstico e literrio, que surgiu oficialmente em 20
de fevereiro de 1909 com a publicao do Manifesto Futurista, pelo
poeta italiano Filippo Marinetti, no jornal francs Le Figaro. Os
adeptos do movimento rejeitavam o moralismo e o passado, e suas
obras baseavam-se fortemente na velocidade e nos
desenvolvimentos tecnolgicos do final do sculo XIX. Os primeiros
futuristas europeus tambm exaltavam a guerra e a violncia. O
Futurismo desenvolveu-se em todas as artes e influenciou diversos
artistas que depois fundaram outros movimentos modernistas

Giacomo Balla - Velocidade de carro 1912


GIACOMO BALLA, em sua obra o pintor italiano tentou endeusar os
novos avanos cientficos e tcnicos por meio de representaes
totalmente desnaturalizadas, embora sem chegar a uma total abstrao.
Mesmo assim, mostrou grande preocupao com o dinamismo das
formas, com a situao da luz e a integrao do espectro cromtico

3.

O DADASMO
Movimento artstico da chamada vanguarda artstica
moderna iniciado em Zurique, em 1915 durante a Primeira Guerra
Mundial, no chamado Cabaret Voltaire. Formado por um grupo de
escritores, poetas e artistas plsticos, dois deles desertores do
servio militar alemo, liderados por Tristan Tzara, Hugo Ball e Hans
Arp. Defendia a oposio a qualquer tipo de equilbrio,
combinao de pessimismo irnico e ingenuidade radical,
ceticismo absoluto e improvisao. Enfatizou o ilgico e o
absurdo. Entretanto, apesar da aparente falta de sentido, o
movimento protestava contra a loucura da guerra. Assim, sua
principal estratgia era mesmo denunciar e escandalizar. A sua
proposta que a arte ficasse solta das amarras racionalistas e fosse
apenas o resultado do automatismo psquico, selecionando e
combinando elementos por acaso. Sendo a negao total da cultura,
o Dadasmo defende o absurdo, a incoerncia, a desordem, o
caos. Politicamente firma-se como um protesto contra uma
civilizao que no conseguiria evitar a guerra. Ready-Made
significa confeccionado, pronto. Expresso criada em 1913 pelo
artista francs Marcel Duchamp para designar qualquer objeto
manufaturado de consumo popular, tratado como objeto de arte por
opo do artista. Como se percebe, possua como caracterstica
principal a ruptura com as formas de arte tradicionais, e portanto,
foi um movimento com forte contedo anrquico. O prprio nome
do movimento deriva de um termo ingls infantil: dad (brinquedo,
cavalo de pau). Da, observa-se a falta de sentido e a quebra com o
tradicional deste movimento.

Receita para fazer um poema Dadasta


Pegue um jornal.
Pegue a tesoura.
Escolha no jornal um artigo do tamanho que voc deseja dar a seu
poema.
Recorte o artigo.
Recorte em seguida com ateno algumas palavras que formam
esse artigo e meta-as num saco.
Agite suavemente.
Tire em seguida cada pedao um aps o outro.
Copie conscienciosamente na ordem em que elas so tiradas do
saco.
O poema se parecer com voc.
E ei-lo um escritor infinitamente original e de uma sensibilidade
graciosa, ainda que incompreendido do pblico.
Tristan Tzara

SECRETARIA DE ESTADO DA EDUCAO

142

QUMICA
LITERATURA

O ready-made manifestao radical da inteno de


Marcel Duchamp de romper com a artesania da operao artstica,
uma vez que se trata de apropriar-se de algo que j est feito:
escolhe produtos industriais, realizados com finalidade prtica e no
artstica (urinol de loua, p, roda de bicicleta), e os eleva
categoria de obra de arte.
4.

O SURREALISMO,
Foi movimento artstico e literrio surgido primeiramente
em Paris dos anos 20, inserido no contexto das vanguardas que
viriam a definir o modernismo no perodo entre as duas Grandes
Guerras Mundiais. Rene artistas anteriormente ligados ao
Dadasmo
ganhando
dimenso
internacional.
Fortemente
influenciado pelas teorias psicanalticas de Sigmund Freud (18561939), mas tambm pelo marxismo, o surrealismo enfatiza o papel
do inconsciente na atividade criativa. Um dos seus objetivos foi
produzir uma arte que, segundo o movimento, estava sendo
destruda pelo racionalismo. O poeta e crtico Andr Breton (18961966) o principal lder e mentor deste movimento. As
caractersticas deste estilo: uma combinao do representativo, do
abstrato, do irreal e do inconsciente. Entre muitas de suas
metodologias esto a colagem e a escrita automtica. Segundo os
surrealistas, a arte deve se libertar das exigncias da lgica e da
razo e ir alm da conscincia cotidiana, buscando expressar o
mundo do inconsciente e dos sonhos.

A Persistncia da Memria - 1931: Salvador Dal


A PERSISTNCIA DA MEMRIA, o mais conhecido dos quadros, um
quadro pequeno (24x33cm), Dal levou apenas 2 horas para realiz-lo. A
flacidez dos relgios dependurados e escorrendo mostram uma
preocupao humana com o tempo e a memria. A cabea adormecida
que aparece nesse quadro, em muitos outros tambm, o prprio Dal
presente.

O EXPRESSIONISMO
Movimento cultural de vanguarda surgido na Alemanha nos
primrdios do sculo XX, de indivduos que estavam mais
interessados na interiorizao da criao artstica do que na sua
exteriorizao, projetando na obra de arte uma reflexo individual e
subjetiva. Ou seja, a obra de arte reflexo direto do mundo interior
do artista expressionista. O Expressionismo a arte do instinto,
trata-se de uma pintura dramtica, subjetiva, expressando
sentimentos humanos. Utilizando cores irreais, d forma
plstica ao amor, ao cime, ao medo, solido, misria
humana, prostituio. Deforma-se a figura, para ressaltar o
sentimento. Predominncia dos valores emocionais sobre os
intelectuais. Corrente artstica concentrada especialmente na
Alemanha entre 1905 e 1930. Principais caractersticas: pesquisa
no domnio psicolgico; cores resplandecentes, vibrantes,
fundidas ou separadas; dinamismo improvisado, abrupto,
inesperado; pasta grossa, martelada, spera; tcnica violenta: o
pincel ou esptula vai e vem, fazendo e refazendo, empastando
ou provocando exploses; preferncia pelo pattico, trgico e
sombrio.

AULA 13: A LITERATURA DA ERA MODERNA - A Semana de


Arte Moderna: trs noites que fizeram histria entre vaias
relinchos e miados que espantaram o pblico.
A Semana de Arte Moderna, tambm chamada de
Semana de 22, ocorreu em So Paulo no ano de 1922, nos dias 13,
15 e 17 de fevereiro, no Teatro Municipal. A Semana de Arte
Moderna representou uma verdadeira renovao de linguagem, na
busca de experimentao, na liberdade criadora da ruptura com
o passado e at corporal, pois a arte passou ento da vanguarda,
para o modernismo. O evento marcou poca ao apresentar novas
ideias e conceitos artsticos, como a poesia atravs da
declamao, que antes era s escrita; a msica por meio de
concertos, que antes s havia cantores sem acompanhamento de
orquestras sinfnicas; e a arte plstica exibida em telas, esculturas e
maquetes de arquitetura, com desenhos arrojados e modernos. O
adjetivo "novo" passou a ser marcado em todas estas manifestaes
que propunha algo no mnimo curioso e de interesse. A Semana de
Arte Moderna ocorreu em uma poca cheia de turbulncias polticas,
sociais, econmicas e culturais. As novas vanguardas estticas
surgiam e o mundo se espantava com as novas linguagens
desprovidas de regras. Alvo de crticas e em parte ignorada, a
Semana no foi bem entendida em sua poca. A Semana de Arte
Moderna se encaixa no contexto da Repblica Velha, controlada
pelas oligarquias cafeeiras e pela poltica do caf com leite. O
capitalismo crescia no Brasil, consolidando a Repblica e a elite
paulista, esta totalmente influenciada pelos padres estticos
europeus mais tradicionais. Seu objetivo era renovar o ambiente
artstico e cultural da cidade com "a perfeita demonstrao do que h
em nosso meio em escultura, arquitetura, msica e literatura sob o
ponto de vista rigorosamente atual", como informava o Correio
Paulistano, rgo do partido governista paulista, em 29 de janeiro de
1922.
A HERANA BRASILEIRA DAS VANGUARDAS
A principal herana das vanguardas para a literatura
brasileira, alm da influncia localizada que algumas delas
exerceram sobre certos poetas e escritores, o impulso de destruir
os modelos arcaicos, desafiar o gosto estabelecido e props um
olhar inovador para o mundo. Ruptura e transformao: dois
termos que definem bem o espirito da primeira gerao modernista,
como veremos adiante

5.

O grito, Edvard Munch - 1895


O GRITO, quadro do expressionista noruegus Edvard Munch de 1895,
uma das obras de arte mais famosas e caras do planeta. Existem quatro
verses da obra, que descrevem visualmente de maneira tocante, toda a
angstia existencial. Uma delas foi leiloada em Setembro de 2012 por
quase 120 milhes de dlares!

SECRETARIA DE ESTADO DA EDUCAO

143

OBJETIVOS DA SEMANA DE ARTE MODERNA DE 1922


ambiente artstico e cultural da cidade com "a perfeita
demonstrao do que h em nosso meio em escultura,
arquitetura, msica e literatura sob o ponto de vista
rigorosamente atual", como informava o Correio Paulistano a 29 de
janeiro de 1922.
A Semana como evento artstico bombstico
Importantes figuras do modernismo, em 1922. Mrio de
Andrade (sentado), Anita Malfatti (sentada, ao centro) e Zina Aita (
esquerda de Anita). Da esquerda para a direita: Pagu, Elsie Lessa,
Tarsila do Amaral, Anita Malfatti e Eugnia lvaro Moreyra. A
Semana, de uma certa maneira, nada mais foi do que uma ebulio
de novas ideias totalmente libertadas, nacionalista em busca de uma
identidade prpria e de uma maneira mais livre de expresso. No se
tinha, porm, um programa definido: sentia-se muito mais um desejo
de experimentar diferentes caminhos do que de definir um nico ideal
moderno.

13 de fevereiro (Segunda-feira) - Casa cheia, abertura


oficial do evento. Espalhadas pelo saguo do Teatro Municipal de
So Paulo, vrias pinturas e esculturas provocam reaes de
espanto e repdio por parte do pblico. O espetculo tem incio com
a confusa conferncia de Graa Aranha, intitulada "A emoo
esttica da Arte Moderna". Tudo transcorreu em certa calma neste
dia.

15 de fevereiro (Quarta-feira) - Guiomar Novais era para


ser a grande atrao da noite. Contra a vontade dos demais artistas
modernistas, aproveitou um intervalo do espetculo para tocar alguns
clssicos consagrados, iniciativa aplaudida pelo pblico. Mas a
"atrao" dessa noite foi a palestra de Menotti del Picchia sobre a
arte esttica. Menotti apresenta os novos escritores dos novos
tempos e surgem vaias e barulhos diversos (miados, latidos,
grunhidos, relinchos) que se alternam e confundem com aplausos.

QUMICA
LITERATURA

Quando Ronald de Carvalho l o poema intitulado Os Sapos de


Manuel Bandeira, (poema criticando abertamente o parnasianismo e
seus adeptos) o pblico faz coro atrapalhando a leitura do texto. A
noite acaba em algazarra. Ronald teve de declamar o poema pois
Bandeira estava impedido de faz-lo por causa de uma crise de
tuberculose.

17 de fevereiro (Sexta-feira) - O dia mais tranquilo da


semana, apresentaes musicais de Villa-Lobos, com participao
de vrios msicos. O pblico em nmero reduzido, portava-se com
mais respeito, at que Villa-Lobos entra de casaca, mas com um p
calado com um sapato, e outro com chinelo; o pblico interpreta a
atitude como futurista e desrespeitosa e vaia o artista
impiedosamente. Mais tarde, o maestro explicaria que no se tratava
de modismo e, sim, de um calo inflamado
Desdobramentos da Semana de Arte Moderna no Brasil
Vale ressaltar, que a Semana em si no teve grande
importncia em sua poca, foi com o tempo que ganhou valor
histrico ao projetar-se ideologicamente ao longo do sculo. Devido
falta de um iderio comum a todos os seus participantes, ela
desdobrou-se em diversos movimentos diferentes, todos eles
declarando levar adiante a sua herana. No Brasil, o
descontentamento com o estilo anterior foi bem mais explorado no
campo da literatura, com maior nfase na poesia. Entre os escritores
modernistas destacam-se: Oswald de Andrade, Guilherme de
Almeida e Manuel Bandeira. Na pintura, destacou-se Anita
Malfatti, que realizou a primeira exposio modernista brasileira em
1917.
Suas obras, influenciadas pelo cubismo, expressionismo e
futurismo, escandalizaram a sociedade da poca. Monteiro Lobato
no poupou crticas pintora, contudo, este episdio serviu como
incentivo para a realizao da Semana de Arte Moderna. A
Semana, na verdade, foi a exploso de ideias inovadoras que
aboliam por completo a perfeio esttica to apreciada no
sculo XIX. Os artistas brasileiros buscavam uma identidade prpria
e a liberdade de expresso; com este propsito, experimentavam
diferentes caminhos sem definir nenhum padro. Isto culminou com a
incompreenso e com a completa insatisfao de todos que foram
assistir a este novo movimento. Logo na abertura, Manuel Bandeira,
ao recitar seu poema Os sapos, foi desaprovado pela plateia atravs
de muitas vaias e gritos. Embora tenha sido alvo de muitas crticas, a
Semana de Arte Moderna s foi adquirir sua real importncia ao
inserir suas ideias ao longo do tempo.
O movimento modernista continuou a expandir-se por
divulgaes atravs da Revista Antropofgica e da Revista
Klaxon, e tambm pelos seguintes movimentos: Movimento PauBrasil, Grupo da Anta, Verde-Amarelismo e pelo Movimento
Antropofgico. Todo novo movimento artstico uma ruptura com
os padres utilizados pelo anterior, isto vale para todas as formas de
expresses, sejam elas atravs da pintura, literatura, escultura,
poesia, etc. Ocorre que nem sempre o novo bem aceito, isto foi
bastante evidente no caso do Modernismo, que, a princpio, chocou
por fugir completamente da esttica europeia tradicional que
influenciava os artistas brasileiros.

Oswald utiliza durante o desenvolvimento do manifesto,


teorias de diversos autores e pensadores mundiais, como Freud,
Marx, Breton, Francis Picabia, Rousseau, Montaigne e Hermann
Keyserling. Combinadas as ideias destes autores e a ideologia
desenvolvida por Oswald, retomam-se caractersticas dos primrdios
da formao cultural brasileira: a combinao das culturas primitivas
(indgena e africana) e da cultura latina, formada pela colonizao
europeia. E forma-se o conceito errneo de caracterizar, perante a
colonizao, o selvagem como elemento agressivo.
A inteno de promover o resgate da cultura primitiva
notvel no manifesto, e o autor o faz por meio de um processo no
harmonioso de tentar promover a assimilao mtua por ambas as
culturas. Oswald, no entanto, no se ope drasticamente
civilizao moderna e industrializada, mas prope um certo tipo de
cautela ao absorver aspectos culturais de outrem, para que a
modernidade no se sobreponha totalmente s culturas primitivas. E
tambm, para que haja maior cuidado ao absorver a cultura de
outros lugares, para que no haja absoro do desnecessrio e a
cultura brasileira vire um amontoado de fragmentos de culturas
exteriores.
No
decorrer
do
manifesto
o
autor
reconta,
metaforicamente, a Histria do Brasil, associando figuras como
1
Padre Vieira, Anchieta, a Me dos Gracos , a corte de D. Joo VI, a
Moral da Cegonha potncia mtica de Jabuti, Guaraci, Jaci e da
Cobra Grande. Oswald caracteriza como idade de ouro a poca do
Brasil no colonizado, com sua prpria lngua e cultura.
O Manifesto Antropofgico foi um marco no Modernismo
brasileiro, pois no somente mudou a forma do brasileiro de encarar
o fluxo de elementos culturais do mundo, mas tambm colocou em
evidncia a produo prpria, a caracterstica brasileira na arte,
ascendendo uma identidade tupiniquim no cenrio artstico mundial.
MANIFESTO ANTROPFAGO
S a ANTROPOFAGIA nos une. Socialmente. Economicamente.
Filosoficamente.
nica lei do mundo. Expresso mascarada de todos os
individualismos, de todos os coletivismos. De todas as religies. De
todos os tratados de paz.
Tupi, or not tupi that is the question.
Contra todas as catequeses. E contra a me dos Gracos.
S me interessa o que no meu. Lei do homem. Lei do
antropfago.

Curiosidades sobre a Semana de Arte Moderna:


- Durante a leitura do poema "Os Sapos", de Manuel Bandeira, o
pblico presente no Teatro Municipal fez coro e atrapalhou a leitura,
mostrando desta forma a desaprovao.
- No dia 17 de fevereiro, Villa-Lobos fez uma apresentao musical.
Entrou no palco calando num p um sapato e em outro um chinelo.
O pblico vaiou, pois considerou a atitude futurista e desrespeitosa.
Depois, foi esclarecido que Villa-Lobos entrou desta forma, pois
estava com um calo no p.
A CRIAO DO MOVIMENTO ANTROPOFGICO
O Manifesto Antropfago ou Antropofgico foi um
manifesto literrio escrito por Oswald de Andrade, publicado em maio
de 1928, que tinha por objetivo repensar a dependncia cultural
brasileira.
O Manifesto foi publicado na primeira edio da Revista de
Antropofagia, meio de comunicao responsvel pela difuso do
movimento antropofgico brasileiro. A linguagem do manifesto
majoritariamente metafrica, contendo fragmentos poticos bemhumorados e torna-se a fonte terica principal do movimento.

SECRETARIA DE ESTADO DA EDUCAO

144

QUMICA

LITERATURA

AULA 14: A LITERATURA DA ERA MODERNA: a Primeira Fase


do Modernismo no Brasil os novos rumos da arte brasileira

A 1 FASE DO MODERNISMO BRASILEIRO (Fase anrquica,


demolidora e de libertao) - ruptura com o passado, demolio
e anarquia na arte a fase de libertao do Modernismo
brasileiro (1922-1930).
A Primeira Fase do Modernismo foi caracterizada pela
tentativa de definir e marcar posies, sendo ela rica em manifestos
e revistas de circulao efmera. Havia a busca pelo moderno,
original e polmico, com o nacionalismo em suas mltiplas
facetas. A volta das origens, atravs da valorizao do indgena e a
lngua falada pelo povo, tambm foram abordados. Contudo, o
nacionalismo foi empregado de duas formas distintas: a crtica,
alinhado a esquerda poltica atravs da denncia da realidade,
e a ufanista, exagerado e de extrema direita. Devido
necessidade de definies e de rompimento com todas as
estruturas do passado foi a fase mais radical, assumindo um carter
anrquico e destruidor. Um ms depois da Semana de Arte
Moderna, o Brasil vivia dois momentos de grande importncia
poltica: as eleies presidenciais e o congresso de fundao do
Partido Comunista em Niteri. Em 1926, surge o Partido
Democrtico, sendo Mrio de Andrade um de seus fundadores. A
Ao Integralista Brasileira, movimento nacionalista radical, tambm
vai ser fundado, em 1932, por Plnio Salgado. Realizada a Semana
de Arte Moderna e ainda sob os ecos das vaias e gritarias, tem
incio uma primeira fase modernista, que se estende de 1922 a
1930, caracterizada pela tentativa de definir e marcar posies.
Constitui, portanto, um perodo rico em manifestos e revistas de
vida efmera: so grupos em busca de definio. Nessa dcada, a
economia mundial caminha para um colapso, que se concretizaria
com a quebra da Bolsa de Valores de Nova Iorque, em 1929. O
Brasil vive os ltimos anos da chamada Repblica Velha, ou seja, o
perodo de domnio poltico das oligarquias ligadas aos grandes
proprietrios rurais. No por mera coincidncia, a partir de 1922,
com a revolta militar do Forte de Copacabana, o Brasil passa por
um momento realmente revolucionrio, que culminaria com a
Revoluo de 1930 e a ascenso de Getlio Vargas.
Nelson Werneck Sodr, ao analisar as dcadas de 1920 e
30 em Histria da literatura brasileira, explica:
Nesse processo verificamos a seriao das manifestaes
poltico-militares iniciadas com os disparos dos canhes de
Copacabana, em 1922, e encerradas com o internamento da
Coluna Prestes na Bolvia. Tais manifestaes, inequivocamente de
classe mdia, assinalavam o crescendo na disputa pelo poder. Nele
verificamos, ainda, a seriao de manifestaes de rebeldia artstica
a que se convencionou chamar Movimento Modernista, tambm
tipicamente de classe mdia."
Caractersticas gerais da FASE DE 22 do Modernismo brasileiro.
Mas esta destruio no apenas continha todos os
germes da atualidade, como era uma convulso profundssima da
realidade brasileira. O que caracteriza esta realidade que o
movimento modernista imps , a meu ver, a fuso de trs princpios
fundamentais: o direito permanente pesquisa esttica; a
atualizao da inteligncia artstica brasileira e a estabilizao de
uma conscincia criadora nacional." Ao mesmo tempo que se
procura o moderno, o original e o polmico, o nacionalismo se
manifesta em suas mltiplas facetas: uma volta s origens, a
pesquisa de fontes quinhentistas, a procura de uma "lngua
brasileira" (a lngua falada pelo povo nas ruas), as pardias - numa
tentativa de repensar a histria e a literatura brasileiras - e a
valorizao do ndio verdadeiramente brasileiro. o tempo do
Manifesto da Poesia Pau-Brasil e do Manifesto Antropfago, ambos
nacionalistas na linha comandada por Oswald de Andrade, e do
Manifesto do Verde-Amarelismo ou da Escola da Anta, que j traz as
sementes do nacionalismo fascista comandado por Plnio Salgado.
Como se percebe j no final da dcada de 20, a postura nacionalista
apresenta duas vertentes distintas: de um lado, um nacionalismo
crtico, consciente, de denncia da realidade brasileira, politicamente
identificado com as esquerdas; de outro, um nacionalismo ufanista,
utpico, exagerado, identificado com as correntes polticas de
extrema direita. Dentre os principais nomes dessa primeira fase do
Modernismo e que continuariam a produzir nas dcadas seguintes,

SECRETARIA DE ESTADO DA EDUCAO

145

destacam-se Mrio de Andrade, Oswald de Andrade, Manuel


Bandeira, Antnio de Alcntara Machado, alm de Menotti del
Picchia, Cassiano Ricardo, Guilherme de Almeida e Plnio Salgado.
Essa primeira fase foi predominantemente potica. Seus autores
pregavam, entre outras ideias, a negao do passado, a
valorizao potica do cotidiano, o nacionalismo e a liberdade
lingustica, basicamente atravs da incorporao da linguagem
coloquial e o Anarquismo. O esprito desses artistas era polmico,
original e debochado. Havia entre eles, como se v, uma posio
anrquica, cuja frase famosa de Mrio de Andrade os justificava:
"No sabemos definir o que queremos mas sabemos o que no
queremos!". Tinham de lutar para demolir a vida social brasileira, que
consideravam colonial, antiga, lusitana. E havia, sobretudo, um
nacionalismo intransigente, que viria a criar Macunama, de Mrio de
Andrade, ou a Poesia Pau-Brasil de Oswald: Sem esses ingnuos e
lutadores artistas, porm fabulosos modernistas de primeira gerao,
os escritores que vieram depois jamais seriam to slidos como
foram. Na loucura desses moos, fundou-se no Brasil o que h de
mais importante: estarmos passo a passo com a modernidade do
mundo. Ou seja, eles eram livres e "internacionais". Como disse
Mrio de Andrade:
Leitor
Est fundado o Desvairismo.
Este prefcio, apesar de interessante, intil.
[...]
Quando sinto a impulso lrica escrevo sem
pensar tudo o que meu inconsciente me grita.
Penso depois: no s para corrigir, como para
justificar o que escrevi. Da a razo deste
("Prefcio Interessantssimo", 1922)

CARACTERSTICAS DA FASE DE 1922 DO MODERNISMO


BRASILEIRO
a. Poemas rpidos, cheios de irreverncia, ironia, coloquialismos.
b. Tematicamente relacionados ao cotidiano e identidade do
povo brasileiro, em especial ao seu jeito de falar.
c. Caractersticas estticas: grande liberdade formal, com o uso de
versos livres (metrificao irregular) e brancos (sem esquemas
de rima), pontuao despreocupada.
d. Primitivismo atravs da volta ao passado colonial com propostas
antropofgicas.
e. Crticas ao academicismo, atravs de poesias em que valoriza o
coloquialismo e a fuso entre poesia e prosa.
f.
Irreverencia, poema piada, uso da pardia, nacionalismo crtico.
g. A sntese, o humor e o poema-relmpago.
h. a valorizao do linguajar brasileiro espontneo, em
detrimento de uma viso de linguagem elitista e o uso enftico
do humor, da ironia e do sarcasmo.

TEXTO PARA DISCUSSO


Pronominais
D-me um cigarro
Diz a gramtica
Do professor e do aluno
E do mulato sabido
Mas o bom negro e o bom branco
Da Nao Brasileira
Dizem todos os dias
Deixa disso camarada
Me d um cigarro
Oswald de Andrade

QUMICA
LITERATURA

AULA 15: A LITERATURA DA ERA MODERNA E


MODERNISMO DE 1930: a Segunda Fase do Modernismo I

O BRASIL DE Vargas: a crise de 1929 e o fim da Repblica Velha


Na segunda fase do Modernismo, a poesia tambm
amadurece. Os poetas abandonam o tom irreverente e polmico dos
primeiros tempos e, com a liberdade esttica conquistada,
desenvolvem plenamente suas prprias tendncias, sem a
preocupao de chocar os tradicionalistas.
"Os camaradas no disseram que havia uma guerra
e era necessrio trazer fogo e alimento."
Carlos Drummond de Andrade

Recebendo como herana todas as conquistas da


gerao de 1922, a segunda fase do Modernismo brasileiro se
estende de 1930 a 1945.
Perodo extremamente rico tanto em termos de produo
potica quanto de prosa, reflete um conturbado momento histrico:
no plano internacional, vive-se a depresso econmica, o
avano do nazifascismo e a II Guerra Mundial; no plano interno,
Getlio Vargas ascende ao poder e se consolida como ditador,
no Estado Novo. Assim, a par das pesquisas estticas, o universo
temtico se amplia, incorporando preocupaes relativas ao destino
dos homens e ao "estar-no-mundo".
Em 1945, ano do fim da guerra, das exploses atmicas,
da criao da ONU e, no plano nacional, da derrubada de Getlio
Vargas, abre-se um novo perodo na histria literria do Brasil.
MOMENTO HISTRICO
O perodo que vai de 1930 a 1945 talvez tenha
testemunhado as maiores transformaes ocorridas neste sculo. A
dcada de 1930 comea sob o forte impacto da crise iniciada com a
quebra da Bolsa de Valores de Nova Iorque, seguida pelo colapso
do sistema financeiro internacional: a Grande Depresso,
caracterizada por paralisaes de fbricas, rupturas nas
relaes comerciais, falncias bancrias, altssimo ndice de
desemprego, fome e misria generalizadas. Assim, cada pas
procura solucionar internamente a crise, mediante a interveno do
Estado na organizao econmica. Ao mesmo tempo, a depresso
leva ao agravamento das questes sociais e ao avano dos partidos
socialistas e comunistas, provocando choques ideolgicos,
principalmente com as burguesias nacionais, que passam a
defender um Estado autoritrio, pautado por um nacionalismo
conservador, por um militarismo crescente c por uma postura
anticomunista e antiparlamentar - ou seja, um Estado fascista. o
que ocorre na Itlia de Mussolini, na Alemanha de Hitler, na
Espanha de Franco e no Portugal de Salazar.
O desenvolvimento do nazifascismo e de sua vocao
expansionista, o crescente militarismo e armamentismo, somados
s frustraes geradas pelas derrotas na I Guerra Mundial: este ,
em linhas gerais, o quadro que levaria o mundo II Guerra
Mundial (1939-1945) e ao horror atmico de Hiroxima e
Nagasqui (agosto de 1945).
No Brasil, 1930 marca o ponto mximo do processo
revolucionrio estudado nos dois captulos anteriores, ou seja, o
fim da Repblica Velha, do domnio das velhas oligarquias
ligadas ao caf e o incio do longo perodo em que Vargas
permaneceu no poder.
A eleio de 1- de maro de 1930 para a sucesso de
Washington Lus representava a disputa entre o candidato Getlio
Vargas, em nome da Aliana Liberal, que reunia Minas Gerais, Rio
Grande do Sul e Paraba, e o candidato oficial Jlio Prestes,
paulista, que contava com o apoio das demais unidades da
Federao. O resultado da eleio foi favorvel a Jlio Prestes;
entretanto, entre a eleio e a posse, que se daria em novembro,
estoura a Revoluo de 30, em 3 de outubro, ao mesmo tempo que
a economia cafeeira sente os primeiros efeitos da crise econmica
mundial.
A Revoluo de 30, que levou Getlio Vargas a um
governo provisrio, contava com o apoio da burguesia industrial,
dos setores mdios e dos tenentes responsveis pelas revoltas na
dcada de 1920 (exceo feita a Lus Carlos Prestes, que, no exlio,

SECRETARIA DE ESTADO DA EDUCAO

146

havia optado claramente pelo comunismo). Desenvolve-se, assim,


uma poltica de incentivo industrializao e entrada de capital
norte-americano, em substituio ao capital ingls.
Uma tentativa contrarrevolucionria partiu de So Paulo,
em 1932, como resultado da frustrao dos paulistas com a
Revoluo de 30: a oligarquia cafeeira sentia-se prejudicada pela
poltica econmica de Vargas; as classes mdias e a burguesia
temiam as agitaes sociais; e, para coroar o descontentamento,
Vargas havia nomeado um interventor pernambucano para So
Paulo. A chamada Revoluo Constitucionalista2 explodiu em 9 de
julho, mas no logrou xito. Se Guilherme de Almeida foi o poeta da
Revoluo paulista, tendo produzido vrios textos ufanistas, Oswald
de Andrade foi seu romancista crtico, como atesta seu livro Marco
zero - a revoluo melanclica.
Ainda em 32, a ideologia fascista encontra ressonncia no
nacionalismo exacerbado do Grupo Verde-Amarelo, liderado por
Plnio Salgado, fundador da Ao Integralista Brasileira. Ao mesmo
tempo crescem no Brasil as foras de esquerda. Em 1934, elas
formam uma frente nica: a ANL - Aliana Nacional Libertadora.
Tornam-se frequentes os choques entre a extrema-direita e os
membros da ANL, at que o governo federal manda fech-la, por
"atividade subversiva de ordem poltica e social", em julho de 1935.
Entretanto, na clandestinidade, a ANL tenta uma revoluo, em
novembro desse mesmo ano, "contra o imperialismo e o fascismo"
e "por um governo popular nacional revolucionrio". Os revoltosos
previam uma rebelio militar imediatamente acompanhada por
revoltas populares, mas o movimento no foi alm de trs unidades
militares, logo derrotadas; milhares de pessoas foram aprisionadas,
e o governo obteve um pretexto para endurecer o regime.
Getlio Vargas, auxiliado pelos integralistas, inicia sua
ditadura em 10 de novembro de 1937. O chamado Estado Novo
ser um longo perodo antidemocrtico, anticomunista, baseado
num nacionalismo conservador e na idolatria de um chefe nico:
Getlio Vargas. Essa situao se prolongar at 29 de outubro de
1945, quando, pressionado, Getlio renuncia.
Diante desses significativos acontecimentos, Carlos
Drummond de Andrade publica, em 1945, um poema intitulado
"Nosso tempo", que revela o estado de nimo da parcela mais
consciente da sociedade:
"Este tempo de partido,
tempo de homens partidos.
Em vo percorremos volumes,
viajamos e nos colorimos.
A hora pressentida esmigalha-se em p na rua.
Os homens pedem carne. Fogo. Sapatos.
As leis no bastam. Os lrios no nascem
da lei. Meu nome tumulto, e escreve-se
na pedra.
(...)"
Carlos Drummond de Andrade

COMO SE DEU A PRODUO LITERRIA DESSA FASE


MODERNISTA: Maturidade e conscincia poltica e social a
Fase de Construo de nosso Modernismo.
Exposto ao horror de duas grandes guerras, o ser
humano vive tempos sombrios em meados do sculo XX. O que
significa estar no mundo? a esperana deve ser depositada nos
indivduos ou projetada na espiritualidade? Confrontada com
questes como estas, a literatura precisa encontrar novos
caminhos, abandonando a relativa leveza que a marcou no incio do
sculo XX. o momento da consolidao de uma esttica, que se
processa diante de uma forte instabilidade social e poltica
relacionada a esse contexto, fazendo com que surjam na literatura
2

A Revoluo Constitucionalista de 1932, Revoluo de 1932 ou Guerra


Paulista, foi o movimento armado ocorrido no Estado de So Paulo, Brasil,
entre os meses de julho e outubro de 1932, tendo por objetivo a derrubada do
Governo Provisrio de Getlio Vargas e a promulgao de uma nova
constituio para o Brasil. Foi uma resposta paulista Revoluo de 1930, a
qual acabou com a autonomia que os estados gozavam durante a vigncia da
Constituio de 1891. A revoluo de 1930 impediu a posse do governador
de So Paulo Jlio Prestes na presidncia da Repblica e derrubou do poder
o presidente Washington Lus, que fora governador de So Paulo de 1920 a
1924, colocando fim Repblica Velha

QUMICA

LITERATURA

propostas de diferentes modos de interpretar a realidade e de


responder s grandes questes humanas. a fase urea do
Modernismo, depois da reforma esttica proposta pelo esprito
irreverente e iconoclasta da fase de 1922. O romance ganha um
impulso extraordinrio. Na poesia, a fase de nacionalismo
contestatrio e de reviso do passado histrico ficou para trs,
agora mais preocupada com reflexes filosficas e existenciais,
numa busca constantes dos poetas e escritores de entender o
mundo contemporneo, utilizando todos os recursos da criao
potica, claro, sem deixar de lado as conquistas da gerao rebelde
da Semana de Arte Moderna.
CARACTERSTICAS GERAIS DO MODERNISMO DE 30
a) Amadurecimento e solidificao da poesia modernista.
b) Mistura do verso livre com formas tradicionais de compor
poemas.
c) Mistura da temtica cotidiana com temtica histrico-social.
d) Revalorizao da poesia simbolista.
e) Reflexo sobre o estar no mundo.
f) Renovao na linguagem.
g) Busca de retratar a condio humana diante da guerra e dos
problemas polticos e sociais.
h) Espiritualidade como forma de soluo metafsica para os
problemas humanos. Isso se d por conta da necessidade do poeta
de entender Deus e de compreender o mundo diante das
atrocidades e da cruel existncia de bombas atmicas e campos de
concentrao.
i) A anlise do ser humano e de suas angstias.
j) O desejo de compreender a relao entre o indivduo e a
sociedade.
l) Forte dimenso social, em que os autores fazem uso da palavra
que apontam para o sentido da modernidade e da
contemporaneidade, mantendo presente em seus textos a reflexo
sobre os acontecimentos que testemunham e para os quais buscam
explicao.
m) Surgimento do romance regionalista psicolgico, em que as
narrativas trazem um carter de denuncia social e politica, numa
espcie de NEO-REALISMO/NATURALISMO.
n) O Nordeste como tema atravs de uma nova forma de
regionalismo.
o) temticas principais dos romances de 30: contrastes humanos
e sociais dos trabalhadores dos engenhos de cana-de-acar e
os habitantes do serto; a condio de subdesenvolvimento de
um Brasil agrrio; os meninos de rua das grandes cidades
brasileiras; a seca no serto nordestino; os conflitos
psicolgicos do homem moderno das grandes cidades; a
problemtica do cacau na Bahia; o cangao, o banditismo.
p) Carter crtico e panfletrio dos romances.
q) A retomada do determinismo cientfico da segunda metade do
sculo XIX, agora apresentando um elemento novo: a fuso do
regional com o psicolgico para atingir o universal.
r) Apresenta como projeto literrio revelar como uma determinada
realidade socioeconmica, no caso, o subdesenvolvimento
brasileiro, pode afetar a vida dos brasileiros.
s) A focalizao da realidade injusta do homem brasileiro.
t) A poesia apresenta como temas: a espiritualidade, a
efemeridade do tempo da vida, o amor, a metalinguagem,
religiosidade crist, engajamento social e socialismo cristo, o
sentimento do mundo e a redefinio do eu.

SECRETARIA DE ESTADO DA EDUCAO

147

EXERCCIO DE FIXAO
Retrato
Eu no tinha este rosto de hoje,
assim calmo, assim triste, assim magro,
nem estes olhos to vazios,
nem o lbio amargo.
Eu no tinha estas mos sem fora,
to paradas e frias e mortas;
eu no tinha este corao
que nem se mostra.
Eu no dei por esta mudana,
to simples, to certa, to fcil:
- Em que espelho ficou perdida a minha face?
Ceclia Meireles
No Vou Me Adaptar
Eu no caibo mais
Nas roupas que eu cabia
Eu no encho mais
A casa de alegria
Os anos se passaram
Enquanto eu dormia
E quem eu queria bem
Me esquecia...
Ser que eu falei
O que ningum ouvia?
Ser que eu escutei
O que ningum dizia?
Eu no vou me adaptar
Me adaptar...
Eu no tenho mais
A cara que eu tinha
No espelho essa cara
No minha
Mas que quando
Eu me toquei
Achei to estranho
A minha barba estava
Desse tamanho...
Arnaldo Antunes
Em ambos os textos pode-se afirmar que:
a)
b)
c)
d)

e)

O eu-lrico focaliza a transitoriedade e fugacidade do tempo.


O primeiro apresenta um tom alegre, enquanto o segundo
apresenta-se melanclico.
O eu-lrico sofre transformaes esto apenas no plano fsico.
Enquanto o primeiro focaliza a passagem da adolescncia para
a vida adulta, o segundo focaliza a passagem da vida adulta
para a morte.
O ser humano no se d conta dos estgios de
desenvolvimento pelos quais passa

QUMICA
LITERATURA

AULA 16: A LITERATURA DA ERA MODERNA E O


MODERNISMO DE 1930 II: a produo literria da Segunda Fase
do Modernismo (1930-1945)
1. INTRODUO
A partir de 1930, as atitudes de rebeldia e transgresso da
gerao de 22 so substitudas por uma nova mentalidade literria:
o momento de construir uma literatura adulta, consciente de sua
prpria identidade. Alis, como vimos na aula anterior, os prprios
participantes da nossa primeira gerao modernista j haviam
passado a se dedicar construo de uma nova literatura em lugar
de concentrar seus esforos em combater o academicismo literrio.
Costumam-se indicar os anos de 1930 e 1945 como limites para
esse subperodo, que constituiu uma das pocas mais frteis de toda
a nossa literatura. Surgiram nesse momento muitos nomes que
passariam a figurar entre os mais importantes de toda a literatura
brasileira.
2.

O SURGIMENTO DO ROMANCE REGIONALISTA DO


NORDESTE: a realidade nordestina e os problemas
humanos a seca, o latifndio, o banditismo e a condio
humana

Do Romantismo ao Modernismo
A prosa de fico da dcada de 1930 se entronca na
tradio da prosa de fico regionalista brasileira proveniente do
sculo XIX. Desde o Romantismo, a valorizao do regional e do
local faz parte das intenes dos ficcionistas. No RealismoNaturalismo, passou-se a observar de forma mais documental essa
realidade regional, procurando-se interpret-la muitas vezes a partir
de esquemas tericos deterministas. Por isso, h quem considere
que o chamado romance nordestino da dcada de 30 teria surgido
independentemente da Semana de Arte Moderna. De qualquer
forma, inegvel que a campanha anti-acadmica da Semana
colaborou para que muitos desses ficcionistas fossem mais
facilmente aceitos.

EM SNTESE: caractersticas do romance regionalista de 30 do


Modernismo contribuio dos autores
GRACILIANO RAMOS: o mestre das palavras secas o
regionalismo universalista e psicolgico.
1. De maneira geral, sus romances caracterizam-se pelo interrelacionamento entre as condies sociais e a psicologia das
personagens; ao que se soma uma linguagem precisa , enxuta
e despojada, de perodos curtos mas de grande fora
expressiva.
2. A linguagem constri o olhar realista: econmica, despojada,
misturando o academicismo do narrador com o coloquialismo
das personagens.
3. As personagens e o meio social poderoso: jaguno X
proprietrio rural.
4. Revelao do ntimo das personagens mostrando sua
deteriorao provocada por situaes traumticas.
5. Processo de zoomorfizao do homem atravs de uma anlise
psicolgica profunda.
6. O cuidado com as palavras.
7. Retrato da destruio individual (So Bernardo).
8. A desumanizao provocada pelo meio: o serto nordestino e a
agruras do meio que ofende homens e animais
1.
2.

A bagaceira: marco literrio


Em 1928, foi publicado o romance A bagaceira, de Jos
Amrico de Almeida. O livro foi, na poca, recebido com muito
entusiasmo e passou a ser considerado um marco literrio: era o
incio de uma literatura regional e social, preocupada em apresentar
e analisar as condies de vida e os costumes dos nordestinos,
particularmente dos trabalhadores rurais cujas vidas eram
atormentadas pela seca e tambm pela explorao econmica.
A bagaceira, como romance, no um livro bem-sucedido. Teve sua
importncia histrica na reabertura de um caminho na prosa de
fico brasileira, o do regionalismo. E foi importante porque
aprofundou o conceito de regionalismo, fazendo-o sair do exotismo
romntico e dos excessos do Naturalismo para uma viso mais
crtica da realidade.
Era o xodo da seca de 1898. Uma ressurreio de
cemitrios antigos - esqueletos redivivos, com o aspecto terroso e o
fedor das covas podres. Os fantasmas estropiados como que iam
danando, de to trpegos e trmulos, num passo arrastado de
quem leva as pernas, em vez de ser levado por elas.
Andavam devagar, olhando para trs, como quem quer
voltar. No tinham pressa em chegar, porque no sabiam onde iam.
Expulsos do seu paraso por espadas de fogo, iam, ao acaso, em
descaminhos, no arrasto dos maus fados.
Fugiam do sol e o sol guiava-os nesse forado nomadismo
Adelgaados na magreira cmica, cresciam, como se o
vento os levantasse. E os braos afinados desciam-lhes aos joelhos,
de mos abanando.
Vinham escoteiros. Menos os hidrpicos - doentes da
alimentao txica - com os fardos das barrigas alarmantes.
No tinham sexo, nem idade, nem condio nenhuma.
Eram os retirantes. Nada mais.
Meninotas, com as pregas da sbita velhice, careteavam,
torcendo as carinhas decrpitas de ex-voto. Os vaqueiros msculos,
como tits alquebrados, em petio de misria. Pequenos
fazendeiros, no arremesso igualitrio, baralhavam-se nesse annimo
aniquilamento.

SECRETARIA DE ESTADO DA EDUCAO

Mais mortos do que vivos. Vivos, vivssimos s no olhar.


Pupilas do sol da seca. Uns olhos espasmdicos de pnico,
assombrados de si prprios. Agnica concentrao de vitalidade
faiscante.
Fariscavam o cheiro enjoativo do melado que lhes
exacerbava os estmagos jejunos. E, em vez de comerem, eram
comidos pela prpria fome numa autofagia erosiva.
Lcio almoava com o sentido nos retirantes. Escondia
cdeas nos bolsos para distribuir com eles, como quem lana
migalhas a aves de arribao.

148

3.
4.
5.

Criador do chamado ciclo da cana-de-acar na literatura de


30 do nosso Modernismo.
Recordaes da infncia de um menino criado na fazenda do
av.
Linguagem construda para dar veracidade s cenas e s
personagens.
Realidade econmica do Nordeste canavieiro e a passagem da
economia agraria, da cana, para o processo de industrializao.
Narrativas memorialistas em que as lembranas pessoais esto
na base de sua criao romanesca.

JORGE AMADO: retrato da diversidade econmica e cultural da


Bahia.
1.
2.

3.

4.

A miscigenao racial e cultural do Brasil.


A emoo, o sentimento e o apelo ao popular atravs de
enredos simples, destacando a presena da mulher, mostrando
uma fuso entre o romntico e o erotismo.
A malicia, a sensualidade e o bom humor do um sabor picante
s cenas em suas obras e constituem tambm fatores
determinantes do grande sucesso do autor.
Subcultura negra e luta de classes, barqueiros, pescadores, o
povo e suas tradies e misticismo, o candombl, meninos de
rua como menores delinquentes, temas socialistas, a temtica
do cacau e a luta pela posse de terras no sul da Bahia,
coronelismo, humor atravs de cenas populares e
malandragens de gente do povo, as lutas politicas, a vitria da
modernidade sobre o medievalismo econmico, a Bahia
pitoresca, mulheres sedutoras, inteligentes e lindas que
provocam transformao em seu meio social e fsico.

RACHEL DE QUEIROZ: Um olhar feminino para o serto.


1. A vida dura do Nordeste e dos retirantes na seca.
2. A seca como motivo literrio. Militante comunista.
3. Linguagem simples como a gente do Nordeste e a focalizao
da tragdia humana desencadeada pelo clima da regio.
4. Inserida no Modernismo (Romance de 30), a prosa regionalista
de Rachel de Queiroz retrata, numa linguagem enxuta e viva, o
Nordeste, mais precisamente o Cear. A autora consegue aliar
a preocupao social (flagelo da seca e coronelismo)
preocupao com os traos psicolgicos das personagens.

QUMICA
LITERATURA

RICO VERSSIMO: O interprete dos gachos no romance de 30.

CECLIA MEIRELES

1.

1.

2.

3.

4.
5.

Retrato da classe mdia gacha e preocupao com a crise


espiritual e moral do homem e da sociedade em que vive.
Investiga a relao entre o presente degradado por crises e
revolues e o passado histrico marcado pelo herosmo do
povo gacho na defesa de seu territrio.
Em O tempo e o vento, o autor aborda o mtico, o histrico e o
social em que mostra a formao de um povo atravs de uma
saga familiar.
Temas polticos.
Veia humanstica em que explora as angustias e os conflitos
vividos pelo homem em um mundo francamente em crise.

DIONLIO MACHADO: as angustias do homem comum.


1.

2.
3.

4.

Autor conhecido pela sua obra Os ratos, em que trata de uma


estranha narrativa que relata minuciosamente as 24 horas da
vida de um funcionrio pblico.
Fico social que mostra o homem urbano e seus conflitos
pessoais.
Estilo seco, enxuto, que transmite ao leitor o pavor e a angustia
de um homem comum.
A crise econmica que afeta o homem e a busca de solues
para sobreviver.

A POESIA DA SEGUNDA FASE DO MODERNISMO


Estendendo-se de 1930 a 1945, a segunda fase foi rica na
produo potica e, tambm, na prosa. O universo temtico ampliase com a preocupao dos artistas com o destino do Homem e no
estar-no-mundo. Ao contrrio da sua antecessora, foi construtiva.
No sendo uma sucesso brusca, a poesia da gerao de 22 e 30
foram contemporneas. A maioria dos poetas de 30 absorveram
experincias de 22, como a liberdade temtica, o gosto da expresso
atualizada ou inventiva, o verso livre e o anti-academicismo Portanto,
ela no precisou ser to combativa quanto a de 22, devido ao
encontro de uma linguagem potica modernista j estruturada.
Passara, ento, a aprimor-la, prosseguindo a tarefa de purificao
de meios e formas direcionando e ampliando a temtica da
inquietao filosfica e religiosa, com Vincius de Moraes, Jorge
de Lima, Augusto Frederico Schmidt, Murilo Mendes, Carlos
Drummond de Andrade.

Primeira mulher a alcanar destaque no cenrio da poesia


brasileira.
2. Ligada corrente espiritualista da gerao de 30 do
Modernismo.
3. Valorizao da intuio e da emoo como maneiras de
interpretara o mundo.
4. Lirismo ligado imagem da natureza (a gua, o mar, o ar, o
vento, o espao, a rosa, etc.), ou seja, do infinito e da
efemeridade, compondo uma atmosfera de sonho e de
fugacidade da vida e do tempo.
5. A vida apresentada metaforicamente como uma rosa.
6. A mostragem dos seres humanos como seres que se apegam
s aparncias.
7. Lembranas do passado, buscando temas como o amor e a
natureza que marca o ritmo da vida.
8. No plano estilstico ao contrrio do coloquialismo dos poetas
modernos h em sua obra uma tendncia linguagem
elevada, sempre carregada de musicalidade. A msica,
algumas vezes, parece ser mais importante que o prprio
sentido dos versos.
9. Temtica neo-simbolista, sobretudo no que se refere busca da
sugesto das palavras.
10. O registro de estados de nimo vagos e quase incorpreos.
Neles predomina uma difusa melancolia e uma noo de perda
amorosa, abandono e solido.
11. A atmosfera de dor existencial que emana dos poemas de
Ceclia Meireles centrada na percepo de que tudo passa e
de que o fluir do tempo dissolve as iluses e os amores, o corpo
e mesmo a memria.
VINICIUS DE MORAIS
1.
2.

3.

4.

CARLOS DRUMMOND DE ANDRADE


1.

2.

3.
4.
5.
6.

Num primeiro momento, sem se deixar envolver, o poeta


mantm um certo distanciamento do mundo sua volta, o que
lhe possibilita brincar e soltar a razo, deixando-a entregue a si
mesma, maquinando incertezas e certezas, mais afeitas a negar
e anular que a construir. Da os temas do cotidiano, da famlia,
do isolamento, da monotonia entendiante das coisas e do viver,
expressos numa linguagem coloquial plena de ironia seca,
sarcasmo e humor desencantado, onde sentimento e emoo
so refreados. So deste primeiro momento os livros Alguma
Poesia e Brejo das Almas.
No segundo momento, sem se distanciar, deixa-se envolver
pela realidade sua volta e canta a impotncia e a solido em
um mundo mecnico, frio e poltico; a decepo e a falta de
perspectiva diante da fragmentao causada pela guerra; o
sofrimento e a solidariedade do ser humano brasileiro e
universal. Temas estes abordados em tons ora esperanosos,
ora desesperanosos, com a mesma ironia, humor e
sobriedade, esto presentes nos livros Sentimento do Mundo,
Jos e A Rosa do Povo.
Questionamento sobre o fazer potico e preocupao com a
funo social do poeta.
A mostragem do poeta desajeitado, torto, perdido em meio aos
problemas existenciais (o poeta gauche)
O poeta apresenta um eu que, por ser deslocado dos outros
homens, tem um modo particular de ver as coisas e o mundo.
Tom pessimista pelo fato de o eu desejar responder s
indagaes sobre o mundo.

SECRETARIA DE ESTADO DA EDUCAO

149

5.

Poeta conhecido por suas canes musicais, iniciando sua


trajetria potica sob forte influncia do Simbolismo.
Em suas obras iniciais, apresenta uma forte religiosidade em
seus versos e uma angstia associada oscilao entre
matria e esprito. Apresenta um profundo misticismo, fruto de
sua fase crist.
Em um segundo momento, sua poesia se volta para temas
relacionados ao cotidiano e para o relacionamento amoroso,
com uma forte oralidade maliciosa.
Forte influencia da poesia camoniana em termos de temtica
amorosa: preferncia por retratar e buscar definir o amor,
gosto pelo soneto como forma potica, uso frequente de
antteses para expressar as contradies prprias do
sentimento amoroso.
Sua poesia varia da passagem do metafsico para o fsico, do
espiritual para o sensual, do sublime para o cotidiano, o poeta
retoma sugestes romnticas (como lua, cidade, samba).
Refugia-se no erotismo: h contemplao do amor, poemas
"sobre a mulher" e adorao pantestica da natureza. Comps
tambm poemas de indignao social, cujos exemplares so:

"Balada dos mortos dos campos de concentrao", "O operrio


em construo" e "A rosa de Hiroxima".
MURILO MENDES: o catlico visionrio.
1.

2.

3.
4.
5.

Sua produo inicial apresenta influncia do olhar irreverente


dos primeiros modernistas (de 22 da fase anrquica), fazendo
releitura do passado histrico brasileiro atravs da pardia.
Estado de confisso transcendente contendo a abundante
inspirao e a contundente inventiva nos versos de intensas
sintaxes e insupervel rigor.
Seguidor da vanguarda surrealista, em que apresenta sonhos
agregados imagens do catolicismo.
A conscincia social e o sentido do que seja humanidade
sempre em consonncia com a dimenso espiritual.
Seus versos so ao mesmo tempo surrealista, mstico,
messinico, anrquico, barroco, catlico e vanguardista. s
vezes, seus poemas buscam entender como se d a relao
entre o eu e o mundo, em temas que se voltam para a
incapacidade de transformao da realidade, em que esse eu
sente rejeitado por Deus, expulso do cu.

QUMICA
LITERATURA

AULA 17: A 3 FASE DO MODERNISMO BRASILEIRO: o mundo


ps-guerra e a busca do sentido da existncia
Ruptura com a 1 e a 2 fase modernista, experimentao
esttica e a busca por uma nova expresso literria foram as
principais caractersticas da terceira gerao modernista.
A terceira gerao modernista, tambm conhecida como
Gerao de 1945, desenvolveu temticas e estticas diversas s
geraes anteriores. Nas fases precedentes, sobretudo na primeira
fase modernista (1922-1930), havia uma grande preocupao em
consolidar a Literatura nacional atravs de elementos que
reforavam a identidade brasileira nas diferentes manifestaes
artsticas. Os escritores da gerao de 45 romperam com esse
padro, apresentando grandes inovaes na pesquisa esttica e nas
formas de expresso literria. Grandes escritores, preocupados
principalmente com a pesquisa em torno da prpria linguagem,
surgiram, pois o contexto poltico, relativamente tranquilo em relao
s geraes anteriores, fomentou o trabalho esttico e lingustico,
pois menos exigidos social e politicamente, puderam explorar com
maior afinco a forma literria, tanto na prosa quanto na poesia. Em
1945, terminada a Segunda Guerra Mundial e, no Brasil, a ditadura
de Vargas, o Brasil vivia um perodo democrtico e
desenvolvimentista, cujo pice ocorreu nos anos de governo do
presidente Juscelino Kubitschek.
1.

CONTEXTO HISTRICO PS 45

O ano de 1945, que assinala o incio da terceira fase do


Modernismo, dos mais marcantes da histria da humanidade.
Exploses atmicas nas cidades japonesas de Hiroshima e
Nagaski, fim da segunda guerra, perodo de reestruturao
geogrfica, poltica e econmica que dividiu o mundo em blocos
capitalistas.
O medo de novos ataques nucleares alimentou a chamada
"guerra fria", que ops pases capitalistas e comunistas ao longo
das dcadas seguintes.
No Brasil, 1945 o ano da queda de Getlio Vargas - que
voltaria ao poder, pelo voto popular, em 1950, onde
permaneceu at suicidar-se, em 1954.
A partir de 1956, o pas vai viver uma poca de democratizao
poltica e de desenvolvimento econmico, que se tornou intenso
durante o governo de Juscelino Kubitschek (1956-1960), cuja
propaganda oficial prometia um avano histrico de "cinquenta
anos em cinco".
Os Planos de Metas de Juscelino para a modernizao do pas
resultaram em impressionante crescimento industrial, que
aumentou os empregos e a renda dos brasileiros. O
desenvolvimento, as grandes realizaes, como a construo
de Braslia, e a estabilidade poltica contriburam para criar a
atmosfera de otimismo dos chamados "anos dourados".

2.

A PRODUO DA FASE DE 45 DO MODERNISMO:


caractersticas gerais.

1.

Preocupao com a materialidade do texto potico (sons,


ritmos, rimas, disposio do verso na pgina, etc.)
Retomadas de formas clssicas como maneira de definir os
limites entre o potico, fundamentado no trabalho com a forma e
o no potico, associado ao mero registro do elemento prosaico
do cotidiano.
Incorporao poesia de ideias relacionadas multiplicidade,
materialidade e fragmentao da sociedade ps-moderna,
renovao esttica, valorizao da tcnica de composio e
busca de temas relacionados ao social, moral e poltico.
Na prosa: explorao da linguagem como matria prima do
texto, reinventado a linguagem e a sintaxe, ruptura com a
estrutura da narrativa tradicional, mergulho no ntimo do ser
humano, atravs do fluxo de conscincia e da introspeco
psicolgica.
Ampliao das possibilidades de significao do texto literrio.
Reviso da funo do narrador.
Ampliao do regionalismo, criando um significado universal,
em que se processam temas ligados a todos os seres humanos,
abordando um mundo repleto de temores, receios, medos,
crenas e fraquezas.

2.

3.

4.

5.
6.
7.

SECRETARIA DE ESTADO DA EDUCAO

150

8.

Reflexes sobre grandes questes universais que atormentam o


ser humano: o bem e o mal, a sanidade e a loucura, o certo e
o errado, o amor e a morte, o acaso e o destino.
9. Investigao dos processos que tornam o ser humano nico,
que lhe do identidade.
10. O ser humano se descobrindo atravs de momentos epifnicos
em que se reconhecem e se reavaliam questionando a sua
submisso s expectativas familiares e sociais.
11. A literatura deve ter, para esses escritores, objetivos profundos
e universalistas, fazendo refletir e questionar um sentido para a
vida e, principalmente, sobre o destino do homem na vida.
Nesse sentido, o escritor no deve ser passivo, em que apenas
recolha os dados da realidade, mas deve ser ativo, estando no
mundo atravs de uma presena ativa em constante
comunicao com que o cerca.
12. Criao de uma prosa que utiliza-se de resgates de termos
arcaicos, criao de neologismos, tentativa de fugir dos clichs,
reconstruo da fala regional criando ritmos inesperados,
inverses surpreendentes, imagens delicadas e breves.
Portanto, a poesia dessa gerao trouxe ao cenrio das
discusses literrias a seguinte proposio: a poesia a arte da
palavra. Esse princpio implicava a alterao de pontos de vista da
poesia de 30, que j tinha sido social, poltica, religiosa e filosfica.
Por conta da extrema preocupao com a forma, chegou a ser
chamada de neo-parnasiana. O estilo culto e levado vai fazer parte
da poesia de alguns desses autores, como o caso de Joo Cabral
de Melo Neto, conforme veremos na prxima aula.
Na prosa, o conto e o romance ganham novos destaques,
a profundando a sondagem psicolgica das personagens, uma
ampliao do regionalismo de 30 agora tomando rumos mais
experimentais, o espao urbano tambm passando a ser objeto de
enfoque.
RESUMINDO
Caractersticas:
- Regionalismo universal;
- Objetividade;
- Aprofundamento do questionamento do eu e da anlise
psicolgica;
- Intimismo;
- Prosa de introspeco psicolgica;
- Presena de neologismos;
- Realismo mgico e fantstico;
- Neoparnasianismo;
- Intertextualidade;
- Uso constante da metalinguagem

QUMICA

LITERATURA

estar-no-mundo e
do sentido da vida a introspeco
psicolgica das personagens.
"Tenho medo de escrever. to perigoso. Quem tentou, sabe.
Perigo de mexer no que est oculto e o mundo no est tona."
(Clarice Lispector)

A Terceira Fase do Modernismo constitui a Gerao de 45,


cuja prosa apresenta uma diversidade grande de temas e de
linguagem. Todavia, os romances e os contos de dois autores
sobrepujam, sobremaneira, os dos outros autores brasileiros que
fizeram literatura a partir dos anos 40 at hoje: Guimares Rosa e
Clarice Lispector.
Apesar das profundas diferenas da obra de cada um,
Guimares Rosa e Clarice Lispector apresentam posturas comuns
como a insubordinao aos processos romanescos de at ento
(no seguiram o romance regionalista, o urbano e o psicolgico
da poca) e a constante pesquisa com a linguagem. Por essas
razes, ambos so chamados de instrumentalistas, o que significa
dizer que se preocupavam muito com o trabalho, isto , com a
melhor elaborao de seus respectivos textos. Alm disso, h nesses
dois escritores um interesse de universalizar o romance brasileiro,
um procedimento bem claro na sondagem do mundo interior das
personagens. A diferena mais evidente entre ambos que em
Guimares Rosa existe ainda a manuteno do enredo com
suspense, ao passo que em Clarice Lispector aparece o abandono
claro do enredo referencial narrativo e o mergulho nas profundezas
da alma de personagens isolados e problemticos.
3.

A NARRATIVA INTIMISTA DE CLARICE LISPECTOR: a


epifania e o fluxo de conscincia.

O objetivo de Clarice, em suas obras, o de atingir as


regies mais profundas da mente das personagens para a
sondar complexos mecanismos psicolgicos. essa procura que
determina as caractersticas especificas de seu estilo.
O enredo tem importncia secundria. As aes - quando
ocorrem - destinam-se a ilustrar caractersticas psicolgicas das
personagens. So comuns em Clarice histrias sem comeo, meio
ou fim. Por isso, ela se dizia, mais que uma escritora, uma
"sentidora", porque registrava em palavras aquilo que sentia. Mais
que histrias, seus livros apresentam impresses.
Predomina em suas obras o tempo psicolgico, visto
que o narrador segue o fluxo do pensamento e o monlogo
interior das personagens. Logo, o enredo pode fragmentar-se. O
espao exterior tambm tem importncia secundria, uma vez
que a narrativa concentra-se no espao mental das
personagens. Caractersticas fsicas das personagens ficam em
segundo plano. Muitas personagens no apresentam sequer nome.
As personagens criadas por Clarice Lispector descobrem-se num
mundo absurdo; esta descoberta d-se normalmente diante de um
fato inusitado - pelo menos inusitado para a personagem. A ocorre a
epifania, classificado como o momento em que a personagem
sente uma luz iluminadora de sua conscincia e que a far despertar
para a vida e situaes a ela pertencentes que em outra instncia
no fariam a menor diferena.
Esse fato provoca um desequilbrio interior que mudar a
vida da personagem para sempre. Para Clarice,
"No fcil escrever. duro quebrar rochas. Mas voam
fascas e lascas como aos espelhados (...) Mas j que se h de
escrever, que ao menos no se esmaguem com palavras as
entrelinhas". "Minha liberdade escrever. A palavra o meu domnio
sobre o mundo."
4.

CLARICE LISPECTOR: a busca incansvel da identidade


atravs da sondagem psicolgica.

1.

Narrativas que tratam da condio feminina, da dificuldade de


relacionamento humano, da hipocrisia dos papeis sociais, da
busca pelo eu.
Esmiuamento dos processos interiores do ser humano.
O cotidiano e o ser humano inserido no mundo da roda viva do
dia-a-dia e deparando-se com determinado evento at chegar
em um momento de epifania (revelao, iluminao em que
toma conscincia da realidade).

2.
3.

SECRETARIA DE ESTADO DA EDUCAO

151

4.

A autodescoberta de sua personagem durante o enredo de suas


narrativas.
5. Personagens em busca de se libertar da opresso social.
6. Metforas incomuns e questionamentos filosficos e
metafsicos.
7. Presena constante de monlogos interiores e discursos
narrativos que buscam a criao de uma suprarrealidade
pessoal e potica (prosa potica intimista, de forte influencia de
James Joyce e de Virginia Woolf).
8. Busca constante de libertao e de mudanas na maneira de
viver por parte das personagens que, no fundo, so a prpria
Clarice.
9. Conscincia da prpria solido, ao perceber que o inferno so
os outros, como disse Sartre, de Heidegger.
10. Descoberta e revelao de si mesmo, busca do significado da
vida, escolhas dos prprios caminhos, convivncia com a
nusea e com a angustia das prprias limitaes, adaptao ao
mundo e busca de um lugar para si.
A PROSA DE GUIMARES ROSA
Joo Guimares Rosa um dos autores de fico
experimental da prosa da terceira gerao do modernismo,
caracterizada pela experimentao potica que abandona as
caractersticas da poesia de 22 como o humor na poesia, para migrar
para uma poesia de perfeio da forma, justificada pela simetria dos
versos e do uso de neologismos como os de Rosa. gerao que
apesar de ser modernista, pode ser chamada de neoparnasiana, que
objetivava a construo de um poema srio e equilibrado.
A obra de Rosa extremamente inovadora e original.
Havia o uso do neologismos em suas obras(a arte de inventar
palavras) que o autor utilizava em seu regionalismo, onde descrevia
o serto mineiro representado principalmente pelos jagunos(bandos
de soldados, como os cangaceiros na Regio Nordeste),saindo da
linguagem inteiramente cotidiana e do serto superficial(as
particularidades e cotidiano de povo),para uma linguagem nutrida
pela fuso da erudio, da formalidade(neologismos) com a
linguagem popular, modificando radicalmente o regionalismo na
literatura.
O regionalismo de Rosa pode ser classificado como
universal pois serto rosiano no apenas um simples serto. O
serto rosiano o prprio mundo dos personagens, no um lugar
que causa a misria, a pobreza dos personagens como os de
Graciliano Ramos, e sim um lugar ideal para a reflexo psicolgica
dos personagens de Rosa sobre o existencialismo da vida, sobre
temas ambguos que esto presentes em todos os homens(como a
existncia do diabo, ou de Deus;). Ou seja, seus personagens no
sero enxergados apenas pelo seu costumes, e sim tambm pelos
seus conflitos internos e sentimentos. Com isso o serto de Rosa
ser um universo mtico e supersticioso, seus personagens tero
crenas politestas e sero umbandistas e espritas. O linguajar de
Rosa ser uma mistura do real, do irreal e universal.
Guimares Rosa transforma o regionalismo da gerao de
30. As marcas regionais so evidentes nos termos utilizados, na
recriao da fala do sertanejo e dos jagunos e de vaqueiros do
interior de Minas. As questes tematizadas, porm, vo muito alm
de uma perspectiva regional. Em suas narrativas, Rosa fala dos
grandes dramas humanos: a dor, a morte, o dio, o amor, o medo.
Indagaes filosficas aparecem na boca de homens simples,
incultos, deixando claro que os grandes fantasmas da existncia
podem ser identificados em qualquer lugar, desde um grande centro
urbano at um minsculo vilarejo nos sertes das Gerais. Essa
incansvel busca de respostas para as angustias humanas faz com
que o regionalismo, na fico de Guimares Rosa, ganhe uma
dimenso universal. Acompanhar a trajetria de Riobaldo,
personagem de Grande serto, veredas, significa aprender que o
serto est em toda parte. Em seus contos e novelas emerge um
mundo sempre marcado pelo confronto de opostos: o arcaico e o
moderno, o rural e o urbano, o oral e o escrito. A infncia, o
amor, a violncia e o misticismo aparecem como temas de grande
interesse em suas narrativas. A sanidade e a loucura aparecem em
seus personagens como forma de refletir sobre a profundidade do
Ser. A capacidade de apresentar o olhar infantil que aparece em
alguns de seus contos. Sintetizando as caractersticas da narrativa
de Guimares Rosa:

QUMICA
LITERATURA

GUIMARES ROSA: o descobridor do serto universal


1.
2.
3.
4.
5.
6.

7.
5.

TEXTO PARA LEITURA E DEBATE

Regionalismo universalista e renovao da linguagem.


Simbiose entre a linguagem padro e o linguajar regional
trabalhado esteticamente.
Musicalidade, ritmo e harmonia fundindo prosa e poesia.
Invenes lxicas, atravs, por exemplo, do neologismo,
utilizando prefixos, sufixos, justaposies e aglutinaes.
Construo de frases e perodos com onomatopeias,
aliteraes, anforas e sinestesias.
Dualismo lingustico que se manifesta na associao do falar
sertanejo com a linguagem de cunho erudito (arcasmos,
estrangeirismos, neologismos cultos)
Emprego de frases exclamativas, truncadas e interrogativas:
repetio de palavras, uso de partculas expletivas.
A POESIA RACIONALISTA DE JOO CABRAL DE MELO
NETO: o poeta engenheiro

Joo Cabral um poeta cerebral. A base de seu fazer


potico , na verdade, o planejamento, a reflexo sobre o prprio
processo de composio. Esses aspectos so os seus pontos de
contato com a gerao de 1945, mas sua preocupao em buscar
extrair a mxima significao de cada palavra e a qualidade literria
de seus poemas foram tambm os fatores que o projetaram para fora
dela e fizeram com que o poeta alcanasse uma importncia muito
grande para a literatura brasileira do sculo XX.
Ao lado de uma poesia formal, tcnica e preparada, Joo Cabral
insere em seus versos um cenrio essencialmente nordestino, com
muitas referncias zona da mata e ao serto. A aridez dos espaos
do Nordeste, onde se criou, faz de sua poesia uma forma de
fidelidade s suas origens. Conforme o poeta mesmo definiu, a
linguagem que utiliza aparece despida de ornamentos, como se
fosse uma espcie de faca s lmina, como aponta um de seus
poemas.
CARACTERSTICAS DA OBRA DE JOO CABRAL DE MELO
NETO

Sempre
guiado pela lgica, pelo raciocnio, seus poemas evitam anlise
e exposio do eu e voltam-se para o universo dos objetos, das
paisagens, dos fatos sociais, jamais apelando para o
sentimentalismo.
O prazer
esttico que sua poesia pode provocar deriva sobretudo de uma
leitura racional, analtica, no do envolvimento emocional com o
texto.
Essas
caractersticas levaram a crtica a ver na obra de Joo Cabral
uma "ruptura com o lirismo" ou a considerar sua expresso
potica como "antilrica".
Mas, no
devemos, entretanto, supor que essa relao do poeta com o
mundo concreto, objetivo, produza apenas textos descritivos.
Na verdade, suas descries ora acabam adquirindo valor
simblico, ora acabam denunciando a crtica social que o poeta
pretende levar a efeito.
A luta de
Joo Cabral contra o individualismo dos poetas que s se
preocupam com seu eu ntimo, levou-o a buscar novas formas
de comunicao.
O poeta
se aproxima das cantigas populares nordestinas, ao cordel,
trovas medievais e uma srie de formas poticas, prprias da
tradio popular, utilizando inclusive as tcnicas de composio
empregadas nessas formas de poesia popular como, por
exemplo, a repetio de palavras, algo muito presente no cordel
nordestino.

SECRETARIA DE ESTADO DA EDUCAO

152

Catar feijo
Catar feijo se limita com escrever:
joga-se os gros na gua do alguidar
e as palavras na folha de papel;
e depois, joga-se fora o que boiar.
Certo, toda palavra boiar no papel,
gua congelada, por chumbo seu verbo:
pois para catar esse feijo, soprar nele,
e jogar fora o leve e oco, palha e eco.
Ora, nesse catar feijo entra um risco:
o de que entre os gros pesados entre
um gro qualquer, pedra ou indigesto,
um gro imastigvel, de quebrar dente.
Certo no, quando ao catar palavras:
a pedra d frase seu gro mais vivo:
obstrui a leitura fluviante, flutual,
aula a ateno, isca-a como o risco.
EXERCCIOS DE FIXAO
01) (UEPB-2009)- Leia as assertivas abaixo a respeito da obra de
Joo Cabral de Melo Neto.
I- Poesia de profunda conscincia formal, que conseguiu aliar as
conquistas poticas do Modernismo a uma reflexo sobre a realidade
social brasileira, sobretudo do homem nordestino.
II- Filiou-se Gerao de 45, sendo um de seus principais expoentes
e o grande disseminador de seus pressupostos estticos e
ideolgicos.
III- Ops-se ao lirismo sentimental, dominante na histria da poesia
brasileira, e construiu uma poesia impessoal, com o intuito de
alcanar pela palavra a dimenso mais concreta possvel da
realidade.
a)
b)
c)
d)
e)

Apenas II correta.
Apenas I e III so corretas.
Apenas III correta.
Todas so corretas.
Nenhuma correta.

02) O experimentalismo ou instrumentalismo lingstico um dos


elementos caractersticos do estilo de Guimares Rosa.
Assinale o item em que tal recurso NO ocorre:
a) Foi o capiauzinho apanhando, estapeado pelos quatro
cacundeiros de Nh Augusto, e empurrando para o denso do povo,
que tambm queria estapear.
b) Passaram entre alas e aclamaes dos outros, que, a, como no
havia mais mulheres, nem brigas, pegaram a debandar ou a cantar.
c) E ela conhecia e temia os repentes de Nh Augusto. Duro, doido
e sem detena, como um bicho grande do mato.
d) ...enquanto Nh Augusto ia indo em busca de qualquer luz em
porta aberta, onde houvesse assombros de homens, para entrar no
meio ou desapartar.
e) ...uma vontade sem calor no corpo, s pensada: como que, se
bebesse e cigarrasse, e ficasse sem trabalhar nem rezar, haveria de
recuperar sua fora de homem...

QUMICA
LITERATURA

AULA 01 : ARTE PR-HISTRICA


EXERCCIOS DE FIXAO
Paleoltico
As primeiras formas de arte at agora
encontradas esto longe de serem
belas. So as chamadas Vnus. Os
traos femininos bastante exagerados
(_____________________________) e
a pouca preocupao com rosto, pernas
e braos nos levam a supor que sejam
formas ligadas ao culto da fertilidade.
Esculpidas em marfim ou pedra, tm
dimenses to pequenas que podem
ser
facilmente
transportadas
e
difundidas.

A _____________________ era feita


em paredes de rochas a cu aberto ou
cavernas. Os primeiros exemplares
dessa arte so identificados em
cavernas de difcil acesso e, muitas
vezes, escuras. , portanto, razovel
imaginar que no foram pintadas
apenas para a apreciao humana,
fazendo parte de um ritual mgico com
vistas a assegurar uma caa bemsucedida.

1) A pintura rupestre acima, que um patrimnio cultural


brasileiro, expressa
a) o conflito entre os povos indgenas e os europeus durante
o processo de colonizao do Brasil.
b) a organizao social e poltica de um povo indgena e a
hierarquia entre seus membros.
c) aspectos da vida cotidiana de grupos que viveram
durante a chamada pr-histria do Brasil.
d) os rituais que envolvem sacrifcios de grandes dinossauros
atualmente extintos.
e) a constante guerra entre diferentes grupos paleondios da
Amrica durante o perodo colonial.
2) Ao observar a ilustrao abaixo podemos afirmar
corretamente:

Neoltico
Nesse perodo, constatamos a representao de figuras
humanas quase sempre estilizadas, nas quais predominam
as cenas de caa.

Muito pouco podemos falar sobre a cermica neoltica, uma vez


que o material bastante frgil. Percebe-se a preocupao com
motivos geomtricos e a simetria a marca registrada.
Em termos de arte pr-histrica, foi durante o Neoltico que
assistimos ao impressionante aparecimento dos monumentos
megalticos. A grandeza e a dificuldade em se criar tais obras
pressupem a utilizao de mo de obra abundante e a estruturao
da sociedade.

I. os homens primitivos utilizavam do recurso da Mo em


Positivo (ou Negativo) para demarcar a posse do territrio;
II. as tintas eram feitas de carvo terra, sangue e minerais
modos e aplicados com as mos, plumas de aves, peles de
animais, lascas de madeira e punhado de musgo
III. muitas das pinturas e insculturas ainda no apresentam
explicaes cientficas totalmente comprovadas, como no
caso da Pedra do Ing, no interior da Paraba;
IV. geralmente atribui-se uma funo ritualista mgica aos
motivos da arte paleoltica.
a) Esto corretas as afirmativas I e II;
b) Esto corretas as afirmativas II e III;
c) Esto corretas as afirmativas I, II e III;
d) Esto corretas as afirmativas II, III e IV;
e) Todas as afirmativas esto corretas.

SECRETARIA DE ESTADO DA EDUCAO

153

QUMICA
ARTES

EXERCCIOS DE FIXAO

AULA 02 : ARTE NA ANTIGUIDADE ORIENTAL


Mesopotmia

A escrita cuneiforme, utilizada pelos mesopotmicos, foi criao


dos antigos sumrios. Entre os mais importantes legados, destaca-se
o ___________________________, que se tornou, posteriormente, a
base do Direito de quase todos os povos semitas.
A escultura desse perodo o que podemos chamar hiertica. No
h preocupao descritiva, salvo pelo rosto. As figuras esculpidas em
baixo-relevo caracterizam-se por um grande realismo.
Com o fim da supremacia sumeriana, acdios e babilnicos
realizaram uma grande renovao artstica. A ourivesaria ganhou
fora e as estelas tornaram-se importantes formas de expresso
artstica. Os desenhos eram mais suaves e a liberdade de
representao aumentou, sem romper com o padro tradicional.

( ____________ )
Apesar de suas caractersticas prprias, a arte fencia tem como
elemento fundamental a sntese entre as muitas tendncias artsticas
existentes na regio. Estilizada, apoiada na ourivesaria e nos
trabalhos em marfim e bronze, a arte fencia no ficou presa a
padres rgidos de beleza. Porm, o grande legado cultural foi a
inveno da escrita fontica, como necessidade de efetuar os
registros de mercadorias, facilitando o intercmbio comercial com
outros povos.

1) Ao observar a imagem acima, podemos notar que a maior


expresso
da
arte
persa
foi
a) a pintura de feitos histricos.
b) a confeco de objetos de cermica.
c) a construo de suntuosos palcios.
d) as necrpoles.
2) A principal manifestao artstica da cultura hebraica foi
caracterizada pela
a) construo de palcios.
b) construo de orculos.
c) disseminao da pintura.
d) literatura.
e) fartura de esculturas
3) A arte fencia possui como principal caracterstica
a) a construo de templos.
b) a construo de pirmides colossais, semelhana dos egpcios.
c) a escultura e o desenvolvimento da pintura na cermica.
d) a escrita, visto terem sido os fencios os criadores do alfabeto
fontico.
e) a sntese entre as vrias tendncias artsticas existentes na
regio, viabilizada pela dinmica atividade mercantil.

BARCO FENCIO

( _____________ )
A cultura persa caracterizada por um __________________ dos
povos conquistados. Os seus tmulos, palcios, joias e trajes
revelam o esplendor de um imprio, organizado de maneira
militarista e autoritria, centrado no poder absoluto do rei e na vasta
burocracia estatal.
Sobre as plancies arenosas mandaram abrir suas cmaras
funerrias, imitando os tmulos dos faras egpcios. Os reis persas,
fascinados pelos grandes palcios, herana dos assrios, ergueram
seus jardins e esculturas decoradas no apenas com baixos-relevos,
mas tambm com tijolos pintados, garantindo um colorido magnfico
que resiste ainda hoje. No h dvida de que os enormes conjuntos
arquitetnicos de Pasrgada, Susa e Perspolis tm grandeza e
revelam a impressionante sensao do poder imperial.

( ___________ )
A Bblia imortalizou o povo hebreu, transformando-se em seu
grande legado cultural. Praticamente inexistiu a arte figurativa e, em
termos arquitetnicos, a economia precria e as inmeras invases
deixaram poucos vestgios, dos quais os mais importantes so as
runas das muralhas de Jeric e a base do templo de Salomo.

SECRETARIA DE ESTADO DA EDUCAO

154

QUMICA
ARTES

AULA 03 : EGITO
De todos os princpios formais e racionalistas da arte do Oriente
Antigo, e especialmente da arte egpcia, o mais saliente e o mais
caracterstico o da frontalidade

Ligadas religio e poltica, podemos dizer que algumas das


mais belas obras de arte dessa civilizao foram executadas
justamente em um momento de ruptura e crise poltica: a tentativa de
implantao do monotesmo organizada pelo fara Amenfis IV.
Romper com o culto anterior significava desenvolver novas formas
de expresso artstica que pudessem estar vinculadas nova
doutrina. Vemos, ento, aparecer uma preocupao muito mais
decorativa; a liberdade artstica foi incentivada e, sobretudo,
financiada pelo fara. O exemplo mais belo que nos chegou desse
perodo o busto da rainha Nefertiti, esposa de Amenfis IV.
O nome dessa rainha significa chegou a bela (ou beleza). Seu
rosto ainda jovem traduzia traos perfeitos que mal conseguiam
dissimular certa melancolia e contemplao. At hoje, considerada
uma das mulheres mais belas que a arte humana conseguiu
perpetuar.
A descoberta da _____________________
durante a campanha militar de Napoleo
Bonaparte ao Egito, possibilitou, em 1822, ao
jovem sbio francs Champollion decifrar a
escrita hieroglfica, desvendando muitos dos
mistrios que cercam essa civilizao

ANTIGUIDADE CLSSICA
Grcia
A Grcia Antiga alcanou um notvel desenvolvimento artstico e
cultural. A arte grega uma arte antro- pocntrica, exprimindo
valores de equilbrio, harmonia, ordem, proporo e medida e
buscando exaltar a beleza e o calor da vida humana, no o mundo
alm-tmulo.
Nessa arte, condenado o mistrio. Os artistas gregos no estavam
sujeitos s limitaes impostas pelos sacerdotes e reis, como na
maioria das civilizaes orientais, e tiraram suas ideias da filosofia
racionalista e humanista dominante.

O pice da escultura grega foi alcanado


durante o Sculo de Pricles. Os grandes
nomes da poca so Fdias, amigo de Pricles e
diretor de todos os seus projetos de construo,
criador das imagens de Zeus em Olmpia e
Atena no Parthenon de Atenas; Prateles, que
se notabilizou pelos retratos de divindades
humanizadas, como a esttua de Hermes, talvez
o mais belo produto da arte grega; Mron,
famoso por retratar os tipos atlticos de Atenas,
destacando-se a figura do Discbulo, que
representa um atleta no instante em que se
prepara para lanar o disco em uma
competio.

SECRETARIA DE ESTADO DA EDUCAO

155

A caracterstica marcante dessa escultura deve-se


engenhosidade de Mron: o movimento do Discbulo deveria
ser
bastante
instvel
e
desequilibrado,
mas,
engenhosamente, o autor usou artifcios para evitar o
problema.
Roma
Os romanos aprenderam a construir, a partir da influncia
deixada pelos etruscos nas tcnicas de utilizao do arco e
da abbada.
Os modelos de construes gregas, apesar de muito
admirados pelos romanos por toda sua beleza, tornaram- se
insuficientes, em vista de raramente os edifcios helnicos
serem feitos para acomodar multides sob seu teto.
Quais as principais formas de edifcios romanos?
_________________________________________________
_________________________________________________
_________________________________________________
EXERCCIOS DE FIXAO

1) A pintura egpcia pode ser caracterizada como uma arte


que
a) definiu os valores passageiros e transitrios como forma
de representao privilegiada.
b) concebeu as imagens como modelo de conduta,
utilizando-as em rituais profanos.
c) adornou os palcios como forma de representao pblica
do poder poltico.
d) valorizou a originalidade na criao artstica como
possibilidade de experimentao de novos estilos.
e) elegeu os valores eternos, presentes nos monumentos
funerrios, como objeto de representao.
2) Na sala, pai e filho esto entretidos com jogos de tabuleiro
e bebem cerveja em um final de tarde de domingo. A perna
engessada de um deles no permitiu que fossem a uma
cervejaria. No quintal, as crianas se divertem brincando de
amarelinha e entre ces de estimao que correm derredor.
Em um dos quartos, duas adolescentes experimentam novos
cosmticos e cremes hidratantes, enquanto conversam sobre
mtodos contraceptivos e o teste de gravidez que a mais
velha far no dia seguinte. No quarto principal, uma mulher
divide seus pensamentos entre a contabilidade de sua
padaria e o divrcio prestes a se concretizar. Para amenizar
a dor de cabea, ela toma remdio base de cido
acetilsaliclico, o princpio ativo da aspirina (Texto extrado
de papiro da XVIII dinastia egpcia, cerca de 3.300 anos
atrs).
3) convincente a prova da sabedoria milenar dessa
sociedade que sobreviveu cerca de 5 mil longos anos.
Sabemos tambm que artisticamente muito foi deixado de
sua cultura, a exemplo das primeiras construes fnebres
sobre a terra dos faras. Estas, que em rabe significam
banco de areia, so denominadas de
a) Mastabas
b) Pirmides
c) Hipogeos
d) Mausolus
e) Vale dos Reis

QUMICA
ARTES

AULA 05 : IDADE MODERNA


Renascimento
O Renascimento caracterizado por um novo esprito que
modificou a mentalidade do pensamento, em uma poca marcada
por grandes descobertas e novas formas de expresso. Os ideais
humanistas conduziram ao individualismo, estimulando a busca de
fama e o desejo de ser conhecido por outros homens, em vida e
depois da morte. Ao mesmo tempo, o homem comeou a olhar o
mundo de uma forma cientfica, buscando explicaes que at ali
tinham sido admitidas sem provas.
Cite outras caractersticas desse perodo:
_______________________________________________________
_______________________________________________________
_______________________________________________________
_______________________________________________________
_______________________________________________________
_______________________________________________________
_____________
Em Florena, no final do sculo XIII, foram lanadas as bases da
pintura europeia que se prolongaria nos sculos seguintes. Buscando
o progresso material, os florentinos favoreceram o desenvolvimento
de Giovanni Cimabue, precursor de um estilo mais realista.

Com ________________________ (1452 a 1519) surgiu o maior


exemplo da cultura universal, fenmeno relativamente frequente,
porm no com uma intensidade to forte.
Exercendo o domnio em vrios outros campos (arquitetura,
escultura, urbanismo, literatura, msica e cincia), Leonardo
pretendia atingir a verdade com experincias objetivas. Essa
universalidade fez com que deixasse vrias pinturas inacabadas,
pois interessava-se por um problema somente at que ele fosse
resolvido. Dentre suas obras mais famosas destacam-se o afresco
da Santa Ceia, a Gioconda e a Virgem dos Rochedos.

Michelangelo (1475 a 1564) exerceu grande domnio na escultura,


pintura e arquitetura, criando obras grandiosas com uma energia
quase sobrenatural. Entre 1508 e 1512 concluiu o teto da Capela
Sistina, no Vaticano, criando uma obra-prima cuja importncia
memorvel.

Barroco
Por volta do final do sculo XVI deu-se uma mudana
impressionante na histria da arte italiana. Os maneiristas
continuavam a pintar, mas sua forma estava fora de moda. Assim,
um maneirismo frio e intelectualista cedeu lugar a um estilo sensual,
emocional e universalmente compreensvel, o Barroco, que surgiu
como reao de uma concepo de arte, em parte intrinsecamente
popular e, em parte, apoiada pela classe cultural dirigente, mas com
considerao pelas massas, contra o exclusivismo intelectual do
perodo precedente.
Emanao direta da Reforma Catlica
(____________________ ) do sculo XVI, o Barroco alcanou a
maturidade no sculo XVII e se prolongou pelo sculo XVIII, com
o nome de _________________. Encontra-se, pois, inserido entre
dois surtos classicistas de ndole greco-romana, um que lhe
antecedeu, o Renascimento, e outro que lhe sucedeu, o
____________________.
O Davi, de Bernini, por exemplo, no ato de matar Golias. O
movimento do corpo jovem do desafiador e a intensidade do seu
olhar sugerem to fortemente a presena de Golias que o centro real
da obra parece estar fora da esttua, na vizinhana do observador.
Dissolveu-se a fronteira entre a obra de arte e o mundo real.

Os artistas barrocos estavam profundamente interessados na


realidade da existncia humana, mas cada um a via a seu modo.
Para Jan Vermeer a realidade residia no detalhe luminoso; para
Rembrandt, no drama esboado das personalidades; Frans Hals
encontrava-as nas faces vigorosas de burgueses e camponeses; e
para Rubens, nos contornos voluptuosos do corpo humano. Mesmo
os altivos aristocratas pintados por Van Dyck, Velsquez e Rigaud
so pessoas reais e os deuses em mrmore de Bernini parecem de
carne e osso.

Neoclassicismo
Uma reao contra o ___________________ .
Os cnones antigos foram reelaborados sob novos princpios de
racionalidade, de proporo, de medida, de ordem e de limpidez.

Apesar de no ser to inovador quanto Leonardo e Michelangelo,


______________________ (1483 a 1520) o pintor mais importante
do Renascimento, ao criar uma arte ao mesmo tempo lrica e
dramtica. Utilizando-se da harmonia e da regularidade de formas e
cores, deu uma extraordinria dimenso pintura. Dentre sua
produo destacam-se
Quais as principais esculturas renascentistas?
_______________________________________________________
_______________________________________________________
_______________________________________________________
_______________________________________________________
_______________________________________________________

A arquitetura do Neoclassicismo tambm apresenta superfcies


planas, na simetria e na exatido geomtrica; por isso, o
convencionalismo e o tecnicismo reinaram nas academias de belasartes at serem questionados pelo advento da arte moderna.
Romantismo

ARTES
SECRETARIA DE ESTADO DA EDUCAO

156

QUMICA

So privilegiados os temas dramticos, sentimentais, inspirados


pela literatura, pela histria e por qualquer anedotismo descritivo.
Procuram-se no contedo, mais do que os valores de arte, os efeitos
emotivos. As bases do Romantismo foram inspiradas na fantasia,
sentimento, paixo e imaginao.
Ao lado dessas caractersticas, outros valores compuseram a
esttica romntica, tais como o sentimento do presente, o
nacionalismo e a valorizao da natureza, esta com uma viso
especial e diferente do neoclssico.
O Romantismo apareceu como reao ao Neoclassicismo, pois,
enquanto este era uma volta aos padres da cultura greco-romana, o
novo estilo dava maior liberdade de expresso aos seus artistas,
valorizando os princpios da criao artstica.
Na Pintura
Diferentemente dos demais estilos de arte, o Romantismo
praticamente s desenvolveu a pintura, com paisagens e quadros
com temas de exaltao nacional.
Um elemento que se pode notar nos quadros romnticos a
composio em diagonal, que sugere instabilidade e dinamismo ao
observador. H uma revalorizao da cor, ressurgindo os contrastes
claro-escuro, o que produz um efeito de dramaticidade.
O Romantismo apareceu como reao ao Neoclassicismo, pois,
enquanto este era uma volta aos padres da cultura greco-romana, o
novo estilo dava maior liberdade de expresso aos seus artistas,
valorizando os princpios da criao artstica.

AULA 06 : VANGUARDAS (SC. XIX E XX)


Art Nouveau
Na segunda metade do sculo XIX, surgiu um novo movimento
artstico, resultante de uma reunio das mais variadas tendncias,
tais como: as ideias da industrializao; o Movimento das Artes e
Ofcios, influenciado pelas ideias de um jovem socilogo e arquiteto
ingls, William Morris (1834-1896); a Arte Oriental, que penetrava na
Europa por causa do Neocolonialismo; as Artes Decorativas e as
Iluminuras Medievais.
Com uma viso ampla, o Art Nouveau procurou preservar o contato
do artista com a _________________ e desenvolver um artesanato
habilidoso. Sua maior conquista foi promover uma unidade das artes
e, sendo assim, os objetos do dia a dia e o prprio edifcio passaram
a ser criados com base em uma mesma tendncia decorativista.
Na arquitetura, manteve-se a tendncia decorativista, segundo a
qual era possvel criar novas formas com o ferro e o vidro. Com isso,
deu-se origem tambm pesquisa em direes diversas na arte de
construir.

Quais as principais inspiraes dos artistas da Art Nouveau?


_______________________________________________________
_______________________________________________________
_______________________________________________________
_______________________________________________________
Impressionismo
Foi o ltimo movimento artstico do sculo XIX, mais
especificamente na sua segunda metade. Foi um movimento que
para muitos autores saiu diretamente do Realismo, passando por
uma curta fase, cujo grande expoente foi ______________________.
Foi do ateli de Gustave Courbet que saiu um quadro que
provocou um escndalo na sociedade da poca. A obra era de um
pintor chamado douard ___________, e o quadro, O Almoo sobre
a Relva, causou um profundo choque na burguesia, pois era
composto de uma mulher nua na companhia de dois senhores de
sobrecasaca, tendo no fundo outra mulher vestida lavando-se no rio,
ou lago.

EXERCCIOS DE FIXAO
1)Observe os quadros abaixo,

Ambos apresentam temticas ,


a) polticas
b) religiosas
c) ldicas
d) aristocrticas
e) filosficas

2) Qual das frases abaixo no condiz com a pintura do


Romantismo?
a) A pintura romntica negou a esttica neoclssica.
b) A pintura romntica aproximou-se das formas barrocas.
c) Na pintura, houve um interesse maior pelos fatos reais da
histria nacional e contempornea de seus artistas.
d) Ora calma, ora agitada, a natureza nas telas do
Romantismo apresenta um dinamismo equivalente s
emoes humanas.
e) Os artistas romnticos tiveram um grande interesse pelos
temas mitolgicos greco-romanos.

SECRETARIA DE ESTADO DA EDUCAO

157

Se Manet abriu a brecha para um novo estilo, foi Claude Monet o


verdadeiro lder da Revoluo Impressionista, apresentando o mundo
como um jogo de superficialidades, com uma forte luminosidade sob
a decomposio cromtica, chegando a tirar a importncia do objeto.
Ele recusou o termo Impressionista para sua pintura, mas a
expresso j havia sido dada por seus discpulos, possivelmente por
um crtico que tenha visto um quadro de Monet intitulado Impresso:
o Nascer do Sol.

QUMICA
ARTES

- Uso de vrios formatos de expresso (objetos do cotidiano, sons,


fotografias, poesias, msicas, jornais, etc) na composio das obras
de artes plsticas;
- Forte carter pessimista e irnico, principalmente com relao aos
acontecimentos polticos do mundo.

A escultura impressionista chegou bem perto da pintura, apesar de


muitos estudiosos da arte terem o costume de afirmar que o
Impressionismo no revitalizou a escultura. Tal afirmao errnea,
pois desde Bernini no aparecia um notvel escultor como Auguste
Rodin (1840-1917), que iria fazer um projeto ambicioso chamado As
Portas do Inferno, inspirado no Inferno de Dante Alighieri, e tem
como principais obras O Pensador e O Beijo. Destaca-se como a sua
mais ousada criao O Monumento a Balzac, de aparncia
fantasmagrica, feita em gesso, com quase 3 metros de altura.

Cubismo
O marco inicial do Cubismo ocorreu em Paris, em 1907, com a tela
Les Demoiselles d'Avignon, pintura que ______________________
levou um ano para finalizar. Nesta obra, este grande artista espanhol
retratou a nudez feminina de uma forma inusitada, onde as formas
reais, naturalmente arredondadas, deram espao a figuras
geomtricas perfeitamente trabalhadas.
Tanto nas obras de Picasso, quanto nas pinturas de outros artistas
que seguiam esta nova tendncia, como, por exemplo, o ex-fauvista
francs _____________________ h uma forte influncia das
esculturas africanas e tambm pelas ltimas pinturas do psimpressionista francs Paul Czanne, que retratava a natureza
atravs de formas bem prximas as geomtricas.
Somente aps a Semana de Arte Moderna de 1922 o movimento
cubista ganhou terreno no Brasil. Mesmo assim, no encontramos
artistas com caractersticas exclusivamente cubistas em nosso pas.
Muitos pintores brasileiros foram influenciados pelo movimento e
apresentaram caractersticas do cubismo em suas obras. Neste
sentido, podemos citar os seguintes artistas: Tarsila do Amaral, Anita
Malfatti, Rego Monteiro e Di Cavalcanti.

_______________________________
Surrealismo
O surrealismo surgiu na Frana na dcada de 1920. Este
movimento foi significativamente influenciado pelas teses
psicanalticas de Sigmund Freud, que mostram a importncia do
inconsciente na criatividade do ser humano.
O marco de incio do surrealismo foi a publicao do Manifesto
Surrealista, feito pelo poeta e psiquiatra francs Andr Breton, em
1924. Neste manifesto, foram declarados os principais princpios do
movimento surrealista: ausncia da lgica, adoo de uma realidade
"maravilhosa" (superior), exaltao da liberdade de criao, entre
outros.
Os artistas do surrealismo que de destacaram mais na dcada de
1920 foram: o escultor italiano Alberto Giacometti, o dramaturgo
francs Antonin Artaud, os pintores espanhis Salvador Dal e Joan
Mir, o belga Ren Magritte, o alemo Max Ernst, e o cineasta
espanhol Luis Buuel e os escritores franceses Paul luard, Louis
Aragon e Jacques Prvert.

_______________________________
Expressionismo
O expressionista inclinado a deformar a realidade de modo cruel,
caricatural, muitas vezes hilrio; o exagero, a distoro e a
dramaticidade das formas, linhas e cores revelam uma atitude
emocional do artista.

_______________________________
Dadasmo
Foi um movimento artstico que surgiu na Europa (cidade suia de
Zurique) no ano de 1916. Possua como caracterstica principal a
ruptura com as formas de arte tradicionais. Portanto, o dadasmo foi
um movimento com forte contedo anrquico. O prprio nome do
movimento deriva de um termo ingls infantil: dad (brinquedo,
cavalo de pau). Da, observa-se a falta de sentido e a quebra com o
tradicional deste movimento.
Caractersticas principais do dadasmo:
- Objetos comuns do cotidiano so apresentados de uma nova forma
e dentro de um contexto artstico;
- Irreverncia artstica;
Combate s formas de arte
institucionalizadas;
- Crtica ao capitalismo e ao consumismo;
- nfase no absurdo e nos temas e contedos sem lgica;

SECRETARIA DE ESTADO DA EDUCAO

158

ARTE CONTEMPORNEA (SC. XX E XXI)


Quais os principais estilos de Arte Contempornea?
A) _____________________________________________
B) _____________________________________________
C) _____________________________________________
D) _____________________________________________

E) _____________________________________________

QUMICA
ARTES

SECRETARIA DE ESTADO DA EDUCAO

159

QUMICA

SECRETARIA DE ESTADO DA EDUCAO

160

QUMICA

AULA 01: TRIGONOMETRIA I

EXERCCIOS DE FIXAO
01. (UFRN) A casa central de uma fazenda situa-se a 9 km,
contados ao longo de um caminho perpendicular estrada reta
que limita a fazenda. Na beira da estrada e a uma distncia de
15 km da casa central, o fazendeiro construiu uma casa para
seu filho. O fazendeiro agora quer construir, na beira da mesma
estrada, um escritrio que fique igualmente distanciado da casa
do filho e da casa central.

TRINGULO RETNGULO
C

Escritrio

Da figura, temos:
hipotenusa: .......
catetos: .............
ngulos: , e 90
relao entre os ngulos agudos: + = ..............
relao entre os lados: ...........................

Casa
do filho

15

Casa central
A distncia comum dever ser:
a) entre 8 e 9 km
b) entre 11 e 12 km
c) entre 12 e 13 km

RAZES TRIGONOMTRICAS
Seno de um ngulo agudo: a razo entre o cateto oposto a
esse ngulo e a hipotenusa.
Cosseno de um ngulo agudo: a razo entre o cateto
adjacente a esse ngulo e a hipotenusa.
Tangente de um ngulo agudo: a razo entre o cateto
oposto e o cateto adjacente a esse ngulo.

d) entre 9 e 10 km
e) entre 9,5 e 10Km

Em relao ao tringulo ABC acima, temos:


sen = ..

cos = .

tg =

sen = ..

cos = .

tg = ..

Concluses imediatas:
I.
O seno de um deles e o cosseno do outro so iguais
II. A tangente de um deles o inverso da tangente do outro
sen
III. tg =
cos
IV.

02. (UFPB/2010) Em parques infantis, comum encontrar um


brinquedo, chamado escorrego, constitudo de uma superfcie
plana inclinada e lisa (rampa), por onde as crianas deslizam, e
de uma escada que d acesso rampa. No parque de certa
praa, h um escorrego, apoiado em um piso plano e horizontal,
cuja escada tem 2m de comprimento e forma um ngulo de 45
com o piso; e a rampa forma um ngulo de 30 com o piso,
conforme ilustrado na figura abaixo.

sen + cos = 1

NGULOS NOTVEIS TABELA IMPORTANTE

ARCO
Chama-se arco a qualquer
compreendida entre dois pontos.

parte

da

circunferncia

As unidades de medidas de arcos so: GRAU, RADIANO ou


GRADO. Vamos trabalhar com grau e radiano por serem os mais
usados.
I. GRAU o arco unitrio que corresponde a 1/360 da
circunferncia. Assim, uma circunferncia tem
360.
II. RADIANO: um arco unitrio cujo comprimento igual ao raio
da circunferncia que o contm. Como o comprimento da
circunferncia de raio r dado por C = 2r, ento, uma
circunferncia ter 2 rad.

De acordo com essas informaes, correto


afirmar que o comprimento (L) da rampa de:
a)

2 m

b) 2 2 m

d) 4 2 m
e) 5 2 m

c) 3 2 m

CONVERSO DE UNIDADES
Para transformarmos arcos de uma unidade para outra basta
fazer uma regra de trs simples com
180 .............

REGRA PRTICA
I.
Para transformarmos um arco de radiano para grau, basta
substituir rad por 180 e fazer as operaes.
II. Para transformarmos um arco de grau para radiano, basta
rad
multiplicarmos o arco por
e fazer as operaes.
180

SECRETARIA DE ESTADO DA EDUCAO

161

QUMICA
GEOMETRIA

AULA 02: TRIGONOMETRIA II

EXERCCIOS DE FIXAO

CICLO TRIGONOMTRICO
uma circunferncia orientada que possui as seguintes
caracteristicas:
Y
(0,1)
+
II Q
IQ
(1,0)

r=1

01. Determine o domnio, o conjunto imagem e o perodo das


funes:

a)

f(x) = 5 3sen(3x + 1)

b)

f(x) = 7 + cos(2x + 3)

(1,0)

III Q

IV Q

(0, 1)

raio: r = 1
origem dos arcos: (1, 0)
sentido positivo: anti-horrio
sentido negativo: horrio
1 quadrante: (0, /2)
2 quadrante: (/2, )
3 quadrante: (, 3/2)
4 quadrante: (3/2, 2)

ARCOS CNGRUOS so arcos que possuem a mesma


extremidade. Diferem um do outro pelo sentido percorrido ou
pelo nmero de voltas.
1 DETERMINAO POSITIVA o menor dos arcos cngruos
positivos.

FUNO SENO
a funo f : IR IR definida por f(x) = sen x

02. (CEFET-PI/2009) No incomum encontramos na natureza


fenmenos cujo comportamento pode ser descrito por funes
matemticas. Dentre estes fenmenos, os que tm
caractersticas cclicas ou de repetio continuada podem ser
expressas em funo do tempo por funes trigonomtricas
peridicas. Considere que o volume de ar, em litros, que tem no
pulmo durante a respirao do porquinho da ndia (Cavia
aparea) muito usada como cobaia de laboratrio, pode ser,
aproximadamente descrito pela expresso v(t) = 8 + 2sen(t)
onde t o tempo dado em minutos. Quando t = 0, o animal se
encontra em repouso sem inspirar nem expirar. Considerando
estas informaes, pode-se afirmar que, aproximadamente, o
volume mximo de ar que cabe no pulmo deste animal , em
litros:
a) 14
b) 8
c) 6
d) 10
e) 12

GRFICO

y
1

/2

3/2
2

-1
Observaes:
Dm(f) =
Im(f) = [1, 1]
Perodo: 2
Paridade: mpar

FUNO COSSENO
a funo f : IR IR definida por f(x) = cos x
GRFICO

y
1

/2

3/2

-1

Observaes:
Dm(f) =
Im(f) = [1, 1]
Perodo: 2
Paridade: par

SECRETARIA DE ESTADO DA EDUCAO

162

QUMICA
GEOMETRIA

AULA 03: TRIGONOMETRIA III

EXERCCIOS DE FIXAO

TANGENTE DE UM ARCO
A tangente do arco AM a medida do segmento AP onde P a
interseco da reta r com o prolongamento do segmento OM.
y

r
M

01. Determine o domnio, o conjunto imagem e o perodo da funo


f(x) = 3tg( + x).

P
+

tg AM = AP

A
0

x
-

Usando a definio vamos encontrar o tangente dos arcos:

3
0, , ,
e 2
2
2
y
/2

0
2

3/2

02. D o sinal do produto sen110.cos350.tg100.


Observe que a tangente varia de a +

FUNO TANGENTE
a funo f: D IR definida por f(x) = tg x, com

D = x IR / x k, k Z
2

GRFICO

2
0

3
2

Observaes:
Dm(f) = {x / x /2 + k, k }
Im(f) =
Perodo:
Paridade: mpar

OUTRAS FUNES
Funo secante: sec x =

1
, cos x 0
cos x

Funo cossecante: cossec x =


Funo cotangente: cotg x =

1
, sen x 0
sen x

1
, tg x 0
tg x

SECRETARIA DE ESTADO DA EDUCAO

163

QUMICA
GEOMETRIA

AULA 04: TRIGONOMETRIA IV

EXERCCIOS DE FIXAO

REDUO AO PRIMEIRO QUADRANTE


Reduzir um arco do 2, 3 ou 4 quadrante ao 1 quadrante
relacion-lo a um arco do 1 quadrante que tenha, em mdulo, o
valor do arco dado.

01. (ESCCAI-MG) O valor de y =


tg x = 3, :
a) 9

b) 27

sec x cos x
, sabendo que
cos sec x sen x

c) 3

d) 1

e) 5

+x

2 x

Observe que na reduo do 2, 3 ou 4 quadrante para o 1


quadrante, temos:
y
180

F
P

x
360
02. Calcule o valor de sen150.tg135 + cos270.tg50.

Do 2 Q para o 1 Q: o que FALTA para 180


Do 3 Q para o 1 Q: o que PASSA de 180
Do 4 Q para o 1 Q: o que FALTA para 360

REGRA PRTICA PARA REDUZIR AO 1 QUADRANTE:


Considerando x um arco do 1 quadrante, temos:
I.
Se o nmero de for inteiro (, 2, 3, ...) mantm-se a
funo trigonomtrica
3 5
II. Se o nmero de for fracionrio ( ,
,
, ...) troca2
2
2
se a funo pela co-funo.
Em ambos os casos, temos que observar o quadrante do arco
inicial para identificar o sinal da funo.

RELAES TRIGONOMTRICAS
So frmulas que relacionam as funes trigonomtricas. As
cinco primeiras so as fundamentais e as trs ltimas so as
decorrentes.
I.
sen x + cos x = 1, x
sen x
II. tg x =
, cos x 0
cos x
cos x
, sen x 0
sen x

III.

cotg x =

IV.

sec x =

V.

cossec x =

VI.

cotg x =

1
, cos x 0
cos x
1
, sen x 0
sen x

1
, sen x.cos x 0
tg x

VII. secx = 1 + tgx, cos x 0


VIII. cossecx = 1 + cotgx, sen x 0

SECRETARIA DE ESTADO DA EDUCAO

164

QUMICA
GEOMETRIA

Exerccios de fixao

AULA 05: GEOMETRIA PLANA

01 - (FUVEST) Na figura adiante, as retas r e s so


paralelas, o ngulo 1 mede 45 e o ngulo 3 :
a) 50
b) 55
c) 60
d) 80
e) 100

ngulo: a abertura formada por duas


semirretas de mesma origem

Tipos de ngulos
a) ngulo reto: o que mede 90
b) ngulo raso: o que mede 180
c) ngulo agudo: menor que 90
d) ngulo obtuso: maior que 90 e menor que 180
e) ngulo reentrante: maior que 180 e
menor que 360
f)
ngulos complementares
g) ngulos suplementares
h) ngulos explementares
i)
ngulos replementares
j)
ngulos consecutivos: possuem o mesmo vrtice
e um lado comum
k) ngulos adjacentes: possuem o mesmo
vrtice e um lado comum entre eles
ngulos opostos pelo mesmo vrtice:

02 - (UEL) Na figura a seguir, as medidas x, y e z so


diretamente
proporcionais aos nmeros 5, 20 e 25,
respectivamente.
O suplemento do ngulo de medida x tem medida
igual a
a) 144
b) 128
c) 116
d) 82
e) 54

ngulos formados por duas paralelas


cortadas por uma transversal

03 - (UNIRIO) As retas r1 e r2 so paralelas. O valor do


ngulo , apresentado na figura a seguir, :
a) 40
b) 45
c) 50
d) 65
e) 130

Bissetriz de um ngulo
uma semirreta com origem no vrtice do
ngulo e que o divide em dois ngulos de
mesma medida

Teorema de Tales
Se duas retas so transversais a um feixe de retas
paralelas, os
segmentos
determinados
nas
transversais so proporcionais.

SECRETARIA DE ESTADO DA EDUCAO

04 - (UNAERP) As retas r e s so interceptadas pela


transversal "t", conforme a figura. O valor de x para que r e s
seja, paralelas :
a) 20
b) 26
c) 28
d) 30
e) 35

165

QUMICA
GEOMETRIA

EXERCCIOS DE FIXAO

AULA 06 : GEOMETRIA PLANA

a)

Classificao:
Quanto aos lados

b)

01 - Um tringulo tem lados iguais a 10 cm e 17 cm. O maior


valor que o terceiro lado poder assumir, expresso por
um nmero natural, em cm, :
a) 26
b) 14
c) 28
d) 32
e) 14

TRINGULOS: Polgonos que possuem trs ngulos

Eqiltero (trs lados de medidas congruentes)


Issceles (dois lados de medidas congruentes)
Escaleno (trs lados de medidas diferentes)

Quanto aos ngulos

Obtusngulo (um ngulo obtuso)

a > b +

c
Acutngulo (trs ngulos agudos) a < b +
c

02 - O permetro de um tringulo 100 cm e um dos lados


vale 36 cm.
Um
tringulo
semelhante,
cujo
lado
homlogo ao lado conhecido de 27 cm, tem por permetro:
a) 63cm b) 72 cm
c) 72cm
d) 400/3 cm e)75 cm

Retngulo (um ngulo reto) a = b + c

ELEMENTOS IMPORTANTES NUM TRINGULO

Altura: um segmento perpendicular


que vai do vrtice at a reta suporte do lado
oposto.
Mediana: um segmento que vai do vrtice
at o meio do lado oposto.
Bissetriz: um segmento que vai do vrtice,
dividindo-o ao meio, at o lado oposto.
Mediatriz: a reta perpendicular a um lado
do tringulo, traada pelo seu ponto mdio.

03 - Na figura abaixo a 100 e b = 110. Quanto mede o ngulo x?

PONTOS NOTVEIS NO TRINGULO (BICO)


a) Baricentro: encontro das medianas
b) Incentro: encontro das bissetrizes
c) Circuncentro: encontro das mediatrizes
d) Ortocentro: encontro das alturas
OBSERVAES:
a) O INCENTRO o centro da circunferncia inscrita ao
tringulo
b) O CIRCUNCENTRO o centro da circunferncia
circunscrita ao tringulo
c) O BARICENTRO est a 1/3 do lado e 2/3 do vrtice
d) O ORTOCENTRO do tringulo retngulo coincide com o
vrtice do ngulo reto

CONGRUNCIA DE TRINGULOS

04 -(Cesgranrio) Na figura a seguir, AB = 8 cm, BC = 10 cm,


AD = 4 cm e o ponto O o centro da circunferncia. O
permetro do tringulo AOC mede, em cm:

Dois tringulos so congruentes se, e somente se, seus lados


e seus ngulos so ordenadamente congruentes

a) 36
b) 45
c) 48
d) 50
e) 54

SEMELHANA DE TRINGULOS
Dois tringulos so semelhantes se, e somente se, possuem
os trs ngulos ordenadamente congruentes e os lados
homlogos proporcionais

BASE MDIA
A base mdia de um tringulo o segmento da reta que liga os
pontos mdios de dois lados de um tringulo.
PROPRIEDADE
A base mdia de um tringulo paralelo ao terceiro lado e
mede a metade deste lado

SECRETARIA DE ESTADO DA EDUCAO

166

QUMICA
GEOMETRIA

Exerccios de fixao

AULA 07 : GEOMETRIA PLANA

01 - Determine a medida dos ngulos

QUADRILTEROS: Polgonos que possuem quatro lados

indicados:
QUADRILTEROS NOTVEIS
a) Trapzio: dois lados paralelos
b) Paralelogramo: lados opostos paralelos
c) Losango: quatro lados congruentes
d) Retngulo: quatro ngulos internos congruentes
e) Quadrado: quatro ngulos internos congruentes e quatro
lados congruentes

a)

ALGUMAS PROPRIEDADES
a)
Em todo paralelogramo os ngulos e os lados
opostos so congruentes
b) Em todo paralelogramo as diagonais se encontram
nos respectivos pontos mdios
c) Em todo losango as diagonais so perpendiculares
e bissetrizes dos ngulos internos
d) Em todo retngulo as diagonais so congruentes

b)

BASE MDIA DE UM TRAPZIO


o segmento de reta que liga os pontos mdios dos
lados no paralelos
02 - No paralelogramo abaixo, determine as medidas de x e y.

Propriedade
A base mdia de um trapzio paralela s bases e
tem medida igual a mdia aritmtica das medidas das
bases do trapzio
POLGONOS
a unio de n (n 3) segmentos de retas consecutivos
O polgono recebe o nome de acordo com o nmero de
lados
Alguns exemplos:
Tringulo (3 lados), quadriltero (4 lados), pentgono (5
lados), hexgono (6 lados), heptgo (7 lados), octgono
(8 lados), enegono (9 lados), decgono (10 lados),
icosgono (20 lados) .

03 - Determine as medidas dos quatro ngulos do


trapzio da figura abaixo:

CLASSIFICAO DOS POLGONOS


a)
Convexo:
quando
dados
dois
pontos
interiores, o segmento formado por eles est
totalmente contido no polgono.
b)
Cncavo:
quando
dados
dois
pontos
interiores,
o segmento formado por eles no est
totalmente contido no polgono.
c)

Regular: quando for eqiltero e

eqingulo

04 - Sabendo que x a medida da base maior, y a


medida da base menor, 5,5 cm a medida da base mdia
de um trapzio e que x - y = 5 cm, determine as medidas de
x e y.

OBSERVAO:
Destaca-se num polgono regular o segmento que vai do
centro ao meio do lado chamado de APTEMA.
Em qualquer polgono de n lados, tem-se

SECRETARIA DE ESTADO DA EDUCAO

167

QUMICA
GEOMETRIA

Exerccios de fixao

AULA 08 : GEOMETRIA PLANA

01 - (USP) No tringulo da figura a seguir, a circunferncia

inscrita tem raio 1 e T o ponto de tangncia. Ento o menor lado do


tringulo mede:

NGULOS NA CIRCUNFERNCIA

a) 3.

b) 20/7.

c) 7/2.

d) 9/2.

e) 30/7.

02 - (PUC) O ngulo x, na figura a seguir, mede:


a) 60

b) 80

c) 90

d) 100

e) 120

03 - (UFES) Nessa figura, BD um dimetro da


circunferncia circunscrita ao tringulo ABC, e os ngulos
ABD e AD medem, respectivamente, 20 e 85.
Assim sendo, o ngulo CBD mede
a) 25
b) 35
c) 30
d) 40
e) 550
RELAES MTRICAS NA CIRCUNFERNCIA

04 - (Fuvest) A medida do ngulo ADC inscrito na


circunferncia de centro O :
a) 125
b) 110
c) 120

SECRETARIA DE ESTADO DA EDUCAO

168

d) 100

e) 135

QUMICA
GEOMETRIA

Exerccios de fixao

AULA 09 : GEOMETRIA PLANA

01 - Um cavalo se encontra preso num cercado de pastagem,


cuja forma um quadrado, com lado medindo 50m. Ele est
amarrado a uma corda de 40m que est fixada num dos
cantos do quadrado. Considerando = 3,14, calcule a rea,
em metros quadrados, da regio do cercado que o cavalo
no conseguir alcanar, porque est amarrado.
a) 1244.
b) 1256.
c) 1422.
d) 1424.
e) 1444.

REAS DAS FGURAS PANAS

02 - Na figura, ABCD um quadrado cujo lado mede a. Um dos


arcos est contido na circunferncia de centro C e raio a, e o outro
uma semicircunferncia de centro no ponto mdio de BC e de
dimetro a. A rea da regio hachurada :
a) Um quarto da rea do crculo de raio a.
b) Um oitavo da rea do crculo de raio a.
c) O dobro da rea do crculo de raio a/2.
d) Igual rea do crculo de raio a/2.
e) A metade da rea do quadrado.

03 - Na figura ao lado, se o dimetro do crculo mede 8 cm


e os raios dos semicrculos medem2 cm, a rea e o
permetro da regio hachurada valem, respectivamente,
a) 8 cm2 e 8 cm
b) 16 cm2 e 8 cm
c) 8 cm2 e 8 cm
d) 16 cm2 e 8

cm

e) 16 cm e 8 cm

SECRETARIA DE ESTADO DA EDUCAO

169

QUMICA

GEOMETRIA

AULA 10 : GEOMETRIA ESPACIAL

EXERCCIOS DE FIXAO
01 - (UFPB) A caracterstica de Euler-Poincar (P) de um
poliedro P definida por (P) = V - A + F, onde V, A
e
F
so, respectivamente, os nmeros de vrtices,
arestas e faces de P. Sendo assim, a caracterstica de EulerPoincar de uma pirmide de base triangular :
a)-2
b)0
c)2
d)-1
e)1

POLIEDROS
DEFINIO

02 -(PUC - PR)Um poliedro convexo formado por


faces quadrangulares e 4 faces triangulares. A soma dos
ngulos de todas as faces igual a 12 retos. Qual o nmero de
arestas desse poliedro?
a) 8
b) 6
c) 4
d) 2
e) 1

POLIEDROS CONVEXOS
DEFINIO

03 - (UFC) Um poliedro convexo s tem faces


triangulares e quadrangulares. Se ele tem 20 arestas e 10
vrtices, ento, o nmero de faces triangulares :
a) 12
b) 11
c) 10
d) 9
e) 8

POLIEDROS CNCAVO
DEFINIO

POLIEDROS REGULARES

04 - (Fuvest) O nmero de faces triangulares de uma pirmide


11. Pode-se, ento, afirmar que esta pirmide possui
a) 33 vrtices e 22 arestas.
b) 12 vrtices e 11 arestas.
c) 22 vrtices e 11 arestas.
d) 11 vrtices e 22 arestas.
e) 12 vrtices e 22 arestas.

RELAO DE EULLER

SOMA DOS ANGULOS DAS FACES

SECRETARIA DE ESTADO DA EDUCAO

170

QUMICA
GEOMETRIA

EXERCCIOS DE FIXAO

AULA 11 : GEOMETRIA ESPACIAL

PRISMA
Sejam
e dois planos paralelos e distintos e ABCDE
uma
segmentos de reta MN paralelos com M um ponto da
de todos os segmentos MN forma um slido chamado de
PRISMA.

01 - No cubo da figura a seguir, as arestas medem 4cm.


Quanto mede a diagonal AB ?
a) 43 cm
b) 23 cm
c) 42 cm
d) 22 cm
e) 2 cm

ELEMENTOS DE UM PRISMA

02 - (UFPB - 97) A diagonal de um cubo mede 12 cm.


Qual o volume desse cubo?

CLASSIFICAO
a) De acordo com o nmero de arestas das bases os
prismas so denominados triangulares (bases tringulos),
quadrangulares (bases quadrilteros), pentagonais (bases
pentgonos) e assim por diante.
b) Prisma reto aquele em que as arestas laterais so
perpendiculares s bases.
03 - (UFPB - 99) Os algarismos 1 e 9 que compem o
nmero 1999, foram confeccionados emendando-se pequenos
cubos de madeira de aresta 0,10 m. Determine o volume
total, em m3, da madeira utilizada na confeco do nmero
1999.

c) Prisma oblquo aquele em que as arestas laterais so


oblquas s bases.
d) Prisma regular aquele em que as bases so polgonos
regulares e reto

DIAGONAL, REA E VOLUME DE UM PARALELEPPEDO


A diagonal (D), a rea total (AT), e o volume (V),
de um paraleleppedo reto-retngulo de dimenses a, b e
c so dados por:

AT = 2(ab + ac + bc)

V = a . b. c

Observao: o CUBO um paraleleppedo regular, ou seja,

d =...........

= ...........

= ...............

SECRETARIA DE ESTADO DA EDUCAO

171

QUMICA
GEOMETRIA

AULA 12 : GEOMETRIA ESPACIAL

EXERCCIOS DE FIXAO

CILINDRO
01 - (Cesgranrio) Um recipiente com a forma de um cilindro
reto, cujo dimetro da base mede 40 cm e altura 100/
cm,
armazena um certo lquido, que ocupa 40% de sua
capacidade. O volume do lquido contido nesse recipiente ,
em litros, aproximadamente, igual a:
a) 16
b) 18
c) 20
d) 3
e) 40

ELEMENTOS DO CILINDRO

02 - (UFRS) Um pedao de cano de 30 cm de comprimento e


10cm de dimetro interno encontra-se na posio vertical e
possui a base inferior vedada. Colocando-se dois litros de gua
em seu interior, e gua.
a) ultrapassa o meio do cano.
b) transborda.
c) no chega ao meio do cano.
d) enche o cano at a borda.
e) atinge exatamente o meio do cano.

CLASSIFICAO

03 - (UFPB - 2003) Depois de desistir de retirar a


pipa do poste, Joo foi jogar futebol no quintal da casa.
Ao chutar a bola com muita fora, fez com que a
mesma casse num reservatrio de gua com a forma
de um cilindro circular reto, cujo dimetro de 96 cm.
Maria percebeu que exatamente a metade da bola ficou
submersa, o que elevou o nvel da gua do reservatrio
em 0,5 cm (ver desenho). O raio dessa bola :

SECO MERIDIANA

a)10 cm
b) 12 cm
c) 14 cm
d)11 cm
e) 13 cm

SECO TRANSVERSAL

04 - (UFPB - 2002) O Sr. Martins foi encarregado de pintar a


lateral de uma caixa dgua cilndrica, com 6 m de raio e
altura de 10 m. Devido ao mau tempo, conseguiu, apenas,
pintar a parte sombreada mostrada na figura ao lado. Assim
sendo, desse trabalho ele executou somente:

a)5%

CILINDRO EQUILTERO

b)15%

c)25%

REAS E VOLUME

d)10%
e) 20%

SECRETARIA DE ESTADO DA EDUCAO

172

QUMICA
GEOMETRIA

AULA 13 : GEOMETRIA ESPACIAL

EXERCCIOS DE FIXAO

01. (UFRGS) Um cone circular reto tal que cada seo obtida pela
interseo de um plano que passa por seu vrtice e pelo centro da
sua base um tringulo retngulo de catetos iguais. Se cortarmos
esse cone ao longo de uma geratriz, abrindo e planificando sua
superfcie lateral, ser obtido um setor circular cujo ngulo central
tem medida x. Ento:

CONE

ELEMENTOS DO CONE

a)

x 180

b)

180 x 200

c)

200 x 220

d)

220 x 240

e)

x 240

02. Uma mistura de leite batido com sorvete servida em um copo,


como na figura. Se na parte superior do copo h uma camada de
espuma de 4cm de altura, ento a porcentagem do volume do copo
ocupada pela espuma est melhor aproximada na alternativa:

CLASSIFICAO

a) 65%
b) 60%
c) 50%
d) 45%
e) 70%

SECO MERIDIANA

03. (FUVEST) Deseja-se construir um cone circular reto com 4cm de


raio da base e 3cm de altura. Para isso, recorta-se, em cartolina, um
setor circular para a superfcie lateral e um crculo para a base. A
medida do ngulo central do setor circular :

a) 144
b) 192
c) 240
d) 288
e) 336

SECO TRANSVERSAL

04 Na figura, a base do cone reto est inscrita na face do cubo.


Supondo =3, se a rea total do cubo 54, ento o volume do cone
:

CONE EQUILTERO

a) 81/2
b) 27/2
c) 9/4
d) 27/4
e) 81/4

REAS E VOLUME

SECRETARIA DE ESTADO DA EDUCAO

173

QUMICA
GEOMETRIA

AULA 14 : GEOMETRIA ESPACIAL

EXERCCIOS DE FIXAO
01. Calcule a medida da rea lateral de uma pirmide
quadrangular regular, sabendo que a rea da base
mede 64m e a altura da pirmide igual a uma das
diagonais da base.

PIRMIDE

02. (UFMG) Em uma indstria de velas, a parafina


armazenada em caixas cbicas, cujo lado mede a.
Depois de derretida, a parafina derramada em
moldes em formato de pirmides de base quadrada,
cuja altura e cuja aresta da base medem, cada uma,

ELEMENTOS DA PIRMIDE

a
. Considerando-se essas informaes, CORRETO
2

afirmar que, com a parafina armazenada em apenas


uma dessas caixas, enche-se um total de:

CLASSIFICAO

a) 6 moldes
b) 8 moldes
c) 24 moldes
d) 32 moldes

SECO MERIDIANA

03. O volume de uma pirmide triangular regular


27m. Calcule a aresta da base, sabendo que a altura
igual ao semipermetro da base.

SECO TRANSVERSAL

04. Uma pirmide tem por base um retngulo cujas


somas das dimenses vale 34cm sendo uma delas

REAS E VOLUME

5
da outra. Determine as dimenses da base e a rea
12
total da pirmide, sabendo que a sua altura mede 5cm
e que a sua projeo sobre a base o ponto de
interseco das diagonais da base.

SECRETARIA DE ESTADO DA EDUCAO

174

QUMICA
GEOMETRIA

AULA 15 : GEOMETRIA ESPACIAL

EXERCCIOS DE FIXAO

ESFERA
01. Calcule o volume de uma esfera inscrita num cone
equiltero cujo volume

72 3 cm 3 .

ESFERA

02. Uma esfera de raio 11cm seccionada por um


plano distante 5cm do seu centro. Calcular as
distncias polares.

SECO TRANSVERSAL

03. Uma esfera seccionada por um plano distante 8


cm de seu centro. Calcule as distncias polares,
sabendo-se que o raio da esfera 10cm.

REA DA SUPERFCIE

VOLUME

04. Calcule a rea da esfera circunscrita ao cone reto


de raio 6cm e altura 18 cm.
REA DO FUSO

05. Se duplicarmos o raio de uma esfera, o que


acontece com o volume? E com a rea da superfcie?

VOLUME DA CUNHA

SECRETARIA DE ESTADO DA EDUCAO

175

QUMICA
GEOMETRIA

AULA 16 : GEOMETRIA ANALTICA

EXERCCIOS DE FIXAO

PONTO

01. (PUC) Os pontos A(3,1), B(4,-2) e C(x,7) so colineares.


Determine o valor de x.

02. (UNESP)Um tringulo tem vrtices P = (2,1), Q = (2,5) e


R = (x,4). Sabendo-se que a rea do tringulo 20, calcule a
abscissa do ponto R.
a) 8 ou 12
b) 9 ou -12
c) 10 ou 9
d) 11 ou -8
e) 12 ou -8

PONTO MDIO
03. (Cesgranrio) A rea do tringulo, cujos vrtices so (1,
2), (3, 4) e (4, -1), igual a:
a) 6.
b) 8.
c) 9.
d) 10.
e) 12.

BARICENTRO

04. UERJ) No sistema de coordenadas cartesianas a seguir,


est representado o tringulo ABC.

REA DO TRINGULO

CONDIO DE ALINHAMENTO
Em relao a esse tringulo, calcule a sua rea.

VOLUME DA CUNHA

SECRETARIA DE ESTADO DA EDUCAO

176

QUMICA
GEOMETRIA

AULA 17: GEOMETRIA ANALTICA

EXERCCIOS DE FIXAO

RETA
01. Se um trimgulo tem como vrtices os pontos
A(2,1) ;B(-2,-4) e C(0,2) determine a equao da reta
suporte da altura relativa ao lado AB do tringulo.

EQUAO GERAL
02. (Unemat 2010) Dada a equao de reta (s): 2x - y
+1 = 0 , a equao de reta paralela a ,que passa pelo
ponto P(1,1) ser:
a) 2x - y = 0
b) 2x + y +1 = 0
c) 2x + y -1 = 0
d) 2x - y -1 = 0
e) 2x - y + 2 = 0

COEFICIENTE ANGULAR

03. (Fatec) Seja a reta r, de equao y=(x/2) +17.


Das equaes a seguir, a que representa uma reta
paralela a r :
a) 2y = (x/2) + 10
b) 2y = - 2x + 5
c) 2y = x + 12
d) y = - 2x + 5
e) y = x + 34

TIPOS DE EQUAES

POSIES RELATIVAS

DISTANCIA DE UM PONTO A UMA RETA

04. (Fgv) As retas de equaes y = - x - 1 e


y = [(-a + 1)/(a - 2)] x + 12 so perpendiculares.
O valor de a :
a) 2
b) 1/2
c) 1
d) -2
e) 3/2

EQUAO DA CINCUNFERNCIA

SECRETARIA DE ESTADO DA EDUCAO

177

QUMICA
GEOMETRIA

AULA 18: MATEMTICA BSICA

EXERCCIOS DE FIXAO
1. (Uerj 2015) Na imagem da etiqueta, informa-se o valor a ser pago
por 0,256 kg de peito de peru.

AULA 07 : RAZES E PROPORO


Definio
Chama-se razo entre os valores a e b (b 0) o quociente de a por
b.
Exemplos - Razo entre
a) 3 e 4 3/4 ou 3 : 4
b) 20m e 4s 20m 1 4s = 5m/s; razo chamada de velocidade
c) 8g e 10cm3 8g / 10cm3 = 0,8 g/cm3; razo chamada de
densidade
Propores
Definio
Proporo a equivalncia entre duas razes. Se os nmeros a, b, c
e d formam, nesta ordem, uma proporo, podemos escrever

ac
b d
Propriedade fundamental das propores
Numa proporo, o produto dos meios igual ao produto dos
extremos.
Se
Exemplo:

b) 25 cm 40 cm.
c) 30 cm 40 cm.

3 2
x 1 7

d) 30 cm 42 cm.
32 cm 44 cm.
e)

Aplicando a propriedade fundamental, temos


2(x + 1) = 3.7

2x + 2 = 21

02. (G1 - ifsp 2014) A fotografia uma forma de representao


artstica. Um fotgrafo deseja ampliar uma fotografia sem a distorcer,
isto , pretende produzir uma imagem semelhante original. Se a
fotografia original possui forma retangular de dimenses
12 cm 16 cm e o fotgrafo pretende utilizar uma constante de
proporcionalidade k = 2,5, ento as dimenses da fotografia
ampliada sero
a) 25 cm 42 cm.

a c ento a d = b c
b d

Calcular o valor de x, em

O valor, em reais, de um quilograma desse produto igual a:


a) 25,60
b) 32,76
c) 40,00
d) 50,00
e) 48,00

x = 19/2

Grandezas direta ou inversamente proporcionais


Vamos considerar que as variveis x e y representem as medidas de
duas grandezas, que podem ser relacionadas entre si. Chamando de
k uma constante real diferente de zero, temos
x diretamente proporcional a y se, e somente se, x = k

x = ky
x inversamente proporcional a y se, e somente se, xy = k

03. (G1 - utfpr 2014) Gabriela gasta por semana R$ 55,00 com
nibus, 3/5 dessa quantia com lanche e 1/11 em xerox. O gasto total
semanal de Gabriela de R$:
a) 58,00.
b) 62,00.
c) 70,00.
d) 81,00.
e) 93,00.

x= x
y

EXEMPLO
Um prmio de R$ 120 000,00 de uma loteria deve ser dividido
proporcional a R$ 2,00; R$ 3,00; R$ 5,00, quantias que cada jogador
apostou. Obter o valor a ser recebido por jogador.
Chamando x, y e z o que cada jogador vai receber, respectivamente,
temos
RESOLUO:

SECRETARIA DE ESTADO DA EDUCAO

178

04. (G1 - cftmg 2014) Seu Wagner, personagem do livro A mocinha


do Mercado Central, contratou uma equipe de artesos para fazer
um lote de bijuterias num prazo de 10 horas. Entretanto, 4
integrantes dessa equipe no puderam comparecer e o servio
demorou 5 horas a mais. Nessa situao, o nmero inicial de
artesos contratados era igual a
a) 8.
b) 12.
c) 16.
d) 20.
e)25

QUMICA
GEOMETRIA

AULA 01 CONJUNTOS

EXERCCIOS DE FIXAO

Conceito primitivo; no necessita, portanto, de definio.


Exemplo: conjunto dos nmeros naturais pares:
P = { 0, 2, 4, 6, 8, 10, 12, ... }.
Esta forma de representar um conjunto, pela enumerao dos seus
elementos, chama-se citao dos elementos (forma de listagem).
O mesmo conjunto tambm poderia ser representado por uma
propriedade dos seus elementos, ou seja, sendo x um elemento
qualquer do conjunto P acima, poderamos escrever:
P = {x | x par e natural} = {0, 2, 4,...}.

01. Uma Instituio de Ensino Superior oferece os cursos A e B. Em


seu processo seletivo o candidato pode optar por inscrever-se nos
dois cursos ou apenas em um curso. Ao final, o nmero de inscries
por curso e o nmero total de candidatos inscritos pode ser
observado no quadro que segue:
N de Inscries
N de Inscries
N Total de
no Curso A
no Curso B
Candidatos Inscritos
480
392
560
Com base nas informaes acima e nas possibilidades de inscries,
pode se afirmar que o nmero de candidatos que optaram por
inscrever-se somente no curso A foi
(A) 80.
(B) 168.
(C) 312.
(D) 480.
(E) 560.

Relao de Pertinncia
Sendo x um elemento do conjunto A, escrevemos x A, onde o
smbolo significa "pertence a".
Sendo y um elemento que no pertence ao conjunto A, indicamos
esse fato com a notao y A.
O conjunto que no possui elementos denominado conjunto vazio
e representado por .
Subconjunto
Se todo elemento de um conjunto A tambm pertence a um
conjunto B, ento dizemos que A subconjunto de B, ou ainda, A
est contido em B, e indicamos por A B.
Notas:
a) Todo conjunto subconjunto de si prprio (A A);
b) O conjunto vazio subconjunto de qualquer conjunto ( A);
c) Se um conjunto A possui m elementos ento ele possui 2m
subconjuntos;
d) O conjunto formado por todos os subconjuntos de um conjunto A
denominado conjunto das partes de A e indicado por P(A).
Assim, se A = {c, d}, o conjunto das partes de A dado por P(A) =
{ , {c}, {d}, {c,d}}.
Operaes com Conjuntos
i) Unio ( ): Dados os conjuntos A e B, define-se o conjunto unio
A B = {x/ x A ou x B}.
Exemplo: {0, 1, 3} {3, 4, 5} = {0, 1, 3, 4, 5}.
ii) Interseo ( ): Dados os conjuntos A e B, define-se o conjunto
interseo A B = {x/ x A e x B}.
Exemplo: {0, 2, 4, 5} {4, 6, 7} = {4}.
Observao: Se A B = , ento dizemos que os conjuntos A e B
so Disjuntos.

02. Em uma reserva ambiental, habitam 40 predadores que tm


predileo por presas dos tipos A, B ou por nenhuma delas.
Sabendo-se que desses predadores 18 preferem presas do tipo A,
22 preferem do tipo B e 6 preferem dos dois tipos, a quantidade de
predadores que no tm predileo por nenhum dos dois tipos de
presas
(A) 3.
(B) 4.
(C) 5.
(D) 6.
(E) 7.
03. Um grupo de estudantes resolveu fazer uma pesquisa sobre as
preferncias dos alunos quanto ao cardpio do Restaurante
Universitrio. Nove alunos optaram somente por carne de frango, 3
somente por peixes, 7 por carne bovina e frango, 9 por peixe e carne
bovina e 4 pelos trs tipos de carne. Considerando que 20 alunos
manifestaram-se vegetarianos, 36 no optaram por carne bovina e
42 no optaram por peixe, o nmero de alunos entrevistados
(A) 38.
(B) 42.
(C) 58.
(D) 62.
(E) 78.
04. Num homicdio praticado na Rua X, a polcia fez as seguintes
anotaes, no boletim de ocorrncia, sobre as pessoas encontradas
no local do crime:
I. Havia 5 mulheres.
II. 5 pessoas usavam culos.
III. 4 homens no usavam culos.
IV. 2 mulheres usavam culos.
Considerando que todas as pessoas encontradas no local do crime
so suspeitas, ento quantos so os suspeitos?
(A) 8
(B) 9
(C) 10
(D) 11
(E) 12
GABARITO:
01. B

02. D

03. C

04. E

iii) Diferena: A B = {x / x A e x B}.


Observe que os elementos da diferena so aqueles que pertencem
ao primeiro conjunto, mas no pertencem ao segundo.
Exemplos:
{0, 5, 7} {0, 7, 3} = {5}.
{1, 2, 3, 4, 5} {1, 2, 3} = {4, 5}.
Obs.: A B B A (a diferena de conjuntos no uma operao
comutativa).
iv) Complementar de um conjunto
Trata-se de um caso particular da diferena entre dois conjuntos.
Assim , que dados dois conjuntos A e B, com a condio de que B
A, a diferena A B chama-se, neste caso, complementar de B
em relao a A. Simbologia: CAB A B .
Caso particular: O complementar de A em relao ao conjunto
universo U, ou seja, U A indicado pelo smbolo A'.

Nmero de Elementos da Unio de

a) 2 conjuntos: n(AB) = n(A) + n(B) n(AB)


b) 3 conjuntos: n(ABC) = n(A) + n(B) + n(C) n(AB) n(AC)
n(BC) + n(ABC)

SECRETARIA DE ESTADO DA EDUCAO

179

QUMICA
GEOMETRIA

AULA 02 CONJUNTOS NUMRICOS

EXERCCIOS DE FIXAO

Entendemos por conjunto numrico, qualquer conjunto cujos


elementos so nmeros. Existem infinitos conjuntos numricos, entre
os quais, os chamados conjuntos numricos fundamentais, a saber:

01. Considere que a tbua abaixo define uma operao , sobre o


conjunto E = {1, 2, 3, 4, 5}

Conjunto dos nmeros naturais


N = {0, 1, 2, 3, 4, 5, 6, ...}

Conjunto dos nmeros inteiros


Z = {..., -4,-3,-2,-1,0,1,2,3,...}
Nota: evidente que N Z.

Mnimo Mltiplo Comum (m.m.c.) de dois ou mais nmeros dado


atravs do produto entre os fatores comuns e no comuns tomados
nos maiores expoentes.

Assim, por exemplo, 5 ( 4 3) = 5 5 = 2


Nessas condies, se x um elemento de E, tal que [(4 3) (
2 5)] x = 1, ento o valor de x
(A) 1.
(B) 2.
(C) 3.
(D) 4.
(E) 5.

Mximo Divisor Comum (m.d.c.) de dois ou mais nmeros dado


atravs do produto entre os fatores comuns tomados nos menores
expoentes.

Conjunto dos nmeros racionais


Q = {x | x = p/q com p Z, q Z e q 0}. (o smbolo | l-se
como "tal que").

Temos ento que nmero racional aquele que pode ser escrito na
forma de uma frao p/q onde p e q so nmeros inteiros, com o
denominador diferente de zero.
Lembre-se que no existe diviso por zero!.
So exemplos de nmeros racionais: 2/3, -3/7, 0,001=1/1000,
0,75=3/4, 0,333... = 1/3, 7 = 7/1, etc.
Notas:
a) evidente que N Z Q.
b) toda dzima peridica um nmero racional, pois sempre
possvel escrever uma dzima peridica na forma de uma frao.

02. Considere a seguinte sequncia de clculos:


11 = 121
111 = 12 321
1 111 = 1234 321
11 111 = 123 454 321
A soma dos algarismos do nmero que se obtm calculando 111
111 111
(A) um quadrado perfeito.
(D) divisvel por 5.
(B) maior que 100.
(E) um nmero primo
(C) menor que 70.

Exemplo: 4/9 = 0,4444... (dzima peridica simples)

Conjunto dos nmeros irracionais


I = Q' = {x | x uma dzima no peridica}. (o smbolo | lse como "tal que").
Exemplos de nmeros irracionais:
= 3,1415926... (nmero pi = razo entre o comprimento
de qualquer circunferncia e o seu dimetro)
2,01001000100001... (dzima no peridica)

03. Sendo x e y nmeros naturais, o resultado da diviso de x por y,


obtido com auxlio de uma calculadora, foi a dzima peridica
3,333...
Dividindo-se y por x nessa calculadora, o resultado obtido ser igual
a
(A) 1,111...
(B) 0,9
(C) 0,333...
(D) 0,3
(E) 0,111...

Conjunto dos nmeros reais


R = { x | x racional ou x irracional }.

Notas:
a) bvio que N Z Q R
b) um nmero real racional ou irracional; no existe outra
hiptese!
c) Q I =

SECRETARIA DE ESTADO DA EDUCAO

04. Em Papua Nova Guin, a contagem feita por meio de uma


correspondncia biunvoca (correspondncia um a um), entre os
objetos contados e os dedos de uma pessoa. Um papua, por
exemplo, exprime certo nmero n como quatro homens mortos, dois
braos completos, uma perna completa e quatro dedos.
No sistema decimal de numerao, o numeral que representa esse
nmero n
(A) 129.
(B) 109.
(C) 104.
(D) 99.
(E) 94.

180

QUMICA
LGEBRA

AULA 03 FUNO
As funes nada mais so que um tipo particular de relao que
possuem uma propriedade especfica.
Para iniciarmos o estudo das funes vamos comear analisando a
relao R1 = {(-3, 9), (0, 0), (3, 9)}, cujo diagrama de flechas pode
ser visto abaixo:

Ele no representa uma funo de A em B, pois o elemento 2 do


conjunto A possui duas imagens, 8 e 8, o que contraria o conceito
de funo.
Se apenas 8 ou 8 recebessem um flechada de 2, a sim teramos
uma funo.
Agora vejamos este outro diagrama de flechas a seguir:

Observe que todos os elementos do conjunto A possuem uma


flecha em direo a um nico elemento do conjunto B.
Em outras palavras, no h no conjunto A qualquer elemento que
no esteja associado a um elemento do conjunto B e os elementos
de A esto associados a apenas um elemento de B.
Por possuir tal propriedade, dizemos que esta relao uma funo
f de A em B representada por f: A B.
Domnio da Funo
Ao conjunto A damos o nome de domnio da funo.
O domnio o conjunto de partida. Ele composto de todos os
elementos do conjunto de partida.
Neste nosso exemplo o domnio da funo f representado por
D(f) = {-3, 0, 3}, ou seja, o domnio desta funo contm todos os
elementos do conjunto A.
Contradomnio da Funo
Ao conjunto B damos o nome de contradomnio da funo.
O contradomnio o conjunto de chegada. Ele composto de todos
os elementos do conjunto de chegada, ou ainda, formado por
todos os possveis resultados a serem obtidos.
Em nosso exemplo o contradomnio da funo f representado por
CD(f) = {0, 9, 18}, isto , o contradomnio desta funo contm
todos os elementos do conjunto B.
Imagem da Funo
A imagem da funo dependendo do caso o prprio
contradomnio, ou ento um subconjunto seu.
Os elementos do conjunto imagem so todos os elementos do
contradomnio que esto associados a algum elemento do domnio,
ou seja, formado pelos resultados obtidos aps aplicao da lei
nos elementos do Domnio. No exemplo que estamos utilizando o
conjunto imagem representado por Im(f) = {0, 9}, pois 0 e 9 so
todos os elementos do CD(f) que esto associados a algum
elemento do D(f).
Nesta funo exemplo o conjunto imagem um subconjunto do
contradomnio [Im(f) CD(f)], pois o elemento 18 de B no est
contido no conjunto imagem, por no estar associado a nenhum
elemento do domnio.
Definio de uma Funo
Esta funo f de A em B, f: A B, definida como:
f: A B, f(x) = x2, ou ainda, f: A B, y = x2.
Veja tambm que representamos f(x) ou y em funo de x. A
varivel f(x) ou y chamada de varivel dependente, pois depende
de x, j a varivel x chamada de varivel independente, pois
independentemente de y, pode representar qualquer elemento do
domnio.
A definio da funo leva em conta tanto o domnio quanto do
contradomnio, relacionando-os. O conjunto imagem Im(f), depende
no s da regra de associao, no caso f(x) = x2, como tambm do
D(f) e do CD(f).
Exemplos de Relao que no Funo
Observe o diagrama de flechas abaixo:

SECRETARIA DE ESTADO DA EDUCAO

181

Veja que no h nenhum elemento do domnio que fleche mais de


um elemento do contradomnio, mas ainda assim no estamos
diante de uma funo. Por qu?
Simplesmente porque o elemento 5 do conjunto A no possui uma
imagem em B.
Observe agora o seguinte grfico no plano cartesiano:

Ele representa ou no uma funo?


Sabemos que em uma funo cada elemento x do domnio deve
estar relacionado a um nico elemento y do contradomnio, ou seja,
deve possuir uma nica imagem.
Note, porm, que neste grfico os pontos (5, 1) e (5, 4), possuem a
mesma abscissa, o que significa dizer que o elemento 5 do domnio
possui duas imagens, ele flecha tanto o elemento 1, quanto o
elemento 4 do contradomnio, portanto tal grfico no representa
uma funo.
Em resumo, levando-se em conta o domnio e o contradomnio da
relao, se no grfico for possvel traar uma reta paralela ao eixo
das ordenadas que passe por mais de um ponto do grfico, ou
ainda que no passe por nenhum dos seus pontos, ento
estaremos diante de um grfico que no representa uma funo.
Zeros ou Razes de uma Funo
Olhe o grfico da funo a seguir e perceba que alguns dos seus
pontos esto localizados sobre o eixo das abscissas.
A abscissa de cada um destes pontos denominada zero da funo
ou raiz da funo.

Todo elemento do domnio da funo que tem como imagem o


elemento 0, uma raiz da funo.
Os elementos do domnio que anulam a funo so as suas razes,
isto significa dizer que dependendo da funo, ela pode no possuir
razes reais, pois pode no existir no seu domnio nenhum elemento
que a anule e sendo assim o seu grfico nunca intercepta o eixo x,
assim como tambm pode possuir infinitas razes reais, pois o seu
grfico intercepta o eixo x infinitas vezes, j que podem existir
infinitos elementos do seu domnio que tornem a funo nula.

QUMICA
LGEBRA

AULA 04 FUNO COMPOSTA E FUNO INVERSA

EXERCCIOS DE FIXAO
01. Na figura abaixo, est representado o grfico de uma funo
f : [2 , 2] R.

I. FUNO COMPOSTA
Chama-se funo composta (ou funo de funo) funo obtida
substituindo-se a varivel independente x, por uma funo.
A funo composta pode ser entendida pela determinao de uma
terceira funo C, formada pela juno das funes A e B.
Matematicamente falando, temos que f A B e g B C,
denomina a formao da funo composta de g com f, h A C.
Dizemos funo g composta com a funo f, representada por gof.
Simbologia: gof (x) = g(f(x)).

O nmero de solues da equao f(x) = 2


(A) 1.
(B) 2.
(D) 4.
(E) 5.

Veja o esquema a seguir:


(C) 3.

02. No grfico a seguir, temos o nvel da gua armazenada em uma


barragem, ao longo de trs anos.

Obs.: Atente para o fato de que fog gof, ou seja, a operao


"composio de funes" no comutativa.

O nvel de 40m foi atingido quantas vezes neste perodo?


(A) 1
(B) 2
(C) 3
(D) 4
(E) 5
03. Um leitor enviou a uma revista a seguinte anlise de um livro
recm lanado, de 400 pginas:
O livro eletrizante, muito envolvente mesmo! A cada pgina
terminada, mais rpido eu lia a prxima! No conseguia parar!
Dentre os grficos apresentados abaixo, o nico que poderia
representar o nmero de pginas lidas pelo leitor (N) em funo do
tempo (t) de modo a refletir corretamente a anlise feita

Exemplo:
Dadas as funes f(x) = 2x + 3 e g(x) = 5x, pede-se determinar
gof(x) e fog(x).
Teremos:
gof(x) = g[f(x)] = g(2x + 3) = 5(2x + 3) = 10x + 15
fog(x) = f[g(x)] = f(5x) = 2(5x) + 3 = 10x + 3
Observe que fog gof.
II. FUNO INVERSA
Dada uma funo f: A B, se f bijetora, ento define-se a funo
inversa f-1 como sendo a funo de B em A , tal que f -1 (y) = x . Veja
a representao a seguir:

bvio ento que:


a) para obter a funo inversa, basta permutar as variveis x e y.
b) o domnio de f -1 igual ao conjunto imagem de f.
c) o conjunto imagem de f -1 igual ao domnio de f.
d) os grficos de f e de f -1 so curvas simtricas em relao reta y
= x, ou seja, bissetriz do primeiro quadrante.
De acordo com a definio:

GABARITO:
01. D

02. B

03. B

SECRETARIA DE ESTADO DA EDUCAO

04. B

182

QUMICA

LGEBRA

Exemplo:
Determine a INVERSA da funo definida por y = 2x + 3.
Permutando as variveis x e y, fica: x = 2y + 3
Explicitando y em funo de x, vem:
2y = x 3 y = (x 3) / 2, que define a funo inversa da funo
dada.

(D)

A funo f ( x ) possui inversa, cujo grfico est


representado na figura a seguir.

(E)

A funo f ( x ) possui inversa, cujo grfico est


representado na figura a seguir.

Os grficos a seguir representam uma funo e a sua inversa.


Observe que as curvas representativas das funes f e f -1, so
simtricas em relao reta y = x, bissetriz do primeiro e terceiro
quadrantes.

EXERCCIOS DE FIXAO
01. Estudando a viabilidade de uma campanha de vacinao, os
tcnicos da Secretria da Sade de um municpio verificaram que o
custo da vacinao de x por cento da populao local era de,
aproximadamente, y = 300x / (400 x) milhares de reais. Nessa
expresso, escrevendo-se x em funo de y, obtm-se x igual a
(A) 4/3.
(B) 300y / (400 y).
(C) 300y / (400 + y).
(D) 400y / (300 y).
(E) 400y / (300 + y).

03. A funo cujo grfico est representado na figura 1 a seguir tem


inversa.

02. Na figura abaixo est representado o grfico de uma funo


f : [ 3 , 3] [1 , 5 ] .
O grfico de sua inversa :

verdade que
(A)

A funo f ( x ) no possui inversa.

(B)

A funo f ( x ) possui inversa, cujo grfico est


representado na figura a seguir.

04. Considere as funes de em definidas por f(x) = x + 1 e g(x)


= x2. A soma das razes da equao f(g(x)) + g(f(x)) 14 = 0,
(A) 3.
(B) 1.
(C) 2.
(D) 7.
(E) 2.
GABARITO:
01. E

(C)

02. E

03. D

04. B

A funo f ( x ) possui inversa, cujo grfico est


representado na figura a seguir.

SECRETARIA DE ESTADO DA EDUCAO

183

QUMICA

LGEBRA

AULA 05 FUNO DO 1 GRAU (FUNO AFIM)


Uma funo dita do 1 grau, quando do tipo y = ax + b, onde a
0.
Exemplos:
a) f(x) = 3x + 5 (a = 3; b = 5)
b) f(x) = -2x + 1 (a = -2; b = 1).

03. Em um determinado concurso, 2000 candidatos inscritos


compareceram s provas realizadas em um grande colgio. O
nmero de candidatos (y) que entraram no colgio, em funo do
horrio de entrada(t), representado por pontos do grfico, sendo t =
0 o instante em que os portes de acesso foram abertos e t = 60, o
instante em que esses portes foram fechados.

Propriedades da funo do 1 grau


1) O grfico de uma funo do 1 grau sempre uma
inclinada aos eixos;
2) Na funo f(x) = ax + b, se b = 0, f dita funo linear, e se alm
disso, a = 1, f dita funo identidade;
3) O grfico intercepta o eixo dos x na raiz da equao f(x) = 0 e,
portanto, no ponto de abscissa x =

4) O grfico intercepta o eixo dos y no ponto (0 , b), onde b


chamado coeficiente linear;
5) Se a > 0, ento f

6) Se a < 0, ento f

7) Quando a funo linear, ou seja, y = f(x) = ax, o grfico uma


reta que sempre passa na origem.

Assim, pode-se afirmar que, quando o nmero de candidatos no


interior do colgio atingiu 1860, o tempo decorrido desde a abertura
dos portes foi igual a
(A) 53 min 20 s.
(B) 53 min 45 s.
(C) 54 min 36 s.
(D) 55 min 20 s.
(E) 55 min 48 s.
04. Sabe-se que o preo a ser pago por uma corrida de txi inclui
uma parcela fixa, que denominada bandeirada, e uma parcela
varivel, que funo da distncia percorrida. Se o preo da
bandeirada R$ 4,60 e o quilmetro rodado R$ 0,96, a distncia
percorrida pelo passageiro que pagou R$ 19,00, para ir de sua casa
ao shopping, de
(A) 5 km.
(B) 10 km.
(C) 15 km.
(D) 20 km.
(E) 25 km.
GABARITO:
01. B

02. C

03. D

04. C

EXERCCIOS DE FIXAO
01. Em certa cidade, acontece anualmente uma corrida, como parte
dos eventos comemorativos pela sua emancipao poltica. Em
2000, o comit organizador da corrida permitiu a participao de
1500 pessoas; e, em 2005, a participao de 1800 pessoas. Devido
s condies de infraestrutura da cidade, o comit decidiu limitar o
nmero de participantes na corrida. Nesse sentido, estudos feitos
concluram que o nmero mximo n(t) de participantes, no ano t,
seria dado pela funo afim n(t) = at + b, onde a e b so constantes.
Com base nessas informaes, conclui-se que, no ano de 2010, o
nmero mximo de participantes na corrida ser igual a
(A) 1900.
(B) 2100.
(C) 2300.
(D) 2500.
(E) 2700.
02. Em um telefone residencial, a conta mensal para as ligaes
locais dada pela funo y = ax + b, onde x um nmero de
chamadas mensais, com durao mxima de 3 minutos, e y o
nmero total a ser pago em reais. No ms de Abril houve 100
chamadas e a conta mensal foi de 170 reais. J no ms de Maio
houve 120 chamadas e a conta mensal foi de 198 reais. Qual o total
a ser pago no ms com 180 chamadas?
(A) R$ 320,00
(B) R$ 305,00
(C) R$ 282,00
(D) R$ 222,00
(E) R$ 251,00

SECRETARIA DE ESTADO DA EDUCAO

184

QUMICA
LGEBRA

AULA 06 FUNO DO 2 GRAU (FUNO QUADRTICA)

EXERCCIOS DE FIXAO

Uma funo f : dita do 2 grau ou quadrtica, quando


do tipo f(x) = ax2 + bx + c, com a 0.

01. No sistema cartesiano ortogonal xOy abaixo, utilizamos parte dos


grficos de funes polinomiais do 2 grau e segmentos de reta para
homenagear a Capital Federal, Braslia. Com base nestes grficos, a
soma S(x) das sentenas que descrevem as funes quadrticas que
tiveram parte de seus grficos utilizados para representar as partes,
cncava e convexa do monumento, dada por:

Exemplos:
f(x) = x2 2x + 1 (a = 1, b = 2, c = 1);
y = x2 (a = 1, b = 0, c = 0)
Grfico da funo do 2 grau y = ax2 + bx + c sempre uma
parbola de eixo vertical.

(A) S(x) = x2 + 2x 3
(C) S(x) = 12x + 1
(E) S(x) = 1

Propriedades do grfico de y = ax + bx + c:
1) Se a > 0 a parbola tem concavidade para
quentemente um ponto de mnimo;

e conse-

2) Se a < 0 a parbola tem concavidade para


sequentemente um ponto de mximo;

con-

3) O vrtice da parbola o ponto V(xv , yv) onde:


xv = b / 2a
yv = / 4a , onde = b2 4ac (discriminante);

(B) S(x) = x2 2x + 5
(D) S(x) = 13x + 3

02. Um mssil foi lanado acidentalmente do ponto A, como mostra


a figura a seguir, tendo como trajetria o grfico da funo

fx x 2 70x , onde x dado em km. Desejando-se destru-lo num


ponto B, que est a uma distncia horizontal de 40 km de A, utilizase um outro mssil que se movimenta numa trajetria descrita,
segundo o grfico da funo g(x) = kx.

4) A parbola intercepta o eixo dos x nos pontos de abscissas x' e


(ou zeros) da equao ax2 + bx + c = 0,

x'', que so as

determinadas atravs de x

b
(Frmula de Bhskara);
2a

5) A parbola intercepta o eixo dos y no ponto

6) O eixo de simetria da parbola uma reta vertical de equao x


= b/2a;
7) ymax = / 4a (a < 0)

Ento, para que ocorra a destruio no ponto determinado, deve-se


tomar k igual a
(A) 20.
(B) 30.
(C) 40.
(D) 50.
(E) 60.

8) ymin = / 4a (a > 0)
9) Im(f) = { y R; y / 4a } (a > 0)
10) Im(f) = { y R; y / 4a } (a < 0)
11) Forma fatorada: sendo x1 e x2 as razes de f(x) = ax2 + bx + c,
ento ela pode ser escrita na forma f(x) = a(x x1).(x x2)
12) Relaes de Girard: Na equao ax2 + bx + c = 0, as razes x1 e
x2 satisfazem as seguintes relaes:
i) Soma (S): x1 + x2 =

b
a

ii) Produto (P): x1 . x2 =

1 2 1
x x , representado na
200
5
figura abaixo, descreve a trajetria de um projtil, lanado a partir
da origem.

03. O grfico da funo y f ( x )

c
a

Concavidade e Razes no Grfico da Funo

Sabendo-se que x e y so dados em quilmetros, a altura mxima


H e o alcance A do projtil so, respectivamente,
(A) 2 km e 40 km.
(B) 40 km e 2 km.
(C) 2 km e 10 km.
(D) 10 km e 2 km.
(E) 2 km e 20 km.
GABARITO:
01. C

SECRETARIA DE ESTADO DA EDUCAO

185

02. B

03. B

QUMICA

LGEBRA

AULA 07 POTENCIAO E FUNO EXPONENCIAL

EXERCCIOS DE FIXAO

Potncias de Base Real com Expoente Inteiro


Nestas condies h quatro situaes em particular que iremos
tratar. A saber, quando o expoente maior que um, quando igual a
um, quando igual a zero e quando negativo.
i) Expoente Maior que 1
De forma geral:
an = a . a . a . ... . a, isto , a multiplicao de n fatores iguais a a.
Este o caso de mais fcil compreenso, pois o conceito da
potenciao est bem claro. Observe a expresso abaixo:
54 = 5 . 5 . 5 . 5 = 625
54, que se l 5 elevado a 4, ou 5 elevado quarta potncia igual
ao produto de quatro fatores todos eles iguais a cinco.
ii) Expoente Igual a 1
Todo nmero elevado a 1 igual ao prprio nmero:
a1 = a ; 31 = 3 ; (2,5)1 = 2,5
iii) Expoente Igual a 0
Todo nmero, diferente de zero, elevado a 0 igual a 1:
a0 = 1 ; 30 = 1 ; (3,12)0 = 1
iv) Expoente Negativo
Qualquer nmero diferente de zero elevado a um expoente negativo
igual ao inverso deste nmero elevado ao oposto do expoente:
3

1
1
3
a 3 ; 2 4 ;
a
2
5

5

3

P.2 -

1
e 3600.
12
1
(B)
e 100.
12
1
(C)
e 64.
12

ax . ay = ax + y

ax

ay

(A)

xy

x y

P.3 P.4 -

a x.y
(a.b) = ax . bx

P.5 -

a
ax
b x , b 0
b

P.6 P.7 P.8 P.9 -

p
q

(E)

a. b a.b

pq

(D) 12 e 5400.

ap
p

a
,b0
b
p.q

02. Em um experimento, verificou-se que uma substncia radioativa


se decompunha de acordo com a regra M(t) = 1215.3t, em que M a
massa (em gramas) restante t horas aps o incio do experimento.
Medindo a massa T horas aps o incio, e novamente uma hora
depois, observou-se uma reduo de 10 gramas na massa. Assim, o
valor de T igual a
(A) 3.
(B) 4.
(C) 5.
(D) 6.
(E) 7.
03. O crescimento de uma cultura de bactrias obedece funo N(t)
= 600.3kt, em que N o nmero de bactrias no instante t, sendo t o
tempo em horas. A produo tem incio em t = 0. Decorridas 12
horas h um total de 1800 bactrias. O valor de k e o nmero de
bactrias, aps 24 horas do incio da produo, so,
respectivamente:

Propriedades das Potncias de Base Real no Nula


P.1 -

01. Em uma empresa de montagem de computadores, foi feito um


estudo e constatou-se que um tcnico com t meses de experincia
conseguia montar no mximo N(t) computadores por ms, onde N(t)
representa o maior nmero inteiro menor ou igual a 600 29t. Para
ser considerado experiente nessa empresa, um tcnico tem que
montar pelo menos 592 computadores por ms.
Admitindo que certo tcnico com t meses de experincia consiga
montar exatamente N(t) computadores por ms, o nmero mnimo de
meses necessrios para que ele seja considerado experiente
(A) 4.
(B) 6.
(C) 8.
(D) 10.
(E) 12.

EQUAO EXPONENCIAL
Equaes exponenciais so aquelas em que a incgnita se encontra
no expoente de pelo menos uma potncia.

1
e 5400.
12

04. O nmero N de habitantes de uma cidade cresce


exponencialmente com o tempo, de modo que, daqui a t anos, esse
nmero ser N = 20.000(1 + k)t, onde k um nmero real. Se daqui a
10 anos a populao for de 24.000 habitantes, daqui a 20 anos ela
ser de
(A) 28.000 habitantes.
(B) 28.200 habitantes.
(C) 28.400 habitantes.
(D) 28.600 habitantes.
(E) 28.800 habitantes.

Exemplos de equaes exponenciais:


10x = 100
2x + 12 = 20
9x = 81
5x+1 = 25
Para resolvermos uma equao exponencial precisamos aplicar
tcnicas para igualar as bases, assim podemos dizer que os
expoentes so iguais. Observe a resoluo da equao exponencial
a seguir:
3x = 2187 (fatorando o nmero 2187 temos: 37)
3x = 37
x=7
O valor de x na equao 7.

SECRETARIA DE ESTADO DA EDUCAO

186

QUMICA
LGEBRA

AULA 08 MATEMTICA FINANCEIRA BSICA

Frmula para o clculo de Juros compostos

Lucro e Prejuzo
i) Se o preo de venda superior ao preo de custo, ento o lucro
nominal, ou monetrio, L dado por: L = V C;
ii) Se o preo de venda V inferior ao preo de custo C, ento o
prejuzo nominal P dado por: P = C V;
Observao: Lucro negativo significa prejuzo e vice-versa
Juros Simples
O regime de juros simples, aquele no qual os juros incidem sempre
sobre o capital inicial.
Considere o capital inicial C aplicado a juros simples de taxa i por
perodo, durante t perodos.
Lembrando que os juros simples incidem sempre sobre o capital
inicial, podemos escrever a seguinte frmula, facilmente
demonstrvel:
J = C . i . t = Cit

Considere o capital inicial (principal C) $1000,00 aplicado a uma taxa


mensal de juros compostos ( i ) de 10% (i = 10% a.m.). Vamos
calcular os montantes (principal + juros), ms a ms:
Aps o 1 ms, teremos: M1 = 1000 x 1,1 = 1100 = 1000(1 + 0,1)
Aps o 2 ms, teremos: M2 = 1100 x 1,1 = 1210 = 1000(1 + 0,1)2
Aps o 3 ms, teremos: M3 = 1210 x 1,1 = 1331 = 1000(1 + 0,1)3
.....................................................................................................
Aps o n (ensimo) ms, sendo M o montante, teremos
evidentemente:
M = 1000(1 + 0,1)n
De uma forma genrica, teremos para um principal P, aplicado a uma
taxa de juros compostos i durante o perodo n:
M = C (1 + i)n
onde M = montante, C = principal, i = taxa de juros e n = nmero de
perodos que o principal C (capital inicial) foi aplicado.
Os juros nominais J, obtidos nesse tipo de aplicao, tambm so
dados por:
J=MC

J = juros produzidos depois de t perodos, do capital C aplicado a


uma taxa de juros por perodo igual a i.
No final de n perodos, claro que o capital ser igual ao capital
inicial adicionado aos juros produzidos no perodo. O capital inicial
adicionado aos juros do perodo denominado MONTANTE (M).
Logo, teramos:
M = C + J = C + C.i.t = C(1 + i.t)
Portanto, M = C(1+it).
Observao: Perceba tambm que: J = M C.

EXERCCIOS DE FIXAO
01. Um capital aplicado no prazo de dois anos, a uma taxa de juros
compostos de 40% ao ano, resulta no montante de R$9.800,00.
Sendo x% a taxa anual de juros simples que, aplicada ao mesmo
capital durante o mesmo prazo, resultar no mesmo montante,
determine x.
(A) 42
(B) 44
(C) 46
(D) 48
(E) 50

Juros Compostos
Aps cada perodo, os juros so incorporados ao principal e passam,
por sua vez, a render juros. Tambm conhecido como "juros sobre
juros".
Vamos ilustrar a diferena entre os crescimentos de um capital
atravs juros simples e juros compostos, com um exemplo:
Suponha que $100,00 so empregados a uma taxa de 10% a.a.
Teremos:

02. Um investidor aplicou a quantia de R$ 500,00 em um fundo de


investimento que opera no regime de juros simples. Aps 6 meses o
investidor verificou que o montante era de R$ 560,00. Qual a taxa de
juros desse fundo de investimento?
(A) 1%
(B) 2%
(C) 3%
(D) 4%
(E) 5%
03. Um automvel 0 km comprado por R$ 32.000,00. Ao final de
cada ano, seu valor diminui 10% em funo da depreciao do bem.
O valor aproximado do automvel, aps seis anos, :
(A) R$ 15.006,00
(B) R$ 19.006,00
(C) R$ 16.006,00
(D) R$ 12.800,00
(E) R$ 17.006,00

Observe que o crescimento do principal segundo juros simples


LINEAR enquanto que o crescimento segundo juros compostos
EXPONENCIAL, e portanto tem um crescimento muito mais "rpido".
Isto poderia ser ilustrado graficamente da seguinte forma:

04. Quando se divide o Produto Interno Bruto (PIB) de um pas pela


sua populao, obtm-se a renda per capita desse pas. Suponha
que a populao de um pas cresa taxa constante de 2% ao ano.
Para que sua renda per capita dobre em 20 anos, o PIB deve crescer
anualmente

taxa
constante
de,
aproximadamente,
(Use: 20 2 1,035 )
(A) 4,2%.
(B) 5,6%.
(C) 6,4%.
(D) 7,5%.
(E) 8,9%.

SECRETARIA DE ESTADO DA EDUCAO

187

QUMICA
LGEBRA

AULA 09 LOGARITMOS

Exemplo: log20 =log(2.10) = log2 + log10 = 0,3010 + 1 = 1,3010.


Observe que como a base no foi especificada, sabemos que ela
igual a 10.

I. DEFINIO
Dados os nmeros reais b (positivo e diferente de 1), N (positivo) e
x, que satisfaam a relao bx = N, dizemos que x o logaritmo de
N na base b. Isto expresso simbolicamente da seguinte forma:
logbN = x bx = N.
Neste caso, dizemos que b a base do sistema de logaritmos, N
o logaritmando ou antilogaritmo e x o logaritmo.
Exemplos:
a) log28 = 3 porque 23 = 8.
b) log41 = 0 porque 40 = 1.
c) log39 = 2 porque 32 = 9.
d) log55 = 1 porque 51 = 5.
Observaes:
1) Quando a base do sistema de logaritmos igual a 10 (Sistema
de Briggs), usamos a expresso logaritmo decimal e na
representao simblica escrevemos somente logN ao invs de
log10N. Assim que quando escrevemos logN = x, devemos
concluir pelo que foi exposto, que 10x = N.
Existe tambm um sistema de logaritmos chamado neperiano (em
homenagem a John Napier matemtico escocs, inventor dos
logaritmos), cuja base o nmero irracional e = 2,7182... (nmero
de Euler) e indicamos este logaritmo pelo smbolo ln.
Assim, logeM = lnM. Este sistema de logaritmos, tambm conhecido
como sistema de logaritmos naturais, tem grande aplicao no
estudo de diversos fenmenos da natureza.
Exemplos:
a) log100 = 2 porque 102 = 100.
b) log1000 = 3 porque 103 = 1000.
c) log2 = 0,3010 porque 100,3010 = 2.
d) log3 = 0,4771 porque 100,4771 = 3.
e) ln e = 1 porque e1 = e = 2,7182...
f) ln 7 = loge7
2) Os logaritmos decimais (base 10) normalmente so nmeros
decimais onde a parte inteira denominada caracterstica e a parte
decimal denominada mantissa.
Assim por exemplo, sendo log20 = 1,3010, 1 a caracterstica e
0,3010 a mantissa.
As mantissas dos logaritmos decimais so tabeladas.
3) Da definio de logaritmo, conclui-se que somente os nmeros
reais positivos possuem logaritmo. Assim, no tm sentido as
expresses log3(-9), log20, etc.
CONSEQUNCIAS DA DEFINIO
fcil demonstrar as seguintes propriedades imediatas dos
logaritmos, todas decorrentes da definio:
P1) O logaritmo da unidade em qualquer base que satisfaa a
condio de existncia, nulo, ou seja, logb1 = 0 porque b0 = 1.
P2) O logaritmo da base sempre igual a 1, ou seja, logbb = 1 ,
porque b1 = b.
P3) logbbk = k, porque bk = bk .
P4) Se logbM = logbN, ento podemos concluir que M = N. Esta
propriedade muito utilizada na soluo de exerccios envolvendo
equaes onde aparecem logaritmos (equaes logartmicas).
log M

P5) b b
igual a M.

M , ou seja, b elevado ao logaritmo de M na base b

P2 LOGARITMO DE UM QUOCIENTE
O logaritmo de uma frao ordinria igual a diferena entre os
logaritmos do numerador da frao e do denominador, ou seja:
logb(M/N) = logbM logbN
Exemplo: log0,02 = log(2/100) = log2 log100 = 0,3010 2,0000 = 1,6990. Do exposto anteriormente, podemos concluir que, sendo
log0,02 = -1,6990 ento 10-1,6990 = 0,02.
Da mesma forma podemos exemplificar:
log5 = log(10/2) = log10 log2 = 1 0,3010 = 0,6990.
Observao: a no indicao da base, subtende-se logaritmos
decimal (base 10).
Nota: Chamamos de cologaritmo de um nmero positivo N numa
base b, ao logaritmo do inverso multiplicativo de N, tambm na base
b. Ou seja:
cologbN = logb(1/N) = logb1 logbN = 0 logbN =
cologbN = logbN = logbN1
Exemplo: colog10 = log10 = 1.
P3 LOGARITMO DE UMA POTNCIA
Temos a seguinte frmula, facilmente demonstrvel: logbMk =
k.logbM.
Exemplo: log5256 = 6.log525 = 6.2 = 12.
Observaes:

m
n
m
. loga b
n

i) logan am
ii) logan bm

P4 MUDANA DE BASE
s vezes, para a soluo de problemas, temos necessidade de
mudar a base de um sistema de logaritmos, ou seja, conhecemos o
logaritmo de N na base b e desejamos obter o logaritmo de N numa
base a. Esta mudana de base, muito importante na soluo de
exerccios, poder ser feita de acordo com a frmula a seguir, cuja
demonstrao no apresenta dificuldades, aplicando-se os
conhecimentos aqui expostos.
loga N
logb N
loga b
Exemplos:
a) log416 = log216 / log24 (2 = 4:2)
b) log864 = log264 / log28 (2 = 6:3)
c) log25125 = log5125 / log525 = 3 / 2 = 1,5. Temos ento que 251,5 =
125.
Observaes:
i) Na resoluo de problemas, sempre muito mais conveniente
mudar um log de uma base maior para uma base menor, pois isto
simplifica os clculos.
ii) Duas consequncias importantes da frmula de mudana de
base so as seguintes:
a) logbN = logN / logb (usando a base comum 10, que no precisa
ser indicada).
b) logba . logab = 1
Exemplos:
a) log37 . log73 = 1
b) log23 = log3 / log2 = 0,4771 / 0,3010 = 1,5850

II. PROPRIEDADES OPERATRIAS DOS LOGARITMOS


P1 LOGARITMO DE UM PRODUTO
O logaritmo de um produto igual soma dos logaritmos dos
fatores, ou seja:
logb(M.N) = logbM + logbN

SECRETARIA DE ESTADO DA EDUCAO

188

QUMICA
LGEBRA

EXERCCIOS DE FIXAO

AULA 10 PROGRESSO ARITMTICA (P.A.)

01. uma tarefa realmente difcil descobrir a magnitude de uma


detonao nuclear subterrnea. O melhor mtodo monitorar a
atividade com sismgrafos, os mesmos instrumentos que detectam
terremotos e, com base na violncia do tremor, obter uma estimativa
aproximada da magnitude da exploso. Ao longo dos anos, os
gelogos aperfeioaram a metodologia de clculo dessas
estimativas.
Depois de observar muitos experimentos, os cientistas
desenvolveram uma excelente equao para calcular a capacidade
de uma arma nuclear detonada com base na intensidade do abalo
ssmico:
mb = 4,45 + 0,75.log(Y),
em que mb a magnitude, na escala, Richter, do tremor causado
pelo teste, e Y a energia liberada pelo artefato nuclear em
quilotons.
SEIFE, Charles. Os nmeros (no) mentem. Editora Zahar.

Chama-se Progresso Aritmtica PA toda sequncia numrica


cujos termos a partir do segundo, so iguais ao anterior somado com
um valor constante denominado razo.
Exemplos:
A = (1, 5, 9, 13, 17, 21, ...) razo = 4 (PA crescente)
B = (3, 12, 21, 30, 39, 48, ...) razo = 9 (PA crescente)
C = (5, 5, 5, 5, 5, 5, 5, ...) razo = 0 (PA constante)
D = (100, 90, 80, 70, 60, 50, ...) razo = -10 ( PA decrescente)

Um terremoto de intensidade 6,0 corresponde a uma bomba de,


aproximadamente
(A) 1 quiloton.
(B) 10 quilotons.
(C) 100 quilotons.
(D) 1.000 quilotons.
(E) 10.000 quilotons.
02. Aproximadamente 6.000 sismos (terremotos) so detectados
anualmente, por sismgrafos, no Planeta Terra, sendo que, de 15 a
20 destes, tem efeitos significativos em termos de destruio. Estes
sismos so mensurados a partir de modelos matemticos ou
tcnicas, sendo que uma das mais conhecidas e utilizadas a
Escala de Richter (em homenagem ao seu inventor, o sismlogo
norte americano, Charles Francis Richter que construiu a escala em
conjunto com o sismlogo Beno Gutemberg). Uma expresso muito
til (elaborada por Richter) na determinao da magnitude de um
terremoto a equao logartmica: Ms = log10(A.f) + 3,30
Em que:
Ms = magnitude do terremoto na Escala Richter.
A = valor numrico da amplitude da onda registrada em um
sismgrafo, em micrmetros (m).
f = valor numrico da frequncia da onda, em Hertz (Hz).
e o produto (A.f) o logaritmando do termo logaritmo.
Utilize a equao logartmica para determinar a magnitude de um
terremoto cuja amplitude de onda registrada pelo sismgrafo de
50.000 m e frequncia da onda 0,2 Hz. Ento a magnitude deste
terremoto de
(A) 5,30.
(B) 6,30.
(C) 7,30.
(D) 8,00.
(E) 9,00.
03. Um paciente de um hospital est recebendo soro por via
intravenosa. O equipamento foi regulado para gotejar x gotas a cada
30 segundos. Sabendo-se que este nmero x a soluo da
equao log4x = log23, e que cada gota tem volume de 0,3ml, podese afirmar que o volume de soro que este paciente recebe em uma
hora de
(A) 800 ml.
(B) 750 ml.
(C) 724 ml.
(D) 500 ml.
(E) 324 ml.

TERMO GERAL
Seja a PA genrica (a1, a2, a3, ... , an, ...) de razo r. De acordo com a
definio podemos escrever:
a2 = a1 + 1.r
a3 = a2 + r = (a1 + r) + r = a1 + 2r
a4 = a3 + r = (a1 + 2r) + r = a1 + 3r
.....................................................
Podemos inferir (deduzir) das igualdades acima que:
an = a1 + (n 1) . r
A expresso an = a1 + (n 1) . r denominada termo geral da PA.
Nesta frmula, temos que an o termo de ordem n (n-simo termo), r
a razo e a1 o primeiro termo da Progresso Aritmtica (PA).
Exemplos:
Qual o milsimo nmero mpar positivo?
Temos a PA: (1, 3, 5, 7, 9, ...), onde o primeiro termo a1= 1, a razo r
= 2 e queremos calcular o milsimo termo a1000. Nestas condies, n
= 1000 e poderemos escrever:
a1000 = a1 + (1000 - 1).2 = 1 + 999.2 = 1 + 1998 = 1999.
Portanto, 1999 o milsimo nmero mpar.
Qual o nmero de termos da PA: ( 100, 98, 96, ... , 22) ?
Temos a1 = 100, r = 98 100 = - 2 e an = 22 e desejamos calcular n.
Substituindo na frmula do termo geral, fica: 22 = 100 + (n 1). (- 2);
logo, 22 100 = - 2n + 2 e, 22 100 2 = - 2n de onde conclui-se
que 80 = - 2n , de onde vem n = 40.
Portanto, a PA possui 40 termos.
Atravs de um tratamento simples e conveniente da frmula do termo
geral de uma PA, podemos generaliza-la da seguinte forma:
Sendo aj o termo de ordem j (j-simo termo) da PA e ak o termo de
ordem k (k-simo termo) da PA, poderemos escrever a seguinte
frmula genrica:
aj = ak + (j - k).r
Exemplos:
Se numa PA o quinto termo 30 e o vigsimo termo 60, qual a
razo?
Temos a5 = 30 e a20 = 60.
Pela frmula anterior, poderemos escrever:
a20 = a5 + (20 5) . r e substituindo fica: 60 = 30 + (20 5).r ;
60 30 = 15r ; logo, r = 2.
Numa PA de razo 5, o vigsimo termo vale 8. Qual o terceiro termo?
Temos r = 5, a20 = 8.
Logo, o termo procurado ser: a3 = a20 + (3 20).5
a3 = 8 17.5 = 8 85 = - 77.
PROPRIEDADES DE UMA P.A.
i) Numa PA, cada termo (a partir do segundo) a mdia
aritmtica dos termos vizinhos deste.
Exemplo:
PA : ( m, n, r ) ; portanto, n = (m + r) / 2
Assim, se lhe apresentarem um problema de PA do tipo:
Trs nmeros esto em PA, ... , a forma mais inteligente de resolver
o problema considerar que a PA do tipo:
(x r, x, x + r), onde r a razo da PA.

SECRETARIA DE ESTADO DA EDUCAO

189

QUMICA
LGEBRA

AULA 11 PROGRESSO GEOMTRICA (P.G.)

ii) Numa PA, a soma dos termos equidistantes dos extremos


constante.
Exemplo:
PA : ( m, n, r, s, t); portanto, m + t = n + s = r + r = 2r
Estas propriedades facilitam sobremaneira a soluo de problemas.

Entenderemos por progresso geomtrica - PG - como qualquer


sequncia de nmeros reais ou complexos, onde cada termo a partir
do segundo, igual ao anterior, multiplicado por uma constante
denominada razo.

SOMA DOS TERMOS DE UMA P.A.

Exemplos:

Seja a PA ( a1, a2, a3, ..., an-1, an).


A soma dos n primeiros termos Sn = a1 + a2 + a3 + ... + an-1 + an ,
pode ser deduzida facilmente, da aplicao da segunda propriedade
acima.
Temos:
Sn = a1 + a2 + a3 + ... + an-1 + an
claro que tambm poderemos escrever a igualdade acima como:
Sn = an + an-1 + ... + a3 + a2 + a1
Somando membro a membro estas duas igualdades, vem:
2. Sn = (a1 + an) + (a2 + an-1) + ... + (an + a1)
Logo, pela segunda propriedade acima, as n parcelas entre
parnteses possuem o mesmo valor ( so iguais soma dos termos
extremos a1 + an), de onde conclumos inevitavelmente que:
2.Sn = (a1 + an).n , onde n o nmero de termos da PA.
Da ento, vem finalmente que:
a an .n
Sn 1
2

(1,2,4,8,16,32, ... ) PG de razo 2


(5,5,5,5,5,5,5, ... ) PG de razo 1
(100,50,25, ... ) PG de razo 1/2
(2,-6,18,-54,162, ...) PG de razo -3

EXERCCIOS DE FIXAO

CLASSIFICAO DAS PROGRESSES GEOMTRICAS

1. Crescente:

2. Decrescente:

3. Alternante ou Oscilatria: quando q < 0.

01. Num laboratrio, foi feito um estudo sobre a evoluo de uma


populao de vrus. Ao final de um minuto do incio das observaes,
existia 1 elemento na populao; ao final de dois minutos, existiam 5,
e assim por diante. A seguinte sequncia de figuras apresenta as
populaes do vrus (representado por um crculo) ao final de cada
um dos quatro primeiros minutos.

4. Constante: quando q = 1
TERMO GERAL DE UMA P.G.
Seja a PG genrica: (a1, a2, a3, a4, ... , a n, ... ) , onde a1 o primeiro
termo, e an o n-simo termo, ou seja, o termo de ordem n. Sendo q
a razo da PG, da definio podemos escrever:
a2 = a1 . q

Supondo que se manteve constante o ritmo de desenvolvimento da


populao, o nmero de vrus no final de 1 hora era de
(A) 241.
(B) 238.
(C) 237.
(D) 233.
(E) 232.
02. A corrida de So Silvestre, em So Paulo, uma das mais
importantes provas de rua disputadas no Brasil. Seu percurso mede
15 km.
Joo, que treina em uma pista circular de 400 m, pretende participar
dessa corrida. Para isso, ele estabeleceu a seguinte estratgia de
treinamento: correr 7000 na primeira semana; depois, a cada
semana, aumentar duas voltas na pista, at atingir a distncia
exigida na prova.
Em que semana do treinamento Joo atingir a distncia exigida na
prova?
(A) 10
(B) 11
(C) 12
(D) 13
(E) 14
03. Um veculo parte de uma cidade A em direo a uma cidade B,
distante 500 km. Na 1 hora do trajeto ele percorre 20 km, na 2 hora
22,5 km, na 3 hora 25 km e assim sucessivamente. Ao completar a
12 hora do percurso, a que distncia, em km, esse veculo estar de
B?
(A) 95
(B) 115
(C) 125
(D) 135
(E) 155

SECRETARIA DE ESTADO DA EDUCAO

190

a3 = a2 . q = (a1 . q) . q = a1 . q2
a4 = a3 . q = (a1 . q2) . q = a1 . q3
................................................
................................................
Infere-se (deduz-se) que: an = a1 . qn-1 , que denominada frmula
do termo geral da PG.
Genericamente, poderemos escrever: aj = ak . qj-k
Exemplos:
a) Dada a PG (2,4,8,... ), pede-se calcular o dcimo termo.
Temos: a1 = 2, q = 4/2 = 8/4 = ... = 2. Para calcular o dcimo termo,
ou seja, a10, vem pela frmula:
a10 = a1 . q9 = 2 . 29 = 2. 512 = 1024
b) Sabe-se que o quarto termo de uma PG crescente igual a 20 e o
oitavo termo igual a 320. Qual a razo desta PG?
Temos a4 = 20 e a8 = 320. Logo, podemos escrever: a8 = a4 . q8-4 .
Da, vem: 320 = 20.q4
Ento q4 =16 e, portanto q = 2.
Nota: Uma PG genrica de 3 termos, pode ser expressa como:
(x/q, x, xq), onde q a razo da PG.
PROPRIEDADES DE UMA P.G.
i) Em toda PG, um termo a mdia geomtrica dos termos
imediatamente anterior e posterior.
Exemplo: PG (A,B,C,D,E,F,G)
Temos ento: B2 = A . C ; C2 = B . D ; D2 = C . E ; E2 = D . F etc.
ii) O produto dos termos equidistantes dos extremos de uma PG
constante.
Exemplo: PG ( A,B,C,D,E,F,G)
Temos ento: A . G = B . F = C . E = D . D = D2

QUMICA
LGEBRA

SOMA DOS n PRIMEIROS TERMOS DE UMA P.G.

EXERCCIOS DE FIXAO

Seja a PG (a1, a2, a3, a4, ... , an , ...) . Para o clculo da soma dos n
primeiros termos Sn, vamos considerar o que segue:
Sn = a1 + a2 + a3 + a4 + ... + an-1 + an
Multiplicando ambos os membros pela razo q vem:
Sn . q = a1 . q + a2 .q + .... + an-1 . q + an .q .
Logo, conforme a definio de PG, podemos reescrever a expresso
acima como:
Sn . q = a2 + a3 + ... + an + an . q
Observe que a2 + a3 + ... + an igual a Sn - a1 . Logo, substituindo,
vem:
Sn . q = Sn - a1 + an . q
Da, simplificando convenientemente, chegaremos seguinte frmula
da soma:
a .q a1
Sn n
q 1
Se substituirmos a n = a1 . qn-1 , obteremos uma nova apresentao
para a frmula da soma, ou seja:

Sn

a1.(qn 1)
q 1

01. As medidas das alturas de trs irmos esto em progresso


geomtrica. Se os dois maiores medem 1,60m e 1,80m, a altura do
menor , em metros, aproximadamente, igual a
(A) 1,30
(B) 1,36
(C) 1,42
(D) 1,48
(E) 1,54
02. Um aluno do curso de biologia estudou durante nove semanas o
crescimento de uma determinada planta, a partir de sua germinao.
Observou que, na primeira semana, a planta havia crescido 16 mm.
Constatou ainda que, em cada uma das oito semanas seguintes, o
crescimento foi sempre a metade do crescimento da semana
anterior.
Dentre os valores a seguir, o que MELHOR aproxima o tamanho
dessa planta, ao final dessas nove semanas, em milmetros,
(A) 48.
(B) 36.
(C) 32.
(D) 30.
(E) 24.
03. Na figura a seguir, temos um ambiente para a realizao de
grandes eventos, no formato de setor circular, onde os assentos
esto dispostos obedecendo a uma lei de formao.

Exemplo:
Calcule a soma dos 10 primeiros termos da PG (1,2,4,8,...)
Temos:
210 1
1023
2 1
Observe que neste caso a1 = 1.
S10

SOMA DOS TERMOS DE UMA P.G. INFINITA CUJA RAZO EM


MDULO MENOR QUE 1
Considere uma PG ILIMITADA (infinitos termos) e decrescente.
Nestas condies, podemos considerar que no limite teremos a n = 0.
Substituindo na frmula anterior, encontraremos:
a
Sn 1
1 q
Exemplo:
Resolva a equao: x + x/2 + x/4 + x/8 + x/16 + ... = 100
Ora, o primeiro membro uma PG de primeiro termo x e razo 1/2.
Logo, substituindo na frmula, vem:
x
100
1
1
2

No vrtice do setor, primeira fileira, coloca-se apenas uma cadeira;


na segunda fileira horizontal posterior, colocam-se duas cadeiras; na
terceira fileira horizontal, colocam-se quatro e assim sucessivamente.
Considere que a lotao mxima de 511 pessoas sentadas. Ento,
o nmero de fileiras nesse ambiente igual a
(A) 7.
(B) 8.
(C) 9.
(D) 10.
(E) 11.
04. A figura abaixo representa o grfico da funo f(x) = 3 x, x 0, e
os primeiros elementos de uma sequncia infinita de retngulos.

Da, vem: x = 100 . 1/2 = 50


PRODUTO DOS n PRIMEIROS TERMOS DE UMA PG

Pn a1n.q

n(n1)
2

A soma das reas de todos os retngulos dessa seqncia infinita


igual a
(A) 1.
(B) 1/2.
(C) 1/3.
(D) 2/3.
(E) 3/2.

SECRETARIA DE ESTADO DA EDUCAO

191

QUMICA
LGEBRA

AULA 12 MATRIZES
I. DEFINIO
Matriz m x n uma tabela de m . n nmeros reais dispostos em m
linhas (filas horizontais) e n colunas (filas verticais). Exemplos:
1 2 3
i) A
uma matriz 2 x 3;
4 2
0
4 0
uma matriz 2 x2;
ii) B
1 1
Como podemos notar nos exemplos 1, 2 e 3 respectivamente, uma
matriz pode ser representada por colchetes, parnteses ou duas
barras verticais.

As matrizes A e B so iguais, pois preenchem todos os requisitos de


igualdade de matrizes.
2 0
2 c
Se A
, B
e A = B, ento c = 0 e b = 3.

1
b

1 3
IV. OPERAES ENTRE MATRIZES
1. Adio
Dadas as matrizes A= aij

m x n

m x n , chamamos de soma

e B = bij

m x n , tal que cij aij bij , para

das matrizes A e B a matriz C = cij


todo 1 i m e todo 1 i n .

II. REPRESENTAO

Notao: A + B = C OBS: A + B existe se, e somente se, A e B so


do mesmo tipo (m x n).

As matrizes costumam ser representadas por letras maisculas e


seus elementos por letras minsculas, acompanhadas de dois
ndices que indicam, respectivamente, a linha e a coluna ocupadas
pelo elemento.

Exemplo:
2 3 0 3 1 1 2 3

0 1 1 1 - 1 2 0 1

3 1
0 1 5 4 1

1 1 1 2 1 0 1

2. Subtrao
Dadas as matrizes A= aij

e B= bij

Exemplo: Uma matriz A do tipo m x n representada por:


a11 a12 a13 a1n

a21 a22 a23 a2 n


A a31 a32 a33 a3 n

a
a
a

m2
m3
mn
m1
ou,

abreviadamente,

mxn ,

A= aij

onde

m x n

a 23

chamamos de

diferena entre as matrizes A e B a soma de A com a matriz oposta


de B
Notao: A B = A + (B)
OBS: A B existe se, e somente se, A e B so do mesmo tipo (m x
n).

representam,

respectivamente, a linha e a coluna que o elemento ocupa,


1 i m
.

1 j n
Por exemplo, na matriz anterior,

m x n ,

o elemento da

segunda linha com o da terceira coluna.

Exemplo:
0 1 2 3
0 1 -2 3 1
0 2 2 2
3

4 7 0 - 2 4 7 0 2 4 0 7 2 4 5
3. Multiplicao de um nmero real por uma matriz
Dados um nmero real x e uma matriz A do tipo m x n, o produto de
x por A uma matriz do tipo m x n, obtida pela multiplicao de cada
elemento de A por x.

2 x 2 , onde aij 2i j :

Exemplo 1: Seja a matriz A= aij

a
a12

Genericamente, temos: A 11
.

a
a
22 2 x 2
21
Utilizando a regra de formao dos elementos dessa matriz, temos:

Notao: B = x.A
OBS.: Cada elemento bij de B tal que bij = x aij

aij 2i j

a21 2(2) 1 5

Exemplo:
2
a) 3.
1

a12 2(1) 2 4

4. Multiplicao entre Matrizes

a11 2(1) 1 3

a22 2(2) 2 6

7 3.2 3.7 6 21

0 3. 1 3.0 3 0

O produto de uma matriz por outra no pode ser determinado


atravs do produto dos seus respectivos elementos. A multiplicao
de matrizes no anloga multiplicao de nmeros reais.
Assim, o produto das matrizes A= aij m x p e B= bij p x n a matriz

3 4
.
Assim, A=
5 6

III. IGUALDADE ENTRE MATRIZES


Para que duas ou mais matrizes sejam consideradas iguais elas
devem obedecer a algumas regras:
Devem ter a mesma ordem, ou seja, o mesmo nmero de linhas e o
mesmo nmero de colunas.
Os elementos devem ser iguais aos seus correspondentes.
Portanto, podemos concluir que:
A matriz A2x2 igual a matriz B se, somente se, a matriz B tiver
tambm a ordem 2x2 e os elementos a11 = b11, a21 = b21, a12 = b12 e
a22 = b22.

m x n , onde cada elemento c ij

C= cij

obtido atravs da soma dos

produtos dos elementos correspondentes da i-sima linha de A pelos


elementos da j-sima coluna de B.
OBS: Elementos correspondentes de matrizes do mesmo tipo m x n,
so os elementos que ocupam a mesma posio nas duas matrizes.
1 6 4
5 0 2
Exemplo: Sejam A
e B
. Os elementos
3
0
2

7 3 4
a13 4 e b13 2 so elementos correspondentes.

Veja um exemplo de matrizes:

SECRETARIA DE ESTADO DA EDUCAO

192

QUMICA
LGEBRA

Decorrncia da definio

02. Uma montadora de automveis prepara trs modelos de carros, a


saber:

A matriz produto A . B existe apenas se o nmero de colunas da


primeira matriz (A) for igual ao nmero de linhas da segunda matriz
(B).
Assim:

Am x p e Bp x n A.Bm x n

Essa montadora divulgou a matriz abaixo em que cada termo a ij


representa a distncia percorrida, em km, pelo modelo i, com um litro
de combustvel, velocidade 10.j km/ h.

Note que a matriz produto ter o nmero de linhas (m) do primeiro


fator e o nmero de colunas (n) do segundo fator.
Exemplos:
a) Se A3 x 2 e B2 x 5 A.B3 x 5
b) Se A4 x 1 e B2 x 3 que no exist eprodut o
c) A4 x 2 e B2 x 1 A.B4 x 1
Exemplo:

2 3
1 2
a) Sendo A=
e B=
, vamos determinar A.B e B.A e
4
1

3 4
comparar os resultados.
Soluo:
2 3 1 2
A.B =
.

4 1 3 4
a

a11 1 linha e 1 coluna = 2.1 + 3.3 = 2 + 9 = 11

Com base nisso, correto dizer que


(A) para motoristas que somente trafegam a 80 km/ h, o carro 1.0 o
mais econmico.
(B) para motoristas que somente trafegam a 50 km/ h, o carro 1.4 o
mais econmico.
(C) para motoristas que somente trafegam a 30 km/ h, o carro 1.4 o
mais econmico.
(D) para motoristas que somente trafegam a 90 km/ h, o carro 1.0 o
menos econmico.
(E) para motoristas que somente trafegam a 70 km/ h, o carro 1.8 o
menos econmico.
03. Considere a produo de certo produto nos tamanhos mdio (M)
e grande (G), nas cores: preto, marrom e branco, nos meses de
maro e abril de um mesmo ano, de acordo com as tabelas a seguir.

a12 1a linhae 2a coluna = 2.2 + 3.4 =4 + 12 = 16

Produo de Maro

a21 2a linhae 1a coluna = 4.1 + 1.3 = 4 + 3 = 7

Tamanho
Cor
Preto
Marrom
Branco

a22 2a linhae 2acoluna = 4.2 + 1.4 = 8 + 4 = 12

2 3 1 2
Assim: A.B =
.
=
4 1 3 4

2.1 3.3 2.2 3.4 11 16

4.1 1.3 4.2 1.4 7 12 2 x 2

1 2
2 3
1.2 2.4 1.3 2.1 10 5
B.A =
.
=

3
4
4
1

2 x 2
2 x 2
3.2 4.4 3.3 4.1 22 13 2 x 2

Comparando os resultados, observamos que A.B B.A, ou seja, a


propriedade comutativa para multiplicao de matrizes no vale.
EXERCCIOS DE FIXAO
01. Um tcnico de basquetebol descreveu o desempenho dos
titulares de sua equipe, em sete jogos, atravs da matriz:
18 17 18 17 21 18 20

15 16 18 18 22 21 18
20 19 20 21 14 14 22

18 22 20 20 18 22 23
19 18 12 14 20 17 18

Cada elemento aij dessa matriz o nmero de pontos marcados pelo


jogador do nmero i no jogo j. Quantos pontos marcou a equipe no
jogo 4?
(A) 20
(B) 90
(C) 128
(D) 143
(E) 186

200
180
120
Produo de Abril
Tamanho

Cor
Preto
Marrom
Branco

190
150
100

220
210
130

205
170
110

A matriz que representa a produo do bimestre de acordo com as


cores e o tamanho, dada por
390 425
420 395

(A) 330 380


(B) 390 320
220 240
250 210
210 198

(C) 195 360


125 105

20 15

(D) 30 20
10 10

10 15

(E) 30 40
20 20

04. Antnio, Bruno e Carlos decidiram passar em uma lanchonete


depois da partida de futebol no clube. A matriz M, a seguir,
representa quantos salgados cada um comeu, alm da forma como a
despesa foi dividida. Sendo Antnio representado pelo nmero 1,
Bruno pelo nmero 2 e Carlos pelo nmero 3, cada elemento aij da
matriz M a seguir indica o nmero de salgados consumidos, em que i
representa a pessoa que comeu e j a pessoa que pagou por essa
quantidade. Por exemplo a23 = 1 indica que Bruno comeu 1 salgado
2 3 2

que Carlos pagou. Sendo M 2 1 1 e R$ 0,80 o valor de cada


0 2 4
salgado, podemos afirmar que
(A) Bruno gastou R$ 4,80 na lanchonete.
(B) Bruno foi quem comeu mais salgados.
(C) Carlos gastou R$ 2,60 a mais que Antnio.
(D) Antnio foi quem teve o maior prejuzo na lanchonete.
(E) Antnio foi quem pagou o maior nmero total de salgados.
GABARITO:
01. B

SECRETARIA DE ESTADO DA EDUCAO

193

02. A

03. B

04. A

QUMICA
LGEBRA

AULA 13 SISTEMAS LINEARES

y
3x 2

2
S2 :
S 1,2
x y 1

3
Como os sistemas admitem a mesma soluo {(1, -2)}, S1 e S2 so
equivalentes.
Um conjunto de p equaes lineares com variveis x1, x2, x3,....,xn
formam um sistema linear com p equaes e n incgnitas.

x 3y 5
S1 :
S 1,2
2 x y 4

EQUAO LINEAR
Toda equao da forma a1x1 a2 x2 ... an x n b denominada
equao linear, em que:
a1, a2 ,..,an so coeficientes
x1, x2 ,...,x n so as incgnitas
b um termo independente

CLASSIFICAO DE UM SISTEMA LINEAR


Exemplos:
a) 2x1 3x2 x3 5 uma equao linear de trs incgnitas.

Todo sistema linear classificado de acordo com o nmero de


solues apresentadas por ele.

b) x y z t 1 uma equao linear de quatro incgnitas.


Observaes:
1) Quando o termo independente b for igual a zero, a equao linear
denomina-se equao linear homognea. Por exemplo: 5x + y = 0.
2) Uma equao linear no apresenta termos da forma x12 , x1.x 2
etc., isto , cada termo da equao tem uma nica incgnita, cujo
expoente sempre 1.
As equaes 3x12 2x2 3 e 4x.y z 2 no so lineares.
3) A soluo de uma equao linear a n incgnitas a seqncia de
nmeros reais ou nupla 1, 2 ,..., n , que, colocados
respectivamente no lugar de
igualdade dada.

x1, x2 ,...,x n , tornam verdadeira a

toda equao que possui variveis e apresenta-se na forma a1x1


+ a2x2 + a3x3 + ...+ anxn = b, em que a1, a2, a3, ....., so os
coeficientes reais e o termo independente representado pelo
nmero real b.
Exemplos:
a) x + y + z = 20
b) 2x 3y + 5z = 6
SISTEMA LINEAR
Denomina-se sistema linear de m equaes nas n incgnitas
x1, x2 ,...,x n todo sistema da forma:
a11x1 a12 x 2 ... a1n x n b1

a21x1 a22 x 2 ... a2 n x n b2

...
...

am1x1 am2 x 2 ... amnx n bn


a11, a12 ,...,a1n, b'1, b'2 ,...,b'n so nmeros reais.

Se o conjunto ordenado de nmeros reais '1, '2 ,..., 'n satisfizer a


todas as equaes do sistema, ser denominado soluo do sistema
linear.
Observaes:
1) Se o termo independente de todas as equaes do sistema for
nulo, isto , b1 b'2 ... b'n 0 , o sistema linear ser dito
homogneo. Veja o exemplo:
2 x y z 0

x y 4 z 0
5 x 2y 3 z 0

Uma soluo evidente do sistema linear homogneo x = y = z = 0.


Esta soluo chama-se soluo trivial do sistema homogneo. Se o
sistema homogneo admitir outra soluo em que as incgnitas no
so todas nulas, a soluo ser chamada soluo no-trivial.
2) Se dois sistemas lineares, S1 e S2, admitem a mesma soluo,
eles so ditos sistemas equivalentes. Veja o exemplo:

SECRETARIA DE ESTADO DA EDUCAO

194

EXERCCIOS DE FIXAO
01. Como se sabe, no jogo de basquete, cada arremesso convertido
de dentro do garrafo vale 2 pontos e, de fora do permetro, vale 3
pontos. Um time combinou com seu clube que receberia $50,00 para
cada arremesso convertido de 3 pontos e $30,00 para cada
arremesso convertido de 2 pontos. Ao final do jogo, o time fez 113
pontos e recebeu $1.760,00. Ento, a quantidade de arremessos
convertidos de 3 pontos foi
(A) 13.
(B) 15.
(C) 16.
(D) 17.
(E) 19.
02. Um cinema recebeu R$ 663,00 (seiscentos e sessenta e trs
reais) pela venda de ingressos (entrada), durante uma nica sesso.
Nessa sesso, o nmero de ingressos vendidos para adultos foi o
triplo do nmero de ingressos vendidos para crianas. O ingresso
para adulto custava R$ 12,00 (doze reais) e o das crianas R$ 3,00
(trs reais). Considere que x seja o nmero de ingressos vendidos
para os adultos e y, o nmero de ingressos vendidos para as
crianas.
Assinale a alternativa que expressa corretamente a equao que
permite determinar o nmero de ingressos vendidos para crianas,
bem como para os adultos.
x 3y
x y 3
(A)
(B)
x y 663
12 x 3y 663
x 3y
x y 3

(C)
(D)
x

663
12
x

3y 663

x 3y

(E)
3 x 12 y 663
03. Trs discos esto soldados como na figura a seguir.

Considerando que as medidas de A, B e C, em centmetros, so


respectivamente, 12, 16 e 18, os dimetros dos discos P, Q e R,
nessa ordem, medem, em centmetros,
(A) 9, 8, e 6.
(B) 5, 7 e 11.
(C) 12, 6, e 4.
(D) 11, 7, e 5.
(E) 4, 6, e 12.
GABARITO:
01. A
02. D
03. D
04. D

QUMICA

LGEBRA

AULA 14 TRATAMENTO DA INFORMAO (ESTATSTICA)

2.2. Grfico em colunas (vertical)


a representao de uma srie por meio de retngulos, dispostos
verticalmente (em colunas).
Os retngulos tm a mesma base e as alturas so proporcionais aos
respectivos dados.

DISTRIBUIO DE FREQUNCIA
1.1. Tabela de distribuio de frequncia
Considere a relao de nmeros abaixo, referente s alturas (em
centmetros) dos alunos de um colgio:
166 160 161 150 162 160 165 167 164 160
162 161 168 163 156 173 160 155 164 168
155 152 163 160 155 155 169 151 170 164
154 161 156 172 153 157 156 158 158 161
Para lidarmos com a lista toda, interessante resumi-la, contando o
nmero de pessoas com cada altura, fazendo uma tabela que
denominamos distribuio de freqncia.

2.3. Grfico em barras (horizontal)


a representao de uma srie por meio de retngulos, dispostos
horizontalmente (em barras).
Os retngulos tm a mesma base e as alturas so proporcionais aos
respectivos dados.

1.2. Elementos de uma distribuio de frequncia


Classe
Classes de frequncia ou, simplesmente, classes so intervalos de
variao da varivel.
Limites de classe
Denominamos limites de classe os extremos de cada classe.
Ex.: limite inferior (li) limite superior (Li)
Amplitude de um intervalo de classe (h)
Amplitude de um intervalo de classe a medida do intervalo que
define a classe.
h = Li li
Amplitude total da distribuio
Amplitude total da distribuio (AT) a diferena entre o limite
superior da ltima classe (limite superior mximo) e o limite inferior
da primeira classe (limite inferior mnimo).
AT = Lmax lmin
Amplitude amostral da distribuio
Amplitude amostral da distribuio (AA) a diferena entre o valor
mximo e o valor mnimo da distribuio.
AA = xmax xmin
Ponto mdio de uma classe
Ponto mdio de uma classe (xi) , como o prprio nome indica, o
ponto que divide o intervalo de classe em duas partes iguais.

2.4. Grfico em setores (popular grfico de pizza)


Este grfico construdo com base em um crculo, e empregado
sempre que desejamos ressaltar a participao do dado no total.

GRFICOS ESTATSTICOS
O grfico estatstico uma forma de apresentao dos dados
estatsticos, cujo objetivo o de produzir, no investigador ou no
pblico em geral, uma impresso mais rpida e viva do fenmeno em
estudo, j que os grficos falam mais rpido compreenso que as
sries.

2.5. Histograma
O Histograma formado por um conjunto de retngulos justapostos,
cujas bases se localizam sobre o eixo horizontal, de tal modo que
seus pontos mdios coincidam com os pontos mdios dos intervalos
de classe.

2.1. Grfico em linhas


Este tipo de grfico se utiliza da linha poligonal para representar a
srie estatstica. O grfico em linha constitui uma aplicao do
processo de representao das funes num sistema de
coordenadas cartesianas.

LGEBRA
MEDIDAS DE CENTRALIDADE
At agora, estudamos de um modo geral, os grupos de valores que
uma varivel pode assumir.
Assim que podemos localizar a maior concentrao de valores de
uma dada distribuio, isto , se ela se localiza no incio, no meio ou
no final, ou ainda, se h uma distribuio por igual.
Porm, para ressaltar as tendncias caractersticas de cada
distribuio, isoladamente, ou em confronto com outras,

SECRETARIA DE ESTADO DA EDUCAO

195

QUMICA
LGEBRA

necessitamos introduzir conceitos que se expressem atravs de


nmeros, que nos permitam traduzir essas tendncias. Esses
conceitos so denominados elementos tpicos da distribuio e
so as:
medidas de posio
medidas de variabilidade ou disperso
medidas de assimetria
As mais importantes das medidas de posio so as medidas de
tendncia central, as quais recebem tal denominao pelos dados
observados tenderem, em geral, a se agrupar em torno dos valores
centrais. Dentre as medidas de tendncia central destacamos:
a mdia aritmtica simples
a mdia aritmtica ponderada
a mediana
a moda

As medidas de disperso medem a variabilidade dos dados em


estudo. Permitem verificar se o conjunto de dados homogneo ou
heterogneo.
Consideremos os seguintes conjuntos de dados:
a) 10 11 11 11 12 12 12 12 13 14 14
b) 1 5 6 9 11 12 12 15 18 21 22
Esses dois conjuntos tm valores iguais para mdia, mediana e
moda, mas existem diferenas entre eles, como podemos verificar
ao coloc-los num diagrama, como mostram as figuras abaixo.

1. Mdia Aritmtica simples X


A mdia aritmtica simples de um conjunto de nmeros igual ao
quociente entre a soma dos valores do conjunto e o nmero total de
valores. o ponto de equilbrio entre os dados.
Podemos estabelecer uma frmula geral para a mdia. Sejam n
nmeros x1, x2, x3, ..., xn. Os nmeros logo abaixo dos diversos x so
chamados ndices. Utilizaremos o smbolo x (x barra) para indicar a
x x2 ... x n
mdia. Podemos, ento, escrever: x 1
n
2. Mdia Aritmtica ponderada
A mdia aritmtica considerada ponderada quando os valores do
conjunto tiverem pesos diferentes. Tratando-se de mdia simples,
todos os valores apresentam igual peso. Obtm-se uma
mdia aritmtica ponderada atravs do quociente entre o produto dos
valores da varivel pelos respectivos pesos e soma dos pesos.

A figura um diagrama mostrando a disperso dos dados.


Isto indica que necessitamos de um outro tipo de medida para
distinguir os dois conjuntos de dados. Observando a figura, podemos
notar que o primeiro conjunto apresenta valores concentrados em
relao mdia, enquanto que o segundo apresenta valores
dispersos (espalhados) em relao mdia. As medidas que tratam
desta caracterstica so chamadas de medidas de disperso.
1. Desvio Mdio ou Desvio Mdio Absoluto
a mdia dos mdulos ou valores absolutos dos desvios. Se
considerssemos somente os desvios, a soma deles seria sempre
zero, pois existem desvios positivos e negativos.

3. Mediana e Moda para dados no agrupados (Dados brutos)


3.1. Mediana (Me)
Outra medida estatstica til a mediana. A mediana de um conjunto
de valores, colocados em rol, o valor situado de tal forma no
conjunto que o separa em dois subconjuntos de mesmo nmero de
elementos (elemento que ocupa a posio central). Em outras
palavras, tendo-se um conjunto de dados ordenados de maneira
crescente (ROL), a mediana o valor que separa os 50% dos
menores dados dos 50% maiores.
OBSERVAO:
A mediana no precisa ser um dos valores da distribuio e nem
deve ser arredondada!
NOTAS:
O valor da mediana pode ou no coincidir com um elemento da
srie, como vimos. Quando o nmero de elementos da srie mpar,
h a coincidncia. O mesmo no acontece, porm, quando esse
nmero par.
A mediana e a mdia aritmtica no tm, necessariamente, o
mesmo valor.
A mediana, como vimos, depende da posio e no dos valores dos
elementos da srie ordenada. Essa uma das diferenas marcantes
entre a mediana e a mdia (que se deixa influenciar, e muito, pelos
valores extremos).

2. Desvio Padro
Ao iniciar as anlises de um agrupamento de dados, a mdia permite
que se estabelea um juzo sobre tal conjunto. Porm, no permite
avaliar a disperso, principalmente para conjunto de dados
numerosos.
O desvio padro foge a falha que ocorre na amplitude, por levar em
conta todos os valores em questo. Portanto, o desvio padro
muito mais conveniente no clculo da disperso.
O desvio padro definido como a raiz quadrada da mdia dos
quadrados dos desvios.

3. Varincia
A varincia nada mais do que o quadrado do desvio padro, ou
seja, 2,

3.2. Moda (Mo)


Outra medida estatstica interessante a moda. A moda de uma
srie de valores o valor de maior frequncia absoluta, ou seja, o
valor que aparece o maior nmero de vezes na distribuio.
FIQUE ATENTO! Moda um valor, ou seja, xi. Moda NO a
frequncia (fi)!
Quanto sua classificao podemos dizer que uma distribuio :
unimodal (possui 1 moda), bimodal (possui 2 modas), trimodal
(possui 3 modas), polimodal (possui mais de 3 modas) e amodal
(no possui moda).
MEDIDAS DE DISPERSO

SECRETARIA DE ESTADO DA EDUCAO

196

QUMICA
LGEBRA

AULA 15 ANLISE COMBINTRIA

EXERCCIOS DE FIXAO
01.

Foi a necessidade de calcular o nmero de possibilidades existentes


nos chamados jogos de azar que levou ao desenvolvimento da
Anlise Combinatria, parte da Matemtica que estuda os mtodos
de contagem.
A Anlise Combinatria visa desenvolver mtodos que permitam
contar - de uma forma indireta - o nmero de elementos de um
conjunto, estando esses elementos agrupados sob certas condies.
Fatorial

Os dados anteriores mostram a taxa de juros cobrada por diversos


pases. Podemos afirmar que a mdia, moda e mediana, so,
respectivamente,
(A) 2,0; 1,55 e 1,0.
(D) 1,55; 1,4 e 2,0.
(B) 1,55; 4,2 e 1,0.
(E) 2,0; 1,4 e 1,55.
(C) 2,0; 1,0 e 1,55.
02.

Seja n um nmero inteiro no negativo. Definimos o fatorial de n


(indicado pelo smbolo n!) como sendo:
n! = n .(n-1) . (n-2) . ... .4.3.2.1 para n 2.
Para n = 0, teremos: 0! = 1.
Para n = 1, teremos: 1! = 1
Exemplos:
a) 6! = 6.5.4.3.2.1 = 720
b) 4! = 4.3.2.1 = 24
c) observe que 6! = 6.5.4!
d) 10! = 10.9.8.7.6.5.4.3.2.1
e) 10! = 10.9.8.7.6.5!
Princpio Fundamental da Contagem PFC
Se determinado acontecimento ocorre em n etapas diferentes, e se a
primeira etapa pode ocorrer de k1 maneiras diferentes, a segunda de
k2 maneiras diferentes, e assim sucessivamente, ento o nmero
total T de maneiras de ocorrer o acontecimento dado por:
T = k1. k2 . k3 . ... . kn

Extrado da Folha de So Paulo, 9 mar. 2013

De acordo com os dados, os aeroportos cujos crescimentos


representam a moda e a mediana podem ser representados,
respectivamente, por
(A) Juazeiro do Norte e Foz do Iguau.
(B) Foz do Iguau e Juazeiro do Norte.
(C) Campinas e Foz do Iguau.
(D) Santos Dumont e Uberaba.
(E) Uberaba e Santos Dumont.
03. Uma escola utiliza um mtodo de avaliao muito diferente dos
tradicionais. O mtodo consiste em:
I. Os alunos so submetidos a nove provas regulares de cada
matria ao longo do ano;
II. O aluno considerado aprovado se a mediana das notas das nove
provas for maior ou igual a sete;
III. Caso o aluno obtenha mediana das notas menor que sete, o
mesmo submetido a uma prova especial de recuperao, na qual
ser aprovado se sua nota for maior ou igual a seis.
As seis primeiras notas do aluno Benedito foram 7, 8, 3, 5, 9, 5.
Sabendo que a mediana das notas de Benedito foi a maior possvel,
ento
(A) Benedito foi reprovado, pois sua mediana foi igual a 4.
(B) Benedito foi reprovado, pois sua mediana foi igual a 6.
(C) Benedito foi aprovado, pois sua mediana foi igual a 6,2.
(D) Benedito foi aprovado, pois sua mediana foi igual a 7.
(E) Benedito foi aprovado, pois sua mediana foi igual a 8.

Exemplo:
O DETRAN decidiu que as placas dos veculos do Brasil sero
codificadas usando-se 3 letras do alfabeto e 4 algarismos. Qual o
nmero mximo de veculos que poder ser licenciado?
Soluo:
Usando o raciocnio anterior, imaginemos uma placa genrica do tipo
PWR-USTZ.
Como o alfabeto possui 26 letras e nosso sistema numrico possui
10 algarismos (de 0 a 9), podemos concluir que: para a 1 posio,
temos 26 alternativas, e como pode haver repetio, para a 2, e 3
tambm teremos 26 alternativas. Com relao aos algarismos,
conclumos facilmente que temos 10 alternativas para cada um dos 4
lugares. Podemos ento afirmar que o nmero total de veculos que
podem ser licenciados ser igual a: 26.26.26.10.10.10.10 que resulta
em 175.760.000. Observe que se no pas existissem 175.760.001
veculos, o sistema de cdigos de emplacamento teria que ser
modificado, j que no existiriam nmeros suficientes para codificar
todos os veculos. Perceberam?
Permutaes Simples
4.1 Permutaes simples de n elementos distintos so os
agrupamentos formados com todos os n elementos e que diferem
uns dos outros pela ordem de seus elementos.
Exemplo: com os elementos A,B,C so possveis as seguintes
permutaes: ABC, ACB, BAC, BCA, CAB e CBA.
4.2 O nmero total de permutaes simples de n elementos
distintos dado por n!, isto
Pn = n! onde n! = n(n-1)(n-2)... .1 .
Exemplos:
a) P6 = 6! = 6.5.4.3.2.1 = 720
b) Calcule o nmero de formas distintas de 5 pessoas ocuparem os
lugares de um banco retangular de cinco lugares.
P5 = 5! = 5.4.3.2.1 = 120
4.3 Denomina-se ANAGRAMA o agrupamento formado pelas letras
de uma palavra, que podem ter ou no significado na linguagem
comum.

SECRETARIA DE ESTADO DA EDUCAO

197

QUMICA
LGEBRA

Exemplo:
Os possveis anagramas da palavra REI so:
REI, RIE, ERI, EIR, IRE e IER.

Nota: o nmero acima tambm conhecido como Nmero Binomial e


indicado por:
n
n!

k k!(n k)!

Permutaes com Elementos Repetidos


Se entre os n elementos de um conjunto, existem a elementos
repetidos, b elementos repetidos, c elementos repetidos e assim
sucessivamente, o nmero total de permutaes que podemos
formar dado por:
n!
Pn(a,b,c,...)
a! b! c!...
Exemplo:
Determine o nmero de anagramas da palavra MATEMTICA. (no
considere o acento)
Soluo:
Temos 10 elementos, com repetio. Observe que a letra M est
repetida duas vezes, a letra A trs, a letra T, duas vezes. Na frmula
anterior, teremos: n = 10, a = 2, b = 3 e c = 2. Sendo k o nmero
procurado, podemos escrever:
k = 10! / (2!.3!.2!) = 151200
Resposta: 151200 anagramas.
Arranjos Simples

Exemplo:
Uma prova consta de 15 questes das quais o aluno deve resolver
10. De quantas formas ele poder escolher as 10 questes?
Soluo:
Observe que a ordem das questes no muda o teste. Logo,
podemos concluir que trata-se de um problema de combinao de 15
elementos com taxa 10.
Aplicando simplesmente a frmula chegaremos a:
C15,10 = 15! / [(15-10)! . 10!] = 15! / (5! . 10!) =
15.14.13.12.11.10! / 5.4.3.2.1.10! = 3003
EXERCCIOS DE FIXAO
01. Num restaurante, so oferecidos 4 tipos de carne, 5 tipos de
massa, 8 tipos de salada e 6 tipos de sobremesa. De quantas
maneiras diferentes podemos escolher uma refeio composta por 1
carne, 1 massa, 1 salada e 1 sobremesa?
(A) 23.
(B) 24.
(C) 401.
(D) 572.
(E) 960.

6.1 Dado um conjunto com n elementos, chama-se arranjo simples


de taxa k, a todo agrupamento de k elementos distintos dispostos
numa certa ordem. Dois arranjos diferem entre si, pela ordem de
colocao dos elementos. Assim, no conjunto E = {a,b,c}, teremos:
a) arranjos de taxa 2: ab, ac, bc, ba, ca, cb.
b) arranjos de taxa 3: abc, acb, bac, bca, cab, cba.
6.2 Representando o nmero total de arranjos de n elementos
tomados k a k (taxa k) por An,k , teremos a seguinte frmula:
n!
An,k
(n k)!

02. O campeonato brasileiro de futebol da srie A tem 20 times que


jogam todos entre si, duas vezes. Ento o nmero total de jogos de
(A) 368. (B) 376.
(C) 380.
(D) 386.
(E) 388.

Obs: fcil perceber que An,n = n! = Pn . (Verifique)


Exemplo:
Um cofre possui um disco marcado com os dgitos 0,1,2,...,9. O
segredo do cofre marcado por uma sequncia de 3 dgitos
distintos. Se uma pessoa tentar abrir o cofre, quantas tentativas
dever fazer (no mximo) para conseguir abri-lo?
Soluo:
As sequncias sero do tipo xyz. Para a primeira posio teremos 10
alternativas, para a segunda, 9 e para a terceira, 8. Podemos aplicar
a frmula de arranjos, mas pelo princpio fundamental de contagem,
chegaremos ao mesmo resultado: 10.9.8 = 720.

03. Observe a tirinha de quadrinhos, a seguir:

Observe que A10,3 = 720.


Combinaes Simples
7.1 Denominamos combinaes simples de n elementos distintos
tomados k a k (taxa k) aos subconjuntos formados por k elementos
distintos escolhidos entre os n elementos dados. Observe que duas
combinaes so diferentes quando possuem elementos distintos,
no importando a ordem em que os elementos so colocados.
Exemplo:
No conjunto E= {a,b,c,d} podemos considerar:
a) combinaes de taxa 2: ab, ac, ad, bc, bd, cd.
b) combinaes de taxa 3: abc, abd, acd, bcd.
c) combinaes de taxa 4: abcd.

A Mnica desafia seus amigos, numa brincadeira de cabo de


guerra.
Supondo que a posio da Mnica pode ser substituda por qualquer
um de seus amigos, e que ela pode ocupar o outro lado, junto com
os demais, mantendo-se em qualquer posio, assim como cada um
dos amigos pode ocupar qualquer posio, o nmero de maneiras
distintas que podem ocorrer nessa brincadeira ser igual a
(A) 60.
(B) 100.
(C) 120. (D) 150.
(E) 600.

7.2 Representando por Cn,k o nmero total de combinaes de n


elementos tomados k a k (taxa k) , temos a seguinte frmula:
n!
Cnk
k!(n k)!

SECRETARIA DE ESTADO DA EDUCAO

198

QUMICA
LGEBRA

AULA 16 PROBABILIDADE

EXERCCIOS DE FIXAO

Experimento Aleatrio
Dizemos que um experimento aleatrio caso seu resultado final
dependa totalmente do acaso, ou seja, aquele cujo resultado
imprevisvel mesmo que haja um grande nmero de repeties do
mesmo experimento.

01. Uma fbrica possui duas mquinas que produzem o mesmo tipo
de pea. Diariamente a mquina M produz 2 000 peas e a mquina
N produz 3 000 peas. Segundo o controle de qualidade da fbrica,
sabe-se que 60 peas, das 2 000 produzidas pela mquina M,
apresentam algum tipo de defeito, enquanto que 120 peas, das 3
000 produzidas pela mquina N, tambm apresentam defeitos. Um
trabalhador da fbrica escolhe ao acaso uma pea, e esta
defeituosa.
Nessas condies, qual a probabilidade de que a pea defeituosa
escolhida tenha sido produzida pela mquina M?
3
1
1
(A)
(B)
(C)
25
100
3
3
2
(D)
(E)
7
3

Espao Amostral
Espao amostral () o conjunto que possui todos os resultados
possveis de um experimento aleatrio.
Evento
Evento (E) um subconjunto qualquer do espao amostral.
Definio
Seja n(E) o nmero de elementos de um evento E e n() o nmero
de elementos do espao amostral que contm E. A probabilidade
do evento E
n(E)
p(E)
n()
Informalmente,
p(E)

nmero de casos favorveis


nmero de casos possveis

02.
Grupo de 200 famlias de So Paulo e do Rio pesquisadas tem
dvida mdia de R$ 1.009 e renda de R$ 2.401
Um mapeamento das famlias endividadas em So Paulo e no Rio
mostra que 46,5% delas fizeram uma dvida nova para pagar uma
antiga. Entre esse grupo de consumidores, comum os que se
desfazem de carros, mquinas de lavar, jogos eletrnicos e joias
para conseguir quitar ou renegociar o quanto devem.

Observao
Se P(A) = 0, dizemos que o evento impossvel.
Se P(A) = 1, dizemos que o evento certo.
0 P(A) 1
Eventos Complementares
O evento complementar de A em ( A ) o subconjunto de todos os
elementos de que no esto em A. Vale que P(A) + P( A ) = 1
Probabilidade da Unio
A probabilidade de ocorrer o evento A ou o evento B
P(A B) = P(A) + P(B) P(A B).
Observao
Dizemos que A e B so eventos mutuamente exclusivos se A B =
0. Nesse caso, P(A B) = 0.

Adaptado de: Folha de So Paulo, 15 ago. 2012.

A probabilidade de uma famlia dessa pesquisa ter pelo menos 4


dvidas
(A) 3%.
(B) 12%.
(C) 15%.
(D) 35%.
(E) 65%.

Probabilidade Condicional
A probabilidade de A, sabendo que j ocorreu B,
P(A B)
P(A / B)
P(B)
Eventos Independentes
Dois eventos so independentes se a ocorrncia de um deles no
altera o resultado do outro, ou seja,
P(A/B) = P(A) e P(B/A) = P(B).
Os eventos A e B so independentes se, e somente se,
P(A B) = P(A).P(B).

03. O resultado do segundo turno das eleies para prefeito de uma


cidade brasileira apresentou os seguintes nmeros:
Candidato A
Candidato B
Votos nulos
Votos em branco

52%
31%
5%
12%

Um eleitor dessa cidade escolhido ao acaso. Sabe-se que ele no


votou no candidato eleito. A probabilidade de que ele tenha votado
em branco
(A) 10%.
(B) 12%.
(C) 15%.
(D) 20%.
(E) 25%.

SECRETARIA DE ESTADO DA EDUCAO

199

QUMICA

LGEBRA

AULA 17 PROPORCIONALIDADE ENTRE GRANDEZAS

EXERCCIOS DE FIXAO

Definimos por grandeza tudo aquilo que pode ser contado e medido,
como o tempo, a velocidade, comprimento, preo, idade, temperatura
entre outros. As grandezas so classificadas em: diretamente
proporcionais e inversamente proporcionais.

01. O painel de um automvel indica o consumo mdio de


combustvel da seguinte forma:
12,5 L / 100 km
Com base nessa informao, podemos inferir que esse automvel
percorre, em mdia, com 1 litro desse combustvel,
(A) 7,2 km.
(B) 7,6 km.
(C) 8,0 km.
(D) 8,4 km.
(E) 8,8 km.

Grandezas diretamente proporcionais


So aquelas grandezas onde a variao de uma provoca a variao
da outra numa mesma razo. Se uma dobra a outra dobra, se uma
triplica a outra triplica, se uma divida em duas partes iguais a outra
tambm divida metade.
Exemplo 1
Se trs cadernos custam R$ 8,00, o preo de seis cadernos custar
R$ 16,00. Observe que se dobramos o nmero de cadernos tambm
dobramos o valor dos cadernos. Confira pela tabela:

Exemplo 2
Para percorrer 300 km, um carro gastou 30 litros de combustvel. Nas
mesmas condies, quantos quilmetros o carro percorrer com 60
litros? E com 120 litros?

Grandezas inversamente proporcionais


Uma grandeza inversamente proporcional quando operaes
inversas so utilizadas nas grandezas. Por exemplo, se dobramos
uma das grandezas temos que dividir a outra por dois, se triplicamos
uma delas devemos dividir a outra por trs e assim sucessivamente.
A velocidade e o tempo so considerados grandezas inversas, pois
aumentarmos a velocidade, o tempo reduzido, e se diminumos a
velocidade, o tempo aumenta.
Exemplo 3
Para encher um tanque so necessrias 30 vasilhas de 6 litros cada
uma. Se forem usadas vasilhas de 3 litros cada, quantas sero
necessrias?

02. O milionrio Pinquio possui um terreno de rea 2.500 m 2, onde


cria 1.000 bois. Com o crescimento do mercado de carne em sua
regio, ele deseja adquirir um terreno maior para aumentar sua
criao e, consequentemente, sua produo. Aps visitar algumas
propriedades, ficou em dvida quanto s propostas dos terrenos dos
senhores Gargamel e Patolino.

A propriedade do senhor Gargamel possui formato


retangular de dimenses 70 m e 80 m;

A propriedade do senhor Patolino tambm possui formato


retangular e suas dimenses so 85 m e 65 m.
Deve-se manter, pelo menos, a rea individual que j existe para
cada animal no terreno atual.
Deste modo, com a inteno de obter o maior aumento possvel no
nmero de animais, o senhor Pinquio deve optar pela compra do
terreno do senhor
(A) Patolino ou do senhor Gargamel, pois o aumento no nmero de
cabeas de gado em ambos os terrenos o mesmo.
(B) Patolino, pois aumentar em 1.210 seu nmero atual de cabeas
de gado.
(C) Patolino, pois aumentar em 1.240 seu nmero atual de cabeas
de gado.
(D) Gargamel, pois aumentar em 1.210 seu nmero atual de
cabeas de gado.
(E) Gargamel, pois aumentar em 1.240 seu nmero atual de
cabeas de gado.
03. Para asfaltar 2 km de estrada so necessrios 30 homens,
durante 12 dias, trabalhando 8 horas por dia. Se dez homens sarem
de frias, quantos dias sero necessrios para asfaltar 4 km de
estrada, trabalhando 12 horas por dia?
(A) 48
(B) 36
(C) 24
(D) 12
(E) 6
04. Certo dia, Adilson, Bento e Celso, funcionrios de uma mesma
empresa, receberam um lote de documentos para arquivar e
dividiram o total de documentos entre eles, na razo inversa de suas
respectivas idades: 24, 30 e 36 anos. Se, ao completarem tal tarefa,
foi observado que a soma dos documentos arquivados por Adilson e
Celso excedia a quantidade arquivada por Bento em 26 unidades,
ento o total de documentos do lote era um nmero
(A) primo.
(B) quadrado perfeito.
(C) mltiplo de 4.
(D) divisvel por 6.
(E) maior do que 60.

Utilizaremos 60 vasilhas, pois se a capacidade da vasilha diminui, o


nmero de vasilhas aumenta no intuito de encher o tanque.
As duas grandezas so muito utilizadas em situaes de
comparao, isto comum no cotidiano. A utilizao da regra de trs
nos casos envolvendo proporcionalidade direta e inversa de
extrema importncia para a obteno dos resultados.

SECRETARIA DE ESTADO DA EDUCAO

200

QUMICA
LGEBRA

AULA 18 REGRA DE TRS


Regra de trs simples
Quando, em uma relao entre duas grandezas, conhecemos trs
valores de um problema e desconhecemos apenas um, poderemos
chegar a sua soluo utilizando os princpios da regra de trs
simples. Para isso, basta que multipliquemos os meios entre si e os
extremos tambm entre si. Acompanhem:

Exemplo: os nmeros 6 e 10 so diretamente proporcionais a 12 e x


respectivamente. Nessas condies, vamos encontrar o valor de x
que torne essa afirmao verdadeira.
6
10

6.x 12 .10 6x 120 x 20


12
x
Exemplos:
a) Um quilo de farinha de trigo suficiente para fazer 12 pes. De
quanta farinha necessito para fazer 18 pes?
Vamos chamar o valor desconhecido de x e montar uma tabela
contendo os valores.

Analisemos as grandezas a fim de saber se so direta ou


inversamente proporcionais entre si.
i) Fixando a grandeza quantidade de homens, vamos relacionar as
grandezas tempo de montagem com nmero de mquinas. Se
dobrarmos o tempo de montagem, dobraremos o nmero de
mquinas. Logo, essas duas grandezas so diretamente
proporcionais;
ii) Fixando a grandeza nmero de mquinas, vamos relacionar as
grandezas quantidade de homens com tempo de montagem. Se
dobrarmos o nmero de homens, teremos reduzido metade o
tempo de montagem. Logo, essas duas grandezas so inversamente
proporcionais.
iii) Sabendo dessas informaes, basta escrevermos a proporo de
acordo com a tabela acima;
iv) Como temos grandezas inversamente proporcionais, devemos
inverter uma das fraes;

12 15 16
12 240
4800

240 .x 400 .12 x


x 20
x
8 50
x
400
240
Concluso: Com 15 homens, sero construdas 50 mquinas em 20
dias.

Inicialmente teremos que analisar se as grandezas quantidade de


farinha de trigo e nmero de pes so inversa ou diretamente
proporcionais.
i) Se duplicarmos a quantidade de farinha de trigo, a quantidade de
pes tambm duplicar. Se triplicarmos a farinha, os pes tambm
sero triplicados, e assim por diante. Sendo assim, somos levados a
concluir que essas duas grandezas so diretamente proporcionais;
ii) Sabendo dessa informao, basta escrevermos a proporo de
acordo com o quadro acima e partir para sua soluo;
iii) As flechas no mesmo sentido indicam que as grandezas so
diretamente proporcionais.
1 12
18
3

12 x 18 x
x , ou seja, x = 1,5 kg.
x 18
12
2
Concluso: para fazer 18 pes precisaremos de 1,5 kg de farinha de
trigo.
Regra de trs composta
Quando trabalhamos com trs grandezas, direta ou inversamente
proporcionais e, num determinado problema, existem seis valores,
dos quais cinco so conhecidos e apenas um desconhecido, podese encontrar o valor da incgnita atravs da regra de trs composta.
Exemplo:
Se 8 homens levam 12 dias montando 16 mquinas, ento, nas
mesmas condies, 15 homens levaro quantos dias para montar 50
mquinas?
Vamos chamar o valor desconhecido de x e montar uma tabela
contendo os valores:

SECRETARIA DE ESTADO DA EDUCAO

201

EXERCCIOS DE FIXAO
01. Para asfaltar 2 km de estrada so necessrios 30 homens,
durante 12 dias, trabalhando 8 horas por dia. Se dez homens sarem
de frias, quantos dias sero necessrios para asfaltar 4 km de
estrada, trabalhando 12 horas por dia?
(A) 48
(B) 36
(C) 24
(D) 12
(E) 6
02. Um trabalhador, para ganhar R$ 2.400,00 em 2 meses,
trabalhou 8 horas por dia. Se tivesse trabalhado 10 horas por dia
durante 5 meses, ento teria que receber o valor de
(A) R$ 7.500,00.
(B) R$ 6.800,00.
(C) R$ 7.680,00.
(D) R$ 7.800,00.
(E) R$ 7.200,00.

03. Em 8 horas, 20 caminhes descarregam 160m3 de areia. Em


5 horas, quantos caminhes sero necessrios para descarregar
125m3?
(A) 20
(B) 25
(C) 30
(D) 32

(E) 35

GABARITO:
01.C

02. A

03. B

QUMICA

LGEBRA

AULA 01 : ESTRATGIAS DE LEITURA


Nesta aula, inicialmente abordaremos algumas estratgias que
facilitaro a leitura e compreenso de textos na Lngua Inglesa. Para
tanto, importante que conheamos os trs nveis bsicos de
compreenso de um texto:
1)General Comprehension
(Compreenso Geral) obtida atravs de uma leitura rpida para
se captar as informaes gerais do texto, ou seja, o que mais
importante o assunto abordado.
2)Main Points Comprehension
(Compreenso dos Pontos Principais) Neste caso necessrio
que nos detenhamos um pouco mais em busca das informaes
principais do texto, observando cada pargrafo para identificar os
dados especficos que mais interessam ao leitor.
3)Detailed Comprehension
(Compreenso Detalhada) Nesse tipo de leitura h necessidade
de mais aprofundamento que os anteriores. Exige a compreenso
detalhada do texto e demanda, por isso, muito mais tempo.
Em quaisquer dos tipos de compreenso abordados acima, as dicas
e habilidades a seguir so de grande valia.
a)Prediction Significa inferir o contedo de um texto atravs de
seu conhecimento prvio sobre o tema
( background ); atravs do contexto semntico (palavras de um
mesmo grupo); contexto lingustico (pistas gramaticais); contexto
no-lingustico (gravuras, grficos, tabelas, nmeros, etc, ...),
conhecimento da estrutura do texto ( lay out, ttulo, subttulo...).

01. Segundo o texto


a) uma jovem namorada estava tentando se casar com um japons..
b) um carteiro estava tentando convencer a namorada a casar-se
com ele.
c) um jovem japons tentou convencer sua namorada a casar-se
com ele.
d) uma jovem japonesa estava tentando convencer seu namorado a
casar-se com ela.
e) um jovem casou-se com sua namorada atravs de 700 cartas de
amor.
02. Baseado no texto,
a) o japons escreveu cerca de 700 cartas de amor para sua
namorada tentando acabar o namoro.
b) a garota aceitou casar-se com o jovem japons apesar de ser
mais jovem que ele.
c) o carteiro, que entregou fielmente todas as cartas a garota, nunca
mais a encontrou.
d) o japons tentou convencer o carteiro a casar-se com sua
namorada.
e) a garota decidiu casar-se com o carteiro que lhe entregou todas as
700 cartas de amor do japons.
03. O verbo convince
no texto acima, poderia ser
substitudo , corretamente, por:
a) resist
d) convey
b) persist
e) understand
c) persuade

b)Cognates Um item muito importante para facilitar a


compreenso de um texto so os cognatos (transparentes) que
representam de 20% a 25% de todas as palavras de um texto. Eles
se dividem em:
b. Idnticos: chocolate, crime, vrus, radio, hospital, social, hotel,...
b. Semelhantes: telephone, apartment, industry, violence,...
b. Vagamente Parecidos: activity, drug, responsible, computer,

TEXTO 02

ATENO!
Os falsos cognates (false friends) so comuns e devemos
traduzi-los, no isoladamente, mas sim, dentro do contexto para
evitarmos possveis atropelos (Parents = pais; Relatives = parentes;
Realize = perceber; Accomplish = realizar; Pretend = fingir, Actually
= realmente; Currently = atualmente...).
c)Repeated Words Se uma palavra aparece vrias vezes no texto,
isto significa que ela importante para a compreenso do mesmo
d)Typographical Evidences So smbolos, letras maisculas,
negrito, itlico, numerais, neologismos ( shopping center, lay out,
outdoor, self service, fast food, ...)
STRATEGIES

It will be spring soon,


It will be spring soon
And I, whose childhood
Is a forgotten boredom
Feel like a child
Who comes on a scene
Of adults reconciling
And can understand nothing
But the unusual laughter
And starts to be happy.
(Philip Larkin)

VOCABULARY
. will be: _____________
. spring:_____________
. soon: ______________
. whose:_____________
. childhood: _________
. forgotten:__________
. boredom:__________
. feel: ______________
. like: ______________
. child:_____________
. come on:__________
. who: ______________
. can: ______________
. understand:________
. unusual: ___________
. laughter: ___________
. starts: _____________
. happy: ____________

a) Skimming - leitura rpida para ter-se uma ideia central do texto


b) Scanning - leitura com o objetivo de encontrar algumas
informaes especficas no texto.
c) Selectivity leitura seletiva, isto ,
selecionar os trechos onde se quer encontrar uma determinada
informao, pargrafos, por exemplo.

SECRETARIA DE ESTADO DA EDUCAO

TEXTO 01
A young Japanese man trying to convince his girlfriend to marry
him, wrote some seven hundred love letters to her.
His try finally brought a positive result: the girl decided to marry the
postman, who faithfully delivered her all the letters.
(slightly adapted from Readers digest)

202

QUMICA
INGLS

AULA 02 : VOCABULRIO
TEXTO 01

VOCABULARY
. thought: ______________________.
. dark: ________________________
. leave alone: ________________________
. all in all: ___________________________
. just: ______________________________
. another: ___________________________
. brick: _____________________________
. wall: ______________________________
02. Estes versos so da cano Another Brick in the Wall
do grupo ingles Pink Floyd. Sucesso internacional, a
cano basicamente contrria a(o):
A) a rigidez das escolas.
B) educao formal.
C) limitar as idias dos alunos.
D) todos os professores e diretores.
E) liberdade excessiva dos alunos.

VOCABULARY
. servings: _____________________.
. bread: _____________________
. rice: _______________________
. pasta______________________
. meat: ______________________
. poultry: ____________________
. fish: _______________________
. dry beans;__________________
. eggs: _____________________
. nuts: ______________________
. cheese: ____________________
. fats: _______________________
. oils: _______________________
. enjoy: _____________________
. sparingly: __________________

TEXTO 03
Brazil's economic boom is attracting European professionals who are
finding it increasingly difficult to find jobs at home. Officials say the
number of visas granted to Europeans grew 20% in the first quarter
of 2011, compared to the same period of 2010. A recently launched
Facebook page offering advice to foreigners willing to work in the
country wooed 20,000 followers in just two days. With eyes set on
the 2014 World Cup and hosting the Olympic Games only two years
later, not even the prospect of a global economic downturn seems to
influence the country's optimism towards the future.
Source:
BBC
(Adapted from: http://www.bbc.co.uk/news/
November/2011)

01. O uso de grficos muito comum para a quantificao de forma


ilustrada. Na figura acima, temos uma pirmide alimentar, que
quantifica as pores ideais de ingerncia de tipos de diferentes
nutrientes. Sabendo-se que a busca por uma alimentao saudvel
constitui um dos principais objetivos do homem atual, e com base na
pirmide alimentar apresentada acima, conclui-se que
A) laticnios e protenas devem ser ingeridos em
quantidade similar.
B) recomendvel ingerir muito mais protenas do que
carboidratos.
C) devemos ingerir no mnimo 8 pores dirias de lquidos
em geral.
D) frutas e verduras podem e devem ser consumidas de
forma irrestrita.
E) doces e alimentos gordurosos so estritamente
proibidos de serem consumidos.
TEXTO 02
Another brick in the wall
We dont need no education
We dont need no thought control
No dark sarcasm in the classroom
Teachers leave us kids alone
Hey! Teacher! Leave us kids alone!
All in all its just another brick in the wall.
All in all youre just another brick in the wall.
FLOYD, Pink. Disponvel em:
<http://www.vagalume.com.br/pink-floyd/
another-brick-in-the-wall-traducao.html>.
Acesso em: 12 out. 2013

SECRETARIA DE ESTADO DA EDUCAO

203

VOCABULARY
. boom: __________________________
. find: ___________________________
. increase: _______________________
. jobs: ___________________________
. granted: ________________________
. grew: __________________________
. quarter: ________________________
. launched: ______________________
. advice: ________________________
. foreigners: _____________________
. willing: ________________________
. woo: __________________________
. followers: ______________________
. hosting: _______________________
. downturn: _____________________.
. seem: ________________________.
. towards: ______________________.
03. Os olhos do mundo esto voltados para o Brasil neste
ano de 2014, quando o pas sediar um dos maiores
eventos esportivos da sua histria, a Copa do Mundo de
Futebol. Dois anos depois ser a vez das Olimpadas.
Com base no texto acima, chega-se a concluso que o
autor
A) encontra-se muito desencorajado com as altas taxas de
inflao nos pases europeus.
B) est bastante preocupado com o grande nmero de
profissionais europeus desempregados.
C) tem certeza de que a crise econmica mundial afetar a
Copa do Mundo e os Jogos Olmpicos no Brasil.
D) pessimista em relao aos resultados brasileiros na
Copa do Mundo de 2014 e nas Olimpadas de 2016.
E) diz que os futuros eventos no Brasil atrairo
profissionais estrangeiros apesar da crise econmica
mundial.

QUMICA
INGLS

AULA 03 : TEXT COMPREHENSION


Passado o calor das discusses em volta das eleies presidenciais
2014 no Brasil, o pas retoma a sua rotina, com o seu governante
para 2015 2018 j definido. Abaixo, voc vai ler depoimentos de
dois eleitores brasileiros, numa reportagem da BBC de Londres.
TEXTO 01
Angelica Mari, Sao Paulo
27 October 2014 Last updated at 13:28 GMT

I voted for Dilma Rousseff. I hope that she will have the guts to be
more daring and implement the changes people - and I mean every
Brazilian - want to see: More investment in health and education,
less bureaucracy and better conditions to do business, and of
course, less corruption.
It has been an exciting couple of months of political campaigning people have discussed politics more than I have seen in my lifetime.
VOCABULARY
. hope: ___________________
. have the guts: ____________
. daring:__________________
. changes: ________________
. I mean: _________________
. health: __________________
. bureaucracy:_____________
. better:__________________
. of course:_______________
. have seen: _____________
. lifetime: ________________

. incumbent: _________________
. challenger: _________________
. win: ______________________
. quite: _____________________
. worried: ___________________
. under: ____________________
. after: _____________________
. curently: __________________
01. Com base nos textos acima, deduz-se que
a) Angelica Mari, eleitora de Dilma, acha que o
pas est vivendo uma excelente fase em todos
os setores.
b) Tarlei Lemos, eleitor de Acio Neves, no
tinha expectativas de que seu candidato viesse a
vencer as eleies.
c) Angelica Mari apia integralmente a gesto de
Dilma Rousseff, achando que esta tem investido
o suficiente em sade e educao.
d) Tarlei Lemos, ex-membro do PT, votou em
Acio Neves na esperana de renovao do
pensamento poltico brasileiro.
e) apesar de terem escolhido candidatos
diferentes, tanto Angelica quanto Tarlei anseiam
por mudanas em diversos setores do pas.
TEXTO 03:

TEXTO 02
Tarlei Lemos Pereira, So Paulo
I think Brazil deserves much more.

Corruption is a cancer here, something simply


unacceptable. There is no real democracy with corruption.
We're living a false democracy.
Our economy isn't growing, health and education services
are extremely poor. I voted against the incumbent
president. I hoped the challenger would win this election.
I'm quite worried about the corrosion of Brazil's institutions
after 12 years under PT. Members of PT are currently
serving their sentences in penitentiaries but corruption is
part of the Brazilian politicians' culture.

O cartum sugere que o professor


A) pressiona psicologicamente os alunos com
relao s suas responsabilidades.
B) estimula a conservao adequada de
embries de porcos e vacas.
C) motiva os estudantes a se prepararem para
os testes.
D) questiona os meios de preservao de certos
tipos de animais.
E) ameaa os estudantes que maltratarem os
animais citados.

(Extracted and adapted from


http://www.bbc.com/news/world-29785043)

VOCABULARY
. deserve: ___________________
. much more: ________________
. unnacceptable: _____________
. were: ____________________
. living: ____________________
. growing: __________________
. poor: ______________________
. against:____________________

SECRETARIA DE ESTADO DA EDUCAO

204

QUMICA
INGLS

AULA 04: TEXT COMPREHENSION

.
.
.
.
.
.

O Ebola um vrus altamente infeccioso que pode atingir


uma taxa de mortalidade de at 90%. O vrus tem cinco
cepas (origem ou linhagem). O mais letal deles, chamado
Zaire, predominante na epidemia, que desde maro de
2014 atinge a frica Ocidental e considerada a maior da
histria.

01. Com base na leitura do texto 01, pode-se


inferir que a respeito do novo medicamento
desenvolvido para tratar o vrus do ebola, os
cientistas tm agora permisso pra test-lo nos
seres humanos.

TEXTO 01
Ebola drug to be tested on humans
Scientists developing a drug to treat the Ebola virus have
been given permission to test it on humans. Authorities in
the United States approved limited clinical trials after the
treatment was shown to be effective in monkeys.
Ebola is even more deadly to monkeys than humans. If
they get it they die. However in tests, the new treatment
cured 60% of them. It also proved 100% effective in dealing
with the closely-related Marburg virus. The hope is that
these results will be replicated or even bettered when the
drug is given to humans.
Ebola has killed around 1,200 people since it was
discovered during the mid-1970s in Zaire, now the
Democratic Republic of Congo. So far, outbreaks have
been limited to Africa where its thought to be carried by
fruit bats. Ebola can be transmitted through contact with
bodily fluids. Symptoms include nausea and vomiting, with
victims suffering internal bleeding and organ failure before
they die. The viruss high mortality rate has led to fears that
it could be used in bioterrorism.
Funding for research into Ebola was stepped up in the
United States following the attacks of September 11th
2001. The new treatment has been developed by the US
Army Medical Research Institute of Infectious Diseases,
along with a private company called AVI BioPharma.
Details of the research appear in the science journal Nature
Medicine.

A) esto cticos quanto ao seu efeito nos


humanos.
B) temem que o medicamento agrave os
sintomas da doena.
C) esto usando nos seres humanos desde
meados dos anos 70.
D) acham que o medicamento ser totalmente
eficaz nos humanos.
E) os cientistas tm agora permisso pra test-lo
nos seres humanos.
02. Aps ler o texto 01, conclui-se que o vrus do
Ebola

MACKENZIE, Lain. Ebola drug to be tested on humans. Disponvel


em:<www.bbc.co.uk/worldservice/learningenglish/wordsinthenews/2010/08/100
823_witn_ebola_page.shtml>. Acesso em: 21 jul. 2013.

A) est matando todos os morcegos frugvoros


da rea.
B) provvel matar menos humanos do que
macacos.
C) provou-se ser to fatal para os macacos
quanto para os humanos.
D) pode ser mais fatal para os humanos do que
para os macacos.
E) geralmente repelido pelo sistema
imunolgico dos macacos.
03. Infere-se pela leitura do texto 01, que o Ebola
passa de uma pessoa para outra atravs de (o)
(a)

VOCABULARY
. treat: _____________________________
. clinical trials: _______________________
. even more: ________________________
. deadly: ___________________________
. was shown: _______________________
. if: _______________________________
. however: _________________________
. also: _____________________________
. dealing: __________________________
. closely related: ___________________
. even: ____________________________
. bettered: _________________________
. is given: __________________________
. has killed: ________________________
. discovered: _______________________
. mid / middle: ______________________
. so far: ___________________________
. outbreaks: ________________________
. its thought: _________________________
. carried: ____________________________
. fruit bats: _________________________
. bodily fluids: ________________________
. suffering: __________________________
. bleeding: __________________________
. failure: ____________________________
. rate: _____________________________
. has led to: ________________________
. fears: ____________________________
. funding: __________________________

SECRETARIA DE ESTADO DA EDUCAO

research:_________________________
stepped up: _______________________
followed: _________________________
along with: ________________________
diseases: _________________________
appear: __________________________

a) contato com o morcego.


b) ar poludo.
c) gua contaminada.
d) contato sexual.
e) sangue ou saliva.

205

QUMICA
INGLS

AULA O5: INTERPRETAO TEXTUAL

02. O sarcstico gato Garfield, junto com seu dono John,


so as estrelas de uma das tirinhas mais famosas da
histria que tem como objetivo fazer humor. Assim, o
humor na tirinha acima consiste em:
a) Apesar de John ter se gabado por abrir a porta para Liz,
ela no gostou da atitude.
b) John fica chateado por Liz no ter expressado
agradecimento quando ele abriu a porta para ela e Garfield
debocha dele.
c) John teve boa inteno com Liz, porm a porta que ele
abriu era do armrio, o que caracteriza burrice para
Garfield.
d) Liz no entende o ato gentil de John, pois ele abriu a
porta do armrio.
e) Garfield debocha do fato de John ter se auto
denominado um cavalheiro, pois ele nunca fez nada gentil.

TEXTO 01
SCHOOL SHOOTINGS
Incidents were classified as school shootings when a
firearm was discharged inside a school building or on
school or campus grounds, as documented in publicly
reported news accounts. This includes assaults, homicides,
suicides, and accidental shootings. Incidents in which guns
were brought into schools but not fired there, or were fired
off school grounds after having been possessed in schools,
were not included. This list includes incidents meeting the
above criteria that were brought to our attention after our
School Shootings Analysis was issued on February 10,
2014. Incidents were identified through media reports, so
this is likely an undercount of the true total.
(fromhttp://U.S.
everytown.org/article/schoolshootings/)

TEXTO 03
Bird Language
Trying to understand the words
Uttered on all sides by birds,
I recognise in what I hear
Noises that betoken fear.

VOCABULARY
. shootings: tiroteios; tiros
. firearm: arma de fogo
. discharged: _______________________
. campus grounds: __________________
. news: ___________________________
. accounts: ________________________
. assaults: _________________________
. were brought: _____________________
. fired: ____________________________
. was issued: _______________________
. meeting: __________________________
. above: ___________________________
. through: __________________________
. likely: ____________________________
. undercount:_______________________

Though some of them,


Im certain, must
Stand for rage, bravado, lust,
All other notes that birds employ
Sound like synonyms for joy.
W. H. Auden

VOCABULARY
. words: __________________________
. uttered: _________________________
. sides: __________________________
. recognise: _______________________
. hear: ___________________________
. noises: __________________________
. betoken: _________________________
. fear: _____________________________
. stand for: _________________________
. rage: ____________________________
. lust: ____________________________
. employ: _________________________
. joy: _____________________________
3. Que grupo de palavras do poema acima (texto 03) tem
uma conexo semntica?
a) Recognise, certain, uttered, hear.
b) Uttered, words, hear, sound.
c) Fear, uttered, hear, sound.
d)Hear, some, uttered, recognise.
e) Uttered, hear, sound, sides.

01. Em relao ao texto 01, pode-se depreender


que o seu contedo
a) define e detalha o fenmeno dos tiroteios nas
escolas americanas.
b) orienta como proceder durante um incidente
grave em uma escola americana.
c) instrui os responsveis pelas escolas como
agir em caso de tiroteio.
d) uma sequncia de aes a serem tomadas
durante um tiroteio.
e) exime a direo das escolas americanas de
toda a culpa pelos tiroteios ocorridos.
TEXTO 02

04. No texto 03, a expresso stand for utilizado como:


a) Advrbio.
d) Verbo Frasal
b) Adjetivo.
e) Preposio
c) Substantivo.
04. O poeta, no texto 03, acredita que a linguagem dos
pssaros expressa
a) mais luxria que ira.
d) apenas bravado e luxria
b) apenas ira.
e) apenas medo e tristeza.
c) mais alegria que ira.

SECRETARIA DE ESTADO DA EDUCAO

206

QUMICA
INGLS

Aula 06 :TEXT COMPREHENSION

. the worst:_____________________________
.highways:______________________________
. drunk:________________________________
.while:_________________________________
.law:__________________________________
.behavioral:____________________________
.against:_______________________________
.behavioral:_____________________________
.instead of:_____________________________
.whatever:______________________________
02. Aps a leitura do texto 02, podemos inferior que a idia
principal do mesmo encontra-se em
A) Uma noite sem dormir pode trazer benefcios.
B) Dormir demais beneficial.
C) Privao do sono faz parte da natureza humana.
D) Espera-se que todo mundo durma igual a um beb.
E) No dormir bem pode ser malfico sade.
TEXTO 03

TEXTO 01

FRAN. Disponvel em: <www.cartoomstock.com>.


Acesso em:21 jul. 2013.

01. A partir da fala do paciente neste cartum, podemos


inferir que ele
A) costuma pesquisar na internet sobre problemas de
sade.
B) no gosta de ir ao medico regularmente.
C) confia 100 por cento nos diagnsticos da internet.
D) no pretende levar em conta o diagnostico do mdico.
E) no tem intimidade com os computadores.
TEXTO 02
SLEEPY PEOPLE

IMAGINE ( J. Lennon)
Imagine theres no heaven,
Its easy if you try.
No hell Bellow us,
Above us only Sky
Imagine all the people
Living for today...

What does a chronic lack of sleep do to us? As well as making us


irritable and unhappy, it also reduces our motivation and ability to
work. This effect has serious implications for society in general.
Doctors, for example, are often chronically sleep deprived, 5
especially when they are on night call and may get less than three
hours sleep Lack of sleep can seriously impair their mood,
judgment, and ability to make decisions. Politicians are often jetlagged after crossing time zones. World summit meetings called to
deal with a crisis often result in decisions being made after marathon
10 sessions when everyone is severely sleep deprived. Human error
caused by tiredness contributed to the worst nuclear accident in
history in Chernobyl in 1986, when tired engineers, in the early hours
of the morning, made a series of mistakes with catastrophic results.
On our streets and highways, lack of sleep kills thousands of people
15 every year. Tests show that a tired driver can be as dangerous as
a drunk driver. However, driving when drunk is against the law while
driving when exhausted isnt.
As the behavioral biologist Paul Martin says, it is ironic that we
admire people who function on very little sleep instead of criticizing
them for being irresponsible. Our world would be a much safer,
happier place if everyone, whatever their job, slept eight hours a
night.
OXENDEN, Clive and Latham-Koenig, Christina American
English File 4, p. 73. [2010?]

VOCABULARY
. sleepy:________________________________
. lack of:_______________________________
. as well as:_____________________________
. deprived:______________________________
. mood:________________________________
. jet-lagged:_____________________________
. summit meetings: _______________________
. to deal:_______________________________
. tiredness:_____________________________

SECRETARIA DE ESTADO DA EDUCAO

207

Imagine theres no country,


It isnt hard to do.
Nothing to kill or die for,
And no religion, too.
Imagine all the people
Living life in peace...
Em apenas duas dcadas, Lennon viveu o
melhor e o pior da fama. Nos anos 1960, era um
dos mais talentosos dos Beatles, um rapaz
alegre e irnico que ditava a moda e o
comportamento da gerao "paz e amor". Nos
anos 1970, j casado com a artista plsticaYoko
Ono, engajou-se em campanhas pacifistas e se
envolveu em diversas polmicas. Nos ltimos
anos de vida, era um pacato pai de famlia.
(UOL, Atualidades - Jos Renato Salatiel
Especial para a Pgina 3 Pedagogia &
Comunicao, discretamente adaptado / questo
adaptada do processo seletivo 2011 das
Faculdades Santo Agostinho pelo Prof Ailton
Motta.)
03. Analisando os versos da cano Imagineapresentada
acima, infere-se que:
a) a letra de John Lennon ainda atual, visto que o mundo
est longe de construir a paz.
b) a letra contraria o texto, pois a gerao paz e amor
contestava princpios morais daquela gerao.
c) a luta de Lennon pela paz no mundo negava os valores
cristos fortemente marcados naquela poca.
d) a letra de Lennon, criticando os valores, atual por
tratar de assuntos polticos.
e) a luta de Lennon pela paz no mundo no encontrou
adeptos, tornando-se ele, ento, um idealista isolado.

QUMICA
INGLS

You might also like